All Lewis chapters except Ch 64 and Ch 66

Lakukan tugas rumah & ujian kamu dengan baik sekarang menggunakan Quizwiz!

Several patients call the outpatient clinic and ask to make an appointment as soon as possible. Which patient should the nurse schedule to be seen first? a. A 44-yr-old with sickle cell anemia who says his eyes always look sort of yellow b. A 23-yr-old with no previous health problems who has a nontender lump in the axilla c. A 50-yr-old with early-stage chronic lymphocytic leukemia who reports chronic fatigue d. A 19-yr-old with hemophilia who wants to learn to self-administer factor VII replacement

B The patient's age and presence of a nontender axillary lump suggest possible lymphoma, which needs rapid diagnosis and treatment. The other patients have questions about treatment or symptoms that are consistent with their diagnosis but do not need to be seen urgently.

When caring for a young adult patient who has abnormalities in the cytochrome P450 (CYP 450) gene, which action will the nurse include in the patient's plan of care? a. Teach that some medications may not work as effectively. b. Teach about genetic risk for cystic fibrosis in any children. c. Suggest that the patient make heart healthy lifestyle choices. d. Discuss the need for screening mammograms starting at age 30.

ANS: A The CYP 450 gene affects the metabolism of many medications, and they may not work as effectively or may have unexpected toxic effects. The CYP 450 gene does not affect risk for breast cancer, cystic fibrosis, or coronary artery disease

5. A 23-year-old patient with a history of muscular dystrophy is hospitalized with pneumonia. Which nursing action will be included in the plan of care? a. Logroll the patient every 2 hours. b. Assist the patient with ambulation. c. Discuss the need for genetic testing with the patient. d. Teach the patient about the muscle biopsy procedure.

ANS: B Because the goal for the patient with muscular dystrophy is to keep the patient active for as long as possible, assisting the patient to ambulate will be part of the care plan. The patient will not require logrolling. Muscle biopsies are necessary to confirm the diagnosis but are not necessary for a patient who already has a diagnosis. There is no need for genetic testing because the patient already knows the diagnosis.

4. The nurse instructs a patient who has osteosarcoma of the tibia about a scheduled above-the-knee amputation. Which statement by a patient indicates that additional patient teaching is needed? a. I will need to participate in physical therapy after surgery. b. I did not have this bone cancer until my leg broke a week ago. c. I wish that I did not have to have chemotherapy after this surgery. d. I can use the patient-controlled analgesia (PCA) to control postoperative pain.

ANS: B Osteogenic sarcoma may be diagnosed following a fracture, but it is not caused by the injury. The other patient statements indicate that patient teaching has been effective.

A 52-yr-old patient has a new diagnosis of pernicious anemia. The nurse determines that the patient understands the teaching about the disorder when the patient states, a. "I need to start eating more red meat and liver." b. "I will stop having a glass of wine with dinner." c. "I could choose nasal spray rather than injections of vitamin B12." d. "I will need to take a proton pump inhibitor such as omeprazole (Prilosec)."

C Because pernicious anemia prevents the absorption of vitamin B12, this patient requires injections or intranasal administration of cobalamin. Alcohol use does not cause cobalamin deficiency. Proton pump inhibitors decrease the absorption of vitamin B12. Eating more foods rich in vitamin B12 is not helpful because the lack of intrinsic factor prevents absorption of the vitamin.

When caring for a patient on the first postoperative day after an abdominal aortic aneurysm repair, which assessment finding is most important for the nurse to communicate to the health care provider? a.Presence of flatus b.Hypoactive bowel sounds c.Maroon-colored liquid stool d.Abdominal pain with palpation

C Loose, bloody (maroon colored) stools at this time may indicate intestinal ischemia or infarction and should be reported immediately because the patient may need an emergency bowel resection. The other findings are normal on the first postoperative day after abdominal surgery.

Which finding about a patient with polycythemia vera is most important for the nurse to report to the health care provider? a. Hematocrit 55% b. Presence of plethora c. Calf swelling and pain d. Platelet count 450,000/mL

C The calf swelling and pain suggest that the patient may have developed a deep vein thrombosis, which will require diagnosis and treatment to avoid complications such as pulmonary embolus. The other findings will also be reported to the health care provider but are expected in a patient with this diagnosis.

Which patient information is most important for the nurse to monitor when evaluating the effectiveness of deferoxamine (Desferal) for a patient with hemochromatosis? a. Skin color c. Liver function b. Hematocrit d. Serum iron level

D Because iron chelating agents are used to lower serum iron levels, the most useful information will be the patient's iron level. The other parameters will also be monitored, but are not the most important to monitor when determining the effectiveness of deferoxamine.

A patient is being evaluated for postthrombotic syndrome. Which assessment will the nurse perform? a.Ask about leg pain with exercise. b.Determine the ankle-brachial index. c.Assess capillary refill in the patient's toes. d.Inspect for presence of lipodermatosclerosis.

D Clinical signs of postthrombotic syndrome include lipodermatosclerosis. In this situation, the skin on the lower leg becomes scarred, and the leg becomes tapered like an "inverted bottle." The other assessments would be done for patients with peripheral arterial disease.

A patient with paraplegia resulting from a T9 spinal cord injury has a neurogenic reflexic bladder. Which action will the nurse include in the plan of care? a. Teach the patient the Credé method. b. Instruct the patient how to self-catheterize. c. Catheterize for residual urine after voiding. d. Assist the patient to the toilet every 2 hours.

Instruct the patient how to self-catheterize. Because the patient's bladder is spastic and will empty in response to overstretching of the bladder wall, the most appropriate method is to avoid incontinence by emptying the bladder at regular intervals through intermittent catheterization. Assisting the patient to the toilet will not be helpful because the bladder will not empty. The Credé method is more appropriate for a bladder that is flaccid, such as occurs with areflexic neurogenic bladder. Catheterization after voiding will not resolve the patient's incontinence.

Which action will the nurse take when caring for a 46-year-old patient who develops tetanus from an injectable substance use? a. Avoid use of sedatives. b. Provide a quiet environment. c. Check pupil reaction to light every 4 hours. d. Provide range-of-motion exercises several times daily.

Provide a quiet environment. In patients with tetanus, painful seizures can be precipitated by jarring, loud noises, or bright lights, so the nurse will minimize noise and avoid shining light into the patient's eyes. Range-of-motion exercises may also stimulate the patient and cause seizures. Although the patient has a history of injectable drug use, sedative medications will be needed to decrease spasms.

While close family members are visiting, a patient has a respiratory arrest, and resuscitation is started. Which action by the nurse is best? a. Tell the family members that watching the resuscitation will be very stressful. b. Ask family members if they wish to remain in the room during the resuscitation. c. Take the family members quickly out of the patient room and remain with them. d. Assign a staff member to wait with family members just outside the patient room.

ANS: B Evidence indicates that many family members want the option of remaining in the room during procedures such as cardiopulmonary resuscitation (CPR) and that this decreases anxiety and facilitates grieving. The other options may be appropriate if the family decides not to remain with the patient.

The nurse estimates the extent of a burn using the rule of nines for a patient who has been admitted with deep partial-thickness burns of the anterior trunk and the entire left arm. What percentage of the patient's total body surface area (TBSA) has been injured?

ANS: 27% When using the rule of nines, the anterior trunk is considered to cover 18% of the patient's body and each arm is 9%.

The nurse is caring for a 33-year-old patient who arrived in the emergency department with acute respiratory distress. Which assessment finding by the nurse requires the most rapid action? a. The patient's PaO2 is 45 mmHg. b. The patient's PaCO2 is 33 mmHg. c. The patient's respirations are shallow. d. The patient's respiratory rate is 32 breaths/minute.

a. The patient's PaO2 is 45 mmHg. -The PaO2 indicates severe hypoxemia and respiratory failure. Rapid action is needed to prevent further deterioration of the patient. Although the shallow breathing, rapid respiratory rate, and low PaCO2 also need to be addressed, the most urgent problem is the patient's poor oxygenation.

A patient with ARDS is placed in the prone position. When prone positioning is used, which information obtained by the nurse indicates that the positioning is effective? a. The patient's PaO2 is 89 mmHg, and the SaO2 is 91%. b. Endotracheal suctioning results in clear mucous return. c. Sputum and blood cultures show no growth after 48 hours. d. The skin on the patient's back is intact and without redness.

a. The patient's PaO2 is 89 mmHg, and the SaO2 is 91%. -The purpose of prone positioning is to improve the patient's oxygenation as indicated by the PaO2 and SaO2. The other information will be collected but does not indicate whether prone positioning has been effective.

The nurse teaches a patient with *chronic bronchitis* about a new prescription for *Advair Diskus (combined fluticasone and salmeterol)*. Which action by the patient would indicate to the nurse that teaching about medication administration has been successful? a. The patient shakes the device before use. b. The patient rapidly inhales the medication. c. The patient attaches a spacer to the Diskus. d. The patient performs huff coughing after inhalation.

b. The patient *rapidly inhales* the medication. The patient should inhale the medication rapidly. Otherwise the dry particles will stick to the tongue and oral mucosa and not get inhaled into the lungs. Advair Diskus is a dry powder inhaler; shaking is not recommended. Spacers are not used with dry powder inhalers.

The HCP is considering the use a sumatriptan (Imitrex) for a 54-year-old male patient with migraine headaches. Which information obtained by the nurse is most important to report to the HCP? a. The patient drinks 1 to 2 cups of coffee daily. b. The patient had recent acute myocardial infarction. c. The patient has had migraine headaches for 30 years. d. The patient has taken topiramate (Topamax) for 2 months.

b. The patient had recent acute myocardial infarction. -The triptans cause coronary artery vasoconstriction and should be avoided in patients with coronary artery disease. The other information will be reported to the hCP, but none of it indicates that sumatriptan would be inappropriate treatment.

The nurse in the emergency department receives arterial blood gas results for four recently admitted patients with obstructive pulmonary disease. The results for which patient will require the most rapid action by the nurse? a. pH 7.28, PaCO2 50 mm Hg, and PaO2 58 mm Hg b. pH 7.48, PaCO2 30 mm Hg, and PaO2 65 mm Hg c. pH 7.34, PaCO2 33 mm Hg, and PaO2 80 mm Hg d. pH 7.31, PaCO2 58 mm Hg, and PaO2 64 mm Hg

a. pH 7.28, PaCO2 50 mm Hg, and PaO2 58 mm Hg The pH, PaCO2, and PaO2 indicate that the patient has severe uncompensated respiratory acidosis and hypoxemia. Rapid action will be required to prevent increasing hypoxemia and correct the acidosis.

The emergency department nurse is evaluating the effectiveness of therapy for a patient who has received treatment during an asthma attack. Which assessment finding is the best indicator that the therapy has been effective? a. No wheezes are audible. b. O2 saturation is >90%. c. Accessory muscle use has decreased. d. Respiratory rate is 16 breaths/minute.

b. O2 saturation is >90%.

A patient with a family history of cystic fibrosis (CF) asks for information about genetic testing. Which response by the nurse is most appropriate? a. Refer the patient to a qualified genetic counselor. b. Ask the patient why genetic testing is so important. c. Remind the patient that genetic testing has many social implications. d. Tell the patient that cystic fibrosis is an autosomal recessive disorder.

ANS: A A genetic counselor is best qualified to address the multiple issues involved in genetic testing for a patient who is considering having children. Although genetic testing does have social implications, the woman will be better served by a genetic counselor who will have more expertise in this area. CF is an autosomal recessive disorder, but the patient might not understand the implications of this statement. Asking why the patient feels genetic testing is important may imply to the patient that the nurse is questioning her value system

26. The nurse admitting a patient with acute diverticulitis explains that the initial plan of care is to a. administer IV fluids. b. give stool softeners and enemas. c. order a diet high in fiber and fluids. d. prepare the patient for colonoscopy.

ANS: A A patient with acute diverticulitis will be NPO and given parenteral fluids. A diet high in fiber and fluids will be implemented before discharge. Bulk-forming laxatives, rather than stool softeners, are usually given, and these will be implemented later in the hospitalization. The patient with acute diverticulitis will not have enemas or a colonoscopy because of the risk for perforation and peritonitis.

13. The nurse evaluating effectiveness of prescribed calcitonin (Cibacalcin) and ibandronate (Boniva) for a patient with Pagets disease will consider the patients a. pain level. b. oral intake. c. daily weight. d. grip strength.

ANS: A Bone pain is one of the common early manifestations of Pagets disease, and the nurse should assess the pain level to determine whether the treatment is effective. The other information will also be collected by the nurse, but will not be used in evaluating the effectiveness of the therapy.

Interleukin-2 (IL-2) is used as adjuvant therapy for a patient with metastatic renal cell carcinoma. Which information should the nurse include when explaining the purpose of this therapy to the patient? a. IL-2 enhances the immunologic response to tumor cells. b. IL-2 stimulates malignant cells in the resting phase to enter mitosis. c. IL-2 prevents the bone marrow depression caused by chemotherapy. d. IL-2 protects normal cells from the harmful effects of chemotherapy.

ANS: A IL-2 enhances the ability of the patient's own immune response to suppress tumor cells. IL-2 does not protect normal cells from damage caused by chemotherapy, stimulate malignant cells to enter mitosis, or prevent bone marrow depression

15. A 51-year-old male patient has a new diagnosis of Crohn's disease after having frequent diarrhea and a weight loss of 10 pounds (4.5 kg) over 2 months. The nurse will plan to teach about a. medication use. b. fluid restriction. c. enteral nutrition. d. activity restrictions.

ANS: A Medications are used to induce and maintain remission in patients with inflammatory bowel disease (IBD). Decreased activity level is indicated only if the patient has severe fatigue and weakness. Fluids are needed to prevent dehydration. There is no advantage to enteral feedings.

A patient arrives in the emergency department (ED) several hours after taking "25 to 30" acetaminophen (Tylenol) tablets. Which action will the nurse plan to take? a. Give N-acetylcysteine. b. Discuss the use of chelation therapy. c. Start oxygen using a non-rebreather mask. d. Have the patient drink large amounts of water.

ANS: A N-acetylcysteine is the recommended treatment to prevent liver damage after acetaminophen overdose. The other actions might be used for other types of poisoning, but they will not be appropriate for a patient with acetaminophen poisoning.

15. Which information in a patient's history indicates to the nurse that the patient is not an appropriate candidate for kidney transplantation? a. The patient has type 1 diabetes. b. The patient has metastatic lung cancer. c. The patient has a history of chronic hepatitis C infection. d. The patient is infected with human immunodeficiency virus.

ANS: B Disseminated malignancies are a contraindication to transplantation. The conditions of the other patients are not contraindications for kidney transplant. DIF: Cognitive Level: Understand (comprehension) REF: 1092 TOP: Nursing Process: Assessment MSC: NCLEX: Physiological Integrity

A 23-year-old has been admitted with acute liver failure. Which assessment data are most important for the nurse to communicate to the health care provider? a. Asterixis and lethargy b. Jaundiced sclera and skin c. Elevated total bilirubin level d. Liver 3 cm below costal margin

ANS: A The patient's findings of asterixis and lethargy are consistent with grade 2 hepatic encephalopathy. Patients with acute liver failure can deteriorate rapidly from grade 1 or 2 to grade 3 or 4 hepatic encephalopathy and need early transfer to a transplant center. The other findings are typical of patients with hepatic failure and would be reported but would not indicate a need for an immediate change in the therapeutic plan.

When evaluating a patient with a central venous catheter, the nurse observes that the insertion site is red and tender to touch and the patient's temperature is 101.8° F. What should the nurse plan to do? a. Discontinue the catheter and culture the tip. b. Use the catheter only for fluid administration. c. Change the flush system and monitor the site. d. Check the site more frequently for any swelling.

ANS: A The information indicates that the patient has a local and systemic infection caused by the catheter, and the catheter should be discontinued to avoid further complications such as endocarditis. Changing the flush system, continued monitoring, or using the line for fluids will not help prevent or treat the infection.

A patient who is orally intubated and receiving mechanical ventilation is anxious and is "fighting" the ventilator. Which action should the nurse take next? a. Verbally coach the patient to breathe with the ventilator. b. Sedate the patient with the ordered PRN lorazepam (Ativan). c. Manually ventilate the patient with a bag-valve-mask device. d. Increase the rate for the ordered propofol (Diprivan) infusion.

ANS: A The initial response by the nurse should be to try to decrease the patient's anxiety by coaching the patient about how to coordinate respirations with the ventilator. The other actions may also be helpful if the verbal coaching is ineffective in reducing the patient's anxiety.

33. The nurse is titrating the IV fluid infusion rate immediately after a patient has had kidney transplantation. Which parameter will be most important for the nurse to consider? a. Heart rate b. Urine output c. Creatinine clearance d. Blood urea nitrogen (BUN) level

ANS: B Fluid volume is replaced based on urine output after transplant because the urine output can be as high as a liter an hour. The other data will be monitored but are not the most important determinants of fluid infusion rate. DIF: Cognitive Level: Analyze (analysis) REF: 1095 OBJ: Special Questions: Prioritization TOP: Nursing Process: Implementation MSC: NCLEX: Physiological Integrity

Gastric lavage and administration of activated charcoal are ordered for an unconscious patient who has been admitted to the emergency department (ED) after ingesting 30 lorazepam (Ativan) tablets. Which prescribed action should the nurse plan to do first? a. Insert a large-bore orogastric tube. b. Assist with intubation of the patient. c. Prepare a 60-mL syringe with saline. d. Give first dose of activated charcoal.

ANS: B In an unresponsive patient, intubation is done before gastric lavage and activated charcoal administration to prevent aspiration. The other actions will be implemented after intubation.

30. A female patient with chronic kidney disease (CKD) is receiving peritoneal dialysis with 2-L inflows. Which information should the nurse report promptly to the health care provider? a. The patient has an outflow volume of 1800 mL. b. The patient's peritoneal effluent appears cloudy. c. The patient's abdomen appears bloated after the inflow. d. The patient has abdominal pain during the inflow phase.

ANS: B Cloudy-appearing peritoneal effluent is a sign of peritonitis and should be reported immediately so that treatment with antibiotics can be started. The other problems can be addressed through nursing interventions such as slowing the inflow and repositioning the patient. DIF: Cognitive Level: Apply (application) REF: 1087 TOP: Nursing Process: Assessment MSC: NCLEX: Physiological Integrity

55. Which prescribed intervention for a 61-year-old female patient with chronic short bowel syndrome will the nurse question? a. Ferrous sulfate (Feosol) 325 mg daily b. Senna (Senokot) 1 tablet every day c. Psyllium (Metamucil) 2.1 grams 3 times daily d. Diphenoxylate with atropine (Lomotil) prn loose stools

ANS: B Patients with short bowel syndrome have diarrhea because of decreased nutrient and fluid absorption and would not need stimulant laxatives. Iron supplements are used to prevent iron-deficiency anemia, bulk-forming laxatives help make stools less watery, and opioid antidiarrheal drugs are helpful in slowing intestinal transit time.

7. After the nurse teaches the patient about the use of carvedilol (Coreg) in preventing anginal episodes, which statement by a patient indicates that the teaching has been effective? a. "Carvedilol will help my heart muscle work harder." b. "It is important not to suddenly stop taking the carvedilol." c. "I can expect to feel short of breath when taking carvedilol." d. "Carvedilol will increase the blood flow to my heart muscle."

ANS: B Patients who have been taking β-adrenergic blockers can develop intense and frequent angina if the medication is suddenly discontinued. Carvedilol (Coreg) decreases myocardial contractility. Shortness of breath that occurs when taking β-adrenergic blockers for angina may be due to bronchospasm and should be reported to the health care provider. Carvedilol works by decreasing myocardial oxygen demand, not by increasing blood flow to the coronary arteries.

Family members of a patient who has a traumatic brain injury ask the nurse about the purpose of the ventriculostomy system being used for intracranial pressure monitoring. Which response by the nurse is best for this situation? a. "This type of monitoring system is complex and it is managed by skilled staff." b. "The monitoring system helps show whether blood flow to the brain is adequate." c. "The ventriculostomy monitoring system helps check for alterations in cerebral perfusion pressure." d. "This monitoring system has multiple benefits including facilitation of cerebrospinal fluid drainage."

ANS: B Short and simple explanations should be given initially to patients and family members. The other explanations are either too complicated to be easily understood or may increase the family members' anxiety.

The nurse should include which food choice when providing dietary teaching for a patient scheduled to receive external beam radiation for abdominal cancer? a. Fresh fruit salad b. Roasted chicken c. Whole wheat toast d. Cream of potato soup

ANS: B To minimize the diarrhea that is commonly associated with bowel radiation, the patient should avoid foods high in roughage, such as fruits and whole grains. Lactose intolerance may develop secondary to radiation, so dairy products should also be avoided

36. A 25-year-old male patient calls the clinic complaining of diarrhea for 24 hours. Which action should the nurse take first? a. Inform the patient that laboratory testing of blood and stools will be necessary. b. Ask the patient to describe the character of the stools and any associated symptoms. c. Suggest that the patient drink clear liquid fluids with electrolytes, such as Gatorade or Pedialyte. d. Advise the patient to use over-the-counter loperamide (Imodium) to slow gastrointestinal (GI) motility.

ANS: B The initial response by the nurse should be further assessment of the patient. The other responses may be appropriate, depending on what is learned in the assessment.

A 55-year-old patient admitted with an abrupt onset of jaundice and nausea has abnormal liver function studies but serologic testing is negative for viral causes of hepatitis. Which question by the nurse is most appropriate? a. "Is there any history of IV drug use?" b. "Do you use any over-the-counter drugs?" c. "Are you taking corticosteroids for any reason?" d. "Have you recently traveled to a foreign country?"

ANS: B The patient's symptoms, lack of antibodies for hepatitis, and the abrupt onset of symptoms suggest toxic hepatitis, which can be caused by commonly used over-the-counter drugs such as acetaminophen (Tylenol). Travel to a foreign country and a history of IV drug use are risk factors for viral hepatitis. Corticosteroid use does not cause the symptoms listed

Which information about a 72-year-old patient who has a new prescription for phenytoin (Dilantin) indicates that the nurse should consult with the HCP before administration of the medication? a. Patient has generalized tonic-clonic seizures. b. Patient experiences an aura before seizures. c. Patient's most recent blood pressure is 156/92 mmHg. d. Patient has minor elevations in the liver function tests.

d. Patient has minor elevations in the liver function tests. -Many older patients (especially with compromised liver function) may not be able to metabolize phenytoin. The HCP may need to choose another antiseizure medication. Phenytoin is an appropriate medication for patients with tonic-clonic seizures, with or without an aura. Hypertension is not a contraindication for phenytoin therapy.

The nurse assessing a 54-year-old female patient with newly diagnosed trigeminal neuralgia will ask the patient about a. visual problems caused by ptosis. b. triggers leading to facial discomfort. c. poor appetite caused by loss of taste. d. weakness on the affected side of the face.

triggers leading to facial discomfort. The major clinical manifestation of trigeminal neuralgia is severe facial pain that is triggered by cutaneous stimulation of the nerve. Ptosis, loss of taste, and facial weakness are not characteristics of trigeminal neuralgia.

32. A patient admitted to the coronary care unit (CCU) with an ST-segment-elevation myocardial infarction (STEMI) is restless and anxious. The blood pressure is 86/40 and heart rate is 123. Based on this information, which nursing diagnosis is a priority for the patient? a. Acute pain related to myocardial infarction b. Anxiety related to perceived threat of death c. Stress overload related to acute change in health d. Decreased cardiac output related to cardiogenic shock

ANS: C All the nursing diagnoses may be appropriate for this patient, but the hypotension and tachycardia indicate decreased cardiac output and shock from the damaged myocardium. This will result in decreased perfusion to all vital organs (e.g., brain, kidney, heart) and is a priority.

1. Which action will the nurse include in the plan of care for a 42-year-old patient who is being admitted with Clostridium difficile? a. Educate the patient about proper food storage. b. Order a diet with no dairy products for the patient. c. Place the patient in a private room on contact isolation. d. Teach the patient about why antibiotics will not be used.

ANS: C Because C. difficile is highly contagious, the patient should be placed in a private room and contact precautions should be used. There is no need to restrict dairy products for this type of diarrhea. Metronidazole (Flagyl) is frequently used to treat C. difficile. Improper food handling and storage do not cause C. difficile.

Which action by the nurse will determine if the therapies ordered for a patient with chronic constrictive pericarditis are most effective? a. Assess for the presence of a paradoxical pulse. b. Monitor for changes in the patient's sedimentation rate. c. Assess for the presence of jugular venous distention (JVD). d. Check the electrocardiogram (ECG) for ST segment changes.

ANS: C Because the most common finding on physical examination for a patient with chronic constrictive pericarditis is jugular venous distention, a decrease in JVD indicates improvement. Paradoxical pulse, ST segment ECG changes, and changes in sedimentation rates occur with acute pericarditis but are not expected in chronic constrictive pericarditis. DIF: Cognitive Level: Apply (application)

A 49-year-old female patient with cirrhosis and esophageal varices has a new prescription for propranolol (Inderal). Which finding is the best indicator that the medication has been effective? a. The patient reports no chest pain. b. Blood pressure is 140/90 mm Hg. c. Stools test negative for occult blood. d. The apical pulse rate is 68 beats/minute.

ANS: C Because the purpose of b-blocker therapy for patients with esophageal varices is to decrease the risk for bleeding from esophageal varices, the best indicator of the effectiveness for propranolol is the lack of blood in the stools. Although propranolol is used to treat hypertension, angina, and tachycardia, the purpose for use in this patient is to decrease the risk for bleeding from esophageal varices

A patient who is on the telemetry unit develops atrial flutter, rate 150, with associated dyspnea and chest pain. Which action that is included in the hospital dysrhythmia protocol should the nurse do first? a. Obtain a 12-lead electrocardiogram (ECG). b. Notify the health care provider of the change in rhythm. c. Give supplemental O2 at 2 to 3 L/min via nasal cannula. d. Assess the patient's vital signs including O2 saturation.

ANS: C Because this patient has dyspnea and chest pain in association with the new rhythm, the nurse's initial actions should be to address the patient's airway, breathing, and circulation (ABC) by starting with O2 administration. The other actions are also important and should be implemented rapidly. DIF: Cognitive Level: Analyze (analysis)

A patient has acute bronchitis with a nonproductive cough and wheezes. Which topic should the nurse plan to include in the teaching plan? a. Purpose of antibiotic therapy b. Ways to limit oral fluid intake c. Appropriate use of cough suppressants d. Safety concerns with home oxygen therapy

ANS: C Cough suppressants are frequently prescribed for acute bronchitis. Because most acute bronchitis is viral in origin, antibiotics are not prescribed unless there are systemic symptoms. Fluid intake is encouraged. Home oxygen is not prescribed for acute bronchitis, although it may be used for chronic bronchitis

21. A nurse who works on the orthopedic unit has just received the change-of-shift report. Which patient should the nurse assess first? a. Patient who reports foot pain after hammertoe surgery b. Patient with low back pain and a positive straight-leg-raise test c. Patient who has not voided 10 hours after having a laminectomy d. Patient with osteomyelitis who has a temperature of 100.5 F (38.1 C)

ANS: C Difficulty in voiding may indicate damage to the spinal nerves and should be assessed and reported to the surgeon immediately. The information about the other patients is consistent with their diagnoses. The nurse will need to assess them as quickly as possible, but the information about them does not indicate a need for immediate intervention.

The home health nurse is visiting a 30-yr-old patient recovering from rheumatic fever without carditis. The nurse establishes the nursing diagnosis of ineffective health maintenance related to lack of knowledge regarding long-term management of rheumatic fever when the patient makes which statement? a. "I will need prophylactic antibiotic therapy for 5 years." b. "I can take aspirin or ibuprofen (Motrin) to relieve my joint pain." c. "I will be immune to future episodes of rheumatic fever after this infection." d. "I should call the health care provider if I am fatigued or have difficulty breathing."

ANS: C Patients with a history of rheumatic fever are more susceptible to a second episode. Patients with rheumatic fever without carditis require prophylaxis until age 20 years and for a minimum of 5 years. The other patient statements are correct and would not support the nursing diagnosis of ineffective health maintenance. DIF: Cognitive Level: Apply (application)

Which information about a 30-yr-old patient who is hospitalized after a traumatic brain injury requires the most rapid action by the nurse? a. Intracranial pressure of 15 mm Hg b. Cerebrospinal fluid (CSF) drainage of 25 mL/hr c. Pressure of oxygen in brain tissue (PbtO2) is 14 mm Hg d. Cardiac monitor shows sinus tachycardia at 120 beats/minute

ANS: C The PbtO2 should be 20 to 40 mm Hg. Lower levels indicate brain ischemia. An intracranial pressure (ICP) of 15 mm Hg is at the upper limit of normal. CSF is produced at a rate of 20 to 30 mL/hr. The reason for the sinus tachycardia should be investigated, but the elevated heart rate is not as concerning as the decrease in PbtO2.

4. A patient who has acute glomerulonephritis is hospitalized with hyperkalemia. Which information will the nurse monitor to evaluate the effectiveness of the prescribed calcium gluconate IV? a. Urine volume b. Calcium level c. Cardiac rhythm d. Neurologic status

ANS: C The calcium gluconate helps prevent dysrhythmias that might be caused by the hyperkalemia. The nurse will monitor the other data as well, but these will not be helpful in determining the effectiveness of the calcium gluconate. DIF: Cognitive Level: Apply (application) REF: 1073 TOP: Nursing Process: Evaluation MSC: NCLEX: Physiological Integrity

The nurse is caring for a patient who has a head injury and fractured right arm after being assaulted. Which assessment information requires rapid action by the nurse? a. The apical pulse is slightly irregular. b. The patient complains of a headache. c. The patient is more difficult to arouse. d. The blood pressure (BP) increases to 140/62 mm Hg.

ANS: C The change in level of consciousness (LOC) is an indicator of increased intracranial pressure (ICP) and suggests that action by the nurse is needed to prevent complications. The change in BP should be monitored but is not an indicator of a need for immediate nursing action. Headache and a slightly irregular apical pulse are not unusual in a patient after a head injury.

During change-of-shift report, the nurse learns about the following four patients. Which patient requires assessment first? a. 40-year-old with chronic pancreatitis who has gnawing abdominal pain b. 58-year-old who has compensated cirrhosis and is complaining of anorexia c. 55-year-old with cirrhosis and ascites who has an oral temperature of 102° F (38.8° C) d. 36-year-old recovering from a laparoscopic cholecystectomy who has severe shoulder pain

ANS: C This patient's history and fever suggest possible spontaneous bacterial peritonitis, which would require rapid assessment and interventions such as antibiotic therapy. The clinical manifestations for the other patients are consistent with their diagnoses and do not indicate complications are occurring

14. A patient with ST-segment elevation in three contiguous electrocardiographic (ECG) leads is admitted to the emergency department (ED) and diagnosed as having an ST-segment-elevation myocardial infarction (STEMI). Which question should the nurse ask to determine whether the patient is a candidate for thrombolytic therapy? a. "Do you have any allergies?" b. "Do you take aspirin on a daily basis?" c. "What time did your chest pain begin?" d. "Can you rate your chest pain using a 0 to 10 scale?"

ANS: C Thrombolytic therapy should be started within 6 hours of the onset of the myocardial infarction (MI), so the time at which the chest pain started is a major determinant of the appropriateness of this treatment. The other information will also be needed, but it will not be a factor in the decision about thrombolytic therapy.

A 42-year-old female patient is scheduled for transsphenoidal hypophysectomy to treat a pituitary adenoma. During preoperative teaching, the nurse instructs the patient about the need to a. cough and deep breathe every 2 hours postoperatively. b. remain on bed rest for the first 48 hours after the surgery. c. avoid brushing teeth for at least 10 days after the surgery. d. be positioned flat with sandbags at the head postoperatively.

ANS: C To avoid disruption of the suture line, the patient should avoid brushing the teeth for 10 days after surgery. It is not necessary to remain on bed rest after this surgery. Coughing is discouraged because it may cause leakage of cerebrospinal fluid (CSF) from the suture line. The head of the bed should be elevated 30 degrees to reduce pressure on the sella turcica and decrease the risk for headaches

42. Which care activity for a patient with a paralytic ileus is appropriate for the registered nurse (RN) to delegate to unlicensed assistive personnel (UAP)? a. Auscultation for bowel sounds b. Nasogastric (NG) tube irrigation c. Applying petroleum jelly to the lips d. Assessment of the nares for irritation

ANS: C UAP education and scope of practice include patient hygiene such as oral care. The other actions require education and scope of practice appropriate to the RN.

20. A patient with diabetes who has bacterial pneumonia is being treated with IV gentamicin 60 mg IV BID. The nurse will monitor for adverse effects of the medication by evaluating the patient's a. blood glucose. b. urine osmolality. c. serum creatinine. d. serum potassium.

ANS: C When a patient at risk for chronic kidney disease (CKD) receives a potentially nephrotoxic medication, it is important to monitor renal function with BUN and creatinine levels. The other laboratory values would not be useful in assessing for the adverse effects of the gentamicin. DIF: Cognitive Level: Apply (application) REF: 1083 TOP: Nursing Process: Evaluation MSC: NCLEX: Physiological Integrity

An intraaortic balloon pump (IABP) is being used for a patient who is in cardiogenic shock. Which assessment data indicate to the nurse that the goals of treatment with the IABP are being met? a. Urine output of 25 mL/hr b. Heart rate of 110 beats/minute c. Cardiac output (CO) of 5 L/min d. Stroke volume (SV) of 40 mL/beat

ANS: C A CO of 5 L/min is normal and indicates that the IABP has been successful in treating the shock. The low SV signifies continued cardiogenic shock. The tachycardia and low urine output also suggest continued cardiogenic shock.

A patient who is unconscious has ineffective cerebral tissue perfusion and cerebral tissue swelling. Which nursing intervention will be included in the plan of care? a. Encourage coughing and deep breathing. b. Position the patient with knees and hips flexed. c. Keep the head of the bed elevated to 30 degrees. d. Cluster nursing interventions to provide rest periods.

ANS: C The patient with increased intracranial pressure (ICP) should be maintained in the head-up position to help reduce ICP. Extreme flexion of the hips and knees increases abdominal pressure, which increases ICP. Because the stimulation associated with nursing interventions increases ICP, clustering interventions will progressively elevate ICP. Coughing increases intrathoracic pressure and ICP.

A 51-year-old woman had an incisional cholecystectomy 6 hours ago. The nurse will place the highest priority on assisting the patient to a. choose low-fat foods from the menu. b. perform leg exercises hourly while awake. c. ambulate the evening of the operative day. d. turn, cough, and deep breathe every 2 hours.

ANS: D Postoperative nursing care after a cholecystectomy focuses on prevention of respiratory complications because the surgical incision is high in the abdomen and impairs coughing and deep breathing. The other nursing actions are also important to implement but are not as high a priority as ensuring adequate ventilation

The nurse provides discharge teaching for a patient who has two fractured ribs from an automobile accident. Which statement, if made by the patient, would indicate that teaching has been effective? a. "I am going to buy a rib binder to wear during the day." b. "I can take shallow breaths to prevent my chest from hurting." c. "I should plan on taking the pain pills only at bedtime so I can sleep." d. "I will use the incentive spirometer every hour or two during the day."

ANS: D Prevention of the complications of atelectasis and pneumonia is a priority after rib fracture. This can be ensured by deep breathing and coughing. Use of a rib binder, shallow breathing, and taking pain medications only at night are likely to result in atelectasis

The nurse has identified a nursing diagnosis of acute pain related to inflammatory process for a patient with acute pericarditis. An appropriate intervention by the nurse for this problem is to a. teach the patient to take deep, slow breaths to control the pain. b. force fluids to 3000 mL/day to decrease fever and inflammation. c. provide a fresh ice bag every hour for the patient to place on the chest. d. place the patient in Fowler's position, leaning forward on the overbed table.

ANS: D Sitting upright and leaning forward frequently will decrease the pain associated with pericarditis. Forcing fluids will not decrease the inflammation or pain. Taking deep breaths will tend to increase pericardial pain. Ice does not decrease this type of inflammation and pain. DIF: Cognitive Level: Apply (application)

12. Which menu choice by a patient with osteoporosis indicates that the nurses teaching about appropriate diet has been effective? a. Pancakes with syrup and bacon b. Whole wheat toast and fresh fruit c. Egg-white omelet and a half grapefruit d. Oatmeal with skim milk and fruit yogurt

ANS: D Skim milk and yogurt are high in calcium. The other choices do not contain any high-calcium foods.

28. Which breakfast choice indicates a patient's good understanding of information about a diet for celiac disease? a. Oatmeal with nonfat milk b. Whole wheat toast with butter c. Bagel with low-fat cream cheese d. Corn tortilla with scrambled eggs

ANS: D Avoidance of gluten-containing foods is the only treatment for celiac disease. Corn does not contain gluten, while oatmeal and wheat do.

Which action will be included in the care for a patient who has recently been diagnosed with asymptomatic nonalcoholic fatty liver disease (NAFLD)? a. Teach symptoms of variceal bleeding. b. Draw blood for hepatitis serology testing. c. Discuss the need to increase caloric intake. d. Review the patient's current medication list.

ANS: D Some medications can increase the risk for NAFLD, and they should be eliminated. NAFLD is not associated with hepatitis, weight loss is usually indicated, and variceal bleeding would not be a concern in a patient with asymptomatic NAFLD

The nurse is caring for a patient with mitral regurgitation. Referring to the figure below, where should the nurse listen to best hear a murmur typical of mitral regurgitation? a. 1st sternal b. 2nd sternal c. 4th sternal d. 5th intercostal space, midclavicular line.

ANS: D Sounds from the mitral valve are best heard at the apex of the heart, fifth intercostal space, midclavicular line. DIF: Cognitive Level: Understand (comprehension)

11. Nadolol (Corgard) is prescribed for a patient with chronic stable angina and left ventricular dysfunction. To determine whether the drug is effective, the nurse will monitor for a. decreased blood pressure and heart rate. b. fewer complaints of having cold hands and feet. c. improvement in the strength of the distal pulses. d. the ability to do daily activities without chest pain.

ANS: D Because the medication is ordered to improve the patient's angina, effectiveness is indicated if the patient is able to accomplish daily activities without chest pain. Blood pressure and heart rate may decrease, but these data do not indicate that the goal of decreased angina has been met. The noncardioselective β-adrenergic blockers can cause peripheral vasoconstriction, so the nurse would not expect an improvement in distal pulse quality or skin temperature.

When analyzing the rhythm of a patient's electrocardiogram (ECG), the nurse will need to investigate further upon finding a(n) a. isoelectric ST segment. c. QT interval of 0.38 second. b. PR interval of 0.18 second. d. QRS interval of 0.14 second.

ANS: D Because the normal QRS interval is less than 0.12 seconds, the patient's QRS interval of 0.14 seconds indicates that the conduction through the ventricular conduction system is prolonged. The PR interval and QT interval are within normal range and ST segment should be isoelectric (flat). DIF: Cognitive Level: Apply (application)

4. Which information given by a patient admitted with chronic stable angina will help the nurse confirm this diagnosis? a. The patient states that the pain "wakes me up at night." b. The patient rates the pain at a level 3 to 5 (0 to 10 scale). c. The patient states that the pain has increased in frequency over the last week. d. The patient states that the pain "goes away" with one sublingual nitroglycerin tablet.

ANS: D Chronic stable angina is typically relieved by rest or nitroglycerin administration. The level of pain is not a consistent indicator of the type of angina. Pain occurring at rest or with increased frequency is typical of unstable angina.

31. The nurse will plan to teach a patient with Crohn's disease who has megaloblastic anemia about the need for a. oral ferrous sulfate tablets. b. regular blood transfusions. c. iron dextran (Imferon) infusions. d. cobalamin (B12) spray or injections.

ANS: D Crohn's disease frequently affects the ileum, where absorption of cobalamin occurs. Cobalamin must be administered regularly by nasal spray or IM to correct the anemia. Iron deficiency does not cause megaloblastic anemia. The patient may need occasional transfusions but not regularly scheduled transfusions.

The nurse obtains a health history from an older patient with a prosthetic mitral valve who has symptoms of infective endocarditis (IE). Which question by the nurse is most focused on identifying a risk factor for IE? a. "Do you have a history of a heart attack?" b. "Is there a family history of endocarditis?" c. "Have you had any recent immunizations?" d. "Have you had dental work done recently?"

ANS: D Dental procedures place the patient with a prosthetic mitral valve at risk for IE. Myocardial infarction, immunizations, and a family history of endocarditis are not risk factors for IE. DIF: Cognitive Level: Apply (application)

A patient who has just been admitted with community-acquired pneumococcal pneumonia has a temperature of 101.6° F with a frequent cough and is complaining of severe pleuritic chest pain. Which prescribed medication should the nurse give first? a. Codeine b. Guaifenesin (Robitussin) c. Acetaminophen (Tylenol) d. Piperacillin/tazobactam (Zosyn)

ANS: D Early initiation of antibiotic therapy has been demonstrated to reduce mortality. The other medications are also appropriate and should be given as soon as possible, but the priority is to start antibiotic therapy

A 19-yr-old patient is brought to the emergency department (ED) with multiple lacerations and tissue avulsion of the left hand. When asked about tetanus immunization, the patient denies having any previous vaccinations. The nurse will anticipate giving a. tetanus immunoglobulin (TIG) only. b. TIG and tetanus-diphtheria toxoid (Td). c. tetanus-diphtheria toxoid and pertussis vaccine (Tdap) only. d. TIG and tetanus-diphtheria toxoid and pertussis vaccine (Tdap).

ANS: D For an adult with no previous tetanus immunizations, TIG and Tdap are recommended. The other immunizations are not sufficient for this patient.

An older adult patient has a prescription for cyclosporine following a kidney transplant. Which information in the patient's health history has the most implications for planning patient teaching about the medication at this time? a. The patient restricts salt to treat prehypertension. b. The patient drinks 3 to 4 quarts of fluids every day. c. The patient has many concerns about the effects of cyclosporine. d. The patient has a glass of grapefruit juice every day for breakfast.

ANS: D Grapefruit juice can increase the toxicity of cyclosporine. The patient should be taught to avoid grapefruit juice. High fluid intake will not affect cyclosporine levels or renal function. Cyclosporine may cause hypertension, and the patient's many concerns should be addressed, but these are not potentially life-threatening problems.

A 24-year-old female contracts hepatitis from contaminated food. During the acute (icteric) phase of the patient's illness, the nurse would expect serologic testing to reveal a. antibody to hepatitis D (anti-HDV). b. hepatitis B surface antigen (HBsAg). c. anti-hepatitis A virus immunoglobulin G (anti-HAV IgG). d. anti-hepatitis A virus immunoglobulin M (anti-HAV IgM).

ANS: D Hepatitis A is transmitted through the oral-fecal route, and antibody to HAV IgM appears during the acute phase of hepatitis A. The patient would not have antigen for hepatitis B or antibody for hepatitis D. Anti-HAV IgG would indicate past infection and lifelong immunity.

The nurse is caring for a patient receiving intravesical bladder chemotherapy. The nurse should monitor for which adverse effect? a. Nausea b. Alopecia c. Mucositis d. Hematuria

ANS: D The adverse effects of intravesical chemotherapy are confined to the bladder. The other adverse effects are associated with systemic chemotherapy

36. A patient had a non-ST-segment-elevation myocardial infarction (NSTEMI) 3 days ago. Which nursing intervention included in the plan of care is most appropriate for the registered nurse (RN) to delegate to an experienced licensed practical/vocational nurse (LPN/LVN)? a. Evaluation of the patient's response to walking in the hallway b. Completion of the referral form for a home health nurse follow-up c. Education of the patient about the pathophysiology of heart disease d. Reinforcement of teaching about the purpose of prescribed medications

ANS: D LPN/LVN education and scope of practice include reinforcing education that has previously been done by the RN. Evaluating the patient response to exercise after a NSTEMI requires more education and should be done by the RN. Teaching and discharge planning/ documentation are higher level skills that require RN education and scope of practice.

Which intervention will the nurse include in the plan of care for a patient who is diagnosed with a lung abscess? a. Teach the patient to avoid the use of over-the-counter expectorants. b. Assist the patient with chest physiotherapy and postural drainage. c. Notify the health care provider immediately about any bloody or foul-smelling sputum. d. Teach about the need for prolonged antibiotic therapy after discharge from the hospital.

ANS: D Long-term antibiotic therapy is needed for effective eradication of the infecting organisms in lung abscess. Chest physiotherapy and postural drainage are not recommended for lung abscess because they may lead to spread of the infection. Foul smelling and bloody sputum are common clinical manifestations in lung abscess. Expectorants may be used because the patient is encouraged to cough

During the primary survey of a patient with severe leg trauma, the nurse observes that the patient's left pedal and posterior tibial pulses are absent and the entire leg is swollen. Which action will the nurse take next? a. Send blood to the lab for a complete blood count. b. Assess further for a cause of the decreased circulation. c. Finish the airway, breathing, circulation, disability survey. d. Start normal saline fluid infusion with a large-bore IV line.

ANS: D The assessment data indicate that the patient may have arterial trauma and hemorrhage. When a possibly life-threatening injury is found during the primary survey, the nurse should immediately start interventions before proceeding with the survey. Although a complete blood count is indicated, administration of IV fluids should be started first. Completion of the primary survey and further assessment should be completed after the IV fluids are initiated.

31. When caring for a patient with acute coronary syndrome who has returned to the coronary care unit after having angioplasty with stent placement, the nurse obtains the following assessment data. Which data indicate the need for immediate action by the nurse? a. Heart rate 102 beats/min b. Pedal pulses 1+ bilaterally c. Blood pressure 103/54 mm Hg d. Chest pain level 7 on a 0 to 10 point scale

ANS: D The patient's chest pain indicates that restenosis of the coronary artery may be occurring and requires immediate actions, such as administration of oxygen and nitroglycerin, by the nurse. The other information indicates a need for ongoing assessments by the nurse.

Which nursing action is needed when preparing to assist with the insertion of a pulmonary artery catheter? a. Determine if the cardiac troponin level is elevated. b. Auscultate heart sounds before and during insertion. c. Place the patient on NPO status before the procedure. d. Attach cardiac monitoring leads before the procedure.

ANS: D Dysrhythmias can occur as the catheter is floated through the right atrium and ventricle, and it is important for the nurse to monitor for these during insertion. Pulmonary artery catheter insertion does not require anesthesia, and the patient will not need to be NPO. Changes in cardiac troponin or heart and breath sounds are not expected during pulmonary artery catheter insertion.

The nurse notes premature ventricular contractions (PVCs) while suctioning a patient's endotracheal tube. Which next action by the nurse is indicated? a. Plan to suction the patient more frequently. b. Decrease the suction pressure to 80 mm Hg. c. Give antidysrhythmic medications per protocol. d. Stop and ventilate the patient with 100% oxygen.

ANS: D Dysrhythmias during suctioning may indicate hypoxemia or sympathetic nervous system stimulation. The nurse should stop suctioning and ventilate the patient with 100% O2. There is no indication that more frequent suctioning is needed. Lowering the suction pressure will decrease the effectiveness of suctioning without improving the hypoxemia. Because the PVCs occurred during suctioning, there is no need for antidysrhythmic medications (which may have adverse effects) unless they recur when the suctioning is stopped and patient is well oxygenated.

The nurse is caring for a patient with a subarachnoid hemorrhage who is intubated and placed on a mechanical ventilator with 10 cm H2O of peak end-expiratory pressure (PEEP). When monitoring the patient, the nurse will need to notify the health care provider immediately if the patient develops a. O2 saturation of 93%. b. green nasogastric tube drainage. c. respirations of 20 breaths/minute. d. increased jugular venous distention.

ANS: D Increases in jugular venous distention in a patient with a subarachnoid hemorrhage may indicate an increase in intracranial pressure (ICP) and that the PEEP setting is too high for this patient. A respiratory rate of 20, O2 saturation of 93%, and green nasogastric tube drainage are within normal limits.

Which collaborative problem will the nurse include in a care plan for a patient admitted to the hospital with idiopathic aplastic anemia? a. Potential complication: seizures b. Potential complication: infection c. Potential complication: neurogenic shock d. Potential complication: pulmonary edema

B Because the patient with aplastic anemia has pancytopenia, the patient is at risk for infection and bleeding. There is no increased risk for seizures, neurogenic shock, or pulmonary edema.

Which assessment finding should the nurse caring for a patient with thrombocytopenia communicate immediately to the health care provider? a. The platelet count is 52,000/μL. b. The patient is difficult to arouse. c. There are purpura on the oral mucosa. d. There are large bruises on the patient's back.

B Difficulty in arousing the patient may indicate a cerebral hemorrhage, which is life threatening and requires immediate action. The other information should be documented and reported but would not be unusual in a patient with thrombocytopenia.

After teaching a patient with newly diagnosed Raynaud's phenomenon about how to manage the condition, which action by the patient best demonstrates that the teaching has been effective? a.The patient exercises indoors during the winter months. b.The patient immerses hands in hot water when they turn pale. c.The patient takes pseudoephedrine (Sudafed) for cold symptoms. d.The patient avoids taking nonsteroidal antiinflammatory drugs (NSAIDs).

A Patients should avoid temperature extremes by exercising indoors when it is cold. To avoid burn injuries, the patient should use warm rather than hot water to warm the hands. Pseudoephedrine is a vasoconstrictor and should be avoided. There is no reason to avoid taking NSAIDs with Raynaud's phenomenon.

A patient in the outpatient clinic has a new diagnosis of peripheral artery disease (PAD). Which group of drugs will the nurse plan to include when teaching about PAD management? a.Statins b.Antibiotics c.Thrombolytics d.Anticoagulants

A Research indicates that statin use by patients with PAD improves multiple outcomes. There is no research that supports the use of the other drug categories in PAD.

The nurse notes new onset confusion in an older patient who is normally alert and oriented. In which order should the nurse take the following actions? (Put a comma and a space between each answer choice [A, B, C, D].) a. Obtain the oxygen saturation. b. Check the patient's pulse rate. c. Document the change in status. d. Notify the health care provider

ANS: A, B, D, C Assessment for physiologic causes of new onset confusion such as pneumonia, infection, or perfusion problems should be the first action by the nurse. Airway and oxygenation should be assessed first, then circulation. After assessing the patient, the nurse should notify the health care provider. Finally, documentation of the assessments and care should be done

A patient undergoing external radiation has developed a dry desquamation of the skin in the treatment area. The nurse teaches the patient about management of the skin reaction. Which statement, if made by the patient, indicates the teaching was effective? a. "I can buy some aloe vera gel to use on the area." b. "I will expose the treatment area to a sun lamp daily." c. "I can use ice packs to relieve itching in the treatment area." d. "I will scrub the area with warm water to remove the scales."

ANS: A Aloe vera gel and cream may be used on the radiated skin area. Ice and sunlamps may injure the skin. Treatment areas should be cleaned gently to avoid further injury

A college athlete is seen in the clinic 6 weeks after a concussion. Which assessment information will the nurse collect to determine whether the patient is developing postconcussion syndrome? a. Short-term memory c. Glasgow Coma Scale b. Muscle coordination d. Pupil reaction to light

ANS: A Decreased short-term memory is one indication of postconcussion syndrome. The other data may be assessed but are not indications of postconcussion syndrome.

An older patient is receiving standard multidrug therapy for tuberculosis (TB). The nurse should notify the health care provider if the patient exhibits which finding? a. Yellow-tinged skin b. Orange-colored sputum c. Thickening of the fingernails d. Difficulty hearing high-pitched voices

ANS: A Noninfectious hepatitis is a toxic effect of isoniazid (INH), rifampin, and pyrazinamide, and patients who develop hepatotoxicity will need to use other medications. Changes in hearing and nail thickening are not expected with the four medications used for initial TB drug therapy. Presbycusis is an expected finding in the older adult patient. Orange discoloration of body fluids is an expected side effect of rifampin and not an indication to call the health care provider

A patient is being evaluated for possible atopic dermatitis. The nurse expects elevation of which laboratory value? a. IgE b. IgA c. Basophils d. Neutrophils

ANS: A Serum IgE is elevated in an allergic response (type 1 hypersensitivity disorders). The eosinophil level will be elevated rather than neutrophil or basophil counts. IgA is located in body secretions and would not be tested when evaluating a patient who has symptoms of atopic dermatitis.

45. A 51-year-old woman with Crohn's disease who is taking infliximab (Remicade) calls the nurse in the outpatient clinic about new symptoms. Which symptom is most important to communicate to the health care provider? a. Fever b. Nausea c. Joint pain d. Headache

ANS: A Since infliximab suppresses the immune response, rapid treatment of infection is essential. The other patient complaints are common side effects of the medication, but they do not indicate any potentially life-threatening complications.

When planning the response to the potential use of smallpox as a biological weapon, the emergency department (ED) nurse manager will plan to obtain adequate quantities of a. vaccine. c. antibiotics. b. atropine. d. whole blood.

ANS: A Smallpox infection can be prevented or ameliorated by the administration of vaccine given rapidly after exposure. The other interventions would be helpful for other agents of terrorism but not for smallpox.

27. A patient who is being admitted to the emergency department with intermittent chest pain gives the following list of medications to the nurse. Which medication has the most immediate implications for the patient's care? a. Sildenafil (Viagra) b. Furosemide (Lasix) c. Captopril (Capoten) d. Warfarin (Coumadin)

ANS: A The nurse will need to avoid giving nitrates to the patient because nitrate administration is contraindicated in patients who are using sildenafil because of the risk of severe hypotension caused by vasodilation. The other home medications also should be documented and reported to the health care provider but do not have as immediate an impact on decisions about the patient's treatment.

A male patient who has possible cerebral edema has a serum sodium level of 116 mEq/L (116 mmol/L) and a decreasing level of consciousness (LOC). He is now complaining of a headache. Which prescribed interventions should the nurse implement first? a. Administer IV 5% hypertonic saline. b. Draw blood for arterial blood gases (ABGs). c. Send patient for computed tomography (CT). d. Administer acetaminophen (Tylenol) 650 mg orally.

ANS: A The patient's low sodium indicates that hyponatremia may be causing the cerebral edema. The nurse's first action should be to correct the low sodium level. Acetaminophen (Tylenol) will have minimal effect on the headache because it is caused by cerebral edema and increased intracranial pressure (ICP). Drawing ABGs and obtaining a CT scan may provide some useful information, but the low sodium level may lead to seizures unless it is addressed quickly.

Which interventions will the nurse plan for a comatose patient who is to begin therapeutic hypothermia (select all that apply)? a. Assist with endotracheal intubation. b. Insert an indwelling urinary catheter. c. Begin continuous cardiac monitoring. d. Obtain an order to restrain the patient. e. Prepare to give sympathomimetic drugs.

ANS: A, B, C Cooling can produce dysrhythmias, so the patient's heart rhythm should be continuously monitored and dysrhythmias treated if necessary. Bladder catheterization and endotracheal intubation are needed during cooling. Sympathomimetic drugs tend to stimulate the heart and increase the risk for fatal dysrhythmias such as ventricular fibrillation. Patients receiving therapeutic hypothermia are comatose or do not follow commands so restraints are not indicated.

Which factors will the nurse consider when calculating the CURB-65 score for a patient with pneumonia (select all that apply)? a. Age b. Blood pressure c. Respiratory rate d. Oxygen saturation e. Presence of confusion f. Blood urea nitrogen (BUN) level

ANS: A, B, C, E, F Data collected for the CURB-65 are mental status (confusion), BUN (elevated), blood pressure (decreased), respiratory rate (increased), and age (65 and older). The other information is also essential to assess, but are not used for CURB-65 scoring

53. The nurse is admitting a 67-year-old patient with new-onset steatorrhea. Which question is most important for the nurse to ask? a. "How much milk do you usually drink?" b. "Have you noticed a recent weight loss?" c. "What time of day do your bowels move?" d. "Do you eat meat or other animal products?"

ANS: B Although all of the questions provide useful information, it is most important to determine if the patient has an imbalance in nutrition because of the steatorrhea.

9. A patient being admitted with an acute exacerbation of ulcerative colitis reports crampy abdominal pain and passing 15 or more bloody stools a day. The nurse will plan to a. administer IV metoclopramide (Reglan). b. discontinue the patient's oral food intake. c. administer cobalamin (vitamin B12) injections. d. teach the patient about total colectomy surgery.

ANS: B An initial therapy for an acute exacerbation of inflammatory bowel disease (IBD) is to rest the bowel by making the patient NPO. Metoclopramide increases peristalsis and will worsen symptoms. Cobalamin (vitamin B12) is absorbed in the ileum, which is not affected by ulcerative colitis. Although total colectomy is needed for some patients, there is no indication that this patient is a candidate.

After endotracheal suctioning, the nurse notes that the intracranial pressure (ICP) for a patient with a traumatic head injury has increased from 14 to 17 mm Hg. Which action should the nurse take first? a. Document the increase in intracranial pressure. b. Ensure that the patient's neck is in neutral position. c. Notify the health care provider about the change in pressure. d. Increase the rate of the prescribed propofol (Diprivan) infusion.

ANS: B Because suctioning will cause a transient increase in ICP, the nurse should initially check for other factors that might be contributing to the increase and observe the patient for a few minutes. Documentation is needed, but this is not the first action. There is no need to notify the health care provider about this expected reaction to suctioning. Propofol is used to control patient anxiety or agitation. There is no indication that anxiety has contributed to the increase in ICP.

3. Which assessment data collected by the nurse who is admitting a patient with chest pain suggest that the pain is caused by an acute myocardial infarction (AMI)? a. The pain increases with deep breathing. b. The pain has lasted longer than 30 minutes. c. The pain is relieved after the patient takes nitroglycerin. d. The pain is reproducible when the patient raises the arms.

ANS: B Chest pain that lasts for 20 minutes or more is characteristic of AMI. Changes in pain that occur with raising the arms or with deep breathing are more typical of musculoskeletal pain or pericarditis. Stable angina is usually relieved when the patient takes nitroglycerin.

The cardiac telemetry unit charge nurse receives status reports from other nursing units about four patients who need cardiac monitoring. Which patient should be transferred to the cardiac unit first? a. Patient with Hashimoto's thyroiditis and a heart rate of 102 b. Patient with tetany who has a new order for IV calcium chloride c. Patient with Cushing syndrome and a blood glucose of 140 mg/dL d. Patient with Addison's disease who takes hydrocortisone twice daily

ANS: B Emergency treatment of tetany requires IV administration of calcium; ECG monitoring will be required because cardiac arrest may occur if high calcium levels result from too-rapid administration. The information about the other patients indicates that they are more stable than the patient with tetany

Which nursing assessment of a 69-year-old patient is most important to make during initiation of thyroid replacement with levothyroxine (Synthroid)? a. Fluid balance b. Apical pulse rate c. Nutritional intake d. Orientation and alertness

ANS: B In older patients, initiation of levothyroxine therapy can increase myocardial oxygen demand and cause angina or dysrhythmias. The medication also is expected to improve mental status and fluid balance and will increase metabolic rate and nutritional needs, but these changes will not result in potentially life-threatening complications

Which question will the nurse in the endocrine clinic ask to help determine a patient's risk factors for goiter? a. "How much milk do you drink?" b. "What medications are you taking?" c. "Are your immunizations up to date?" d. "Have you had any recent neck injuries?"

ANS: B Medications that contain thyroid-inhibiting substances can cause goiter. Milk intake, neck injury, and immunization history are not risk factors for goiter

A 34-year old patient with chronic hepatitis C infection has several medications prescribed. Which medication requires further discussion with the health care provider before administration? a. Ribavirin (Rebetol, Copegus) 600 mg PO bid b. Pegylated a-interferon (PEG-Intron, Pegasys) SQ daily c. Diphenhydramine (Benadryl) 25 mg PO every 4 hours PRN itching d. Dimenhydrinate (Dramamine) 50 mg PO every 6 hours PRN nausea

ANS: B Pegylated a-interferon is administered weekly. The other medications are appropriate for a patient with chronic hepatitis C infection

The public health nurse is planning a program to decrease the incidence of meningitis in teenagers and young adults. Which action is most likely to be effective? a. Emphasize the importance of hand washing. b. Immunize adolescents and college freshman. c. Support serving healthy nutritional options in the college cafeteria. d. Encourage adolescents and young adults to avoid crowds in the winter.

ANS: B The Neisseria meningitides vaccination is recommended for children ages 11 and 12 years, unvaccinated teens entering high school, and college freshmen. Hand washing may help decrease the spread of bacteria, and good nutrition may increase resistance to infection. but those are not as effective as immunization. Because adolescents and young adults are in school or the workplace, avoiding crowds is not realistic.

28. Which assessment finding by the nurse caring for a patient who has had coronary artery bypass grafting using a right radial artery graft is most important to communicate to the health care provider? a. Complaints of incisional chest pain b. Pallor and weakness of the right hand c. Fine crackles heard at both lung bases d. Redness on both sides of the sternal incision

ANS: B The changes in the right hand indicate compromised blood flow, which requires immediate evaluation and actions such as prescribed calcium channel blockers or surgery. The other changes are expected and/or require nursing interventions.

The nurse admitting a patient who has a right frontal lobe tumor would expect the patient may have a. expressive aphasia. c. right-sided weakness. b. impaired judgment. d. difficulty swallowing.

ANS: B The frontal lobe controls intellectual activities such as judgment. Speech is controlled in the parietal lobe. Weakness and hemiplegia occur on the contralateral side from the tumor. Swallowing is controlled by the brainstem.

When developing a community health program to decrease the incidence of rheumatic fever, which action should the community health nurse include? a. Vaccinate high-risk groups in the community with streptococcal vaccine. b. Teach community members to seek treatment for streptococcal pharyngitis. c. Teach about the importance of monitoring temperature when sore throats occur. d. Teach about prophylactic antibiotics to those with a family history of rheumatic fever.

ANS: B The incidence of rheumatic fever is decreased by treatment of streptococcal infections with antibiotics. Family history is not a risk factor for rheumatic fever. There is no immunization that is effective in decreasing the incidence of rheumatic fever. Teaching about monitoring temperature will not decrease the incidence of rheumatic fever. DIF: Cognitive Level: Apply (application)

2. A 71-year-old male patient tells the nurse that growing old causes constipation so he has been using a suppository for constipation every morning. Which action should the nurse take first? a. Encourage the patient to increase oral fluid intake. b. Assess the patient about risk factors for constipation. c. Suggest that the patient increase intake of high-fiber foods. d. Teach the patient that a daily bowel movement is unnecessary.

ANS: B The nurse's initial action should be further assessment of the patient for risk factors for constipation and for his usual bowel pattern. The other actions may be appropriate but will be based on the assessment.

A patient with newly diagnosed lung cancer tells the nurse, "I don't think I'm going to live to see my next birthday." Which response by the nurse is best? a. "Would you like to talk to the hospital chaplain about your feelings?" b. "Can you tell me what it is that makes you think you will die so soon?" c. "Are you afraid that the treatment for your cancer will not be effective?" d. "Do you think that taking an antidepressant medication would be helpful?"

ANS: B The nurse's initial response should be to collect more assessment data about the patient's statement. The answer beginning "Can you tell me what it is" is the most open-ended question and will offer the best opportunity for obtaining more data. The answer beginning, "Are you afraid" implies that the patient thinks that the cancer will be immediately fatal, although the patient's statement may not be related to the cancer diagnosis. The remaining two answers offer interventions that may be helpful to the patient, but more assessment is needed to determine whether these interventions are appropriate

After receiving information about four patients during change-of-shift report, which patient should the nurse assess first? a. Patient with acute pericarditis who has a pericardial friction rub b. Patient who has just returned to the unit after balloon valvuloplasty c. Patient who has hypertrophic cardiomyopathy and a heart rate of 116 d. Patient with a mitral valve replacement who has an anticoagulant scheduled

ANS: B The patient who has just arrived after balloon valvuloplasty will need assessment for complications such as bleeding and hypotension. The information about the other patients is consistent with their diagnoses and does not indicate any complications or need for urgent assessment or intervention. DIF: Cognitive Level: Analyze (analysis)

A 63-year-old patient with primary hyperparathyroidism has a serum phosphorus level of 1.7 mg/dL (0.55 mmol/L) and calcium of 14 mg/dL (3.5 mmol/L). Which nursing action should be included in the plan of care? a. Restrict the patient to bed rest. b. Encourage 4000 mL of fluids daily. c. Institute routine seizure precautions. d. Assess for positive Chvostek's sign.

ANS: B The patient with hypercalcemia is at risk for kidney stones, which may be prevented by a high fluid intake. Seizure precautions and monitoring for Chvostek's or Trousseau's sign are appropriate for hypocalcemic patients. The patient should engage in weight-bearing exercise to decrease calcium loss from bone

A patient whose heart monitor shows sinus tachycardia, rate 132, is apneic, and has no palpable pulses. What action should the nurse take next? a. Perform synchronized cardioversion. b. Start cardiopulmonary resuscitation (CPR). c. Give atropine per agency dysrhythmia protocol. d. Provide supplemental O2 via non-rebreather mask.

ANS: B The patient's clinical manifestations indicate pulseless electrical activity, and the nurse should immediately start CPR. The other actions would not be of benefit to this patient. DIF: Cognitive Level: Apply (application)

Administration of hepatitis B vaccine to a healthy 18-year-old patient has been effective when a specimen of the patient's blood reveals a. HBsAg. b. anti-HBs. c. anti-HBc IgG. d. anti-HBc IgM.

ANS: B The presence of surface antibody to HBV (anti-HBs) is a marker of a positive response to the vaccine. The other laboratory values indicate current infection with HBV

A patient who is suspected of having an epidural hematoma is admitted to the emergency department. Which action will the nurse expect to take? a. Administer IV furosemide (Lasix). b. Prepare the patient for craniotomy. c. Initiate high-dose barbiturate therapy. d. Type and crossmatch for blood transfusion.

ANS: B The principal treatment for epidural hematoma is rapid surgery to remove the hematoma and prevent herniation. If intracranial pressure is elevated after surgery, furosemide or high-dose barbiturate therapy may be needed, but these will not be of benefit unless the hematoma is removed. Minimal blood loss occurs with head injuries, and transfusion is usually not necessary.

To detect possible complications in a patient with severe cirrhosis who has bleeding esophageal varices, it is most important for the nurse to monitor a. bilirubin levels. b. ammonia levels. c. potassium levels. d. prothrombin time.

ANS: B The protein in the blood in the gastrointestinal (GI) tract will be absorbed and may result in an increase in the ammonia level because the liver cannot metabolize protein very well. The prothrombin time, bilirubin, and potassium levels should also be monitored, but they will not be affected by the bleeding episode

While the patient's full-thickness burn wounds to the face are exposed, what is the best nursing action to prevent cross contamination? a. Use sterile gloves when removing old dressings. b. Wear gowns, caps, masks, and gloves during all care of the patient. c. Administer IV antibiotics to prevent bacterial colonization of wounds. d. Turn the room temperature up to at least 70° F (20° C) during dressing changes.

ANS: B Use of gowns, caps, masks, and gloves during all patient care will decrease the possibility of wound contamination for a patient whose burns are not covered. When removing contaminated dressings and washing the dirty wound, use nonsterile, disposable gloves. The room temperature should be kept at approximately 85° F for patients with open burn wounds to prevent shivering. Systemic antibiotics are not well absorbed into deep burns because of the lack of circulation

The nurse notes that a patient's heart monitor shows that every other beat is earlier than expected, has no visible P wave, and has a QRS complex that is wide and bizarre in shape. How will the nurse document the rhythm? a. Ventricular couplets b. Ventricular bigeminy c. Ventricular R-on-T phenomenon d. Multifocal premature ventricular contractions

ANS: B Ventricular bigeminy describes a rhythm in which every other QRS complex is wide and bizarre looking. Pairs of wide QRS complexes are described as ventricular couplets. There is no indication that the premature ventricular contractions are multifocal or that the R-on-T phenomenon is occurring. DIF: Cognitive Level: Apply (application)

Which hemodynamic parameter best reflects the effectiveness of drugs that the nurse gives to reduce a patient's left ventricular afterload? a. Mean arterial pressure (MAP) b. Systemic vascular resistance (SVR) c. Pulmonary vascular resistance (PVR) d. Pulmonary artery wedge pressure (PAWP)

ANS: B SVR reflects the resistance to ventricular ejection, or afterload. The other parameters may be monitored but do not reflect afterload as directly.

A patient with burns covering 40% total body surface area (TBSA) is in the acute phase of burn treatment. Which snack would be best for the nurse to offer to this patient? a. Bananas b. Orange gelatin c. Vanilla milkshake d. Whole grain bagel

ANS: C A patient with a burn injury needs high protein and calorie food intake, and the milkshake is the highest in these nutrients. The other choices are not as nutrient-dense as the milkshake. Gelatin is likely high in sugar. The bagel is a good carbohydrate choice, but low in protein. Bananas are a good source of potassium, but are not high in protein and calories

A patient has just arrived on the unit after a thyroidectomy. Which action should the nurse take first? a. Observe the dressing for bleeding. b. Check the blood pressure and pulse. c. Assess the patient's respiratory effort. d. Support the patient's head with pillows.

ANS: C Airway obstruction is a possible complication after thyroidectomy because of swelling or bleeding at the site or tetany. The priority nursing action is to assess the airway. The other actions are also part of the standard nursing care postthyroidectomy but are not as high of a priority

Which laboratory test result will the nurse monitor when evaluating the effects of therapy for a 62-year-old female patient who has acute pancreatitis? a. Calcium b. Bilirubin c. Amylase d. Potassium

ANS: C Amylase is elevated in acute pancreatitis. Although changes in the other values may occur, they would not be useful in evaluating whether the prescribed therapies have been effective

When caring for a patient with infective endocarditis of the tricuspid valve, the nurse should monitor the patient for the development of a. flank pain. c. shortness of breath. b. splenomegaly. d. mental status changes.

ANS: C Embolization from the tricuspid valve would cause symptoms of pulmonary embolus. Flank pain, changes in mental status, and splenomegaly would be associated with embolization from the left-sided valves. DIF: Cognitive Level: Apply (application)

Which response by the nurse best explains the purpose of ranitidine (Zantac) for a patient admitted with bleeding esophageal varices? a. The medication will reduce the risk for aspiration. b. The medication will inhibit development of gastric ulcers. c. The medication will prevent irritation of the enlarged veins. d. The medication will decrease nausea and improve the appetite.

ANS: C Esophageal varices are dilated submucosal veins. The therapeutic action of H2-receptor blockers in patients with esophageal varices is to prevent irritation and bleeding from the varices caused by reflux of acid gastric contents. Although ranitidine does decrease the risk for peptic ulcers, reduce nausea, and help prevent aspiration pneumonia, these are not the primary purposes for H2-receptor blockade in this patient

The nurse is reviewing laboratory results on a patient who had a large burn 48 hours ago. Which result requires priority action by the nurse? a. Hematocrit 53% b. Serum sodium 147 mEq/L c. Serum potassium 6.1 mEq/L d. Blood urea nitrogen 37 mg/dL

ANS: C Hyperkalemia can lead to fatal dysrhythmias and indicates that the patient requires cardiac monitoring and immediate treatment to lower the potassium level. The other laboratory values are also abnormal and require changes in treatment, but they are not as immediately life threatening as the elevated potassium level

A patient has a junctional escape rhythm on the monitor. The nurse will expect the patient to have a heart rate of _____ beats/min. a. 15 to 20 c. 40 to 60 b. 20 to 40 d. 60 to 100

ANS: C If the sinoatrial (SA) node fails to discharge, the atrioventricular (AV) node will automatically discharge at the normal rate of 40 to 60 beats/minute. The slower rates are typical of the bundle of His and Purkinje system and may be seen with failure of both the SA and AV node to discharge. The normal SA node rate is 60 to 100 beats/min.

A 21-yr-old woman is scheduled for percutaneous transluminal balloon valvuloplasty to treat mitral stenosis. Which information should the nurse include when explaining the advantages of valvuloplasty over valve replacement to the patient? a. Biologic valves will require immunosuppressive drugs after surgery. b. Mechanical mitral valves need to be replaced sooner than biologic valves. c. Lifelong anticoagulant therapy is needed after mechanical valve replacement. d. Ongoing cardiac care by a health care provider is not necessary after valvuloplasty.

ANS: C Long-term anticoagulation therapy is needed after mechanical valve replacement, and this would restrict decisions about career and childbearing in this patient. Mechanical valves are durable and last longer than biologic valves. All valve repair procedures are palliative, not curative, and require lifelong health care. Biologic valves do not activate the immune system and immunosuppressive therapy is not needed. DIF: Cognitive Level: Apply (application)

The nurse has administered prescribed IV mannitol (Osmitrol) to an unconscious patient. Which parameter should the nurse monitor to determine the medication's effectiveness? a. Blood pressure c. Intracranial pressure b. Oxygen saturation d. Hemoglobin and hematocrit

ANS: C Mannitol is an osmotic diuretic and will reduce cerebral edema and intracranial pressure. It may initially reduce hematocrit and increase blood pressure, but these are not the best parameters for evaluation of the effectiveness of the drug. O2 saturation will not directly improve as a result of mannitol administration.

The nurse will teach a patient with chronic pancreatitis to take the prescribed pancrelipase (Viokase) a. at bedtime. b. in the morning. c. with each meal. d. for abdominal pain.

ANS: C Pancreatic enzymes are used to help with digestion of nutrients and should be taken with every meal

A nurse has obtained donor tissue typing information about a patient who is waiting for a kidney transplant. Which results should be reported to the transplant surgeon? a. Patient is Rh positive and donor is Rh negative b. Six antigen matches are present in HLA typing c. Results of patient-donor cross matching are positive d. Panel of reactive antibodies (PRA) percentage is low

ANS: C Positive crossmatching is an absolute contraindication to kidney transplantation, since a hyperacute rejection will occur after the transplant. The other information indicates that the tissue match between the patient and potential donor is acceptable

An occupational health nurse works at a manufacturing plant where there is potential exposure to inhaled dust. Which action, if recommended by the nurse, will be most helpful in reducing the incidence of lung disease? a. Treat workers with pulmonary fibrosis. b. Teach about symptoms of lung disease. c. Require the use of protective equipment. d. Monitor workers for coughing and wheezing.

ANS: C Prevention of lung disease requires the use of appropriate protective equipment such as masks. The other actions will help in recognition or early treatment of lung disease but will not be effective in prevention of lung damage. Repeated exposure eventually results in diffuse pulmonary fibrosis. Fibrosis is the result of tissue repair after inflammation

A 37-year-old patient is being admitted with a diagnosis of Cushing syndrome. Which findings will the nurse expect during the assessment? a. Chronically low blood pressure b. Bronzed appearance of the skin c. Purplish streaks on the abdomen d. Decreased axillary and pubic hair

ANS: C Purplish-red striae on the abdomen are a common clinical manifestation of Cushing syndrome. Hypotension and bronzed-appearing skin are manifestations of Addison's disease. Decreased axillary and pubic hair occur with androgen deficiency

A patient who was admitted with a myocardial infarction experiences a 45-second episode of ventricular tachycardia, then converts to sinus rhythm with a heart rate of 98 beats/min. Which action should the nurse take next? a. Immediately notify the health care provider. b. Document the rhythm and continue to monitor the patient. c. Prepare to give IV amiodarone per agency dysrhythmia protocol. d. Perform synchronized cardioversion per agency dysrhythmia protocol.

ANS: C The burst of sustained ventricular tachycardia indicates that the patient has significant ventricular irritability, and antidysrhythmic medication administration is needed to prevent further episodes. The nurse should notify the health care provider after the medication is started. Cardioversion is not indicated given that the patient has returned to a sinus rhythm. Documentation and continued monitoring are not adequate responses to this situation. DIF: Cognitive Level: Analyze (analysis)

During the teaching session for a patient who has a new diagnosis of acute leukemia the patient is restless and is looking away, never making eye contact. After teaching about the complications associated with chemotherapy, the patient asks the nurse to repeat all of the information. Based on this assessment, which nursing diagnosis is most appropriate for the patient? a. Risk for ineffective adherence to treatment related to denial of need for chemotherapy b. Acute confusion related to infiltration of leukemia cells into the central nervous system c. Risk for ineffective health maintenance related to anxiety about new leukemia diagnosis d. Deficient knowledge: chemotherapy related to a lack of interest in learning about treatment

ANS: C The patient who has a new cancer diagnosis is likely to have high anxiety, which may impact learning and require that the nurse repeat and reinforce information. The patient's history of a recent diagnosis suggests that infiltration of the leukemia is not a likely cause of the confusion. The patient asks for the information to be repeated, indicating that lack of interest in learning and denial are not etiologic factors

When taking the blood pressure (BP) on the right arm of a patient with severe acute pancreatitis, the nurse notices carpal spasms of the patient's right hand. Which action should the nurse take next? a. Ask the patient about any arm pain. b. Retake the patient's blood pressure. c. Check the calcium level in the chart. d. Notify the health care provider immediately.

ANS: C The patient with acute pancreatitis is at risk for hypocalcemia, and the assessment data indicate a positive Trousseau's sign. The health care provider should be notified after the nurse checks the patient's calcium level. There is no indication that the patient needs to have the BP rechecked or that there is any arm pain

Which assessment finding for a 33-year-old female patient admitted with Graves' disease requires the most rapid intervention by the nurse? a. Bilateral exophthalmos b. Heart rate 136 beats/minute c. Temperature 103.8° F (40.4° C) d. Blood pressure 166/100 mm Hg

ANS: C The patient's temperature indicates that the patient may have thyrotoxic crisis and that interventions to lower the temperature are needed immediately. The other findings also require intervention but do not indicate potentially life-threatening complications

When caring for a patient with pulmonary hypertension, which parameter will the nurse use to directly evaluate the effectiveness of the treatment? a. Central venous pressure (CVP) b. Systemic vascular resistance (SVR) c. Pulmonary vascular resistance (PVR) d. Pulmonary artery wedge pressure (PAWP)

ANS: C PVR is a major contributor to pulmonary hypertension, and a decrease would indicate that pulmonary hypertension was improving. The other parameters may also be monitored but do not directly assess for pulmonary hypertension

A 23-year-old patient is admitted with diabetes insipidus. Which action will be most appropriate for the registered nurse (RN) to delegate to an experienced licensed practical/vocational nurse (LPN/LVN)? a. Titrate the infusion of 5% dextrose in water. b. Teach the patient how to use desmopressin (DDAVP) nasal spray. c. Assess the patient's hydration status every 8 hours. d. Administer subcutaneous DDAVP.

ANS: D Administration of medications is included in LPN/LVN education and scope of practice. Assessments, patient teaching, and titrating fluid infusions are more complex skills and should be done by the RN.

The nurse will ask a 64-year-old patient being admitted with acute pancreatitis specifically about a history of a. diabetes mellitus. b. high-protein diet. c. cigarette smoking. d. alcohol consumption.

ANS: D Alcohol use is one of the most common risk factors for pancreatitis in the United States. Cigarette smoking, diabetes, and high-protein diets are not risk factors

A patient has been assigned the nursing diagnosis of imbalanced nutrition: less than body requirements related to painful oral ulcers. Which nursing action will be most effective in improving oral intake? a. Offer the patient frequent small snacks between meals. b. Assist the patient to choose favorite foods from the menu. c. Provide teaching about the importance of nutritional intake. d. Apply the ordered anesthetic gel to oral lesions before meals.

ANS: D Because the etiology of the patient's poor nutrition is the painful oral ulcers, the best intervention is to apply anesthetic gel to the lesions before the patient eats. The other actions might be helpful for other patients with impaired nutrition, but would not be as helpful for this patient

The nurse supervises unlicensed assistive personnel (UAP) who are providing care for a patient with right lower lobe pneumonia. The nurse should intervene if which action by UAP is observed? a. UAP splint the patient's chest during coughing. b. UAP assist the patient to ambulate to the bathroom. c. UAP help the patient to a bedside chair for meals. d. UAP lower the head of the patient's bed to 15 degrees.

ANS: D Positioning the patient with the head of the bed lowered will decrease ventilation. The other actions are appropriate for a patient with pneumonia

24. A patient with hyperlipidemia has a new order for colesevelam (Welchol). Which nursing action is most appropriate when giving the medication? a. Have the patient take this medication with an aspirin. b. Administer the medication at the patient's usual bedtime. c. Have the patient take the colesevelam with a sip of water. d. Give the patient's other medications 2 hours after the colesevelam.

ANS: D The bile acid sequestrants interfere with the absorption of many other drugs, and giving other medications at the same time should be avoided. Taking an aspirin concurrently with the colesevelam may increase the incidence of gastrointestinal side effects such as heartburn. An increased fluid intake is encouraged for patients taking the bile acid sequestrants to reduce the risk for constipation. For maximum effect, colesevelam should be administered with meals.

A nurse is considering which patient to admit to the same room as a patient who had a liver transplant 3 weeks ago and is now hospitalized with acute rejection. Which patient would be the best choice? a. Patient who is receiving chemotherapy for liver cancer b. Patient who is receiving treatment for acute hepatitis C c. Patient who has a wound infection after cholecystectomy d. Patient who requires pain management for chronic pancreatitis

ANS: D The patient with chronic pancreatitis does not present an infection risk to the immunosuppressed patient who had a liver transplant. The other patients either are at risk for infection or currently have an infection, which will place the immunosuppressed patient at risk for infection.

While assisting with the placement of a pulmonary artery (PA) catheter, the nurse notes that the catheter is correctly placed when the balloon is inflated and the monitor shows a a. typical PA pressure waveform. b. tracing of the systemic arterial pressure. c. tracing of the systemic vascular resistance. d. typical PA wedge pressure (PAWP) tracing.

ANS: D The purpose of a PA line is to measure PAWP, so the catheter is floated through the pulmonary artery until the dilated balloon wedges in a distal branch of the pulmonary artery, and the PAWP readings are available. After insertion, the balloon is deflated and the PA waveform will be observed. Systemic arterial pressures are obtained using an arterial line, and the systemic vascular resistance is a calculated value, not a waveform.

When assisting with oral intubation of a patient who is having respiratory distress, in which order will the nurse take these actions? (Put a comma and a space between each answer choice [A, B, C, D, E].) a. Obtain a portable chest-x-ray. b. Position the patient in the supine position. c. Inflate the cuff of the endotracheal tube after insertion. d. Attach an end-tidal CO2 detector to the endotracheal tube. e. Oxygenate the patient with a bag-valve-mask device for several minutes.

ANS: E, B, C, D, A The patient is pre-oxygenated with a bag-valve-mask system for 3 to 5 minutes before intubation and then placed in a supine position. After the intubation, the cuff on the endotracheal tube is inflated to occlude and protect the airway. Tube placement is assessed first with an end-tidal CO2 sensor and then with chest x-ray examination.

Which action will the nurse include in the plan of care for a 62-year-old patient who is experiencing pain from trigeminal neuralgia? a. Assess fluid and dietary intake. b. Apply ice packs for 20 minutes. c. Teach facial relaxation techniques. d. Spend time talking with the patient.

Assess fluid and dietary intake. The patient with an acute episode of trigeminal neuralgia may be unwilling to eat or drink, so assessment of nutritional and hydration status is important. Because stimulation by touch is the precipitating factor for pain, relaxation of the facial muscles will not improve symptoms. Application of ice is likely to precipitate pain. The patient will not want to engage in conversation, which may precipitate attacks.

A patient admitted with dermal ulcers who has a history of a T3 spinal cord injury tells the nurse, "I have a pounding headache and I feel sick to my stomach." Which action should the nurse take first? a. Check for a fecal impaction. b. Give the prescribed analgesic. c. Assess the blood pressure (BP). d. Notify the health care provider.

Assess the blood pressure (BP). The BP should be assessed immediately in a patient with an injury at the T6 level or higher who complains of a headache to determine whether autonomic dysreflexia is occurring. Notification of the patient's health care provider is appropriate after the BP is obtained. Administration of an antiemetic is indicated after autonomic dysreflexia is ruled out as the cause of the nausea. After checking the BP, the nurse may assess for a fecal impaction using lidocaine jelly to prevent further increased BP.

Which nursing action has the highest priority for a patient who was admitted 16 hours previously with a C5 spinal cord injury? a. Cardiac monitoring for bradycardia b. Assessment of respiratory rate and effort c. Application of pneumatic compression devices to legs d. Administration of methylprednisolone (Solu-Medrol) infusion

Assessment of respiratory rate and effort Edema around the area of injury may lead to damage above the C4 level, so the highest priority is assessment of the patient's respiratory function. Methylprednisolone (Solu-Medrol) is no longer recommended for the treatment of spinal cord injuries. The other actions also are appropriate but are not as important as assessment of respiratory effort.

Which nursing action will the home health nurse include in the plan of care for a patient with paraplegia at the T4 level in order to prevent autonomic dysreflexia? a. Support selection of a high-protein diet. b. Discuss options for sexuality and fertility. c. Assist in planning a prescribed bowel program. d. Use quad coughing to strengthen cough efforts.

Assist in planning a prescribed bowel program. Fecal impaction is a common stimulus for autonomic dysreflexia. Dietary protein, coughing, and discussing sexuality/fertility should be included in the plan of care but will not reduce the risk for autonomic dysreflexia.

Which intervention will be included in the nursing care plan for a patient with immune thrombocytopenic purpura? a. Assign the patient to a private room. b. Avoid intramuscular (IM) injections. c. Use rinses rather than a soft toothbrush for oral care. d. Restrict activity to passive and active range of motion.

B IM or subcutaneous injections should be avoided because of the risk for bleeding. A soft toothbrush can be used for oral care. There is no need to restrict activity or place the patient in a private room.

When a patient with splenomegaly is scheduled for splenectomy, which action will the nurse include in the preoperative plan of care? a. Discourage deep breathing to reduce risk for splenic rupture. b. Teach the patient to use ibuprofen for left upper quadrant pain. c. Schedule immunization with the pneumococcal vaccine (e.g., Pneumovax). d. Avoid the use of acetaminophen (Tylenol) for at least 2 weeks prior to surgery.

C Asplenic patients are at high risk for infection with pneumococcal infections and immunization reduces this risk. There is no need to avoid acetaminophen use before surgery, but nonsteroidal antiinflammatory drugs (NSAIDs) may increase bleeding risk and should be avoided. The enlarged spleen may decrease respiratory depth, and the patient should be encouraged to take deep breaths.

Following successful treatment of Hodgkin's lymphoma for a 55-yr-old woman, which topic will the nurse include in patient teaching? a. Potential impact of chemotherapy treatment on fertility b. Application of soothing lotions to treat residual pruritus c. Use of maintenance chemotherapy to maintain remission d. Need for follow-up appointments to screen for malignancy

D The chemotherapy used in treating Hodgkin's lymphoma results in a high incidence of secondary malignancies; follow-up screening is needed. The fertility of a 55-yr-old woman will not be impacted by chemotherapy. Maintenance chemotherapy is not used for Hodgkin's lymphoma. Pruritus is a clinical manifestation of lymphoma but should not be a concern after treatment.

Which finding in a patient with a spinal cord tumor is most important for the nurse to report to the health care provider? a. Back pain that increases with coughing b. Depression about the diagnosis of a tumor c. Decreasing sensation and ability to move the legs d. Anxiety about scheduled surgery to remove the tumor

Decreasing sensation and ability to move the legs Decreasing sensation and leg movement indicates spinal cord compression, an emergency that will require rapid action (such as surgery) to prevent paralysis. The other findings will also require nursing action but are not emergencies.

A patient is admitted with possible botulism poisoning after eating home-canned green beans. Which intervention ordered by health care provider will the nurse question? a. Encourage oral fluids to 3 L/day b. Document neurologic symptoms c. Position patient lying on the side d. Observe respiratory status closely

Encourage oral fluids to 3 L/day The patient should be maintained on NPO status because neuromuscular weakness increases risk for aspiration. Side-lying position is not contraindicated. Assessment of neurologic and respiratory status is appropriate.

The nurse is admitting a patient with a neck fracture at the C6 level to the intensive care unit. Which assessment finding(s) indicate(s) neurogenic shock? a. Hyperactive reflex activity below the level of injury b. Involuntary, spastic movements of the arms and legs c. Hypotension, bradycardia, and warm, pink extremities d. Lack of sensation or movement below the level of injury

Hypotension, bradycardia, and warm, pink extremities Neurogenic shock is characterized by hypotension, bradycardia, and vasodilation leading to warm skin temperature. Spasticity and hyperactive reflexes do not occur at this stage of spinal cord injury. Lack of movement and sensation indicate spinal cord injury, but not neurogenic shock.

Which action should the nurse take when assessing a patient with trigeminal neuralgia? a. Have the patient clench the jaws. b. Inspect the oral mucosa and teeth. c. Palpate the face to compare skin temperature bilaterally. d. Identify trigger zones by lightly touching the affected side.

Inspect the oral mucosa and teeth. Oral hygiene is frequently neglected because of fear of triggering facial pain. Having the patient clench the facial muscles will not be useful because the sensory branches of the nerve are affected by trigeminal neuralgia. Light touch and palpation may be triggers for pain and should be avoided.

A 33-year-old patient with a T4 spinal cord injury asks the nurse whether he will be able to be sexually active. Which initial response by the nurse is best? a. Reflex erections frequently occur, but orgasm may not be possible. b. Sildenafil (Viagra) is used by many patients with spinal cord injury. c. Multiple options are available to maintain sexuality after spinal cord injury. d. Penile injection, prostheses, or vacuum suction devices are possible options.

Multiple options are available to maintain sexuality after spinal cord injury. Although sexuality will be changed by the patient's spinal cord injury, there are options for expression of sexuality and for fertility. The other information also is correct, but the choices will depend on the degrees of injury and the patient's individual feelings about sexuality.

An expected nursing diagnosis for a 30-year-old patient admitted to the hospital with symptoms of diabetes insipidus is a. excess fluid volume related to intake greater than output. b. impaired gas exchange related to fluid retention in lungs. c. sleep pattern disturbance related to frequent waking to void. d. risk for impaired skin integrity related to generalized edema.

Nocturia occurs as a result of the polyuria caused by diabetes insipidus. Edema, excess fluid volume, and fluid retention are not expected

Which of these nursing actions for a 64-year-old patient with Guillain-Barré syndrome is most appropriate for the nurse to delegate to an experienced unlicensed assistive personnel (UAP)? a. Nasogastric tube feeding q4hr b. Artificial tear administration q2hr c. Assessment for bladder distention q2hr d. Passive range of motion to extremities q4hr

Passive range of motion to extremities q4hr Assisting a patient with movement is included in UAP education and scope of practice. Administration of tube feedings, administration of ordered medications, and assessment are skills requiring more education and scope of practice, and the RN should perform these skills.

A 20-year-old patient who sustained a T2 spinal cord injury 10 days ago angrily tells the nurse "I want to be transferred to a hospital where the nurses know what they are doing!" Which action by the nurse is best? a. Clarify that abusive language will not be tolerated. b. Request that the patient provide input for the plan of care. c. Perform care without responding to the patient's comments. d. Reassure the patient about the competence of the nursing staff.

Request that the patient provide input for the plan of care. The patient is demonstrating behaviors consistent with the anger phase of the grief process, and the nurse should allow expression of anger and seek the patient's input into care. Expression of anger is appropriate at this stage, and should be accepted by the nurse. Reassurance about the competency of the staff will not be helpful in responding to the patient's anger. Ignoring the patient's comments will increase the patient's anger and sense of helplessness.

A 39-year-old patient is being evaluated for a possible spinal cord tumor. Which finding by the nurse requires the most immediate action? a. The patient has new onset weakness of both legs. b. The patient complains of chronic severe back pain. c. The patient starts to cry and says, "I feel hopeless." d. The patient expresses anxiety about having surgery.

The patient has new onset weakness of both legs. The new onset of symptoms indicates cord compression, which is an emergency that requires rapid treatment to avoid permanent loss of function. The other patient assessments also indicate a need for nursing action but do not require intervention as rapidly as the new onset weakness.

Which assessment data for a patient who has Guillain-Barré syndrome will require the nurse's most immediate action? a. The patient's triceps reflexes are absent. b. The patient is continuously drooling saliva. c. The patient complains of severe pain in the feet. d. The patient's blood pressure (BP) is 150/82 mm Hg.

The patient is continuously drooling saliva. Drooling indicates decreased ability to swallow, which places the patient at risk for aspiration and requires rapid nursing and collaborative actions such as suctioning and possible endotracheal intubation. The foot pain should be treated with appropriate analgesics, and the BP requires ongoing monitoring, but these actions are not as urgently needed as maintenance of respiratory function. Absence of the reflexes should be documented, but this is a common finding in Guillain-Barré syndrome.

A 31-year-old woman who has multiple sclerosis (MS) asks the nurse about risks associated with pregnancy. Which response by the nurse is accurate? a. "MS symptoms may be worse after the pregnancy." b. "Women with MS frequently have premature labor." c. "MS is associated with an increased risk for congenital defects." d. "Symptoms of MS are likely to become worse during pregnancy."

a. "MS symptoms may be worse after the pregnancy." -During the postpartum period, women with MS are at greater risk for exacerbation of symptoms. There is no increased risk for congenital defects in infants born of mothers with MS. Symptoms of MS may improve during pregnancy. Onset of labor is not affected by MS.

The nurse observes a patient ambulating in the hospital hall when the patient's arms and legs suddenly jerk and the patient falls to the floor. The nurse will first a. Assess the patient for a possible head injury. b. Give the scheduled dose of divalproex (Depakote). c. Document the timing and description of the seizure. d. Notify the patient's hHCP about the seizure.

a. Assess the patient for a possible head injury. -The patient who has had a myoclonic seizure is at risk for head injury and should first be evaluated and treated for this possible complication. Documentation of the seizure, notification of the seizure, and administration of antiseizure medications are also appropriate actions, but the initial action should be assessment for injury.

A 64-year-old patient who has amyotrophic lateral sclerosis (ALS) is hospitalized with pneumonia. Which nursing action will be included in the plan of care? a. Assist with active range of motion (ROM). b. Observe for agitation and paranoia. c. Give muscle relaxants as needed to reduce spasms. d. Use simple words and phrases to explain procedures.

a. Assist with active range of motion (ROM). -ALS causes progressive muscle weakness, but assisting the patient to perform active ROM will help maintain strength as long as possible. Psychotic manifestations such as agitation and paranoia are not associated with ALS. Cognitive function is not affected by ALS, and the patient's ability to understand procedures will not be impaired. Muscle relaxants will further increase muscle weakness and depress respirations.

A patient has been taking phenytoin (Dilantin) for 2 years. Which action will the nurse take when evaluating for adverse effects of the medication? a. Inspect the oral mucosa. b. Listen to the lung sounds. c. Auscultate the bowel tones. d. Check pupil reaction to light.

a. Inspect the oral mucosa. -Phenytoin can cause gingival hyperplasia, but does not affect bowel tones, lung sounds, or pupil reaction to light.

A patient who is experiencing an acute asthma attack is admitted to the emergency department. Which assessment should the nurse complete first? a. Listen to the patient's breath sounds. b. Ask about inhaled corticosteroid use. c. Determine when the dyspnea started. d. Obtain the forced expiratory volume (FEV) flow rate.

a. Listen to the patient's breath sounds.

A patient who has been experiencing an asthma attack develops bradycardia and a decrease in wheezing. Which action should the nurse take first? a. Notify the health care provider. b. Document changes in respiratory status. c. Encourage the patient to cough and deep breathe. d. Administer IV methylprednisolone (Solu-Medrol).

a. Notify the health care provider. The patient's assessment indicates impending respiratory failure, and the nurse should prepare to assist with intubation and mechanical ventilation after notifying the health care provider.

Which nursing action for a patient with chronic obstructive pulmonary disease (COPD) could the nurse delegate to experienced unlicensed assistive personnel (UAP)? a. Obtain O2 saturation using pulse oximetry. b. Monitor for increased O2 need with exercise. c. Teach the patient about safe use of O2 at home. d. Adjust O2 to keep saturation in prescribed parameters.

a. Obtain O2 saturation using pulse oximetry.

After change-of-shift report, which patient should the nurse assess first? a. Patient with myasthenia gravis who is reporting increased muscle weakness. b. Patient with a bilateral headache described as "like a band around my head." c. Patient with seizures who is scheduled to receive a dose of phenytoin (Dilantin). d. Patient with Parkinson's disease who has developed cogwheel rigidity of the arms.

a. Patient with myasthenia gravis who is reporting increased muscle weakness. -Because increased muscle weakness may indicate the onset of myasthenic crisis, the nurse should assess this patient first. The other patients should also be assessed, but do not appear to need immediate nursing assessments or actions to prevent life-threatning complications.

A hospitalized 31-year-old patient with a history of cluster headache awakens during the night with a severe stabbing headache. Which action should the nurse take first? a. Start the ordered PRN oxygen at 6 L/min. b. Put a moist hot pack on the patient's neck. c. Give the ordered PRN acetaminophen (Tylenol). d. Notify the patient's HCP immediately.

a. Start the ordered PRN oxygen at 6 L/min. -Acute treatment for cluster headache is administration of 100% oxygen at 6 to 8 L/min. If the patient obtains relief with the oxygen, there is no immediate need to notify the HCP. Cluster headaches last only 60 to 90 minutes, so oral pain medications have minimal effect. Hot packs are helpful for tension headaches but are not likely to reduce pain associated with a cluster headache.

When evaluating outcomes of a glycerol rhizotomy for a patient with trigeminal neuralgia, the nurse will a. assess whether the patient is doing daily facial exercises. b. question whether the patient is using an eye shield at night. c. ask the patient about social activities with family and friends. d. remind the patient to chew on the unaffected side of the mouth.

ask the patient about social activities with family and friends. Because withdrawal from social activities is a common manifestation of trigeminal neuralgia, asking about social activities will help in evaluating whether the patient's symptoms have improved. Glycerol rhizotomy does not damage the corneal reflex or motor functions of the trigeminal nerve, so there is no need to use an eye shield, do facial exercises, or take precautions with chewing.

The nurse reviews the medication administration record (MAR) for a patient having an acute asthma attack. Which medication should the nurse administer first? a. Methylprednisolone (Solu-Medrol) 60 mg IV b. Albuterol (Ventolin HFA) 2.5 mg per nebulizer c. Salmeterol (Serevent) 50 mcg per dry-powder inhaler (DPI) d. Ipratropium (Atrovent) 2 puffs per metered-dose inhaler (MDI)

b. Albuterol (Ventolin HFA) 2.5 mg per nebulizer

A patient with severe chronic obstructive pulmonary disease (COPD) tells the nurse, "I wish I were dead! I'm just a burden on everybody." Based on this information, which nursing diagnosis is most appropriate? a. Complicated grieving related to expectation of death b. Chronic low self-esteem related to physical dependence c. Ineffective coping related to unknown outcome of illness d. Deficient knowledge related to lack of education about COPD

b. Chronic low self-esteem related to physical dependence

A patient with chronic obstructive pulmonary disease (COPD) has poor gas exchange. Which action by the nurse would support the patient's ventilation? a. Have the patient rest in bed with the head elevated to 15 to 20 degrees. b. Encourage the patient to sit up at the bedside in a chair and lean forward. c. Ask the patient to rest in bed in a high-Fowler's position with the knees flexed. d. Place the patient in the Trendelenburg position with pillows behind the head.

b. Encourage the patient to sit up at the bedside in a chair and lean forward. Patients with *COPD* improve the mechanics of breathing by sitting up in the *"tripod" position*.

Which assessment finding in a patient who has received omalizumab (Xolair) is most important to report immediately to the health care provider? a. Pain at injection site c. Peak flow reading 75% of normal b. Flushing and dizziness d. Respiratory rate 24 breaths/minute

b. Flushing and dizziness Flushing and dizziness may indicate that the patient is experiencing an anaphylactic reaction, and immediate intervention is needed.

A patient in the clinic with *cystic fibrosis (CF)* reports increased *sweating and weakness* during the *summer* months. Which action by the nurse would be most appropriate? a. Teach the patient signs of hypoglycemia. b. Have the patient add dietary salt to meals. c. Suggest decreasing intake of dietary fat and calories. d. Instruct the patient about pancreatic enzyme replacements.

b. Have the patient add dietary salt to meals. Added dietary salt is indicated whenever sweating is excessive, such as during hot weather, when fever is present, or from intense physical activity.

The nurse is admitting a patient diagnosed with an acute exacerbation of chronic obstructive pulmonary disease (COPD). How should the nurse determine the appropriate O2 flow rate? a. Minimize O2 use to avoid O2 dependency. b. Maintain the pulse oximetry level at 90% or greater. c. Administer O2 according to the patient's level of dyspnea. d. Avoid administration of O2 at a rate of more than 2 L/min.

b. Maintain the pulse oximetry level at 90% or greater.

The home health registered nurse is planning care for a patient with a seizure disorder related to a recent head injury. Which nursing action can be delegated to a LPN/LVN? a. Make referrals to appropriate community agencies. b. Place medications in the home medication organizer. c. Teach the patient and family how to manage seizures. d. Assess for use of medications that may precipitate seizures.

b. Place medications in the home medication organizer. -LPN/LVN education includes administration of medications.The other activities require RN education and scope of practice.

Which actions should the nurse initiate to reduce the risk for ventilator-associated pneumonia (VAP) (Select all that apply)? a. Obtain arterial blood gases daily. b. Provide a "sedation holiday" daily. c. Elevate the head of the bed to at least 30 degrees. d. Give prescribed pantoprazole (Protonix). e. Provide oral care with chlorhexidine (0.12%) solution daily.

b. Provide a "sedation holiday" daily. c. Elevate the head of the bed to at least 30 degrees. d. Give prescribed pantoprazole (Protonix). e. Provide oral care with chlorhexidine (0.12%) solution daily. -All of these interventions are part of the ventilator bundle that is recommended to prevent VAP. Arterial blood gases may be done daily but are not always necessary and do not help prevent VAP.

Which intervention will the nurse include in the plan of care for a patient with primary restless legs syndrome (RLS) who is having difficulty sleeping? a. Teach about the use of antihistamines to improve sleep. b. Suggest that the patient exercise regularly during the day. c. Make a referral to a massage therapist for deep massage of the legs. d. Assure the patient that the problem is transient and likely to resolve.

b. Suggest that the patient exercise regularly during the day. -Nondrug interventions such as getting regular exercise are initially suggested to improve sleep quality in patients with RLS. Antihistamines may aggravate RLS. Massage does not alleviate RLS symptoms and RLS is likely to progress in most patients.

A 73-year-old patient with Parkinson's disease has a nursing diagnosis of impaired physical mobility related to bradykinesia. Which action will the nurse include in the plan of care? a. Instruct the patient in activities that can be done while lying or sitting. b. Suggest that the patient rock from side to side to initiate leg movement. c. Have the patient take small steps in a straight line directly in from the the feet. d. Teach the patient to keep the feet in contact with the floor and slide them forward.

b. Suggest that the patient rock from side to side to initiate leg movement. -Rocking the body from side to side stimulates balance and improves mobility. The patient will be encouraged to continue exercising because this will maintain functional abilities. Maintaining a wide base of support will help with balance. The patient should lift the feet and avoid a shuffling gait.

A 55-yr-old patient with increasing dyspnea is being evaluated for a possible diagnosis of chronic obstructive pulmonary disease (COPD). When teaching a patient about pulmonary spirometry for this condition, what is the most important question the nurse should ask? a. "Are you claustrophobic?" b. "Are you allergic to shellfish?" c. "Have you taken any bronchodilators today?" d. "Do you have any metal implants or prostheses?"

c. "Have you taken any bronchodilators today?"

Postural drainage with percussion and vibration is ordered twice daily for a patient with chronic bronchitis. Which intervention should the nurse include in the plan of care? a. Schedule the procedure 1 hour after the patient eats. b. Maintain the patient in the lateral position for 20 minutes. c. Give the prescribed albuterol (Ventolin HFA) before the therapy. d. Perform percussion before assisting the patient to the drainage position.

c. Give the prescribed albuterol (Ventolin HFA) before the therapy. Bronchodilators are administered before chest physiotherapy.

The nurse is caring for a patient with *cor pulmonale*. The nurse should monitor the patient for which expected finding? a. Chest pain c. Peripheral edema b. Finger clubbing d. Elevated temperature

c. Peripheral edema *Cor pulmonale* causes clinical manifestations of *right ventricular failure*, such as *peripheral edema*.

A patient with cystic fibrosis (CF) has blood glucose levels that are consistently between 180 to 250 mg/dL. Which nursing action will the nurse plan to implement? a. Discuss the role of diet in blood glucose control. b. Evaluate the patient's use of pancreatic enzymes. c. Teach the patient about administration of insulin. d. Give oral hypoglycemic medications before meals.

c. Teach the patient about administration of insulin. The glucose levels indicate that the patient has developed CF-related diabetes, and insulin therapy is required. Because the etiology of diabetes in CF is inadequate insulin production, oral hypoglycemic agents are not effective.

The nurse teaches a patient how to administer *formoterol (Perforomist)* through a *nebulizer*. Which action by the patient indicates good understanding of the teaching? a. The patient attaches a spacer before using the inhaler. b. The patient coughs vigorously after using the inhaler. c. The patient removes the facial mask when misting stops. d. The patient activates the inhaler at the onset of expiration.

c. The patient removes the facial mask when misting stops. A *nebulizer* is used to administer aerosolized medication. A *mist* is seen when the medication is aerosolized, and when all of the medication has been used, the misting stops.

The nurse assesses a patient with a history of asthma. Which assessment finding indicates that the nurse should take immediate action? a. Pulse oximetry reading of 91% b. Respiratory rate of 26 breaths/min c. Use of accessory muscles in breathing d. Peak expiratory flow rate of 240 L/min

c. Use of accessory muscles in breathing

A patient is scheduled for *spirometry*. Which action should the nurse take to prepare the patient for this procedure? a. Give the rescue medication immediately before testing. b. Administer oral corticosteroids 2 hours before the procedure. c. Withhold bronchodilators for 6 to 12 hours before the examination. d. Ensure that the patient has been NPO for several hours before the test.

c. Withhold bronchodilators for 6 to 12 hours before the examination. *Bronchodilators* are *held before spirometry* so that a baseline assessment of airway function can be determined

Which prescribed intervention will the nurse implement first for a patient in the emergency department who is experiencing continuous tonic-clonic-seizures? a. Give phenytoin (Dilantin) 100 mg IV. b. Monitor level of consciousness (LOC). c. Obtain computed tomography (CT) scan. d. Administer lorazepam (Ativan) 4 mg IV.

d. Administer lorazepam (Ativan) 4 mg IV. -To prevent ongoing seizures, the nurse should administer rapidly acting antiseizure mediations such as the benzodiazepines. A CT scan is appropriate, but prevention of any seizure activity during the CT scan is necessary. Phenytoin will also be administered, but it is not rapidly acting. Patient who are experiencing tonic-clonic seizures are non responsive, although the nurse should assess LOC after the seizure.

Which medication taken by a patient with restless leg syndrome should the nurse discuss with the patient? a. Multivitamin (Stresstabs). b. Acetaminophen (Tylenol). c. Ibuprofen (Motrin, Advil). d. Diphenhydramine (Benadryl)/

d. Diphenhydramine (Benadryl)/ -Antihistamines can aggravate restless leg syndrome. The other medications will not contribute to restless leg syndrome.

Which nursing diagnosis is of highest priority for a patient with Parkinson's disease who is unable to move the facial muscles. a. Activity intolerance. b. Self-care deficit: toileting. c. Ineffective self-health management. d. Imbalanced nutrition: less than body requirements.

d. Imbalanced nutrition: less than body requirements. -The data about the patient indicate that poor nutrition will be a concern because of decreased swallowing. The other diagnoses may also be appropriate for a patient with Parkinson's disease, but the data do not indicate that they are current problems for this patient.

The nurse will explain to the patient who has a T2 spinal cord transection injury that a. use of the shoulders will be limited. b. function of both arms should be retained. c. total loss of respiratory function may occur. d. tachycardia is common with this type of injury.

function of both arms should be retained. The patient with a T2 injury can expect to retain full motor and sensory function of the arms. Use of only the shoulders is associated with cervical spine injury. Loss of respiratory function occurs with cervical spine injuries. Bradycardia is associated with injuries above the T6 level.

A 27-year-old patient is hospitalized with new onset of Guillain-Barré syndrome. The most essential assessment for the nurse to carry out is a. determining level of consciousness. b. checking strength of the extremities. c. observing respiratory rate and effort. d. monitoring the cardiac rate and rhythm.

observing respiratory rate and effort. The most serious complication of Guillain-Barré syndrome is respiratory failure, and the nurse should monitor respiratory function continuously. The other assessments will also be included in nursing care, but they are not as important as respiratory assessment.

Which menu choice indicates that the patient understands the nurse's teaching about recommended dietary choices for iron-deficiency anemia? a. Omelet and whole wheat toast b. Cantaloupe and cottage cheese c. Strawberry and banana fruit plate d. Cornmeal muffin and orange juice

A Eggs and whole grain breads are high in iron. The other choices are appropriate for other nutritional deficiencies but are not the best choice for a patient with iron-deficiency anemia.

The nurse is caring for a 78-year-old patient who was hospitalized 2 days earlier with community-acquired pneumonia. Which assessment information is most important to communicate to the HCP? a. Scattered crackles bilaterally in the posterior lung bases. b. Persistent cough that is productive of blood-tinged sputum. c. Temperature of 101.5 F (38.6 C) after 2 days of IV antibiotic therapy. d. Decreased oxygen saturation to 90% with 100% O2 by non-rebreather mask.

d. Decreased oxygen saturation to 90% with 100% O2 by non-rebreather mask. -The patient's low SpO2 despite receiving a high fraction of inspired oxygen (FIO2) indicates the possibility of ARDS. The patient's blood-tinged sputum and scattered crackles are not unusual in a patient with pneumonia, although they do require continued monitoring. The continued temperature elevation indicates a possible need to change antibiotics, but this is not as urgent a concern as the progression toward hypoxemia despite an increase in O2 flow rate.

Which nursing interventions included in the care of a mechanically ventilated patient with acute respiratory failure can the RN delegate to an experienced licensed practical/vocational (LPN/LVN) working in the ICU? a. Assess breath sounds every hour. b. Monitor central venous pressures. c. Place the patient in prone position. d. Insert an indwelling urinary catheter.

d. Insert an indwelling urinary catheter. -Insertion of indwelling urinary catheters is included in LPN/LVN education and scope of practice and can be safely delegated t an LPN/LVN who is experienced in caring for critically ill patients. Placing a patient who is on a ventilator in the prone position requires multiple staff, and should be supervised by an RN. Assessment of breath sounds and obtaining central venous pressures require advanced assessment skills and should be done by the RN caring for a critically ill patient.

A patient with ARDS who is intubated and receiving mechanical ventilation develops a right pneumothorax. Which action will the nurse anticipate taking next? a. Increase the tidal volume and respiratory rate. b. Increase the fraction of inspired oxygen (FIO2). c. Perform endotracheal suctioning more frequently. d. Lower the PEEP.

d. Lower the PEEP. -Because barotrauma is associated with high airway pressures, the level of PEEP should be decreased. The other actions will not decrease the risk for pneumothorax.

The nurse identifies a patient with type 1 diabetes and a history of herpes simplex infection as being at risk for Bell's palsy. Which information should the nurse include in teaching the patient? a. "You may be able to prevent Bell's palsy by doing facial exercises regularly." b. "Prophylactic treatment of herpes with antiviral agents prevents Bell's palsy." c. "Medications to treat Bell's palsy work only if started before paralysis onset." d. "Call the doctor if you experience pain or develop herpes lesions near the ear."

"Call the doctor if you experience pain or develop herpes lesions near the ear." Pain or herpes lesions near the ear may indicate the onset of Bell's palsy and rapid corticosteroid treatment may reduce the duration of Bell's palsy symptoms. Antiviral therapy for herpes simplex does not reduce the risk for Bell's palsy. Corticosteroid therapy will be most effective in reducing symptoms if started before paralysis is complete but will still be somewhat effective when started later. Facial exercises do not prevent Bell's palsy.

When caring for a patient who experienced a T2 spinal cord transection 24 hours ago, which collaborative and nursing actions will the nurse include in the plan of care (select all that apply)? a. Urinary catheter care b. Nasogastric (NG) tube feeding c. Continuous cardiac monitoring d. Maintain a warm room temperature e. Administration of H2 receptor blockers

-Urinary catheter care -Continuous cardiac monitoring -Maintain a warm room temperature -Administration of H2 receptor blockers

) are a first-line therapy for this type of angina. Lipid-lowering drugs help reduce atherosclerosis (i.e., plaque formation), and β-adrenergic blockers decrease sympathetic stimulation of the heart (i.e., palpitations). Medications or activities that increase myocardial contractility will increase the incidence of angina by increasing oxygen demand.

...

When assessing a patient with possible peripheral artery disease (PAD), the nurse obtains a brachial blood pressure (BP) of 147/82 mm Hg and an ankle pressure of 112/74 mm Hg. The nurse calculates the patient's ankle-brachial index (ABI) as ________ (round up to the nearest hundredth).

0.76 The ABI is calculated by dividing the ankle systolic BP by the brachial systolic BP.

Which action will the nurse include in the plan of care for a patient admitted with multiple myeloma? a. Monitor fluid intake and output. b. Administer calcium supplements. c. Assess lymph nodes for enlargement. d. Limit weight bearing and ambulation.

A A high fluid intake and urine output helps prevent the complications of kidney stones caused by hypercalcemia and renal failure caused by deposition of Bence-Jones protein in the renal tubules. Weight bearing and ambulation are encouraged to help bone retain calcium. Lymph nodes are not enlarged with multiple myeloma. Calcium supplements will further increase the patient's calcium level and are not used.

An appropriate nursing intervention for a patient with non-Hodgkin's lymphoma whose platelet count drops to 18,000/μL during chemotherapy is to a. check all stools for occult blood. b. encourage fluids to 3000 mL/day. c. provide oral hygiene every 2 hours. d. check the temperature every 4 hours.

A Because the patient is at risk for spontaneous bleeding, the nurse should check stools for occult blood. A low platelet count does not require an increased fluid intake. Oral hygiene is important, but it is not necessary to provide oral care every 2 hours. The low platelet count does not increase risk for infection, so frequent temperature monitoring is not indicated.

Which information obtained by the nurse assessing a patient admitted with multiple myeloma is most important to report to the health care provider? a. Serum calcium level is 15 mg/dL. b. Patient reports no stool for 5 days. c. Urine sample has Bence-Jones protein. d. Patient is complaining of severe back pain.

A Hypercalcemia may lead to complications such as dysrhythmias or seizures, and should be addressed quickly. The other patient findings will also be discussed with the health care provider but are not life threatening.

Which patient statement to the nurse indicates a need for additional instruction about taking oral ferrous sulfate? a. "I will call my health care provider if my stools turn black." b. "I will take a stool softener if I feel constipated occasionally." c. "I should take the iron with orange juice about an hour before eating." d. "I should increase my fluid and fiber intake while I am taking iron tablets."

A It is normal for the stools to appear black when a patient is taking iron, and the patient should not call the health care provider about this. The other patient statements are correct.

Which patient should the nurse assign as the roommate for a patient who has aplastic anemia? a. A patient with chronic heart failure b. A patient who has viral pneumonia c. A patient who has right leg cellulitis d. A patient with multiple abdominal drains

A Patients with aplastic anemia are at risk for infection because of the low white blood cell production associated with this type of anemia, so the nurse should avoid assigning a roommate with any possible infectious process.

A patient with immune thrombocytopenic purpura (ITP) has an order for a platelet transfusion. Which information indicates that the nurse should consult with the health care provider before obtaining and administering platelets? a. Platelet count is 42,000/mL. b. Petechiae are present on the chest. c. Blood pressure (BP) is 94/56 mm Hg. d. Blood is oozing from the venipuncture site.

A Platelet transfusions are not usually indicated until the platelet count is below 10,000 to 20,000/mL unless the patient is actively bleeding. Therefore the nurse should clarify the order with the health care provider before giving the transfusion. The other data all indicate that bleeding caused by ITP may be occurring and that the platelet transfusion is appropriate.

The nurse has obtained the health history, physical assessment data, and laboratory results shown in the accompanying figure for a patient admitted with aplastic anemia. Which information is most important to communicate to the health care provider? History Physical Assessment Laboratory Results · Fatigue, which has increased over last month · Frequent constipation · Conjunctiva pale pink, moist · Multiple bruises · Clear lung sounds · Hct 33% · WBC 1500/μL · Platelets 70,000/ μL a. Neutropenia c. Increasing fatigue b. Constipation d. Thrombocytopenia

A The low white blood cell count indicates that the patient is at high risk for infection and needs immediate actions to diagnose and treat the cause of the leukopenia. The other information may require further assessment or treatment but does not place the patient at immediate risk for complications.

The nurse is developing a discharge teaching plan for a patient diagnosed with thromboangiitis obliterans (Buerger's disease). Which expected outcome has the highest priority for this patient? a.Cessation of all tobacco use b.Control of serum lipid levels c.Maintenance of appropriate weight d.Demonstration of meticulous foot care

A Absolute cessation of nicotine use is needed to reduce the risk for amputation in patients with Buerger's disease. Other therapies have limited success in treatment of this disease.

Which patient statement to the nurse is most consistent with the diagnosis of venous insufficiency? a."I can't get my shoes on at the end of the day." b."I can't ever seem to get my feet warm enough." c."I have burning leg pains after I walk two blocks." d."I wake up during the night because my legs hurt."

A Because the edema associated with venous insufficiency increases when the patient has been standing, shoes will feel tighter at the end of the day. The other patient statements are characteristic of peripheral artery disease.

An older patient with a history of an abdominal aortic aneurysm arrives at the emergency department (ED) with severe back pain and absent pedal pulses. Which action should the nurse take first? a.Check the blood pressure. b.Draw blood for laboratory testing. c.Assess for the presence of an abdominal bruit. d.Determine any family history of heart disease.

A Because the patient appears to be experiencing aortic dissection, the nurse's first action should be to determine the hemodynamic status by assessing blood pressure. The other actions may also be done, but they will not provide information to determine what interventions are needed immediately.

When evaluating the discharge teaching for a patient with chronic peripheral artery disease (PAD), the nurse determines a need for further instruction when the patient says, "I will a.use a heating pad on my feet at night to increase the circulation." b.buy some loose clothes that do not bind across my legs or waist." c.walk to the point of pain, rest, and walk again for at least 30 minutes 3 times a week." d.change my position every hour and avoid long periods of sitting with my legs crossed."

A Because the patient has impaired circulation and sensation to the feet, the use of a heating pad could lead to burns. The other patient statements are correct and indicate that teaching has been successful.

The nurse is caring for a patient with critical limb ischemia who has just arrived on the nursing unit after having percutaneous transluminal balloon angioplasty. Which action should the nurse perform first? a.Obtain vital signs. b.Teach wound care. c.Assess pedal pulses. d.Check the wound site.

A Bleeding is a possible complication after catheterization of the femoral artery, so the nurse's first action should be to assess for changes in vital signs that might indicate hemorrhage. The other actions are also appropriate but can be done after determining that bleeding is not occurring.

The nurse is caring for a patient immediately after repair of an abdominal aortic aneurysm. On assessment, the patient has absent popliteal, posterior tibial, and dorsalis pedis pulses. The legs are cool and mottled. Which action should the nurse take first? a.Notify the surgeon and anesthesiologist. b.Wrap both the legs in a warming blanket. c.Document the findings and recheck in 15 minutes. d.Compare findings to the preoperative assessment of the pulses.

A Lower extremity pulses may be absent for a short time after surgery because of vasospasm and hypothermia. Decreased or absent pulses together with a cool and mottled extremity may indicate embolization or graft occlusion. These findings should be reported to the surgeon immediately because this is an emergency situation. Because pulses are marked before surgery, the nurse would know whether pulses were present before surgery before notifying the health care providers about the absent pulses. Because the patient's symptoms may indicate graft occlusion or multiple emboli and a possible need to return to surgery, it is not appropriate to wait 15 minutes before taking action. A warming blanket will not improve the circulation to the patient's legs.

A patient who is 2 days post femoral popliteal bypass graft to the right leg is being cared for on the vascular unit. Which action by a licensed practical/vocational nurse (LPN/LVN) caring for the patient requires the registered nurse (RN) to intervene? a.The LPN/LVN has the patient to sit in a chair for 2 hours. b.The LPN/LVN gives the prescribed aspirin after breakfast. c.The LPN/LVN assists the patient to walk 40 feet in the hallway. d.The LPN/LVN places the patient in Fowler's position for meals.

A The patient should avoid sitting for long periods because of the increased stress on the suture line caused by leg edema and because of the risk for venous thromboembolism (VTE). The other actions by the LPN/LVN are appropriate.

A patient is to receive methylprednisolone (Solu-Medrol) 100 mg. The label on the medication states: methylprednisolone 125 mg in 2 mL. How many milliliters will the nurse administer?

ANS: 1.6 A concentration of 125 mg in 2 mL will result in 100 mg in 1.6 mL.

A patient is to receive an infusion of 250 mL of platelets over 2 hours through tubing that is labeled: 1 mL equals 10 drops. How many drops per minute will the nurse infuse?

ANS: 21 To infuse 250 mL over 2 hours, the calculated drip rate is 20.8 drops/min or 21 drops/min.

The following four patients arrive in the emergency department (ED) after a motor vehicle collision. In which order should the nurse assess them? (Put a comma and a space between each answer choice [A, B, C, D, E].) a. A 74-yr-old patient with palpitations and chest pain b. A 43-yr-old patient complaining of 7/10 abdominal pain c. A 21-yr-old patient with multiple fractures of the face and jaw d. A 37-yr-old patient with a misaligned lower left leg with intact pulses

ANS: C, A, B, D The highest priority is to assess the 21-yr-old patient for airway obstruction, which is the most life-threatening injury. The 74-yr-old patient may have chest pain from cardiac ischemia and should be assessed and have diagnostic testing for this pain. The 43-yr-old patient may have abdominal trauma or bleeding and should be seen next to assess circulatory status. The 37-yr-old patient appears to have a possible fracture of the left leg and should be seen soon, but this patient has the least life-threatening injury.

When analyzing an electrocardiographic (ECG) rhythm strip of a patient with a regular heart rhythm, the nurse counts 30 small blocks from one R wave to the next. The nurse calculates the patient's heart rate as ____.

ANS: 50 There are 1500 small blocks in a minute, and the nurse will divide 1500 by 30. DIF: Cognitive Level: Apply (application)

An 80-kg patient with burns over 30% of total body surface area (TBSA) is admitted to the burn unit. Using the Parkland formula of 4 mL/kg/%TBSA, what is the IV infusion rate (mL/hour) for lactated Ringer's solution that the nurse will administer during the first 8 hours?

ANS: 600 mL The Parkland formula states that patients should receive 4 mL/kg/%TBSA burned during the first 24 hours. Half of the total volume is given in the first 8 hours and then the last half is given over 16 hours: 4 ´ 80 ´ 30 = 9600 mL total volume; 9600/2 = 4800 mL in the first 8 hours; 4800 mL/8 hr = 600 mL/hr.

An unconscious patient with a traumatic head injury has a blood pressure of 130/76 mm Hg and an intracranial pressure (ICP) of 20 mm Hg. The nurse will calculate the cerebral perfusion pressure (CPP) as ____ mm Hg.

ANS: 74 Calculate the CPP: (CPP = Mean arterial pressure [MAP] - ICP). MAP = DBP + 1/3 (Systolic blood pressure [SBP] - Diastolic blood pressure [DBP]). The MAP is 94. The CPP is 74.

1. A patient in the oliguric phase after an acute kidney injury has had a 250-mL urine output and an emesis of 100 mL in the past 24 hours. What is the patient's fluid restriction for the next 24 hours?

ANS: 950 mL The general rule for calculating fluid restrictions is to add all fluid losses for the previous 24 hours, plus 600 mL for insensible losses: (250 + 100 + 600 = 950 mL). DIF: Cognitive Level: Understand (comprehension) REF: 1073 TOP: Nursing Process: Implementation MSC: NCLEX: Physiological Integrity

When preparing to defibrillate a patient, in which order will the nurse perform the following steps? (Put a comma and a space between each answer choice [A, B, C, D, E].) a. Turn the defibrillator on. b. Deliver the electrical charge. c. Select the appropriate energy level. d. Place the hands-free, multifunction defibrillator pads on the patient's chest. e. Check the location of other staff and call out "all clear."

ANS: A, C, D, E, B This order will result in rapid defibrillation without endangering hospital staff. DIF: Cognitive Level: Analyze (analysis)

In which order will the nurse take these actions when doing a dressing change for a partial-thickness burn wound on a patient's chest? (Put a comma and a space between each answer choice [A, B, C, D, E].) a. Apply sterile gauze dressing. b. Document wound appearance. c. Apply silver sulfadiazine cream. d. Administer IV fentanyl (Sublimaze). e. Clean wound with saline-soaked gauze.

ANS: D, E, C, A, B Because partial-thickness burns are very painful, the nurse's first action should be to administer pain medications. The wound will then be cleaned, antibacterial cream applied, and covered with a new sterile dressing. The last action should be to document the appearance of the wound

A patient's vital signs are pulse 87, respirations 24, and BP 128/64 mm Hg, and cardiac output is 4.7 L/min. The patient's stroke volume is _____ mL. (Round to the nearest whole number.)

ANS: 54 Stroke volume = Cardiac output/heart rate

30. A 50-year-old female patient calls the clinic to report a new onset of severe diarrhea. The nurse anticipates that the patient will need to a. collect a stool specimen. b. prepare for colonoscopy. c. schedule a barium enema. d. have blood cultures drawn.

ANS: A Acute diarrhea is usually caused by an infectious process, and stool specimens are obtained for culture and examined for parasites or white blood cells. There is no indication that the patient needs a colonoscopy, blood cultures, or a barium enema.

51. Which menu choice by the patient with diverticulosis is best for preventing diverticulitis? a. Navy bean soup and vegetable salad b. Whole grain pasta with tomato sauce c. Baked potato with low-fat sour cream d. Roast beef sandwich on whole wheat bread

ANS: A A diet high in fiber and low in fats and red meat is recommended to prevent diverticulitis. Although all of the choices have some fiber, the bean soup and salad will be the highest in fiber and the lowest in fat.

A patient arrives in the emergency department with facial and chest burns caused by a house fire. Which action should the nurse take first? a. Auscultate the patient's lung sounds. b. Determine the extent and depth of the burns. c. Infuse the ordered lactated Ringer's solution. d. Administer the ordered hydromorphone (Dilaudid).

ANS: A A patient with facial and chest burns is at risk for inhalation injury, and assessment of airway and breathing is the priority. The other actions will be completed after airway management is assured

A patient is admitted to the hospital with acute rejection of a kidney transplant. Which intervention will the nurse prepare for this patient? a. Administration of immunosuppressant medications b. Insertion of an arteriovenous graft for hemodialysis c. Placement of the patient on the transplant waiting list d. A blood draw for human leukocyte antigen (HLA) matching

ANS: A Acute rejection is treated with the administration of additional immunosuppressant drugs such as corticosteroids. Because acute rejection is potentially reversible, there is no indication that the patient will require another transplant or hemodialysis. There is no indication for repeat HLA testing.

29. When caring for a patient who has just arrived on the medical-surgical unit after having cardiac catheterization, which nursing intervention should the nurse delegate to a licensed practical/vocational nurse (LPN/LVN)? a. Give the scheduled aspirin and lipid-lowering medication. b. Perform the initial assessment of the catheter insertion site. c. Teach the patient about the usual postprocedure plan of care. d. Titrate the heparin infusion according to the agency protocol.

ANS: A Administration of oral medications is within the scope of practice for LPNs/LVNs. The initial assessment of the patient, patient teaching, and titration of IV anticoagulant medications should be done by the registered nurse (RN).

25. A 34-year-old female patient with a new ileostomy asks how much drainage to expect. The nurse explains that after the bowel adjusts to the ileostomy, the usual drainage will be about _____ cups. a. 2 b. 3 c. 4 d. 5

ANS: A After the proximal small bowel adapts to reabsorb more fluid, the average amount of ileostomy drainage is about 500 mL daily. One cup is about 240 mL.

A 68-yr-old male patient is brought to the emergency department (ED) by ambulance after being found unconscious on the bathroom floor by his spouse. Which action will the nurse take first? a. Check oxygen saturation. b. Assess pupil reaction to light. c. Palpate the head for injuries d. Verify Glasgow Coma Scale (GCS) score.

ANS: A Airway patency and breathing are the most vital functions and should be assessed first. The neurologic assessments should be accomplished next and additional assessment after that.

5. A patient complains of gas pains and abdominal distention two days after a small bowel resection. Which nursing action is best to take? a. Encourage the patient to ambulate. b. Instill a mineral oil retention enema. c. Administer the ordered IV morphine sulfate. d. Offer the ordered promethazine (Phenergan) suppository.

ANS: A Ambulation will improve peristalsis and help the patient eliminate flatus and reduce gas pain. A mineral oil retention enema is helpful for constipation with hard stool. A return-flow enema might be used to relieve persistent gas pains. Morphine will further reduce peristalsis. Promethazine (Phenergan) is used as an antiemetic rather than to decrease gas pains or distention.

Which patient is most appropriate for the burn unit charge nurse to assign to a registered nurse (RN) who has floated from the hospital medical unit? a. A 34-year-old patient who has a weight loss of 15% from admission and requires enteral feedings. b. A 67-year-old patient who has blebs under an autograft on the thigh and has an order for bleb aspiration c. A 46-year-old patient who has just come back to the unit after having a cultured epithelial autograft to the chest d. A 65-year-old patient who has twice-daily burn debridements and dressing changes to partial-thickness facial burns

ANS: A An RN from a medical unit would be familiar with malnutrition and with administration and evaluation of response to enteral feedings. The other patients require burn assessment and care that is more appropriate for staff who regularly care for burned patients

Which patient is most appropriate for the intensive care unit (ICU) charge nurse to assign to a registered nurse (RN) who has floated from the medical unit? a. A 45-yr-old patient receiving IV antibiotics for meningococcal meningitis b. A 35-yr-old patient with intracranial pressure (ICP) monitoring after a head injury c. A 25-yr-old patient admitted with a skull fracture and craniotomy the previous day d. A 55-yr-old patient who has increased intracranial pressure (ICP) and is receiving hyperventilation therapy

ANS: A An RN who works on a medical unit will be familiar with administration of IV antibiotics and with meningitis. The patient recovering from a craniotomy, the patient with an ICP monitor, and the patient on a ventilator should be assigned to an RN familiar with the care of critically ill patients.

Which finding for a patient who has hypothyroidism and hypertension indicates that the nurse should contact the health care provider before administering levothyroxine (Synthroid)? a. Increased thyroxine (T4) level b. Blood pressure 112/62 mm Hg c. Distant and difficult to hear heart sounds d. Elevated thyroid stimulating hormone level

ANS: A An increased thyroxine level indicates the levothyroxine dose needs to be decreased. The other data are consistent with hypothyroidism and the nurse should administer the levothyroxine

48. A female patient is awaiting surgery for acute peritonitis. Which action will the nurse include in the plan of care? a. Position patient with the knees flexed. b. Avoid use of opioids or sedative drugs. c. Offer frequent small sips of clear liquids. d. Assist patient to breathe deeply and cough.

ANS: A There is less peritoneal irritation with the knees flexed, which will help decrease pain. Opioids and sedatives are typically given to control pain and anxiety. Preoperative patients with peritonitis are given IV fluids for hydration. Deep breathing and coughing will increase the patient's discomfort.

11. A patient will need vascular access for hemodialysis. Which statement by the nurse accurately describes an advantage of a fistula over a graft? a. A fistula is much less likely to clot. b. A fistula increases patient mobility. c. A fistula can accommodate larger needles. d. A fistula can be used sooner after surgery.

ANS: A Arteriovenous (AV) fistulas are much less likely to clot than grafts, although it takes longer for them to mature to the point where they can be used for dialysis. The choice of an AV fistula or a graft does not have an impact on needle size or patient mobility. DIF: Cognitive Level: Understand (comprehension) REF: 1088 TOP: Nursing Process: Implementation MSC: NCLEX: Physiological Integrity

A patient with dilated cardiomyopathy has new-onset atrial fibrillation that has been unresponsive to drug therapy for several days. Teaching for this patient would include information about a. anticoagulant therapy. c. emergency cardioversion. b. permanent pacemakers. d. IV adenosine (Adenocard).

ANS: A Atrial fibrillation therapy that has persisted for more than 48 hours requires anticoagulant treatment for 3 weeks before attempting cardioversion. This is done to prevent embolization of clots from the atria. Cardioversion may be done after several weeks of anticoagulation therapy. Adenosine is not used to treat atrial fibrillation. Pacemakers are routinely used for patients with bradydysrhythmias. Information does not indicate that the patient has a slow heart rate. DIF: Cognitive Level: Apply (application)

After the nurse gives IV atropine to a patient with symptomatic type 1, second-degree atrioventricular (AV) block, which finding indicates that the drug has been effective? a. Increase in the patient's heart rate b. Increase in strength of peripheral pulses c. Decrease in premature atrial contractions d. Decrease in premature ventricular contractions

ANS: A Atropine will increase the heart rate and conduction through the AV node. Because the drug increases electrical conduction, not cardiac contractility, the quality of the peripheral pulses is not used to evaluate the drug effectiveness. The patient does not have premature atrial or ventricular contractions. DIF: Cognitive Level: Apply (application)

The urgent care center protocol for tick bites includes the following actions. Which action will the nurse take firstwhen caring for a patient with a tick bite? a. Use tweezers to remove any remaining ticks. b. Check the vital signs, including temperature. c. Give doxycycline (Vibramycin) 100 mg orally. d. Obtain information about recent outdoor activities.

ANS: A Because neurotoxic venom is released as long as the tick is attached to the patient, the initial action should be to remove any ticks using tweezers or forceps. The other actions are also appropriate, but the priority is to minimize venom release.

8. Which nursing action will be included in the plan of care for a 27-year-old male patient with bowel irregularity and a new diagnosis of irritable bowel syndrome (IBS)? a. Encourage the patient to express concerns and ask questions about IBS. b. Suggest that the patient increase the intake of milk and other dairy products. c. Educate the patient about the use of alosetron (Lotronex) to reduce symptoms. d. Teach the patient to avoid using nonsteroidal antiinflammatory drugs (NSAIDs).

ANS: A Because psychologic and emotional factors can affect the symptoms for IBS, encouraging the patient to discuss emotions and ask questions is an important intervention. Alosetron has serious side effects, and is used only for female patients who have not responded to other therapies. Although yogurt may be beneficial, milk is avoided because lactose intolerance can contribute to symptoms in some patients. NSAIDs can be used by patients with IBS.

The clinic nurse teaches a patient with a 42 pack-year history of cigarette smoking about lung disease. Which information will be most important for the nurse to include? a. Options for smoking cessation b. Reasons for annual sputum cytology testing c. Erlotinib (Tarceva) therapy to prevent tumor risk d. Computed tomography (CT) screening for lung cancer

ANS: A Because smoking is the major cause of lung cancer, the most important role for the nurse is teaching patients about the benefits of and means of smoking cessation. CT scanning is currently being investigated as a screening test for high-risk patients. However, if there is a positive finding, the person already has lung cancer. Erlotinib may be used in patients who have lung cancer, but it is not used to reduce the risk of developing cancer

A patient who is diagnosed with cervical cancer that is classified as Tis, N0, M0 asks the nurse what the letters and numbers mean. Which response by the nurse is most appropriate? a. "The cancer involves only the cervix." b. "The cancer cells look almost like normal cells." c. "Further testing is needed to determine the spread of the cancer." d. "It is difficult to determine the original site of the cervical cancer."

ANS: A Cancer in situ indicates that the cancer is localized to the cervix and is not invasive at this time. Cell differentiation is not indicated by clinical staging. Because the cancer is in situ, the origin is the cervix. Further testing is not indicated given that the cancer has not spread

Which finding by the nurse when assessing a patient with Hashimoto's thyroiditis and a goiter will require the most immediate action? a. New-onset changes in the patient's voice b. Apical pulse rate at rest 112 beats/minute c. Elevation in the patient's T3 and T4 levels d. Bruit audible bilaterally over the thyroid gland

ANS: A Changes in the patient's voice indicate that the goiter is compressing the laryngeal nerve and may lead to airway compression. The other findings will also be reported but are expected with Hashimoto's thyroiditis and do not require immediate action

The nurse is caring for a patient with aortic stenosis. Which assessment data obtained by the nurse would be most important to report to the health care provider? a. The patient complains of chest pressure when ambulating. b. A loud systolic murmur is heard along the right sternal border. c. A thrill is palpated at the second intercostal space, right sternal border. d. The point of maximum impulse (PMI) is at the left midclavicular line.

ANS: A Chest pressure (or pain) occurring with aortic stenosis is caused by cardiac ischemia, and reporting this information would be a priority. A systolic murmur and thrill are expected in a patient with aortic stenosis. A PMI at the left midclavicular line is normal. DIF: Cognitive Level: Analyze (analysis)

Which finding indicates to the nurse that the current therapies are effective for a patient with acute adrenal insufficiency? a. Increasing serum sodium levels b. Decreasing blood glucose levels c. Decreasing serum chloride levels d. Increasing serum potassium levels

ANS: A Clinical manifestations of Addison's disease include hyponatremia and an increase in sodium level indicates improvement. The other values indicate that treatment has not been effective

34. Which information about a patient who has been receiving thrombolytic therapy for an acute myocardial infarction (AMI) is most important for the nurse to communicate to the health care provider? a. No change in the patient's chest pain b. An increase in troponin levels from baseline c. A large bruise at the patient's IV insertion site d. A decrease in ST-segment elevation on the electrocardiogram

ANS: A Continued chest pain suggests that the thrombolytic therapy is not effective and that other interventions such as percutaneous coronary intervention (PCI) may be needed. Bruising is a possible side effect of thrombolytic therapy, but it is not an indication that therapy should be discontinued. The decrease of the ST-segment elevation indicates that thrombolysis is occurring and perfusion is returning to the injured myocardium. An increase in troponin levels is expected with reperfusion and is related to the washout of cardiac markers into the circulation as the blocked vessel is opened.

The nurse administers prescribed therapies for a patient with cor pulmonale and right-sided heart failure. Which assessment would best evaluate the effectiveness of the therapies? a. Observe for distended neck veins. b. Auscultate for crackles in the lungs. c. Palpate for heaves or thrills over the heart. d. Review hemoglobin and hematocrit values.

ANS: A Cor pulmonale is right ventricular failure caused by pulmonary hypertension, so clinical manifestations of right ventricular failure such as peripheral edema, jugular venous distention, and right upper-quadrant abdominal tenderness would be expected. Crackles in the lungs are likely to be heard with left-sided heart failure. Findings in cor pulmonale include evidence of right ventricular hypertrophy on electrocardiogram ECG and an increase in intensity of the second heart sound. Heaves or thrills are not common with cor pulmonale. Chronic hypoxemia leads to polycythemia and increased total blood volume and viscosity of the blood. The hemoglobin and hematocrit values are more likely to be elevated with cor pulmonale than decreased

A patient with bacterial pneumonia has rhonchi and thick sputum. What is the nurse's most appropriate action to promote airway clearance? a. Assist the patient to splint the chest when coughing. b. Teach the patient about the need for fluid restrictions. c. Encourage the patient to wear the nasal oxygen cannula. d. Instruct the patient on the pursed lip breathing technique.

ANS: A Coughing is less painful and more likely to be effective when the patient splints the chest during coughing. Fluids should be encouraged to help liquefy secretions. Nasal oxygen will improve gas exchange, but will not improve airway clearance. Pursed lip breathing is used to improve gas exchange in patients with COPD, but will not improve airway clearance

The nurse teaches a patient about drug therapy after a kidney transplant. Which statement by the patient would indicate a need for further instructions? a. "After a couple of years, it is likely that I will be able to stop taking the cyclosporine." b. "If I develop an acute rejection episode, I will need to have other types of drugs given IV." c. "I need to be monitored closely because I have a greater chance of developing malignant tumors." d. "The drugs are given in combination because they inhibit different ways the kidney can be rejected."

ANS: A Cyclosporine, a calcineurin inhibitor, will need to be continued for life. The other patient statements are accurate and indicate that no further teaching is necessary about those topics.

25. The nurse is caring for a patient who was admitted to the coronary care unit following an acute myocardial infarction (AMI) and percutaneous coronary intervention the previous day. Teaching for this patient would include a. when cardiac rehabilitation will begin. b. the typical emotional responses to AMI. c. information regarding discharge medications. d. the pathophysiology of coronary artery disease.

ANS: A Early after an AMI, the patient will want to know when resumption of usual activities can be expected. At this time, the patient's anxiety level or denial will interfere with good understanding of complex information such as the pathophysiology of coronary artery disease (CAD). Teaching about discharge medications should be done closer to discharge. The nurse should support the patient by decreasing anxiety rather than discussing the typical emotional responses to myocardial infarction (MI).

A patient has just been admitted with a 40% total body surface area (TBSA) burn injury. To maintain adequate nutrition, the nurse should plan to take which action? a. Insert a feeding tube and initiate enteral feedings. b. Infuse total parenteral nutrition via a central catheter. c. Encourage an oral intake of at least 5000 kcal per day. d. Administer multiple vitamins and minerals in the IV solution.

ANS: A Enteral feedings can usually be initiated during the emergent phase at low rates and increased over 24 to 48 hours to the goal rate. During the emergent phase, the patient will be unable to eat enough calories to meet nutritional needs and may have a paralytic ileus that prevents adequate nutrient absorption. Vitamins and minerals may be administered during the emergent phase, but these will not assist in meeting the patient's caloric needs. Parenteral nutrition increases the infection risk, does not help preserve gastrointestinal function, and is not routinely used in burn patients

A 36-year-old male patient in the outpatient clinic is diagnosed with acute hepatitis C (HCV) infection. Which action by the nurse is appropriate? a. Schedule the patient for HCV genotype testing. b. Administer the HCV vaccine and immune globulin. c. Teach the patient about ribavirin (Rebetol) treatment. d. Explain that the infection will resolve over a few months.

ANS: A Genotyping of HCV has an important role in managing treatment and is done before drug therapy is initiated. Because most patients with acute HCV infection convert to the chronic state, the nurse should not teach the patient that the HCV will resolve in a few months. Immune globulin or vaccine is not available for HCV. Ribavirin is used for chronic HCV infection

A 56-year-old female patient has an adrenocortical adenoma, causing hyperaldosteronism. The nurse providing care should a. monitor the blood pressure every 4 hours. b. elevate the patient's legs to relieve edema. c. monitor blood glucose level every 4 hours. d. order the patient a potassium-restricted diet.

ANS: A Hypertension caused by sodium retention is a common complication of hyperaldosteronism. Hyperaldosteronism does not cause an elevation in blood glucose. The patient will be hypokalemic and require potassium supplementation before surgery. Edema does not usually occur with hyperaldosteronism

1. A patient with acute osteomyelitis of the left femur is hospitalized for regional antibiotic irrigation. Which intervention will be included in the initial plan of care? a. Immobilization of the left leg b. Positioning the left leg in flexion c. Assisted weight-bearing ambulation d. Quadriceps-setting exercise repetitions

ANS: A Immobilization of the affected leg helps decrease pain and reduce the risk for pathologic fractures. Weightbearing exercise increases the risk for pathologic fractures. Flexion of the affected limb is avoided to prevent contractures.

A 20-yr-old patient has a mandatory electrocardiogram (ECG) before participating on a college soccer team and is found to have sinus bradycardia, rate 52. Blood pressure (BP) is 114/54 mm Hg, and the student denies any health problems. What action by the nurse is most appropriate? a. Allow the student to participate on the soccer team. b. Refer the student to a cardiologist for further testing. c. Tell the student to stop playing immediately if any dyspnea occurs. d. Obtain more detailed information about the student's family health history.

ANS: A In an aerobically trained individual, sinus bradycardia is normal. The student's normal BP and negative health history indicate that there is no need for a cardiology referral or for more detailed information about the family's health history. Dyspnea during an aerobic activity such as soccer is normal. DIF: Cognitive Level: Apply (application)

The nurse assesses the chest of a patient with pneumococcal pneumonia. Which finding would the nurse expect? a. Increased tactile fremitus b. Dry, nonproductive cough c. Hyperresonance to percussion d. A grating sound on auscultation

ANS: A Increased tactile fremitus over the area of pulmonary consolidation is expected with bacterial pneumonias. Dullness to percussion would be expected. Pneumococcal pneumonia typically presents with a loose, productive cough. Adventitious breath sounds such as crackles and wheezes are typical. A grating sound is more representative of a pleural friction rub rather than pneumonia

26. A patient who has recently started taking pravastatin (Pravachol) and niacin (Nicobid) reports the following symptoms to the nurse. Which is most important to communicate to the health care provider? a. Generalized muscle aches and pains b. Dizziness when changing positions quickly c. Nausea when taking the drugs before eating d. Flushing and pruritus after taking the medications

ANS: A Muscle aches and pains may indicate myopathy and rhabdomyolysis, which have caused acute kidney injury and death in some patients who have taken the statin medications. These symptoms indicate that the pravastatin may need to be discontinued. The other symptoms are common side effects when taking niacin, and although the nurse should follow-up with the health care provider, they do not indicate that a change in medication is needed.

19. Which action will the nurse take when caring for a patient with osteomalacia? a. Teach about the use of vitamin D supplements. b. Educate about the need for weight-bearing exercise. c. Discuss the use of medications such as bisphosphonates. d. Emphasize the importance of sunscreen use when outside.

ANS: A Osteomalacia is caused by inadequate intake or absorption of vitamin D. Weight-bearing exercise and bisphosphonate administration may be used for osteoporosis but will not be beneficial for osteomalacia. Because ultraviolet light is needed for the body to synthesize vitamin D, the patient might be taught that 20 minutes/day of sun exposure is beneficial.

10. An assessment finding for a 55-year-old patient that alerts the nurse to the presence of osteoporosis is a. a measurable loss of height. b. the presence of bowed legs. c. the aversion to dairy products. d. a statement about frequent falls.

ANS: A Osteoporosis occurring in the vertebrae produces a gradual loss of height. Bowed legs are associated with osteomalacia. Low intake of dairy products is a risk factor for osteoporosis, but it does not indicate that osteoporosis is present. Frequent falls increase the risk for fractures but are not an indicator of osteoporosis.

The nurse cares for a patient who has just had a thoracentesis. Which assessment information obtained by the nurse is a priority to communicate to the health care provider? a. Oxygen saturation is 88%. b. Blood pressure is 145/90 mm Hg. c. Respiratory rate is 22 breaths/minute when lying flat. d. Pain level is 5 (on 0 to 10 scale) with a deep breath.

ANS: A Oxygen saturation would be expected to improve after a thoracentesis. A saturation of 88% indicates that a complication such as pneumothorax may be occurring. The other assessment data also indicate a need for ongoing assessment or intervention, but the low oxygen saturation is the priority

A patient experiences a chest wall contusion as a result of being struck in the chest with a baseball bat. The emergency department nurse would be most concerned if which finding is observed during the initial assessment? a. Paradoxic chest movement b. Complaint of chest wall pain c. Heart rate of 110 beats/minute d. Large bruised area on the chest

ANS: A Paradoxic chest movement indicates that the patient may have flail chest, which can severely compromise gas exchange and can rapidly lead to hypoxemia. Chest wall pain, a slightly elevated pulse rate, and chest bruising all require further assessment or intervention, but the priority concern is poor gas exchange

The nurse determines that additional instruction is needed for a 60-year-old patient with chronic syndrome of inappropriate antidiuretic hormone (SIADH) when the patient says which of the following? a. "I need to shop for foods low in sodium and avoid adding salt to food." b. "I should weigh myself daily and report any sudden weight loss or gain." c. "I need to limit my fluid intake to no more than 1 quart of liquids a day." d. "I will eat foods high in potassium because diuretics cause potassium loss."

ANS: A Patients with SIADH are at risk for hyponatremia, and a sodium supplement may be prescribed. The other patient statements are correct and indicate successful teaching has occurred

A patient who has bacterial meningitis is disoriented and anxious. Which nursing action will be included in the plan of care? a. Encourage family members to remain at the bedside. b. Apply soft restraints to protect the patient from injury. c. Keep the room well-lighted to improve patient orientation. d. Minimize contact with the patient to decrease sensory input.

ANS: A Patients with meningitis and disorientation will be calmed by the presence of someone familiar at the bedside. Restraints should be avoided because they increase agitation and anxiety. The patient requires frequent assessment for complications. The use of touch and a soothing voice will decrease anxiety for most patients. The patient will have photophobia, so the light should be dim.

The nurse is planning care for a 48-year-old woman with acute severe pancreatitis. The highest priority patient outcome is a. maintaining normal respiratory function. b. expressing satisfaction with pain control. c. developing no ongoing pancreatic disease. d. having adequate fluid and electrolyte balance.

ANS: A Respiratory failure can occur as a complication of acute pancreatitis, and maintenance of adequate respiratory function is the priority goal. The other outcomes would also be appropriate for the patient

While caring for a patient with aortic stenosis, the nurse identifies a nursing diagnosis of acute pain related to decreased coronary blood flow. An appropriate nursing intervention for this patient would be to a. promote rest to decrease myocardial oxygen demand. b. teach the patient about the need for anticoagulant therapy. c. teach the patient to use sublingual nitroglycerin for chest pain. d. raise the head of the bed 60 degrees to decrease venous return.

ANS: A Rest is recommended to balance myocardial oxygen supply and demand and to decrease chest pain. The patient with aortic stenosis requires higher preload to maintain cardiac output, so nitroglycerin and measures to decrease venous return are contraindicated. Anticoagulation is not recommended unless the patient has atrial fibrillation. DIF: Cognitive Level: Apply (application)

The nurse assesses a patient with non-Hodgkin's lymphoma who is receiving an infusion of rituximab (Rituxan). Which assessment finding would require the most rapid action by the nurse? a. Shortness of breath b. Temperature 100.2° F (37.9° C) c. Shivering and complaint of chills d. Generalized muscle aches and pains

ANS: A Rituximab (Rituxan) is a monoclonal antibody. Shortness of breath should be investigated rapidly because anaphylaxis is a possible reaction to monoclonal antibody administration. The nurse will need to rapidly take actions such as stopping the infusion, assessing the patient further, and notifying the health care provider. The other findings will also require action by the nurse, but are not indicative of life-threatening complications

A patient with circumferential burns of both legs develops a decrease in dorsalis pedis pulse strength and numbness in the toes. Which action should the nurse take? a. Notify the health care provider. b. Monitor the pulses every 2 hours. c. Elevate both legs above heart level with pillows. d. Encourage the patient to flex and extend the toes on both feet.

ANS: A The decrease in pulse in a patient with circumferential burns indicates decreased circulation to the legs and the need for an escharotomy. Monitoring the pulses is not an adequate response to the decrease in circulation. Elevating the legs or increasing toe movement will not improve the patient's circulation

7. Sodium polystyrene sulfonate (Kayexalate) is ordered for a patient with hyperkalemia. Before administering the medication, the nurse should assess the a. bowel sounds. b. blood glucose. c. blood urea nitrogen (BUN). d. level of consciousness (LOC).

ANS: A Sodium polystyrene sulfonate (Kayexalate) should not be given to a patient with a paralytic ileus (as indicated by absent bowel sounds) because bowel necrosis can occur. The BUN and creatinine, blood glucose, and LOC would not affect the nurse's decision to give the medication. DIF: Cognitive Level: Apply (application) REF: 1080 TOP: Nursing Process: Assessment MSC: NCLEX: Physiological Integrity

The charge nurse observes the following actions being taken by a new nurse on the burn unit. Which action by the new nurse would require an intervention by the charge nurse? a. The new nurse uses clean latex gloves when applying antibacterial cream to a burn wound. b. The new nurse obtains burn cultures when the patient has a temperature of 95.2° F (35.1° C). c. The new nurse administers PRN fentanyl (Sublimaze) IV to a patient 5 minutes before a dressing change. d. The new nurse calls the health care provider for a possible insulin order when a nondiabetic patient's serum glucose is elevated.

ANS: A Sterile gloves should be worn when applying medications or dressings to a burn. Hypothermia is an indicator of possible sepsis, and cultures are appropriate. Nondiabetic patients may require insulin because stress and high calorie intake may lead to temporary hyperglycemia. Fentanyl peaks 5 minutes after IV administration, and should be used just before and during dressing changes for pain management

Admission vital signs for a brain-injured patient are blood pressure of 128/68 mm Hg, pulse of 110 beats/min, and of respirations 26 breaths/min. Which set of vital signs, if taken 1 hour later, will be of most concern to the nurse? a. Blood pressure of 154/68 mm Hg, pulse of 56 beats/min, respirations of 12 breaths/min b. Blood pressure of 134/72 mm Hg, pulse of 90 beats/min, respirations of 32 breaths/min c. Blood pressure of 148/78 mm Hg, pulse of 112 beats/min, respirations of 28 breaths/min d. Blood pressure of 110/70 mm Hg, pulse of 120 beats/min, respirations of 30 breaths/min

ANS: A Systolic hypertension with widening pulse pressure, bradycardia, and respiratory changes represent Cushing's triad. These findings indicate that the intracranial pressure (ICP) has increased, and brain herniation may be imminent unless immediate action is taken to reduce ICP. The other vital signs may indicate the need for changes in treatment, but they are not indicative of an immediately life-threatening process.

The nurse is caring for a patient who has a head injury. Which finding, when reported to the health care provider, should the nurse expect will result in new prescribed interventions? a. Pale yellow urine output of 1200 mL over the past 2 hours. b. Ventriculostomy drained 40 mL of fluid in the past 2 hours. c. Intracranial pressure spikes to 16 mm Hg when patient is turned. d. LICOX brain tissue oxygenation catheter shows PbtO2 of 38 mm Hg.

ANS: A The high urine output indicates that diabetes insipidus may be developing, and interventions to prevent dehydration need to be rapidly implemented. The other data do not indicate a need for any change in therapy.

A patient who has received allergen testing using the cutaneous scratch method has developed itching and swelling at the skin site. Which action should the nurse take first? a. Administer epinephrine. b. Apply topical hydrocortisone. c. Monitor the patient for lower extremity edema. d. Ask the patient about exposure to any new lotions or soaps.

ANS: A The initial symptoms of anaphylaxis are itching and edema at the site of the exposure. Hypotension, tachycardia, dilated pupils, and wheezes occur later. Rapid administration of epinephrine when excessive itching or swelling at the skin site is observed can prevent the progression to anaphylaxis. Topical hydrocortisone would not deter an anaphylactic reaction. Exposure to lotions and soaps does not address the immediate concern of a possible anaphylactic reaction. The nurse should not wait and observe for edema. The nurse should act immediately in order to prevent progression to anaphylaxis.

19. Three days after experiencing a myocardial infarction (MI), a patient who is scheduled for discharge asks for assistance with hygiene activities, saying, "I am too nervous to take care of myself." Based on this information, which nursing diagnosis is appropriate? a. Ineffective coping related to anxiety b. Activity intolerance related to weakness c. Denial related to lack of acceptance of the MI d. Disturbed personal identity related to understanding of illness

ANS: A The patient data indicate that ineffective coping after the MI caused by anxiety about the impact of the MI is a concern. The other nursing diagnoses may be appropriate for some patients after an MI, but the data for this patient do not support denial, activity intolerance, or disturbed personal identity.

The nurse is caring for a patient with colon cancer who is scheduled for external radiation therapy to the abdomen. Which information obtained by the nurse would indicate a need for patient teaching? a. The patient swims a mile 3 days a week. b. The patient snacks frequently during the day. c. The patient showers everyday with a mild soap. d. The patient has a history of dental caries with amalgam fillings.

ANS: A The patient is instructed to avoid swimming in salt water or chlorinated pools during the treatment period. The patient does not need to change habits of eating frequently or showering with a mild soap. A history of dental caries will not impact the patient who is scheduled for abdominal radiation.

The following interventions are part of the emergency department (ED) protocol for a patient who has been admitted with multiple bee stings to the hands. Which action should the nurse take first? a. Remove the patient's rings. b. Apply ice packs to both hands. c. Apply calamine lotion to itching areas. d. Give diphenhydramine (Benadryl) 50 mg PO.

ANS: A The patient's rings should be removed first because it might not be possible to remove them if swelling develops. The other orders should also be implemented as rapidly as possible after the nurse has removed the jewelry.

A patient with possible viral meningitis is admitted to the nursing unit after lumbar puncture was performed in the emergency department. Which action prescribed by the health care provider should the nurse question? a. Restrict oral fluids to 1000 mL/day. b. Elevate the head of the bed 20 degrees. c. Administer ceftriaxone (Rocephin) 1 g IV every 12 hours. d. Give ibuprofen (Motrin) 400 mg every 6 hours as needed for headache.

ANS: A The patient with meningitis has increased fluid needs, so oral fluids should be encouraged. The other actions are appropriate. Slight elevation of the head of the bed will decrease headache without causing leakage of cerebrospinal fluid from the lumbar puncture site. Antibiotics should be administered until bacterial meningitis is ruled out by the cerebrospinal fluid analysis.

Two days after an acute myocardial infarction (MI), a patient complains of stabbing chest pain that increases with a deep breath. Which action will the nurse take first? a. Auscultate the heart sounds. b. Check the patient's temperature. c. Give the PRN acetaminophen (Tylenol). d. Notify the patient's health care provider.

ANS: A The patient's clinical manifestations and history are consistent with pericarditis, and the first action by the nurse should be to listen for a pericardial friction rub. Checking the temperature and notifying the health care provider are also appropriate actions but would not be done before listening for a rub. Acetaminophen (Tylenol) is not very effective for pericarditis pain, and an analgesic would not be given before assessment of a new symptom. DIF: Cognitive Level: Analyze (analysis)

The nurse is caring for a patient admitted with diabetes insipidus (DI). Which information is most important to report to the health care provider? a. The patient is confused and lethargic. b. The patient reports a recent head injury. c. The patient has a urine output of 400 mL/hr. d. The patient's urine specific gravity is 1.003.

ANS: A The patient's confusion and lethargy may indicate hypernatremia and should be addressed quickly. In addition, patients with DI compensate for fluid losses by drinking copious amounts of fluids, but a patient who is lethargic will be unable to drink enough fluids and will become hypovolemic. A high urine output, low urine specific gravity, and history of a recent head injury are consistent with diabetes insipidus, but they do not require immediate nursing action to avoid life-threatening complications

24. A 72-yr-old patient with a history of benign prostatic hyperplasia (BPH) is admitted with acute urinary retention and elevated blood urea nitrogen (BUN) and creatinine levels. Which prescribed therapy should the nurse implement first? a. Insert urethral catheter. b. Obtain renal ultrasound. c. Draw a complete blood count. d. Infuse normal saline at 50 mL/hour.

ANS: A The patient's elevation in BUN is most likely associated with hydronephrosis caused by the acute urinary retention, so the insertion of a retention catheter is the first action to prevent ongoing postrenal failure for this patient. The other actions also are appropriate but should be implemented after the retention catheter. DIF: Cognitive Level: Analyze (analysis) REF: 1071 OBJ: Special Questions: Prioritization TOP: Nursing Process: Implementation MSC: NCLEX: Physiological Integrity

Ten days after receiving a bone marrow transplant, a patient develops a skin rash. What would the nurse suspect is the cause of this patient's skin rash? a. The donor T cells are attacking the patient's skin cells. b. The patient's antibodies are rejecting the donor bone marrow. c. The patient is experiencing a delayed hypersensitivity reaction. d. The patient will need treatment to prevent hyperacute rejection.

ANS: A The patient's history and symptoms indicate that the patient is experiencing graft-versus-host disease, in which the donated T cells attack the patient's tissues. The history and symptoms are not consistent with rejection or delayed hypersensitivity

A patient with rheumatic fever has subcutaneous nodules, erythema marginatum, and polyarthritis. The patient reports that discomfort in the joints prevents favorite activities such as taking a daily walk and working on sewing projects. Based on these findings, which nursing diagnosis statement would be appropriate? a. Activity intolerance related to arthralgia b. Anxiety related to permanent joint fixation c. Altered body image related to polyarthritis d. Social isolation related to pain and swelling

ANS: A The patient's joint pain will lead to difficulty with activity. Although acute joint pain will be a problem for this patient, joint inflammation is a temporary clinical manifestation of rheumatic fever and is not associated with permanent joint changes. This patient did not provide any data to support a diagnosis of social isolation, anxiety, or altered body image. DIF: Cognitive Level: Apply (application)

A triage nurse in a busy emergency department (ED) assesses a patient who complains of 7/10 abdominal pain and states, "I had a temperature of 103.9° F (39.9° C) at home." The nurse's first action should be to a. assess the patient's current vital signs. b. give acetaminophen (Tylenol) per agency protocol. c. ask the patient to provide a clean-catch urine for urinalysis. d. tell the patient that it will be 1 to 2 hours before seeing a health care provider.

ANS: A The patient's pain and statement about an elevated temperature indicate that the nurse should obtain vital signs before deciding how rapidly the patient should be seen by the health care provider. A urinalysis may be appropriate, but this would be done after the vital signs are taken. The nurse will not give acetaminophen before confirming a current temperature elevation.

The nurse administering a-interferon and ribavirin (Rebetol) to a patient with chronic hepatitis C will plan to monitor for a. leukopenia. b. hypokalemia. c. polycythemia. d. hypoglycemia.

ANS: A Therapy with ribavirin and a-interferon may cause leukopenia. The other problems are not associated with this drug therapy

A patient who was admitted the previous day with pneumonia complains of a sharp pain of 7 (based on 0 to 10 scale) "whenever I take a deep breath." Which action will the nurse take next? a. Auscultate breath sounds. b. Administer the PRN morphine. c. Have the patient cough forcefully. d. Notify the patient's health care provider.

ANS: A The patient's statement indicates that pleurisy or a pleural effusion may have developed and the nurse will need to listen for a pleural friction rub and/or decreased breath sounds. Assessment should occur before administration of pain medications. The patient is unlikely to be able to cough forcefully until pain medication has been administered. The nurse will want to obtain more assessment data before calling the health care provider

The nurse caring for a patient admitted with burns over 30% of the body surface assesses that urine output has dramatically increased. Which action by the nurse would best ensure adequate kidney function? a. Continue to monitor the urine output. b. Monitor for increased white blood cells (WBCs). c. Assess that blisters and edema have subsided. d. Prepare the patient for discharge from the burn unit.

ANS: A The patient's urine output indicates that the patient is entering the acute phase of the burn injury and moving on from the emergent stage. At the end of the emergent phase, capillary permeability normalizes and the patient begins to diurese large amounts of urine with a low specific gravity. Although this may occur at about 48 hours, it may be longer in some patients. Blisters and edema begin to resolve, but this process requires more time. White blood cells may increase or decrease, based on the patient's immune status and any infectious processes. The WBC count does not indicate kidney function. The patient will likely remain in the burn unit during the acute stage of burn injury

12. When caring for a patient with a left arm arteriovenous fistula, which action will the nurse include in the plan of care to maintain the patency of the fistula? a. Auscultate for a bruit at the fistula site. b. Assess the quality of the left radial pulse. c. Compare blood pressures in the left and right arms. d. Irrigate the fistula site with saline every 8 to 12 hours.

ANS: A The presence of a thrill and bruit indicates adequate blood flow through the fistula. Pulse rate and quality are not good indicators of fistula patency. Blood pressures should never be obtained on the arm with a fistula. Irrigation of the fistula might damage the fistula, and typically only dialysis staff would access the fistula. DIF: Cognitive Level: Understand (comprehension) REF: 1087 TOP: Nursing Process: Planning MSC: NCLEX: Physiological Integrity

An older adult patient who is having an annual check-up tells the nurse, "I feel fine, and I don't want to pay for all these unnecessary cancer screening tests!" Which information should the nurse plan to teach this patient? a. Consequences of aging on cell-mediated immunity b. Decrease in antibody production associated with aging c. Impact of poor nutrition on immune function in older people d. Incidence of cancer-stimulating infections in older individuals

ANS: A The primary impact of aging on immune function is on T cells, which are important for immune surveillance and tumor immunity. Antibody function is not affected as much by aging. Poor nutrition can also contribute to decreased immunity, but there is no evidence that it is a contributing factor for this patient. Although some types of cancer are associated with specific infections, this patient does not have an active infection.

An unresponsive patient is admitted to the emergency department (ED) after falling through the ice while ice skating. Which assessment will the nurse obtain first? a. Pulse c. Breath sounds b. Heart rhythm d. Body temperature

ANS: A The priority assessment in an unresponsive patient relates to CAB (circulation, airway, breathing) so a pulse check should be performed first. While assessing the pulse, the nurse should look for signs of breathing. The other data will also be collected rapidly but are not as essential as determining if there is a pulse.

An unresponsive 79-yr-old patient is admitted to the emergency department (ED) during a summer heat wave. The patient's core temperature is 105.4° F (40.8° C), blood pressure (BP) is 88/50 mm Hg, and pulse is 112 beats/min. The nurse will plan to a. apply wet sheets and a fan to the patient. b. provide O2 at 2 L/min with a nasal cannula. c. start lactated Ringer's solution at 1000 mL/hr. d. give acetaminophen (Tylenol) rectal suppository.

ANS: A The priority intervention is to cool the patient. Antipyretics are not effective in decreasing temperature in heat stroke and 100% O2 should be given, which requires a high flow rate through a non-rebreather mask. An older patient would be at risk for developing complications such as pulmonary edema if given fluids at 1000 mL/hr.

Which finding indicates to the nurse that lactulose (Cephulac) is effective for a 72-year-old man who has advanced cirrhosis? a. The patient is alert and oriented. b. The patient denies nausea or anorexia. c. The patient's bilirubin level decreases. d. The patient has at least one stool daily.

ANS: A The purpose of lactulose in the patient with cirrhosis is to lower ammonia levels and prevent encephalopathy. Although lactulose may be used to treat constipation, that is not the purpose for this patient. Lactulose will not decrease nausea and vomiting or lower bilirubin levels.

Following an earthquake, patients are triaged by emergency medical personnel and transported to the emergency department (ED). Which patient will the nurse need to assess first? a. A patient with a red tag c. A patient with a black tag b. A patient with a blue tag d. A patient with a yellow tag

ANS: A The red tag indicates a patient with a life-threatening injury requiring rapid treatment. The other tags indicate patients with less urgent injuries or those who are likely to die.

While admitting a 42-yr-old patient with a possible brain injury after a car accident to the emergency department (ED), the nurse obtains the following information. Which finding is most important to report to the health care provider? a. The patient takes warfarin (Coumadin) daily. b. The patient's blood pressure is 162/94 mm Hg. c. The patient is unable to remember the accident. d. The patient complains of a severe dull headache.

ANS: A The use of anticoagulants increases the risk for intracranial hemorrhage and should be immediately reported. The other information would not be unusual in a patient with a head injury who had just arrived in the ED.

Following assessment of a patient with pneumonia, the nurse identifies a nursing diagnosis of ineffective airway clearance. Which assessment data best supports this diagnosis? a. Weak, nonproductive cough effort b. Large amounts of greenish sputum c. Respiratory rate of 28 breaths/minute d. Resting pulse oximetry (SpO2) of 85%

ANS: A The weak, nonproductive cough indicates that the patient is unable to clear the airway effectively. The other data would be used to support diagnoses such as impaired gas exchange and ineffective breathing pattern

Which patient should the nurse assess first? a. A patient with smoke inhalation who has wheezes and altered mental status b. A patient with full-thickness leg burns who has a dressing change scheduled c. A patient with abdominal burns who is complaining of level 8 (0 to 10 scale) pain d. A patient with 40% total body surface area (TBSA) burns who is receiving IV fluids at 500 mL/hour

ANS: A This patient has evidence of lower airway injury and hypoxemia and should be assessed immediately to determine the need for oxygen or intubation. The other patients should also be assessed as rapidly as possible, but they do not have evidence of life-threatening complications

A widowed mother of four school-age children is hospitalized with metastatic ovarian cancer. The patient is crying and tells the nurse that she does not know what will happen to her children when she dies. Which response by the nurse is most appropriate? a. "Why don't we talk about the options you have for the care of your children?" b. "I'm sure you have friends that will take the children when you can't care for them." c. "For now you need to concentrate on getting well and not worrying about your children." d. "Many patients with cancer live for a long time, so there is still time to plan for your children."

ANS: A This response expresses the nurse's willingness to listen and recognizes the patient's concern. The responses beginning "Many patients with cancer live for a long time" and "For now you need to concentrate on getting well" close off discussion of the topic and indicate that the nurse is uncomfortable with the topic. In addition, the patient with metastatic ovarian cancer may not have a long time to plan. Although it is possible that the patient's friends will take the children, more assessment information is needed before making plans

When planning care for a patient hospitalized with a streptococcal infective endocarditis (IE), which intervention is most appropriate for the nurse to include? a. Arrange for placement of a long-term IV catheter. b. Monitor labs for levels of streptococcal antibodies. c. Teach the importance of completing all oral antibiotics. d. Encourage the patient to begin regular aerobic exercise.

ANS: A Treatment for IE involves 4 to 6 weeks of IV antibiotic therapy to eradicate the bacteria, which will require a long-term IV catheter such as a peripherally inserted central catheter (PICC) line. Rest periods and limiting physical activity to a moderate level are recommended during the treatment for IE. Oral antibiotics are not effective in eradicating the infective bacteria that cause IE. Blood cultures, rather than antibody levels, are used to monitor the effectiveness of antibiotic therapy. DIF: Cognitive Level: Apply (application)

The nurse is caring for a patient who has a right-sided chest tube after a right lower lobectomy. Which nursing action can the nurse delegate to the unlicensed assistive personnel (UAP)? a. Document the amount of drainage every eight hours. b. Obtain samples of drainage for culture from the system. c. Assess patient pain level associated with the chest tube. d. Check the water-seal chamber for the correct fluid level.

ANS: A UAP education includes documentation of intake and output. The other actions are within the scope of practice and education of licensed nursing personnel

When caring for a patient who is pancytopenic, which action by unlicensed assistive personnel (UAP) indicates a need for the nurse to intervene? a. The UAP assists the patient to use dental floss after eating. b. The UAP adds baking soda to the patient's saline oral rinses. c. The UAP puts fluoride toothpaste on the patient's toothbrush. d. The UAP has the patient rinse after meals with a saline solution.

ANS: A Use of dental floss is avoided in patients with pancytopenia because of the risk for infection and bleeding. The other actions are appropriate for oral care of a pancytopenic patient

21. A few days after experiencing a myocardial infarction (MI) and successful percutaneous coronary intervention, the patient states, "I just had a little chest pain. As soon as I get out of here, I'm going for my vacation as planned." Which reply would be most appropriate for the nurse to make? a. "What do you think caused your chest pain?" b. "Where are you planning to go for your vacation?" c. "Sometimes plans need to change after a heart attack." d. "Recovery from a heart attack takes at least a few weeks."

ANS: A When the patient is experiencing denial, the nurse should assist the patient in testing reality until the patient has progressed beyond this step of the emotional adjustment to MI. Asking the patient about vacation plans reinforces the patient's plan, which is not appropriate in the immediate post-MI period. Reminding the patient in denial about the MI is likely to make the patient angry and lead to distrust of the nursing staff.

After the return of spontaneous circulation following the resuscitation of a patient who had a cardiac arrest, therapeutic hypothermia is ordered. Which action will the nurse include in the plan of care? a. Initiate cooling per protocol. b. Avoid the use of sedative drugs. c. Check mental status every 15 minutes. d. Rewarm if temperature is below 91° F (32.8° C).

ANS: A When therapeutic hypothermia is used postresuscitation, external cooling devices or cold normal saline infusions are used to rapidly lower body temperature to 89.6° F to 93.2° F (32° C to 34° C). Because hypothermia will decrease brain activity, assessing mental status every 15 minutes is not done at this stage. Sedative drugs are given during therapeutic hypothermia.

An employee spills industrial acids on both arms and legs at work. What is the priority action that the occupational health nurse at the facility should take? a. Remove nonadherent clothing and watch. b. Apply an alkaline solution to the affected area. c. Place cool compresses on the area of exposure. d. Cover the affected area with dry, sterile dressings.

ANS: A With chemical burns, the initial action is to remove the chemical from contact with the skin as quickly as possible. Remove nonadherent clothing, shoes, watches, jewelry, glasses, or contact lenses (if face was exposed). Flush chemical from wound and surrounding area with copious amounts of saline solution or water. Covering the affected area or placing cool compresses on the area will leave the chemical in contact with the skin. Application of an alkaline solution is not recommended

A patient who was admitted with myxedema coma and diagnosed with hypothyroidism is improving and expected to be discharged in 2 days. Which teaching strategy will be best for the nurse to use? a. Provide written reminders of self-care information. b. Offer multiple options for management of therapies. c. Ensure privacy for teaching by asking visitors to leave. d. Delay teaching until patient discharge date is confirmed.

ANS: A Written instructions will be helpful to the patient because initially the hypothyroid patient may be unable to remember to take medications and other aspects of self-care. Because the treatment regimen is somewhat complex, teaching should be initiated well before discharge. Family members or friends should be included in teaching because the hypothyroid patient is likely to forget some aspects of the treatment plan. A simpler regimen will be easier to understand until the patient is euthyroid

Which prescribed medication should the nurse administer first to a 60-year-old patient admitted to the emergency department in thyroid storm? a. Propranolol (Inderal) b. Propylthiouracil (PTU) c. Methimazole (Tapazole) d. Iodine (Lugol's solution)

ANS: A b-Adrenergic blockers work rapidly to decrease the cardiovascular manifestations of thyroid storm. The other medications take days to weeks to have an impact on thyroid function

Which nursing action is a priority for a patient who has suffered a burn injury while working on an electrical power line? a. Obtain the blood pressure. b. Stabilize the cervical spine. c. Assess for the contact points. d. Check alertness and orientation.

ANS: B Cervical spine injuries are commonly associated with electrical burns. Therefore stabilization of the cervical spine takes precedence after airway management. The other actions are also included in the emergent care after electrical burns, but the most important action is to avoid spinal cord injury

A patient who is receiving positive pressure ventilation is scheduled for a spontaneous breathing trial (SBT). Which finding by the nurse is most likely to result in postponing the SBT? a. New ST segment elevation is noted on the cardiac monitor. b. Enteral feedings are being given through an orogastric tube. c. Scattered rhonchi are heard when auscultating breath sounds. d. hydromorphone (Dilaudid) is being used to treat postoperative pain.

ANS: A Myocardial ischemia is a contraindication for ventilator weaning. The ST segment elevation is an indication that weaning should be postponed until further investigation and/or treatment for myocardial ischemia can be done. Ventilator weaning can proceed when opioids are used for pain management, abnormal lung sounds are present, or enteral feedings are being used

Which assessment finding obtained by the nurse when caring for a patient with a right radial arterial line indicates a need for the nurse to take action? a. The right hand feels cooler than the left hand. b. The mean arterial pressure (MAP) is 77 mm Hg. c. The system is delivering 3 mL of flush solution per hour. d. The flush bag and tubing were last changed 2 days previously.

ANS: A The change in temperature of the right hand suggests that blood flow to the right hand is impaired. The flush system needs to be changed every 96 hours. A mean arterial pressure (MAP) of 75 mm Hg is normal. Flush systems for hemodynamic monitoring are set up to deliver 3 to 6 mL/hr of flush solution

A patient with respiratory failure has arterial pressure-based cardiac output (APCO) monitoring and is receiving mechanical ventilation with peak end-expiratory pressure (PEEP) of 12 cm H2O. Which information indicates that a change in the ventilator settings may be required? a. The arterial pressure is 90/46. b. The stroke volume is increased. c. The heart rate is 58 beats/minute. d. The stroke volume variation is 12%.

ANS: A The hypotension suggests that the high intrathoracic pressure caused by the PEEP may be decreasing venous return and (potentially) cardiac output. The other assessment data would not be a direct result of PEEP and mechanical ventilation

After an unimmunized individual is exposed to hepatitis B through a needle-stick injury, which actions will the nurse plan to take (select all that apply)? a. Administer hepatitis B vaccine. b. Test for antibodies to hepatitis B. c. Teach about a-interferon therapy. d. Give hepatitis B immune globulin. e. Teach about choices for oral antiviral therapy.

ANS: A, B, D The recommendations for hepatitis B exposure include both vaccination and immune globulin administration. In addition, baseline testing for hepatitis B antibodies will be needed. Interferon and oral antivirals are not used for hepatitis B prophylaxis

The nurse is caring for a patient who has an intraaortic balloon pump (IABP) after a massive heart attack. When assessing the patient, the nurse notices blood backing up into the IABP catheter. In which order should the nurse take the following actions? (Put a comma and a space between each answer choice [A, B, C, D].) a. Confirm that the IABP console has turned off. b. Assess the patient's vital signs and orientation. c. Obtain supplies for insertion of a new IABP catheter. d. Notify the health care provider of the IABP malfunction.

ANS: A, B, D, C Blood in the IABP catheter indicates a possible tear in the balloon. The console should shut off automatically to prevent complications such as air embolism. Next, the nurse will assess the patient and communicate with the health care provider about the patient's assessment and the IABP problem. Finally, supplies for insertion of a new IABP catheter may be needed based on the patient assessment and the decision of the health care provider.

56. Which information will the nurse include when teaching a patient how to avoid chronic constipation (select all that apply)? a. Many over-the-counter (OTC) medications can cause constipation. b. Stimulant and saline laxatives can be used regularly. c. Bulk-forming laxatives are an excellent source of fiber. d. Walking or cycling frequently will help bowel motility. e. A good time for a bowel movement may be after breakfast.

ANS: A, C, D, E Stimulant and saline laxatives should be used infrequently. Use of bulk-forming laxatives, regular early morning timing of defecation, regular exercise, and avoiding many OTC medications will help the patient avoid constipation.

1. Which information will be included when the nurse is teaching self-management to a patient who is receiving peritoneal dialysis (select all that apply)? a. Avoid commercial salt substitutes. b. Restrict fluid intake to 1000 mL daily. c. Take phosphate binders with each meal. d. Choose high-protein foods for most meals. e. Have several servings of dairy products daily.

ANS: A, C, D Patients who are receiving peritoneal dialysis should have a high-protein diet. Phosphate binders are taken with meals to help control serum phosphate and calcium levels. Commercial salt substitutes are high in potassium and should be avoided. Fluid intake is not limited unless weight and blood pressure are not controlled. Dairy products are high in phosphate and usually are limited. DIF: Cognitive Level: Apply (application) REF: 1087 TOP: Nursing Process: Planning MSC: NCLEX: Physiological Integrity

The nurse at the clinic is interviewing a 64-year-old woman who is 5 feet, 3 inches tall and weighs 125 pounds (57 kg). The patient has not seen a health care provider for 20 years. She walks 5 miles most days and has a glass of wine 2 or 3 times a week. Which topics will the nurse plan to include in patient teaching about cancer screening and decreasing cancer risk (select all that apply)? a. Pap testing b. Tobacco use c. Sunscreen use d. Mammography e. Colorectal screening

ANS: A, C, D, E The patient's age, gender, and history indicate a need for screening and/or teaching about colorectal cancer, mammography, Pap smears, and sunscreen. The patient does not use excessive alcohol or tobacco, she is physically active, and her body weight is healthy

A patient develops neutropenia after receiving chemotherapy. Which information about ways to prevent infection will the nurse include in the teaching plan (select all that apply)? a. Cook food thoroughly before eating. b. Choose low fiber, low residue foods. c. Avoid public transportation such as buses. d. Use rectal suppositories if needed for constipation. e. Talk to the oncologist before having any dental work done.

ANS: A, C, E Eating only cooked food and avoiding public transportation will decrease infection risk. A high-fiber diet is recommended for neutropenic patients to decrease constipation. Because bacteria may enter the circulation during dental work or oral surgery, the patient may need to postpone dental work or take antibiotics

1. Which actions will the nurse include in the plan of care when caring for a patient with metastatic bone cancer of the left femur (select all that apply)? a. Monitor serum calcium level. b. Teach about the need for strict bed rest. c. Avoid use of sustained-release opioids for pain. d. Support the left leg when repositioning the patient. e. Support family as they discuss the prognosis of patient

ANS: A, D, E The nurse will monitor for hypercalcemia caused by bone decalcification. Support of the leg helps reduce the risk for pathologic fractures. Although the patient may be reluctant to exercise, activity is important to maintain function and avoid the complications associated with immobility. Adequate pain medication, including sustained-release and rapidly acting opioids, is needed for the severe pain that is frequently associated with bone cancer. The prognosis for metastatic bone cancer is poor so the patient and family need to be supported as they deal with the reality of the situation.

When caring for a patient who is hospitalized with active tuberculosis (TB), the nurse observes a student nurse who is assigned to take care of a patient. Which action, if performed by the student nurse, would require an intervention by the nurse? a. The patient is offered a tissue from the box at the bedside. b. A surgical face mask is applied before visiting the patient. c. A snack is brought to the patient from the unit refrigerator. d. Hand washing is performed before entering the patient's room.

ANS: B A high-efficiency particulate-absorbing (HEPA) mask, rather than a standard surgical mask, should be used when entering the patient's room because the HEPA mask can filter out 100% of small airborne particles. Hand washing before entering the patient's room is appropriate. Because anorexia and weight loss are frequent problems in patients with TB, bringing food to the patient is appropriate. The student nurse should perform hand washing after handling a tissue that the patient has used, but no precautions are necessary when giving the patient an unused tissue

46. A 33-year-old male patient with a gunshot wound to the abdomen undergoes surgery, and a colostomy is formed as shown in the accompanying figure. Which information will be included in patient teaching? a. Stool will be expelled from both stomas. b. This type of colostomy is usually temporary. c. Soft, formed stool can be expected as drainage. d. Irrigations can regulate drainage from the stomas.

ANS: B A loop, or double-barrel stoma, is usually temporary. Stool will be expelled from the proximal stoma only. The stool from the transverse colon will be liquid and regulation through irrigations will not be possible.

Which action will the emergency department nurse anticipate for a patient diagnosed with a concussion who did not lose consciousness? a. Coordinate the transfer of the patient to the operating room. b. Provide discharge instructions about monitoring neurologic status. c. Transport the patient to radiology for magnetic resonance imaging (MRI). d. Arrange to admit the patient to the neurologic unit for 24 hours of observation.

ANS: B A patient with a minor head trauma is usually discharged with instructions about neurologic monitoring and the need to return if neurologic status deteriorates. MRI, hospital admission, and surgery are not usually indicated in a patient with a concussion.

Which action will the nurse include in the plan of care for a patient who was admitted with syncopal episodes of unknown origin? a. Explain the association between dysrhythmias and syncope. b. Instruct the patient to call for assistance before getting out of bed. c. Teach the patient about the need to avoid caffeine and other stimulants. d. Tell the patient about the benefits of implantable cardioverter-defibrillators.

ANS: B A patient with fainting episodes is at risk for falls. The nurse will plan to minimize the risk by having assistance whenever the patient is up. The other actions may be needed if dysrhythmias are found to be the cause of the patient's syncope but are not appropriate for syncope of unknown origin. DIF: Cognitive Level: Apply (application)

A 45-year-old male patient with suspected acromegaly is seen at the clinic. To assist in making the diagnosis, which question should the nurse ask? a. "Have you had a recent head injury?" b. "Do you have to wear larger shoes now?" c. "Is there a family history of acromegaly?" d. "Are you experiencing tremors or anxiety?"

ANS: B Acromegaly causes an enlargement of the hands and feet. Head injury and family history are not risk factors for acromegaly. Tremors and anxiety are not clinical manifestations of acromegaly

Which action by a new registered nurse (RN) who is orienting to the telemetry unit indicates a good understanding of the treatment of heart dysrhythmias? a. Prepares defibrillator settings at 360 joules for a patient whose monitor shows asystole. b. Injects IV adenosine (Adenocard) over 2 seconds to a patient with supraventricular tachycardia c. Turns the synchronizer switch to the "on" position before defibrillating a patient with ventricular fibrillation d. Gives the prescribed dose of diltiazem (Cardizem) to a patient with new-onset type II second degree AV block

ANS: B Adenosine must be given over 1 to 2 seconds to be effective. The other actions indicate a need for more teaching about treatment of heart dysrhythmias. The RN should hold the diltiazem until discussing it with the health care provider. The treatment for asystole is immediate CPR. The synchronizer switch should be "off" when defibrillating. DIF: Cognitive Level: Analyze (analysis)

Which statement by patient who is being discharged from the emergency department (ED) after a concussion indicates a need for intervention by the nurse? a. "I will return if I feel dizzy or nauseated." b. "I am going to drive home and go to bed." c. "I do not even remember being in an accident." d. "I can take acetaminophen (Tylenol) for my headache."

ANS: B After a head injury, the patient should avoid driving and operating heavy machinery. Retrograde amnesia is common after a concussion. The patient can take acetaminophen for headache and should return if symptoms of increased intracranial pressure such as dizziness or nausea occur.

The nurse recognizes that teaching a 44-year-old woman following a laparoscopic cholecystectomy has been effective when the patient states which of the following? a. "I can expect yellow-green drainage from the incision for a few days." b. "I can remove the bandages on my incisions tomorrow and take a shower." c. "I should plan to limit my activities and not return to work for 4 to 6 weeks." d. "I will always need to maintain a low-fat diet since I no longer have a gallbladder."

ANS: B After a laparoscopic cholecystectomy, the patient will have Band-Aids in place over the incisions. Patients are discharged the same (or next) day and have few restrictions on activities of daily living. Drainage from the incisions would be abnormal, and the patient should be instructed to call the health care provider if this occurs. A low-fat diet may be recommended for a few weeks after surgery but will not be a life-long requirement

The nurse is planning postoperative care for a patient who is being admitted to the surgical unit form the recovery room after transsphenoidal resection of a pituitary tumor. Which nursing action should be included? a. Palpate extremities for edema. b. Measure urine volume every hour. c. Check hematocrit every 2 hours for 8 hours. d. Monitor continuous pulse oximetry for 24 hours.

ANS: B After pituitary surgery, the patient is at risk for diabetes insipidus caused by cerebral edema. Monitoring of urine output and urine specific gravity is essential. Hemorrhage is not a common problem. There is no need to check the hematocrit hourly. The patient is at risk for dehydration, not volume overload. The patient is not at high risk for problems with oxygenation, and continuous pulse oximetry is not needed

19. Before administration of captopril to a patient with stage 2 chronic kidney disease (CKD), the nurse will check the patient's a. glucose. b. potassium. c. creatinine. d. phosphate.

ANS: B Angiotensin-converting enzyme (ACE) inhibitors are frequently used in patients with CKD because they delay the progression of the CKD, but they cause potassium retention. Therefore careful monitoring of potassium levels is needed in patients who are at risk for hyperkalemia. The other laboratory values would also be monitored in patients with CKD but would not affect whether the captopril was given or not. DIF: Cognitive Level: Apply (application) REF: 1075 TOP: Nursing Process: Assessment MSC: NCLEX: Physiological Integrity

Which information given by a 70-year-old patient during a health history indicates to the nurse that the patient should be screened for hepatitis C? a. The patient had a blood transfusion in 2005. b. The patient used IV drugs about 20 years ago. c. The patient frequently eats in fast-food restaurants. d. The patient traveled to a country with poor sanitation.

ANS: B Any patient with a history of IV drug use should be tested for hepatitis C. Blood transfusions given after 1992 (when an antibody test for hepatitis C became available) do not pose a risk for hepatitis C. Hepatitis C is not spread by the oral-fecal route and therefore is not caused by contaminated food or by traveling in underdeveloped countries

The nurse is caring for a 73-year-old man who has cirrhosis. Which data obtained by the nurse during the assessment will be of most concern? a. The patient complains of right upper-quadrant pain with palpation. b. The patient's hands flap back and forth when the arms are extended. c. The patient has ascites and a 2-kg weight gain from the previous day. d. The patient's skin has multiple spider-shaped blood vessels on the abdomen.

ANS: B Asterixis indicates that the patient has hepatic encephalopathy, and hepatic coma may occur. The spider angiomas and right upper quadrant abdominal pain are not unusual for the patient with cirrhosis and do not require a change in treatment. The ascites and weight gain indicate the need for treatment but not as urgently as the changes in neurologic status

18. The nurse preparing for the annual physical exam of a 50-year-old man will plan to teach the patient about a. endoscopy. b. colonoscopy. c. computerized tomography screening. d. carcinoembryonic antigen (CEA) testing.

ANS: B At age 50, individuals with an average risk for colorectal cancer (CRC) should begin screening for CRC. Colonoscopy is the gold standard for CRC screening. The other diagnostic tests are not recommended as part of a routine annual physical exam at age 50.

A patient with idiopathic pulmonary arterial hypertension (IPAH) is receiving nifedipine (Procardia). Which assessment would best indicate to the nurse that the patient's condition is improving? a. Blood pressure (BP) is less than 140/90 mm Hg. b. Patient reports decreased exertional dyspnea. c. Heart rate is between 60 and 100 beats/minute. d. Patient's chest x-ray indicates clear lung fields.

ANS: B Because a major symptom of IPAH is exertional dyspnea, an improvement in this symptom would indicate that the medication was effective. Nifedipine will affect BP and heart rate, but these parameters would not be used to monitor the effectiveness of therapy for a patient with IPAH. The chest x-ray will show clear lung fields even if the therapy is not effective

10. Which nursing action will the nurse include in the plan of care for a 35-year-old male patient admitted with an exacerbation of inflammatory bowel disease (IBD)? a. Restrict oral fluid intake. b. Monitor stools for blood. c. Ambulate four times daily. d. Increase dietary fiber intake.

ANS: B Because anemia or hemorrhage may occur with IBD, stools should be assessed for the presence of blood. The other actions would not be appropriate for the patient with IBD. Because dietary fiber may increase gastrointestinal (GI) motility and exacerbate the diarrhea, severe fatigue is common with IBD exacerbations, and dehydration may occur.

33. When admitting a patient with a non-ST-segment-elevation myocardial infarction (NSTEMI) to the intensive care unit, which action should the nurse perform first? a. Obtain the blood pressure. b. Attach the cardiac monitor. c. Assess the peripheral pulses. d. Auscultate the breath sounds.

ANS: B Because dysrhythmias are the most common complication of myocardial infarction (MI), the first action should be to place the patient on a cardiac monitor. The other actions also are important and should be accomplished as quickly as possible.

27. A 42-yr-old patient admitted with acute kidney injury due to dehydration has oliguria, anemia, and hyperkalemia. Which prescribed action should the nurse take first? a. Insert a urinary retention catheter. b. Place the patient on a cardiac monitor. c. Administer epoetin alfa (Epogen, Procrit). d. Give sodium polystyrene sulfonate (Kayexalate).

ANS: B Because hyperkalemia can cause fatal cardiac dysrhythmias, the initial action should be to monitor the cardiac rhythm. Kayexalate and Epogen will take time to correct the hyperkalemia and anemia. The catheter allows monitoring of the urine output but does not correct the cause of the renal failure. DIF: Cognitive Level: Analyze (analysis) REF: 1073 OBJ: Special Questions: Prioritization TOP: Nursing Process: Implementation MSC: NCLEX: Physiological Integrity

39. To improve the physical activity level for a mildly obese 71-year-old patient, which action should the nurse plan to take? a. Stress that weight loss is a major benefit of increased exercise. b. Determine what kind of physical activities the patient usually enjoys. c. Tell the patient that older adults should exercise for no more than 20 minutes at a time. d. Teach the patient to include a short warm-up period at the beginning of physical activity.

ANS: B Because patients are more likely to continue physical activities that they already enjoy, the nurse will plan to ask the patient about preferred activities. The goal for older adults is 30 minutes of moderate activity on most days. Older adults should plan for a longer warm-up period. Benefits of exercises, such as improved activity tolerance, should be emphasized rather than aiming for significant weight loss in older mildly obese adults.

The nurse supervises the care of a patient with a temporary radioactive cervical implant. Which action by unlicensed assistive personnel (UAP), if observed by the nurse, would require an intervention? a. The UAP flushes the toilet once after emptying the patient's bedpan. b. The UAP stands by the patient's bed for 30 minutes talking with the patient. c. The UAP places the patient's bedding in the laundry container in the hallway. d. The UAP gives the patient an alcohol-containing mouthwash to use for oral care.

ANS: B Because patients with temporary implants emit radioactivity while the implants are in place, exposure to the patient is limited. Laundry and urine/feces do not have any radioactivity and do not require special precautions. Cervical radiation will not affect the oral mucosa, and alcohol-based mouthwash is not contraindicated

35. A patient in the emergency department has just been diagnosed with peritonitis caused by a ruptured diverticulum. Which prescribed intervention will the nurse implement first? a. Insert a urinary catheter to drainage. b. Infuse metronidazole (Flagyl) 500 mg IV. c. Send the patient for a computerized tomography scan. d. Place a nasogastric (NG) tube to intermittent low suction.

ANS: B Because peritonitis can be fatal if treatment is delayed, the initial action should be to start antibiotic therapy (after any ordered cultures are obtained). The other actions can be done after antibiotic therapy is initiated.

The home health nurse cares for a patient who has been receiving interferon therapy for treatment of cancer. Which statement by the patient indicates a need for further assessment? a. "I have frequent muscle aches and pains." b. "I rarely have the energy to get out of bed." c. "I experience chills after I inject the interferon." d. "I take acetaminophen (Tylenol) every 4 hours."

ANS: B Fatigue can be a dose-limiting toxicity for use of biologic therapies. Flulike symptoms, such as muscle aches and chills, are common side effects with interferon use. Patients are advised to use acetaminophen every 4 hours

A 36-year-old female patient is receiving treatment for chronic hepatitis C with pegylated interferon (PEG-Intron, Pegasys), ribavirin (Rebetol), and telaprevir (Incivek). Which finding is most important to communicate to the health care provider? a. Weight loss of 2 lb (1 kg) b. Positive urine pregnancy test c. Hemoglobin level of 10.4 g/dL d. Complaints of nausea and anorexia

ANS: B Because ribavirin is teratogenic, the medication will need to be discontinued immediately. Anemia, weight loss, and nausea are common adverse effects of the prescribed regimen and may require actions such as patient teaching, but they would not require immediate cessation of the therapy.

Which question will the nurse ask a patient who has been admitted with a benign occipital lobe tumor to assess for functional deficits? a. "Do you have difficulty in hearing?" b. "Are you experiencing visual problems?" c. "Are you having any trouble with your balance?" d. "Have you developed any weakness on one side?"

ANS: B Because the occipital lobe is responsible for visual reception, the patient with a tumor in this area is likely to have problems with vision. The other questions will be better for assessing function of the temporal lobe, cerebellum, and frontal lobe.

41. Which information obtained by the nurse interviewing a 30-year-old male patient is most important to communicate to the health care provider? a. The patient has a history of constipation. b. The patient has noticed blood in the stools. c. The patient had an appendectomy at age 27. d. The patient smokes a pack/day of cigarettes.

ANS: B Blood in the stools is a possible clinical manifestation of colorectal cancer and requires further assessment by the health care provider. The other patient information will also be communicated to the health care provider, but does not indicate an urgent need for further testing or intervention.

20. A 74-year-old patient preparing to undergo a colon resection for cancer of the colon asks about the elevated carcinoembryonic antigen (CEA) test result. The nurse explains that the test is used to a. identify any metastasis of the cancer. b. monitor the tumor status after surgery. c. confirm the diagnosis of a specific type of cancer. d. determine the need for postoperative chemotherapy.

ANS: B CEA is used to monitor for cancer recurrence after surgery. CEA levels do not help to determine whether there is metastasis of the cancer. Confirmation of the diagnosis is made on the basis of biopsy. Chemotherapy use is based on factors other than CEA.

6. Which information will the nurse monitor in order to determine the effectiveness of prescribed calcium carbonate (Caltrate) for a patient with chronic kidney disease (CKD)? a. Blood pressure b. Phosphate level c. Neurologic status d. Creatinine clearance

ANS: B Calcium carbonate is prescribed to bind phosphorus and prevent mineral and bone disease in patients with CKD. The other data will not be helpful in evaluating the effectiveness of calcium carbonate. DIF: Cognitive Level: Apply (application) REF: 1081 TOP: Nursing Process: Evaluation MSC: NCLEX: Physiological Integrity

A patient admitted with a diffuse axonal injury has a systemic blood pressure (BP) of 106/52 mm Hg and an intracranial pressure (ICP) of 14 mm Hg. Which action should the nurse take first? a. Document the BP and ICP in the patient's record. b. Report the BP and ICP to the health care provider. c. Elevate the head of the patient's bed to 60 degrees. d. Continue to monitor the patient's vital signs and ICP.

ANS: B Calculate the cerebral perfusion pressure (CPP): (CPP = Mean arterial pressure [MAP] - ICP). MAP = DBP + 1/3 (Systolic blood pressure [SBP] - Diastolic blood pressure [DBP]). Therefore the MAP is 70, and the CPP is 56 mm Hg, which are below the normal values of 60 to 100 mm Hg and are approaching the level of ischemia and neuronal death. Immediate changes in the patient's therapy such as fluid infusion or vasopressor administration are needed to improve the CPP. Adjustments in the head elevation should only be done after consulting with the health care provider. Continued monitoring and documentation will also be done, but they are not the first actions that the nurse should take.

The nurse provides discharge instructions to a patient who has an immune deficiency involving the T lymphocytes. Which screening should the nurse include in the teaching plan for this patient? a. Screening for allergies b. Screening for malignancy c. Antibody deficiency screening d. Screening for autoimmune disorders

ANS: B Cell-mediated immunity is responsible for the recognition and destruction of cancer cells. Allergic reactions, autoimmune disorders, and antibody deficiencies are mediated primarily by B lymphocytes and humoral immunity.

The nurse is assessing a 41-year-old African American male patient diagnosed with a pituitary tumor causing panhypopituitarism. Assessment findings consistent with panhypopituitarism include a. high blood pressure. b. decreased facial hair. c. elevated blood glucose. d. tachycardia and cardiac palpitations.

ANS: B Changes in male secondary sex characteristics such as decreased facial hair, testicular atrophy, diminished spermatogenesis, loss of libido, impotence, and decreased muscle mass are associated with decreases in follicle stimulating hormone (FSH) and luteinizing hormone (LH). Fasting hypoglycemia and hypotension occur in panhypopituitarism as a result of decreases in adrenocorticotropic hormone (ACTH) and cortisol. Bradycardia is likely due to the decrease in thyroid stimulating hormone (TSH) and thyroid hormones associated with panhypopituitarism

A 20-yr-old male patient is admitted with a head injury after a collision while playing football. After noting that the patient has developed clear nasal drainage, which action should the nurse take? a. Have the patient gently blow the nose. b. Check the drainage for glucose content. c. Teach the patient that rhinorrhea is expected after a head injury. d. Obtain a specimen of the fluid to send for culture and sensitivity.

ANS: B Clear nasal drainage in a patient with a head injury suggests a dural tear and cerebrospinal fluid (CSF) leakage. If the drainage is CSF, it will test positive for glucose. Fluid leaking from the nose will have normal nasal flora, so culture and sensitivity will not be useful. Blowing the nose is avoided to prevent CSF leakage.

The nurse is caring for a 64-yr-old patient admitted with mitral valve regurgitation. Which information obtained by the nurse when assessing the patient should be communicated to the health care provider immediately? a. The patient has 4+ peripheral edema. b. The patient has diffuse bilateral crackles. c. The patient has a loud systolic murmur across the precordium. d. The patient has a palpable thrill felt over the left anterior chest.

ANS: B Crackles that are audible throughout the lungs indicate that the patient is experiencing severe left ventricular failure with pulmonary congestion and needs immediate interventions such as diuretics. A systolic murmur and palpable thrill would be expected in a patient with mitral regurgitation. Although 4+ peripheral edema indicates a need for a change in therapy, it does not need to be addressed urgently. DIF: Cognitive Level: Analyze (analysis)

32. The nurse is assessing a 31-year-old female patient with abdominal pain. Th nurse,who notes that there is ecchymosis around the area of umbilicus, will document this finding as a. Cullen sign. b. Rovsing sign. c. McBurney sign. d. Grey-Turner's signt.

ANS: B Cullen sign is ecchymosis around the umbilicus. Rovsing sign occurs when palpation of the left lower quadrant causes pain in the right lower quadrant. Deep tenderness at McBurney's point (halfway between the umbilicus and the right iliac crest), known as McBurney's sign, is a sign of acute appendicitis.

The nurse identifies the nursing diagnosis of decreased cardiac output related to valvular insufficiency for the patient with infective endocarditis (IE) based on which assessment finding(s)? a. Fever, chills, and diaphoresis b. Urine output less than 30 mL/hr c. Petechiae on the inside of the mouth and conjunctiva d. Increase in heart rate of 15 beats/minute with walking

ANS: B Decreased renal perfusion caused by inadequate cardiac output will lead to decreased urine output. Petechiae, fever, chills, and diaphoresis are symptoms of IE but are not caused by decreased cardiac output. An increase in pulse rate of 15 beats/min is normal with exercise. DIF: Cognitive Level: Apply (application)

The nurse determines that demeclocycline (Declomycin) is effective for a patient with syndrome of inappropriate antidiuretic hormone (SIADH) based on finding that the patient's a. weight has increased. b. urinary output is increased. c. peripheral edema is decreased. d. urine specific gravity is increased.

ANS: B Demeclocycline blocks the action of antidiuretic hormone (ADH) on the renal tubules and increases urine output. An increase in weight or an increase in urine specific gravity indicates that the SIADH is not corrected. Peripheral edema does not occur with SIADH. A sudden weight gain without edema is a common clinical manifestation of this disorder

20. When caring for a patient who is recovering from a sudden cardiac death (SCD) event and has no evidence of an acute myocardial infarction (AMI), the nurse will anticipate teaching the patient that a. sudden cardiac death events rarely reoccur. b. additional diagnostic testing will be required. c. long-term anticoagulation therapy will be needed. d. limited physical activity after discharge will be needed to prevent future events.

ANS: B Diagnostic testing (e.g., stress test, Holter monitor, electrophysiologic studies, cardiac catheterization) is used to determine the possible cause of the SCD and treatment options. SCD is likely to recur. Anticoagulation therapy will not have any effect on the incidence of SCD, and SCD can occur even when the patient is resting.

28. A patient has arrived for a scheduled hemodialysis session. Which nursing action is most appropriate for the registered nurse (RN) to delegate to a dialysis technician? a. Teach the patient about fluid restrictions. b. Check blood pressure before starting dialysis. c. Assess for causes of an increase in predialysis weight. d. Determine the ultrafiltration rate for the hemodialysis.

ANS: B Dialysis technicians are educated in monitoring for blood pressure. Assessment, adjustment of the appropriate ultrafiltration rate, and patient teaching require the education and scope of practice of an RN. DIF: Cognitive Level: Apply (application) REF: 1089 OBJ: Special Questions: Delegation TOP: Nursing Process: Planning MSC: NCLEX: Safe and Effective Care Environment

A patient tells the nurse, "I would like to use a home genetic test to see if I will develop breast cancer." Which response by the nurse is best? a. "Home genetic testing is very expensive." b. "Are you concerned about developing breast cancer?" c. "Won't you be depressed if the testing shows a positive result?" d. "Genetic testing can only determine if you are at higher risk for breast cancer."

ANS: B This response uses the communication technique of clarifying to further assess the patient's concerns. The other options accurately indicate information about genetic testing, but the initial response by the nurse should be focused on assessment

A patient is admitted to the hospital with possible acute pericarditis. The nurse should plan to teach the patient about the purpose of a. blood cultures. c. cardiac catheterization. b. echocardiography. d. 24-hour Holter monitor.

ANS: B Echocardiograms are useful in detecting the presence of the pericardial effusions associated with pericarditis. Blood cultures are not indicated unless the patient has evidence of sepsis. Cardiac catheterization and 24-hour Holter monitor are not diagnostic procedures for pericarditis. DIF: Cognitive Level: Apply (application)

14. After a total proctocolectomy and permanent ileostomy, the patient tells the nurse, "I cannot manage all these changes. I don't want to look at the stoma." What is the best action by the nurse? a. Reassure the patient that ileostomy care will become easier. b. Ask the patient about the concerns with stoma management. c. Develop a detailed written list of ostomy care tasks for the patient. d. Postpone any teaching until the patient adjusts to the ileostomy.

ANS: B Encouraging the patient to share concerns assists in helping the patient adjust to the body changes. Acknowledgment of the patient's feelings and concerns is important rather than offering false reassurance. Because the patient indicates that the feelings about the ostomy are the reason for the difficulty with the many changes, development of a detailed ostomy care plan will not improve the patient's ability to manage the ostomy. Although detailed ostomy teaching may be postponed, the nurse should offer teaching about some aspects of living with an ostomy.

Which action will the public health nurse take to reduce the incidence of epidemic encephalitis in a community? a. Teach about prophylactic antibiotics after exposure to encephalitis. b. Encourage the use of effective insect repellent during mosquito season. c. Remind patients that most cases of viral encephalitis can be cared for at home. d. Arrange to screen school-age children for West Nile virus during the school year.

ANS: B Epidemic encephalitis is usually spread by mosquitoes and ticks. Use of insect repellent is effective in reducing risk. Encephalitis frequently requires that the patient be hospitalized in an intensive care unit during the initial stages. Antibiotic prophylaxis is not used to prevent encephalitis because most encephalitis is viral. West Nile virus is most common in adults over age 50 during the summer and early fall.

During the primary assessment of a victim of a motor vehicle collision, the nurse determines that the patient has an unobstructed airway. Which action should the nurse take next? a. Palpate extremities for bilateral pulses. b. Observe the patient's respiratory effort. c. Check the patient's level of consciousness. d. Examine the patient for any external bleeding.

ANS: B Even with a patent airway, patients can have other problems that compromise ventilation, so the next action is to assess the patient's breathing. The other actions are also part of the initial survey but assessment of breathing should be done immediately after assessing for airway patency.

7. A 39-year-old patient whose work involves frequent lifting has a history of chronic back pain. After the nurse has taught the patient about correct body mechanics, which patient statement indicates that the teaching has been effective? a. I will keep my back straight to lift anything higher than my waist. b. I will begin doing exercises to strengthen the muscles of my back. c. I can try to sleep with my hips and knees extended to prevent back strain. d. I can tell my boss that I need to change to a job where I can work at a desk.

ANS: B Exercises can help strengthen the muscles that support the back. Flexion of the hips and knees places less strain on the back. Modifications in the way the patient lifts boxes are needed, but sitting for prolonged periods can aggravate back pain. The patient should not lift above the level of the elbows.

Which action should the nurse take to evaluate treatment effectiveness for a patient who has hepatic encephalopathy? a. Request that the patient stand on one foot. b. Ask the patient to extend both arms forward. c. Request that the patient walk with eyes closed. d. Ask the patient to perform the Valsalva maneuver.

ANS: B Extending the arms allows the nurse to check for asterixis, a classic sign of hepatic encephalopathy. The other tests might also be done as part of the neurologic assessment but would not be diagnostic for hepatic encephalopathy.

33. A 54-year-old critically ill patient with sepsis is frequently incontinent of watery stools. What action by the nurse will prevent complications associated with ongoing incontinence? a. Apply incontinence briefs. b. Use a fecal management system c. Insert a rectal tube with a drainage bag. d. Assist the patient to a commode frequently.

ANS: B Fecal management systems are designed to contain loose stools and can be in place for as long as 4 weeks without causing damage to the rectum or anal sphincters. Although incontinence briefs may be helpful, unless they are changed frequently, they are likely to increase the risk for skin breakdown. Rectal tubes are avoided because of possible damage to the anal sphincter and ulceration of the rectal mucosa. A critically ill patient will not be able to tolerate getting up frequently to use the commode or bathroom.

A patient has a sinus rhythm and a heart rate of 72 beats/min. The nurse determines that the PR interval is 0.24 seconds. The most appropriate intervention by the nurse would be to a. notify the a provider immediately. b. document the finding and monitor the patient. c. give atropine per agency dysrhythmia protocol. d. prepare the patient for temporary pacemaker insertion.

ANS: B First-degree atrioventricular block is asymptomatic and requires ongoing monitoring because it may progress to more serious forms of heart block. The rate is normal, so there is no indication that atropine is needed. Immediate notification of the health care provider about an asymptomatic rhythm is not necessary. DIF: Cognitive Level: Apply (application)

16. A 24-year-old woman with Crohn's disease develops a fever and symptoms of a urinary tract infection (UTI) with tan, fecal-smelling urine. What information will the nurse add to a general teaching plan about UTIs in order to individualize the teaching for this patient? a. Bacteria in the perianal area can enter the urethra. b. Fistulas can form between the bowel and bladder. c. Drink adequate fluids to maintain normal hydration. d. Empty the bladder before and after sexual intercourse.

ANS: B Fistulas between the bowel and bladder occur in Crohn's disease and can lead to UTI. Teaching for UTI prevention in general includes good hygiene, adequate fluid intake, and voiding before and after intercourse.

A patient who has experienced blunt abdominal trauma during a motor vehicle collision is complaining of increasing abdominal pain. The nurse will plan to teach the patient about the purpose of a. peritoneal lavage. b. abdominal ultrasonography. c. nasogastric (NG) tube placement. d. magnetic resonance imaging (MRI).

ANS: B For patients who are at risk for intraabdominal bleeding, focused abdominal ultrasonography is the preferred method to assess for intraperitoneal bleeding. An MRI would not be used. Peritoneal lavage is an alternative, but it is more invasive. An NG tube would not be helpful in the diagnosis of intraabdominal bleeding.

A hospitalized patient who has received chemotherapy for leukemia develops neutropenia. Which observation by the nurse would indicate a need for further teaching? a. The patient ambulates several times a day in the room. b. The patient's visitors bring in some fresh peaches from home. c. The patient cleans with a warm washcloth after having a stool. d. The patient uses soap and shampoo to shower every other day.

ANS: B Fresh, thinned-skin fruits are not permitted in a neutropenic diet because of the risk of bacteria being present. The patient should ambulate in the room rather than the hospital hallway to avoid exposure to other patients or visitors. Because overuse of soap can dry the skin and increase infection risk, showering every other day is acceptable. Careful cleaning after having a bowel movement will help prevent skin breakdown and infection

An experienced nurse instructs a new nurse about how to care for a patient with dyspnea caused by a pulmonary fungal infection. Which action by the new nurse indicates a need for further teaching? a. Listening to the patient's lung sounds several times during the shift b. Placing the patient on droplet precautions and in a private hospital room c. Increasing the oxygen flow rate to keep the oxygen saturation above 90% d. Monitoring patient serology results to identify the specific infecting organism

ANS: B Fungal infections are not transmitted from person to person. Therefore no isolation procedures are necessary. The other actions by the new nurse are appropriate

14. Which action should the nurse take before administering gentamicin (Garamycin) to a patient who has acute osteomyelitis? a. Ask the patient about any nausea. b. Review the patients creatinine level. c. Obtain the patients oral temperature. d. Change the prescribed wet-to-dry dressing.

ANS: B Gentamicin is nephrotoxic and can cause renal failure. Monitoring the patients temperature before gentamicin administration is not necessary. Nausea is not a common side effect of IV gentamicin. There is no need to change the dressing before gentamicin administration.

23. Which information will the nurse include in teaching a patient who had a proctocolectomy and ileostomy for ulcerative colitis? a. Restrict fluid intake to prevent constant liquid drainage from the stoma. b. Use care when eating high-fiber foods to avoid obstruction of the ileum. c. Irrigate the ileostomy daily to avoid having to wear a drainage appliance. d. Change the pouch every day to prevent leakage of contents onto the skin.

ANS: B High-fiber foods are introduced gradually and should be well chewed to avoid obstruction of the ileostomy. Patients with ileostomies lose the absorption of water in the colon and need to take in increased amounts of fluid. The pouch should be drained frequently but is changed every 5 to 7 days. The drainage from an ileostomy is liquid and continuous, so control by irrigation is not possible.

After receiving change-of-shift report on four patients, which patient should the nurse assess first? a. Patient with rheumatic fever who has sharp chest pain with a deep breath b. Patient with acute aortic regurgitation whose blood pressure is 86/54 mm Hg c. Patient with infective endocarditis who has a murmur and splinter hemorrhages d. Patient with dilated cardiomyopathy who has bilateral crackles at the lung bases

ANS: B Hypotension in patients with acute aortic regurgitation may indicate cardiogenic shock. The nurse should immediately assess this patient for other findings such as dyspnea, chest pain or tachycardia. The findings in the other patients are typical of their diagnoses and do not indicate a need for urgent assessment and intervention. DIF: Cognitive Level: Analyze (analysis)

After evacuation of an epidural hematoma, a patient's intracranial pressure (ICP) is being monitored with an intraventricular catheter. Which information obtained by the nurse requires urgent communication with the health care provider? a. Pulse of 102 beats/min b. Temperature of 101.6° F c. Intracranial pressure of 15 mm Hg d. Mean arterial pressure of 90 mm Hg

ANS: B Infection is a serious consideration with ICP monitoring, especially with intraventricular catheters. The temperature indicates the need for antibiotics or removal of the monitor. The ICP, arterial pressure, and apical pulse only require ongoing monitoring at this time.

A patient has just been admitted with probable bacterial pneumonia and sepsis. Which order should the nurse implement first? a. Chest x-ray via stretcher b. Blood cultures from two sites c. Ciprofloxacin (Cipro) 400 mg IV d. Acetaminophen (Tylenol) rectal suppository

ANS: B Initiating antibiotic therapy rapidly is essential, but it is important that the cultures be obtained before antibiotic administration. The chest x-ray and acetaminophen administration can be done last

After receiving change-of-shift report about the following four patients, which patient should the nurse assess first? a. A 31-year-old female with Cushing syndrome and a blood glucose level of 244 mg/dL b. A 70-year-old female taking levothyroxine (Synthroid) who has an irregular pulse of 134 c. A 53-year-old male who has Addison's disease and is due for a scheduled dose of hydrocortisone (Solu-Cortef). d. A 22-year-old male admitted with syndrome of inappropriate antidiuretic hormone (SIADH) who has a serum sodium level of 130 mEq/L

ANS: B Initiation of thyroid replacement in older adults may cause angina and cardiac dysrhythmias. The patient's high pulse rate needs rapid investigation by the nurse to assess for and intervene with any cardiac problems. The other patients also require nursing assessment and/or actions but are not at risk for life-threatening complications

54. Which information will the nurse teach a 23-year-old patient with lactose intolerance? a. Ice cream is relatively low in lactose. b. Live-culture yogurt is usually tolerated. c. Heating milk will break down the lactose. d. Nonfat milk is a better choice than whole milk.

ANS: B Lactose-intolerant individuals can usually eat yogurt without experiencing discomfort. Ice cream, nonfat milk, and milk that has been heated are all high in lactose.

A patient has ST-segment changes that suggest an acute inferior wall myocardial infarction. Which lead would be best for monitoring the patient? a. I c. V2 b. II d. V6

ANS: B Leads II, III, and AVF reflect the inferior area of the heart and the ST segment changes. Lead II will best capture any electrocardiographic changes that indicate further damage to the myocardium. The other leads do not reflect the inferior part of the myocardial wall and will not provide data about further ischemic changes in that area. DIF: Cognitive Level: Analyze (analysis)

The nurse will plan to teach the patient diagnosed with acute hepatitis B about a. side effects of nucleotide analogs. b. measures for improving the appetite. c. ways to increase activity and exercise. d. administering a-interferon (Intron A).

ANS: B Maintaining adequate nutritional intake is important for regeneration of hepatocytes. Interferon and antivirals may be used for chronic hepatitis B, but they are not prescribed for acute hepatitis B infection. Rest is recommended

The nurse is assessing a patient with myocarditis before giving the scheduled dose of digoxin (Lanoxin). Which finding is most important for the nurse to communicate to the health care provider? a. Leukocytosis c. Generalized myalgia b. Irregular pulse d. Complaint of fatigue

ANS: B Myocarditis predisposes the heart to digoxin-associated dysrhythmias and toxicity. The other findings are common symptoms of myocarditis and there is no urgent need to report these. DIF: Cognitive Level: Analyze (analysis)

During the assessment of a young adult patient with infective endocarditis (IE), the nurse would expect to find a. substernal chest pressure. c. a pruritic rash on the chest. b. a new regurgitant murmur. d. involuntary muscle movement.

ANS: B New regurgitant murmurs occur in IE because vegetations on the valves prevent valve closure. Substernal chest discomfort, rashes, and involuntary muscle movement are clinical manifestations of other cardiac disorders such as angina and rheumatic fever. DIF: Cognitive Level: Understand (comprehension)

A patient who had radical neck surgery to remove a malignant tumor developed hypoparathyroidism. The nurse should plan to teach the patient about a. bisphosphonates to reduce bone demineralization. b. calcium supplements to normalize serum calcium levels. c. increasing fluid intake to decrease risk for nephrolithiasis. d. including whole grains in the diet to prevent constipation.

ANS: B Oral calcium supplements are used to maintain the serum calcium in normal range and prevent the complications of hypocalcemia. Whole grain foods decrease calcium absorption and will not be recommended. Bisphosphonates will lower serum calcium levels further by preventing calcium from being reabsorbed from bone. Kidney stones are not a complication of hypoparathyroidism and low calcium levels

52. After change-of-shift report, which patient should the nurse assess first? a. 40-year-old male with celiac disease who has frequent frothy diarrhea b. 30-year-old female with a femoral hernia who has abdominal pain and vomiting c. 30-year-old male with ulcerative colitis who has severe perianal skin breakdown d. 40-year-old female with a colostomy bag that is pulling away from the adhesive wafer

ANS: B Pain and vomiting with a femoral hernia suggest possible strangulation, which will necessitate emergency surgery. The other patients have less urgent problems.

The nurse receives change-of-shift report on the following four patients. Which patient should the nurse assess first? a. A 23-year-old patient with cystic fibrosis who has pulmonary function testing scheduled b. A 46-year-old patient on bed rest who is complaining of sudden onset of shortness of breath c. A 77-year-old patient with tuberculosis (TB) who has four antitubercular medications due in 15 minutes d. A 35-year-old patient who was admitted the previous day with pneumonia and has a temperature of 100.2° F (37.8° C)

ANS: B Patients on bed rest who are immobile are at high risk for deep vein thrombosis (DVT). Sudden onset of shortness of breath in a patient with a DVT suggests a pulmonary embolism and requires immediate assessment and action such as oxygen administration. The other patients should also be assessed as soon as possible, but there is no indication that they may need immediate action to prevent clinical deterioration

The nurse provides discharge instructions to a patient who was hospitalized for pneumonia. Which statement, if made by the patient, indicates a good understanding of the instructions? a. "I will call the doctor if I still feel tired after a week." b. "I will continue to do the deep breathing and coughing exercises at home." c. "I will schedule two appointments for the pneumonia and influenza vaccines." d. "I'll cancel my chest x-ray appointment if I'm feeling better in a couple weeks."

ANS: B Patients should continue to cough and deep breathe after discharge. Fatigue is expected for several weeks. The Pneumovax and influenza vaccines can be given at the same time in different arms. Explain that a follow-up chest x-ray needs to be done in 6 to 8 weeks to evaluate resolution of pneumonia

10. The nurse will suspect that the patient with stable angina is experiencing a side effect of the prescribed metoprolol (Lopressor) if the a. patient is restless and agitated. b. blood pressure is 90/54 mm Hg. c. patient complains about feeling anxious. d. cardiac monitor shows a heart rate of 61 beats/minute.

ANS: B Patients taking β-adrenergic blockers should be monitored for hypotension and bradycardia. Because this class of medication inhibits the sympathetic nervous system, restlessness, agitation, hypertension, and anxiety will not be side effects.

50. A new 19-year-old male patient has familial adenomatous polyposis (FAP). Which action will the nurse in the gastrointestinal clinic include in the plan of care? a. Obtain blood samples for DNA analysis. b. Schedule the patient for yearly colonoscopy. c. Provide preoperative teaching about total colectomy. d. Discuss lifestyle modifications to decrease cancer risk.

ANS: B Patients with FAP should have annual colonoscopy starting at age 16 and usually have total colectomy by age 25 to avoid developing colorectal cancer. DNA analysis is used to make the diagnosis, but is not needed now for this patient. Lifestyle modifications will not decrease cancer risk for this patient.

Which action will the nurse include in the plan of care for a patient who has been diagnosed with chronic hepatitis B? a. Advise limiting alcohol intake to 1 drink daily. b. Schedule for liver cancer screening every 6 months. c. Initiate administration of the hepatitis C vaccine series. d. Monitor anti-hepatitis B surface antigen (anti-HBs) levels annually.

ANS: B Patients with chronic hepatitis are at higher risk for development of liver cancer, and should be screened for liver cancer every 6 to 12 months. Patients with chronic hepatitis are advised to completely avoid alcohol. There is no hepatitis C vaccine. Because anti-HBs is present whenever there has been a past hepatitis B infection or vaccination, there is no need to regularly monitor for this antibody.

2. When a patient with acute kidney injury (AKI) has an arterial blood pH of 7.30, the nurse will expect an assessment finding of a. persistent skin tenting b. rapid, deep respirations. c. hot, flushed face and neck. d. bounding peripheral pulses.

ANS: B Patients with metabolic acidosis caused by AKI may have Kussmaul respirations as the lungs try to regulate carbon dioxide. Bounding pulses and vasodilation are not associated with metabolic acidosis. Because the patient is likely to have fluid retention, poor skin turgor would not be a finding in AKI. DIF: Cognitive Level: Apply (application) REF: 1072 TOP: Nursing Process: Assessment MSC: NCLEX: Physiological Integrity

To assess the patient with pericarditis for evidence of a pericardial friction rub, the nurse should a. listen for a rumbling, low-pitched, systolic murmur over the left anterior chest. b. auscultate with the diaphragm of the stethoscope on the lower left sternal border. c. ask the patient to cough during auscultation to distinguish the sound from a pleural friction rub. d. feel the precordial area with the palm of the hand to detect vibrations with cardiac contraction.

ANS: B Pericardial friction rubs are best heard with the diaphragm at the lower left sternal border. The nurse should ask the patient to hold his or her breath during auscultation to distinguish the sounds from a pleural friction rub. Friction rubs are not typically low pitched or rumbling and are not confined to systole. Rubs are not assessed by palpation. DIF: Cognitive Level: Understand (comprehension)

The nurse teaches a patient diagnosed with systemic lupus erythematosus (SLE) about plasmapheresis. What instructions about plasmapheresis should the nurse include in the teaching plan? a. Plasmapheresis will eliminate eosinophils and basophils from blood. b. Plasmapheresis will remove antibody-antigen complexes from circulation. c. Plasmapheresis will prevent foreign antibodies from damaging various body tissues. d. Plasmapheresis will decrease the damage to organs caused by attacking T lymphocytes.

ANS: B Plasmapheresis is used in SLE to remove antibodies, antibody-antigen complexes, and complement from blood. T lymphocytes, foreign antibodies, eosinophils, and basophils do not directly contribute to the tissue damage in SLE.

9. Diltiazem (Cardizem) is ordered for a patient with newly diagnosed Prinzmetal's (variant) angina. When teaching the patient, the nurse will include the information that diltiazem will a. reduce heart palpitations. b. decrease spasm of the coronary arteries. c. increase the force of the heart contractions. d. help prevent plaque from forming in the coronary arteries.

ANS: B Prinzmetal's angina is caused by coronary artery spasm. Calcium channel blockers (e.g., diltiazem, amlodipine [Norvasc

For a patient with cirrhosis, which of the following nursing actions can the registered nurse (RN) delegate to unlicensed assistive personnel (UAP)? a. Assessing the patient for jaundice b. Providing oral hygiene after a meal c. Palpating the abdomen for distention d. Assisting the patient to choose the diet

ANS: B Providing oral hygiene is within the scope of UAP. Assessments and assisting patients to choose therapeutic diets are nursing actions that require higher-level nursing education and scope of practice and would be delegated to licensed practical/vocational nurses (LPNs/LVNs) or RNs

The nurse suspects cardiac tamponade in a patient who has acute pericarditis. To assess for the presence of pulsus paradoxus, the nurse should a. subtract the diastolic blood pressure from the systolic blood pressure. b. note when Korotkoff sounds are auscultated during both inspiration and expiration. c. check the electrocardiogram (ECG) for variations in rate during the respiratory cycle. d. listen for a pericardial friction rub that persists when the patient is instructed to stop breathing.

ANS: B Pulsus paradoxus exists when there is a gap of greater than 10 mm Hg between when Korotkoff sounds can be heard during only expiration and when they can be heard throughout the respiratory cycle. The other methods described would not be useful in determining the presence of pulsus paradoxus. DIF: Cognitive Level: Apply (application)

6. A 58-year-old man with blunt abdominal trauma from a motor vehicle crash undergoes peritoneal lavage. If the lavage returns brown fecal drainage, which action will the nurse plan to take next? a. Auscultate the bowel sounds. b. Prepare the patient for surgery. c. Check the patient's oral temperature. d. Obtain information about the accident.

ANS: B Return of brown drainage and fecal material suggests perforation of the bowel and the need for immediate surgery. Auscultation of bowel sounds, checking the temperature, and obtaining information about the accident are appropriate actions, but the priority is to prepare to send the patient for emergency surgery.

The nurse is admitting a patient with possible rheumatic fever. Which question on the admission health history focuses on a pertinent risk factor for rheumatic fever? a. "Do you use any illegal IV drugs?" b. "Have you had a recent sore throat?" c. "Have you injured your chest in the last few weeks?" d. "Do you have a family history of congenital heart disease?"

ANS: B Rheumatic fever occurs as a result of an abnormal immune response to a streptococcal infection. Although illicit IV drug use should be discussed with the patient before discharge, it is not a risk factor for rheumatic fever, and it would not be as pertinent when admitting the patient. Family history is not a risk factor for rheumatic fever. Chest injury would cause musculoskeletal chest pain rather than rheumatic fever. DIF: Cognitive Level: Apply (application)

The nurse is admitting a patient with a basal skull fracture. The nurse notes ecchymoses around both eyes and clear drainage from the patient's nose. Which admission order should the nurse question? a. Keep the head of bed elevated. b. Insert nasogastric tube to low suction. c. Turn patient side to side every 2 hours. d. Apply cold packs intermittently to face.

ANS: B Rhinorrhea may indicate a dural tear with cerebrospinal fluid leakage. Insertion of a nasogastric tube will increase the risk for infections such as meningitis. Turning the patient, elevating the head, and applying cold packs are appropriate orders.

23. A patient who is recovering from an acute myocardial infarction (AMI) asks the nurse about when sexual intercourse can be resumed. Which response by the nurse is best? a. "Most patients are able to enjoy intercourse without any complications." b. "Sexual activity uses about as much energy as climbing two flights of stairs." c. "The doctor will provide sexual guidelines when your heart is strong enough." d. "Holding and cuddling are good ways to maintain intimacy after a heart attack."

ANS: B Sexual activity places about as much physical stress on the cardiovascular system as most moderate-energy activities such as climbing two flights of stairs. The other responses do not directly address the patient's question or may not be accurate for this patient.

Which information will the nurse include when teaching a 50-year-old male patient about somatropin (Genotropin)? a. The medication will be needed for 3 to 6 months. b. Inject the medication subcutaneously every day. c. Blood glucose levels may decrease when taking the medication. d. Stop taking the medication if swelling of the hands or feet occurs.

ANS: B Somatropin is injected subcutaneously on a daily basis, preferably in the evening. The patient will need to continue on somatropin for life. If swelling or other common adverse effects occur, the health care provider should be notified. Growth hormone will increase blood glucose levels

A patient who is being treated for stage IV lung cancer tells the nurse about new-onset back pain. Which action should the nurse take first? a. Give the patient the prescribed PRN opioid. b. Assess for sensation and strength in the legs. c. Notify the health care provider about the symptoms. d. Teach the patient how to use relaxation to reduce pain.

ANS: B Spinal cord compression, an oncologic emergency, can occur with invasion of tumor into the epidural space. The nurse will need to assess the patient further for symptoms such as decreased leg sensation and strength and then notify the health care provider. Administration of opioids or use of relaxation may be appropriate but only after the nurse has assessed for possible spinal cord compression

A serum potassium level of 3.2 mEq/L (3.2 mmol/L) is reported for a patient with cirrhosis who has scheduled doses of spironolactone (Aldactone) and furosemide (Lasix). due. Which action should the nurse take? a. Administer both drugs. b. Administer the spironolactone. c. Withhold the spironolactone and administer the furosemide. d. Withhold both drugs until discussed with the health care provider.

ANS: B Spironolactone is a potassium-sparing diuretic and will help increase the patient's potassium level. The nurse does not need to talk with the doctor before giving the spironolactone, although the health care provider should be notified about the low potassium value. The furosemide will further decrease the patient's potassium level and should be held until the nurse talks with the health care provider.

Which intervention will the nurse include in the plan of care for a 52-year-old male patient with syndrome of inappropriate antidiuretic hormone (SIADH)? a. Monitor for peripheral edema. b. Offer patient hard candies to suck on. c. Encourage fluids to 2 to 3 liters per day. d. Keep head of bed elevated to 30 degrees.

ANS: B Sucking on hard candies decreases thirst for a patient on fluid restriction. Patients with SIADH are on fluid restrictions of 800 to 1000 mL/day. Peripheral edema is not seen with SIADH. The head of the bed is elevated no more than 10 degrees to increase left atrial filling pressure and decrease antidiuretic hormone (ADH) release

Which assessment finding in a patient who is admitted with infective endocarditis (IE) is most important to communicate to the health care provider? a. Generalized muscle aching c. Janeway's lesions on the palms b. Sudden onset right flank pain d. Temperature 100.7°F (38.1°C)

ANS: B Sudden onset of flank pain indicates possible embolization to the kidney and may require diagnostic testing such as a renal arteriogram and interventions to improve renal perfusion. The other findings are typically found in IE but do not require any new interventions. DIF: Cognitive Level: Analyze (analysis)

11. Which patient statement indicates that the nurse's teaching about sulfasalazine (Azulfidine) for ulcerative colitis has been effective? a. "The medication will be tapered if I need surgery." b. "I will need to use a sunscreen when I am outdoors." c. "I will need to avoid contact with people who are sick." d. "The medication will prevent infections that cause the diarrhea."

ANS: B Sulfasalazine may cause photosensitivity in some patients. It is not used to treat infections. Sulfasalazine does not reduce immune function. Unlike corticosteroids, tapering of sulfasalazine is not needed.

The nurse administers an IV vesicant chemotherapeutic agent to a patient. Which action is most important for the nurse to take? a. Infuse the medication over a short period of time. b. Stop the infusion if swelling is observed at the site. c. Administer the chemotherapy through a small-bore catheter. d. Hold the medication unless a central venous line is available.

ANS: B Swelling at the site may indicate extravasation, and the IV should be stopped immediately. The medication generally should be given slowly to avoid irritation of the vein. The size of the catheter is not as important as administration of vesicants into a running IV line to allow dilution of the chemotherapeutic drug. These medications can be given through peripheral lines, although central vascular access devices (CVADs) are preferred

Which assessment information will be most important for the nurse to report to the health care provider about a patient with acute cholecystitis? a. The patient's urine is bright yellow. b. The patient's stools are tan colored. c. The patient has increased pain after eating. d. The patient complains of chronic heartburn.

ANS: B Tan or grey stools indicate biliary obstruction, which requires rapid intervention to resolve. The other data are not unusual for a patient with this diagnosis, although the nurse would also report the other assessment information to the health care provider

The nurse teaches a patient about the transmission of pulmonary tuberculosis (TB). Which statement, if made by the patient, indicates that teaching was effective? a. "I will avoid being outdoors whenever possible." b. "My husband will be sleeping in the guest bedroom." c. "I will take the bus instead of driving to visit my friends." d. "I will keep the windows closed at home to contain the germs."

ANS: B Teach the patient how to minimize exposure to close contacts and household members. Homes should be well ventilated, especially the areas where the infected person spends a lot of time. While still infectious, the patient should sleep alone, spend as much time as possible outdoors, and minimize time in congregate settings or on public transportation

A patient who is unconscious after a fall from a ladder is transported to the emergency department by emergency medical personnel. During the primary survey of the patient, the nurse should a. obtain a complete set of vital signs. b. obtain a Glasgow Coma Scale score. c. attach an electrocardiogram monitor. d. ask about chronic medical conditions.

ANS: B The Glasgow Coma Scale is included when assessing for disability during the primary survey. The other information is part of the secondary survey.

The nurse will plan to monitor a patient diagnosed with a pheochromocytoma for a. flushing. b. headache. c. bradycardia. d. hypoglycemia.

ANS: B The classic clinical manifestations of pheochromocytoma are hypertension, tachycardia, severe headache, diaphoresis, and abdominal or chest pain. Elevated blood glucose may also occur because of sympathetic nervous system stimulation. Bradycardia and flushing would not be expected

5. a 48 year old patient with stage 2 chronic kidney disease is scheduled for an intravenous pyelogram. which order for the patient will the nurse question? a. NPO for 6 hours before procedure b. Ibuprofen (Advil) 400 mg PO RN for pain c. Dulcolax suppository 4 hours before procedure d. Nurmal saline 500 mL IV infused before procedure

ANS: B The contrast dye used in IVPs is potentially nephrotoxin, and concurrent use of other nephrotoxic medications such as the nonsteroidal anti-inflammatory drugs (NSAIDS) should be avoided. The suppository and NPO status are necessary to ensure adequate visualization during the IVP. IV fluids are used to ensure adequate hydration, which helps reduce the risk for contrast-induced renal failure

31. The nurse is assessing a patient 4 hours after a kidney transplant. Which information is most important to communicate to the health care provider? a. The urine output is 900 to 1100 mL/hr. b. The patient's central venous pressure (CVP) is decreased. c. The patient has a level 7 (0- to 10-point scale) incisional pain. d. The blood urea nitrogen (BUN) and creatinine levels are elevated.

ANS: B The decrease in CVP suggests hypovolemia, which must be rapidly corrected to prevent renal hypoperfusion and acute tubular necrosis. The other information is not unusual in a patient after a transplant. DIF: Cognitive Level: Analyze (analysis) REF: 1095 OBJ: Special Questions: Prioritization TOP: Nursing Process: Assessment MSC: NCLEX: Physiological Integrity

Which topic is most important to include in patient teaching for a 41-year-old patient diagnosed with early alcoholic cirrhosis? a. Maintaining good nutrition b. Avoiding alcohol ingestion c. Taking lactulose (Cephulac) d. Using vitamin B supplements

ANS: B The disease progression can be stopped or reversed by alcohol abstinence. The other interventions may be used when cirrhosis becomes more severe to decrease symptoms or complications, but the priority for this patient is to stop the progression of the disease

A patient is admitted to the emergency department with an open stab wound to the left chest. What is the first action that the nurse should take? a. Position the patient so that the left chest is dependent. b. Tape a nonporous dressing on three sides over the chest wound. c. Cover the sucking chest wound firmly with an occlusive dressing. d. Keep the head of the patient's bed at no more than 30 degrees elevation.

ANS: B The dressing taped on three sides will allow air to escape when intrapleural pressure increases during expiration, but it will prevent air from moving into the pleural space during inspiration. Placing the patient on the left side or covering the chest wound with an occlusive dressing will allow trapped air in the pleural space and cause tension pneumothorax. The head of the bed should be elevated to 30 to 45 degrees to facilitate breathing

After 2 months of tuberculosis (TB) treatment with isoniazid (INH), rifampin (Rifadin), pyrazinamide (PZA), and ethambutol, a patient continues to have positive sputum smears for acid-fast bacilli (AFB). Which action should the nurse take next? a. Teach about treatment for drug-resistant TB treatment. b. Ask the patient whether medications have been taken as directed. c. Schedule the patient for directly observed therapy three times weekly. d. Discuss with the health care provider the need for the patient to use an injectable antibiotic.

ANS: B The first action should be to determine whether the patient has been compliant with drug therapy because negative sputum smears would be expected if the TB bacillus is susceptible to the medications and if the medications have been taken correctly. Assessment is the first step in the nursing process. Depending on whether the patient has been compliant or not, different medications or directly observed therapy may be indicated. The other options are interventions based on assumptions until an assessment has been completed

13. When titrating IV nitroglycerin (Tridil) for a patient with a myocardial infarction (MI), which action will the nurse take to evaluate the effectiveness of the medication? a. Monitor heart rate. b. Ask about chest pain. c. Check blood pressure. d. Observe for dysrhythmias.

ANS: B The goal of IV nitroglycerin administration in MI is relief of chest pain by improving the balance between myocardial oxygen supply and demand. The nurse also will monitor heart rate and blood pressure (BP) and observe for dysrhythmias, but these parameters will not indicate whether the medication is effective.

25. A 62-yr-old female patient has been hospitalized for 4 days with acute kidney injury (AKI) caused by dehydration. Which information will be most important for the nurse to report to the health care provider? a. The creatinine level is 3.0 mg/dL. b. Urine output over an 8-hour period is 2500 mL. c. The blood urea nitrogen (BUN) level is 67 mg/dL. d. The glomerular filtration rate is less than 30 mL/min/1.73 m2.

ANS: B The high urine output indicates a need to increase fluid intake to prevent hypovolemia. The other information is typical of AKI and will not require a change in therapy. DIF: Cognitive Level: Analyze (analysis) REF: 1072 OBJ: Special Questions: Prioritization TOP: Nursing Process: Assessment MSC: NCLEX: Physiological Integrity

Which goal has the highest priority in the plan of care for a 26-year-old homeless patient admitted with viral hepatitis who has severe anorexia and fatigue? a. Increase activity level. b. Maintain adequate nutrition. c. Establish a stable environment. d. Identify sources of hepatitis exposure.

ANS: B The highest priority outcome is to maintain nutrition because adequate nutrition is needed for hepatocyte regeneration. Finding a home for the patient and identifying the source of the infection would be appropriate activities, but they do not have as high a priority as ensuring adequate nutrition. Although the patient's activity level will be gradually increased, rest is indicated during the acute phase of hepatitis

40. Which patient at the cardiovascular clinic requires the most immediate action by the nurse? a. Patient with type 2 diabetes whose current blood glucose level is 145 mg/dL b. Patient with stable angina whose chest pain has recently increased in frequency c. Patient with familial hypercholesterolemia and a total cholesterol of 465 mg/dL d. Patient with chronic hypertension whose blood pressure today is 172/98 mm Hg

ANS: B The history of more frequent chest pain suggests that the patient may have unstable angina, which is part of the acute coronary syndrome spectrum. This will require rapid implementation of actions such as cardiac catheterization and possible percutaneous coronary intervention. The data about the other patients suggest that their conditions are stable.

23. A 25-yr-old male patient has been admitted with a severe crushing injury after an industrial accident. Which laboratory result will be most important to report to the health care provider? a. Serum creatinine level of 2.1 mg/dL b. Serum potassium level of 6.5 mEq/L c. White blood cell count of 11,500/µL d. Blood urea nitrogen (BUN) of 56 mg/dL

ANS: B The hyperkalemia associated with crushing injuries may cause cardiac arrest and should be treated immediately. The nurse also will report the other laboratory values, but abnormalities in these are not immediately life threatening. DIF: Cognitive Level: Analyze (analysis) REF: 1072 OBJ: Special Questions: Prioritization TOP: Nursing Process: Assessment MSC: NCLEX: Physiological Integrity

A patient who is anxious and has difficulty breathing seeks treatment after being stung by a wasp. What is the nurse's priority action? a. Have the patient lie down. b. Assess the patient's airway. c. Administer high-flow oxygen. d. Remove the stinger from the site.

ANS: B The initial action with any patient with difficulty breathing is to assess and maintain the airway. The other actions also are part of the emergency management protocol for anaphylaxis, but the priority is airway maintenance.

24. The nurse will determine that teaching a 67-year-old man to irrigate his new colostomy has been effective if the patient a. inserts the irrigation tubing 4 to 6 inches into the stoma. b. hangs the irrigating container 18 inches above the stoma. c. stops the irrigation and removes the irrigating cone if cramping occurs. d. fills the irrigating container with 1000 to 2000 mL of lukewarm tap water.

ANS: B The irrigating container should be hung 18 to 24 inches above the stoma. If cramping occurs, the irrigation should be temporarily stopped and the cone left in place. Five hundred to 1000 mL of water should be used for irrigation. An irrigation cone, rather than tubing, should be inserted into the stoma; 4 to 6 inches would be too far for safe insertion.

The nurse monitors a patient after chest tube placement for a hemopneumothorax. The nurse is most concerned if which assessment finding is observed? a. A large air leak in the water-seal chamber b. 400 mL of blood in the collection chamber c. Complaint of pain with each deep inspiration d. Subcutaneous emphysema at the insertion site

ANS: B The large amount of blood may indicate that the patient is in danger of developing hypovolemic shock. An air leak would be expected immediately after chest tube placement for a pneumothorax. Initially, brisk bubbling of air occurs in this chamber when a pneumothorax is evacuated. The pain should be treated but is not as urgent a concern as the possibility of continued hemorrhage. Subcutaneous emphysema should be monitored but is not unusual in a patient with pneumothorax. A small amount of subcutaneous air is harmless and will be reabsorbed

A 53-year-old patient is being treated for bleeding esophageal varices with balloon tamponade. Which nursing action will be included in the plan of care? a. Instruct the patient to cough every hour. b. Monitor the patient for shortness of breath. c. Verify the position of the balloon every 4 hours. d. Deflate the gastric balloon if the patient reports nausea.

ANS: B The most common complication of balloon tamponade is aspiration pneumonia. In addition, if the gastric balloon ruptures, the esophageal balloon may slip upward and occlude the airway. Coughing increases the pressure on the varices and increases the risk for bleeding. Balloon position is verified after insertion and does not require further verification. The esophageal balloon is deflated every 8 to 12 hours to avoid necrosis, but if the gastric balloon is deflated, the esophageal balloon may occlude the airway

A 37-year-old patient has just arrived in the postanesthesia recovery unit (PACU) after a thyroidectomy. Which information is most important to communicate to the surgeon? a. The patient reports 7/10 incisional pain. b. The patient has increasing neck swelling. c. The patient is sleepy and difficult to arouse. d. The patient's cardiac rate is 112 beats/minute.

ANS: B The neck swelling may lead to respiratory difficulty, and rapid intervention is needed to prevent airway obstruction. The incisional pain should be treated but is not unusual after surgery. A heart rate of 112 is not unusual in a patient who has been hyperthyroid and has just arrived in the PACU from surgery. Sleepiness in the immediate postoperative period is expected.

A 38-year-old male patient is admitted to the hospital in Addisonian crisis. Which patient statement supports a nursing diagnosis of ineffective self-health management related to lack of knowledge about management of Addison's disease? a. "I frequently eat at restaurants, and my food has a lot of added salt." b. "I had the stomach flu earlier this week, so I couldn't take the hydrocortisone." c. "I always double my dose of hydrocortisone on the days that I go for a long run." d. "I take twice as much hydrocortisone in the morning dose as I do in the afternoon."

ANS: B The need for hydrocortisone replacement is increased with stressors such as illness, and the patient needs to be taught to call the health care provider because medication and IV fluids and electrolytes may need to be given. The other patient statements indicate appropriate management of the Addison's disease.

35. A 74-yr-old patient who is progressing to stage 5 chronic kidney disease asks the nurse, "Do you think I should go on dialysis? Which initial response by the nurse is best? a. "It depends on which type of dialysis you are considering." b. "Tell me more about what you are thinking regarding dialysis." c. "You are the only one who can make the decision about dialysis." d. "Many people your age use dialysis and have a good quality of life."

ANS: B The nurse should initially clarify the patient's concerns and questions about dialysis. The patient is the one responsible for the decision, and many people using dialysis do have good quality of life, but these responses block further assessment of the patient's concerns. Referring to which type of dialysis the patient might use only indirectly responds to the patient's question. DIF: Cognitive Level: Analyze (analysis) REF: 1091 OBJ: Special Questions: Prioritization TOP: Nursing Process: Assessment MSC: NCLEX: Psychosocial Integrity

When assessing an older patient admitted to the emergency department (ED) with a broken arm and facial bruises, the nurse observes several additional bruises in various stages of healing. Which statement or question by the nurse should be first? a. "You should not go home." b. "Do you feel safe at home?" c. "Would you like to see a social worker?" d. "I need to report my concerns to the police."

ANS: B The nurse's initial response should be to further assess the patient's situation. Telling the patient not to return home may be an option once further assessment is done. A social worker or police report may be appropriate once further assessment is completed.

20. Which action will the nurse take first when a patient is seen in the outpatient clinic with neck pain? a. Provide information about therapeutic neck exercises. b. Ask about numbness or tingling of the hands and arms. c. Suggest that the patient alternate the use of heat and cold to the neck to treat the pain. d. Teach about the use of nonsteroidal antiinflammatory drugs such as ibuprofen (Advil).

ANS: B The nurses initial action should be further assessment of the pain because cervical nerve root compression will require different treatment than musculoskeletal neck pain. The other actions may also be appropriate, depending on the assessment findings.

A patient with a head injury opens his eyes to verbal stimulation, curses when stimulated, and does not respond to a verbal command to move but attempts to push away a painful stimulus. The nurse records the patient's Glasgow Coma Scale score as a. 9. c. 13. b. 11. d. 15.

ANS: B The patient has scores of 3 for eye opening, 3 for best verbal response, and 5 for best motor response.

A chemotherapy drug that causes alopecia is prescribed for a patient. Which action should the nurse take to maintain the patient's self-esteem? a. Tell the patient to limit social contacts until regrowth of the hair occurs. b. Encourage the patient to purchase a wig or hat and wear it once hair loss begins. c. Teach the patient to gently wash hair with a mild shampoo to minimize hair loss. d. Inform the patient that hair usually grows back once the chemotherapy is complete.

ANS: B The patient is taught to anticipate hair loss and to be prepared with wigs, scarves, or hats. Limiting social contacts is not appropriate at a time when the patient is likely to need a good social support system. The damage occurs at the hair follicle and will occur regardless of gentle washing or use of a mild shampoo. The information that the hair will grow back is not immediately helpful in maintaining the patient's self-esteem

An adolescent patient seeks care in the emergency department after sharing needles for heroin injection with a friend who has hepatitis B. To provide immediate protection from infection, what medication will the nurse administer? a. Corticosteroids b. Gamma globulin c. Hepatitis B vaccine d. Fresh frozen plasma

ANS: B The patient should first receive antibodies for hepatitis B from injection of gamma globulin. The hepatitis B vaccination series should be started to provide active immunity. Fresh frozen plasma and corticosteroids will not be effective in preventing hepatitis B in the patient

Which nursing action will be included in the plan of care for a 55-year-old patient with Graves' disease who has exophthalmos? a. Place cold packs on the eyes to relieve pain and swelling. b. Elevate the head of the patient's bed to reduce periorbital fluid. c. Apply alternating eye patches to protect the corneas from irritation. d. Teach the patient to blink every few seconds to lubricate the corneas.

ANS: B The patient should sit upright as much as possible to promote fluid drainage from the periorbital area. With exophthalmos, the patient is unable to close the eyes completely to blink. Lubrication of the eyes, rather than eye patches, will protect the eyes from developing corneal scarring. The swelling of the eye is not caused by excessive blood flow to the eye, so cold packs will not be helpful

2. A 50-year-old patient is being discharged after a week of IV antibiotic therapy for acute osteomyelitis in the right leg. Which information will be included in the discharge teaching? a. How to apply warm packs to the leg to reduce pain b. How to monitor and care for the long-term IV catheter c. The need for daily aerobic exercise to help maintain muscle strength d. The reason for taking oral antibiotics for 7 to 10 days after discharge

ANS: B The patient will be on IV antibiotics for several months, and the patient will need to recognize signs of infection at the IV site and how to care for the catheter during daily activities such as bathing. IV antibiotics rather than oral antibiotics are used for acute osteomyelitis. Patients are instructed to avoid exercise and heat application because these will increase swelling and the risk for spreading infection.

22. Which intervention will be included in the plan of care for a patient with acute kidney injury (AKI) who has a temporary vascular access catheter in the left femoral vein? a. Start continuous pulse oximetry. b. Restrict physical activity to bed rest. c. Restrict the patient's oral protein intake. d. Discontinue the urethral retention catheter.

ANS: B The patient with a femoral vein catheter must be on bed rest to prevent trauma to the vein. Protein intake is likely to be increased when the patient is receiving dialysis. The retention catheter is likely to remain in place because accurate measurement of output will be needed. There is no indication that the patient needs continuous pulse oximetry. DIF: Cognitive Level: Apply (application) REF: 1088 TOP: Nursing Process: Planning MSC: NCLEX: Physiological Integrity

A patient who had a subtotal thyroidectomy earlier today develops laryngeal stridor and a cramp in the right hand upon returning to the surgical nursing unit. Which collaborative action will the nurse anticipate next? a. Suction the patient's airway. b. Administer IV calcium gluconate. c. Plan for emergency tracheostomy. d. Prepare for endotracheal intubation.

ANS: B The patient's clinical manifestations of stridor and cramping are consistent with tetany caused by hypocalcemia resulting from damage to the parathyroid glands during surgery. Endotracheal intubation or tracheostomy may be needed if the calcium does not resolve the stridor. Suctioning will not correct the stridor

32. During routine hemodialysis, a patient complains of nausea and dizziness. Which action should the nurse take first? a. Slow down the rate of dialysis. b. Check the blood pressure (BP). c. Review the hematocrit (Hct) level. d. Give prescribed PRN antiemetic drugs.

ANS: B The patient's complaints of nausea and dizziness suggest hypotension, so the initial action should be to check the BP. The other actions may also be appropriate based on the blood pressure obtained. DIF: Cognitive Level: Analyze (analysis) REF: 1090 OBJ: Special Questions: Prioritization TOP: Nursing Process: Implementation MSC: NCLEX: Physiological Integrity

A patient is admitted to the burn unit with burns to the head, face, and hands. Initially, wheezes are heard, but an hour later, the lung sounds are decreased and no wheezes are audible. What is the best action for the nurse to take? a. Encourage the patient to cough and auscultate the lungs again. b. Notify the health care provider and prepare for endotracheal intubation. c. Document the results and continue to monitor the patient's respiratory rate. d. Reposition the patient in high-Fowler's position and reassess breath sounds.

ANS: B The patient's history and clinical manifestations suggest airway edema and the health care provider should be notified immediately, so that intubation can be done rapidly. Placing the patient in a more upright position or having the patient cough will not address the problem of airway edema. Continuing to monitor is inappropriate because immediate action should occur

A patient's heart monitor shows a pattern of undulations of varying contours and amplitude with no measurable ECG pattern. The patient is unconscious, apneic, and pulseless. Which action should the nurse take first? a. Give epinephrine (Adrenalin) IV. b. Perform immediate defibrillation. c. Prepare for endotracheal intubation. d. Ventilate with a bag-valve-mask device.

ANS: B The patient's rhythm and assessment indicate ventricular fibrillation and cardiac arrest; the initial action should be to defibrillate. If a defibrillator is not immediately available or is unsuccessful in converting the patient to a better rhythm, begin chest compressions. The other actions may also be appropriate but not first. DIF: Cognitive Level: Analyze (analysis)

16. Following laminectomy with a spinal fusion to treat a herniated disc, a patient reports numbness and tingling of the right lower leg. The first action that the nurse should take is to a. report the patients complaint to the surgeon. b. check the chart for preoperative assessment data. c. check the vital signs for indications of hemorrhage. d. turn the patient to the side to relieve pressure on the right leg.

ANS: B The postoperative movement and sensation of the extremities should be unchanged (or improved) from the preoperative assessment. If the numbness and tingling are new, this information should be immediately reported to the surgeon. Numbness and tingling are not symptoms associated with hemorrhage at the site. Turning the patient will not relieve the numbness.

3. The nurse is planning care for a patient with severe heart failure who has developed elevated blood urea nitrogen (BUN) and creatinine levels. The primary treatment goal in the plan will be a. augmenting fluid volume. b. maintaining cardiac output. c. diluting nephrotoxic substances. d. preventing systemic hypertension.

ANS: B The primary goal of treatment for acute kidney injury (AKI) is to eliminate the cause and provide supportive care while the kidneys recover. Because this patient's heart failure is causing AKI, the care will be directed toward treatment of the heart failure. For renal failure caused by hypertension, hypovolemia, or nephrotoxins, the other responses would be correct. DIF: Cognitive Level: Apply (application) REF: 1073 TOP: Nursing Process: Planning MSC: NCLEX: Physiological Integrity

A nurse is caring for a patient who has burns of the ears, head, neck, and right arm and hand. The nurse should place the patient in which position? a. Place the right arm and hand flexed in a position of comfort. b. Elevate the right arm and hand on pillows and extend the fingers. c. Assist the patient to a supine position with a small pillow under the head. d. Position the patient in a side-lying position with rolled towel under the neck.

ANS: B The right hand and arm should be elevated to reduce swelling and the fingers extended to avoid flexion contractures (even though this position may not be comfortable for the patient). The patient with burns of the ears should not use a pillow for the head because this will put pressure on the ears, and the pillow may stick to the ears. Patients with neck burns should not use a pillow because the head should be maintained in an extended position in order to avoid contractures

The nurse develops a plan of care to prevent aspiration in a high-risk patient. Which nursing action will be most effective? a. Turn and reposition immobile patients at least every 2 hours. b. Place patients with altered consciousness in side-lying positions. c. Monitor for respiratory symptoms in patients who are immunosuppressed. d. Insert nasogastric tube for feedings for patients with swallowing problems.

ANS: B The risk for aspiration is decreased when patients with a decreased level of consciousness are placed in a side-lying or upright position. Frequent turning prevents pooling of secretions in immobilized patients but will not decrease the risk for aspiration in patients at risk. Monitoring of parameters such as breath sounds and oxygen saturation will help detect pneumonia in immunocompromised patients, but it will not decrease the risk for aspiration. Conditions that increase the risk of aspiration include decreased level of consciousness (e.g., seizure, anesthesia, head injury, stroke, alcohol intake), difficulty swallowing, and nasogastric intubation with or without tube feeding. With loss of consciousness, the gag and cough reflexes are depressed, and aspiration is more likely to occur. Other high-risk groups are those who are seriously ill, have poor dentition, or are receiving acid-reducing medications

A young adult patient who is in the rehabilitation phase 6 months after a severe face and neck burn tells the nurse, "I'm sorry that I'm still alive. My life will never be normal again." Which response by the nurse is best? a. "Most people recover after a burn and feel satisfied with their lives." b. "It's true that your life may be different. What concerns you the most?" c. "It is really too early to know how much your life will be changed by the burn." d. "Why do you feel that way? You will be able to adapt as your recovery progresses."

ANS: B This response acknowledges the patient's feelings and asks for more assessment data that will help in developing an appropriate plan of care to assist the patient with the emotional response to the burn injury. The other statements are accurate, but do not acknowledge the anxiety and depression that the patient is expressing

17. When administering alendronate (Fosamax) to a patient with osteoporosis, the nurse will a. ask about any leg cramps or hot flashes. b. assist the patient to sit up at the bedside. c. be sure that the patient has recently eaten. d. administer the ordered calcium carbonate.

ANS: B To avoid esophageal erosions, the patient taking bisphosphonates should be upright for at least 30 minutes after taking the medication. Fosamax should be taken on an empty stomach, not after taking other medications or eating. Leg cramps and hot flashes are not side effects of bisphosphonates.

Which admission order written by the health care provider for a patient admitted with infective endocarditis (IE) and a fever would be a priority for the nurse to implement? a. Administer ceftriaxone 1 g IV. b. Order blood cultures drawn from two sites. c. Give acetaminophen (Tylenol) PRN for fever. d. Arrange for a transesophageal echocardiogram.

ANS: B Treatment of the IE with antibiotics should be started as quickly as possible, but it is essential to obtain blood cultures before starting antibiotic therapy to obtain accurate sensitivity results. The echocardiogram and acetaminophen administration also should be implemented rapidly, but the blood cultures (and then administration of the antibiotic) have the highest priority. DIF: Cognitive Level: Analyze (analysis)

Which nursing action can the registered nurse (RN) delegate to experienced unlicensed assistive personnel (UAP) working as telemetry technicians on the cardiac care unit? a. Decide whether a patient's heart rate of 116 requires urgent treatment. b. Observe heart rhythms for multiple patients who have telemetry monitoring. c. Monitor a patient's level of consciousness during synchronized cardioversion. d. Select the best lead for monitoring a patient admitted with acute coronary syndrome.

ANS: B UAP serving as telemetry technicians can monitor heart rhythms for individuals or groups of patients. Nursing actions such as assessment and choice of the most appropriate lead based on ST segment elevation location require RN-level education and scope of practice. DIF: Cognitive Level: Apply (application)

A patient with Hodgkin's lymphoma who is undergoing external radiation therapy tells the nurse, "I am so tired I can hardly get out of bed in the morning." Which intervention should the nurse add to the plan of care? a. Minimize activity until the treatment is completed. b. Establish time to take a short walk almost every day. c. Consult with a psychiatrist for treatment of depression. d. Arrange for delivery of a hospital bed to the patient's home.

ANS: B Walking programs are used to keep the patient active without excessive fatigue. Having a hospital bed does not necessarily address the fatigue. The better option is to stay as active as possible while combating fatigue. Fatigue is expected during treatment and is not an indication of depression. Minimizing activity may lead to weakness and other complications of immobility

Which action should the nurse perform when preparing a patient with supraventricular tachycardia for cardioversion who is alert and has a blood pressure of 110/66 mm Hg? a. Turn the synchronizer switch to the "off" position. b. Give a sedative before cardioversion is implemented. c. Set the defibrillator/cardioverter energy to 360 joules. d. Provide assisted ventilations with a bag-valve-mask device.

ANS: B When a patient has a nonemergency cardioversion, sedation is used just before the procedure. The synchronizer switch is turned "on" for cardioversion. The initial level of joules for cardioversion is low (e.g., 50). Assisted ventilations are not indicated for this patient. DIF: Cognitive Level: Apply (application)

When counseling a couple in which the man has an autosomal recessive disorder and the woman has no gene for the disorder, the nurse uses Punnett squares to show the couple the probability of their having a child with the disorder. Which statement by the nurse is most appropriate? a. "You should consider adoption." b. "Your children will be carriers of the disorder." c. "Your female children will display characteristics of the disorder." d. "Your first-born child will likely display characteristics of the disorder."

ANS: B When one parent has an autosomal recessive disorder and the other parent has no genes for the autosomal recessive disorder, the children will not display characteristics of the disorder. However, the children will be carriers of the autosomal recessive disorder

22. When evaluating the effectiveness of preoperative teaching with a patient scheduled for coronary artery bypass graft (CABG) surgery using the internal mammary artery, the nurse determines that additional teaching is needed when the patient says which of the following? a. "They will circulate my blood with a machine during the surgery." b. "I will have small incisions in my leg where they will remove the vein." c. "They will use an artery near my heart to go around the area that is blocked." d. "I will need to take an aspirin every day after the surgery to keep the graft open."

ANS: B When the internal mammary artery is used there is no need to have a saphenous vein removed from the leg. The other statements by the patient are accurate and indicate that the teaching has been effective.

A patient with a pleural effusion is scheduled for a thoracentesis. Which action should the nurse take to prepare the patient for the procedure? a. Start a peripheral IV line to administer the necessary sedative drugs. b. Position the patient sitting upright on the edge of the bed and leaning forward. c. Obtain a large collection device to hold 2 to 3 liters of pleural fluid at one time. d. Remove the water pitcher and remind the patient not to eat or drink anything for 6 hours.

ANS: B When the patient is sitting up, fluid accumulates in the pleural space at the lung bases and can more easily be located and removed. The patient does not usually require sedation for the procedure, and there are no restrictions on oral intake because the patient is not sedated or unconscious. Usually only 1000 to 1200 mL of pleural fluid is removed at one time. Rapid removal of a large volume can result in hypotension, hypoxemia, or pulmonary edema

13. A patient who has had progressive chronic kidney disease (CKD) for several years has just begun regular hemodialysis. Which information about diet will the nurse include in patient teaching? a. Increased calories are needed because glucose is lost during hemodialysis. b. More protein is allowed because urea and creatinine are removed by dialysis. c. Dietary potassium is not restricted because the level is normalized by dialysis. d. Unlimited fluids are allowed because retained fluid is removed during dialysis.

ANS: B When the patient is started on dialysis and nitrogenous wastes are removed, more protein in the diet is encouraged. Fluids are still restricted to avoid excessive weight gain and complications such as shortness of breath. Glucose is not lost during hemodialysis. Sodium and potassium intake continues to be restricted to avoid the complications associated with high levels of these electrolytes. DIF: Cognitive Level: Apply (application) REF: 1087 TOP: Nursing Process: Implementation MSC: NCLEX: Physiological Integrity

A 56-year-old patient who is disoriented and reports a headache and muscle cramps is hospitalized with possible syndrome of inappropriate antidiuretic hormone (SIADH). The nurse would expect the initial laboratory results to include a(n) a. elevated hematocrit. b. decreased serum sodium. c. low urine specific gravity. d. increased serum chloride.

ANS: B When water is retained, the serum sodium level will drop below normal, causing the clinical manifestations reported by the patient. The hematocrit will decrease because of the dilution caused by water retention. Urine will be more concentrated with a higher specific gravity. The serum chloride level will usually decrease along with the sodium level

When assessing a patient who spilled hot oil on the right leg and foot, the nurse notes that the skin is dry, pale, hard skin. The patient states that the burn is not painful. What term would the nurse use to document the burn depth? a. First-degree skin destruction b. Full-thickness skin destruction c. Deep partial-thickness skin destruction d. Superficial partial-thickness skin destruction

ANS: B With full-thickness skin destruction, the appearance is pale and dry or leathery and the area is painless because of the associated nerve destruction. Erythema, swelling, and blisters point to a deep partial-thickness burn. With superficial partial-thickness burns, the area is red, but no blisters are present. First-degree burns exhibit erythema, blanching, and pain.

The nurse teaches a patient with cancer of the liver about high-protein, high-calorie diet choices. Which snack choice by the patient indicates that the teaching has been effective? a. Lime sherbet b. Blueberry yogurt c. Cream cheese bagel d. Fresh strawberries and bananas

ANS: B Yogurt has high biologic value because of the protein and fat content. Fruit salad does not have high amounts of protein or fat. Lime sherbet is lower in fat and protein than yogurt. Cream cheese is low in protein

After surgery for an abdominal aortic aneurysm, a patient's central venous pressure (CVP) monitor indicates low pressures. Which action should the nurse take? a. Administer IV diuretic medications. b. Increase the IV fluid infusion per protocol. c. Increase the infusion rate of IV vasodilators. d. Elevate the head of the patient's bed to 45 degrees.

ANS: B A low CVP indicates hypovolemia and a need for an increase in the infusion rate. Diuretic administration will contribute to hypovolemia and elevation of the head or increasing vasodilators may decrease cerebral perfusion.

A patient who has been in the intensive care unit for 4 days has disturbed sensory perception from sleep deprivation. Which action should the nurse include in the plan of care? a. Administer prescribed sedatives or opioids at bedtime to promote sleep. b. Cluster nursing activities so that the patient has uninterrupted rest periods. c. Silence the alarms on the cardiac monitors to allow 30- to 40-minute naps. d. Eliminate assessments between 2200 and 0600 to allow uninterrupted sleep.

ANS: B Clustering nursing activities and providing uninterrupted rest periods will minimize sleep-cycle disruption. Sedative and opioid medications tend to decrease the amount of rapid eye movement (REM) sleep and can contribute to sleep disturbance and disturbed sensory perception. Silencing the alarms on the cardiac monitors would be unsafe in a critically ill patient, as would discontinuing all assessments during the night.

The central venous oxygen saturation (ScvO2) is decreasing in a patient who has severe pancreatitis. To determine the possible cause of the decreased ScvO2, the nurse assesses the patient's a. lipase level. b. temperature. c. urinary output. d. body mass index.

ANS: B Elevated temperature increases metabolic demands and O2 use by tissues, resulting in a drop in O2 saturation of central venous blood. Information about the patient's body mass index, urinary output, and lipase will not help in determining the cause of the patient's drop in ScvO2.

To verify the correct placement of an oral endotracheal tube (ET) after insertion, the best initial action by the nurse is to a. obtain a portable chest x-ray. b. use an end-tidal CO2 monitor. c. auscultate for bilateral breath sounds. d. observe for symmetrical chest movement.

ANS: B End-tidal CO2 monitors are currently recommended for rapid verification of ET placement. Auscultation for bilateral breath sounds and checking chest expansion are also used, but they are not as accurate as end-tidal CO2 monitoring. A chest x-ray confirms the placement but is done after the tube is secured.

The intensive care unit (ICU) nurse educator determines that teaching a new staff nurse about arterial pressure monitoring has been effective when the nurse a. balances and calibrates the monitoring equipment every 2 hours. b. positions the zero-reference stopcock line level with the phlebostatic axis. c. ensures that the patient is supine with the head of the bed flat for all readings. d. rechecks the location of the phlebostatic axis with changes in the patient's position.

ANS: B For accurate measurement of pressures, the zero-reference level should be at the phlebostatic axis. There is no need to rebalance and recalibrate monitoring equipment every 2 hours. Accurate hemodynamic readings are possible with the patient's head raised to 45 degrees or in the prone position. The anatomic position of the phlebostatic axis does not change when patients are repositioned.

The nurse is caring for a patient who has an intraaortic balloon pump in place. Which action should be included in the plan of care? a. Avoid the use of anticoagulant medications. b. Measure the patient's urinary output every hour. c. Provide passive range of motion for all extremities. d. Position the patient supine with head flat at all times.

ANS: B Monitoring urine output will help determine whether the patient's cardiac output has improved and also help monitor for balloon displacement blocking the renal arteries. The head of the bed can be elevated up to 30 degrees. Heparin is used to prevent thrombus formation. Limited movement is allowed for the extremity with the balloon insertion site to prevent displacement of the balloon.

The nurse is caring for a patient who has an arterial catheter in the left radial artery for arterial pressure-based cardiac output (APCO) monitoring. Which information obtained by the nurse requires a report to the health care provider? a. The patient has a positive Allen test result. b. There is redness at the catheter insertion site. c. The mean arterial pressure (MAP) is 86 mm Hg. d. The dicrotic notch is visible in the arterial waveform.

ANS: B Redness at the catheter insertion site indicates possible infection. The Allen test is performed before arterial line insertion, and a positive test result indicates normal ulnar artery perfusion. A MAP of 86 mm Hg is normal, and the dicrotic notch is normally present on the arterial waveform.

The nurse educator is evaluating the performance of a new registered nurse (RN) who is providing care to a patient who is receiving mechanical ventilation with 15 cm H2O of peak end-expiratory pressure (PEEP). Which action indicates that the new RN is safe? a. The RN plans to suction the patient every 1 to 2 hours. b. The RN uses a closed-suction technique to suction the patient. c. The RN tapes the connection between the ventilator tubing and the ET. d. The RN changes the ventilator circuit tubing routinely every 48 hours.

ANS: B The closed-suction technique is used when patients require high levels of PEEP (>10 cm H2O) to prevent the loss of PEEP that occurs when disconnecting the patient from the ventilator. Suctioning should not be scheduled routinely, but it should be done only when patient assessment data indicate the need for suctioning. Taping connections between the ET and ventilator tubing would restrict the ability of the tubing to swivel in response to patient repositioning. Ventilator tubing changes increase the risk for ventilator-associated pneumonia and are not indicated routinely.

An 81-yr-old patient who has been in the intensive care unit (ICU) for a week is now stable and transfer to the progressive care unit is planned. On rounds, the nurse notices that the patient has new onset confusion. The nurse will plan to a. give PRN lorazepam (Ativan) and cancel the transfer. b. inform the receiving nurse and then transfer the patient. c. notify the health care provider and postpone the transfer. d. obtain an order for restraints as needed and transfer the patient.

ANS: B The patient's history and symptoms most likely indicate delirium associated with the sleep deprivation and sensory overload in the ICU environment. Informing the receiving nurse and transferring the patient is appropriate. Postponing the transfer is likely to prolong the delirium. Benzodiazepines and restraints contribute to delirium and agitation.

A 29-year-old woman with systemic lupus erythematosus has been prescribed 2 weeks of high-dose prednisone therapy. Which information about the prednisone is most important for the nurse to include? a. "Weigh yourself daily to monitor for weight gain caused by increased appetite." b. "A weight-bearing exercise program will help minimize the risk for osteoporosis." c. "The prednisone dose should be decreased gradually rather than stopped suddenly." d. "Call the health care provider if you experience mood alterations with the prednisone."

ANS: C Acute adrenal insufficiency may occur if exogenous corticosteroids are suddenly stopped. Mood alterations and weight gain are possible adverse effects of corticosteroid use, but these are not life-threatening effects. Osteoporosis occurs when patients take corticosteroids for longer periods.

The nurse teaches a patient who is scheduled for a prostate needle biopsy about the procedure. Which statement, if made by the patient, indicates that teaching was effective? a. "The biopsy will remove the cancer in my prostate gland." b. "The biopsy will determine how much longer I have to live." c. "The biopsy will help decide the treatment for my enlarged prostate." d. "The biopsy will indicate whether the cancer has spread to other organs."

ANS: C A biopsy is used to determine whether the prostate enlargement is benign or malignant, and determines the type of treatment that will be needed. A biopsy does not give information about metastasis, life expectancy, or the impact of cancer on the patient's life

19. The nurse is providing preoperative teaching for a 61-year-old man scheduled for an abdominal-perineal resection. Which information will the nurse include? a. Another surgery in 8 to 12 weeks will be used to create an ileal-anal reservoir. b. The patient will begin sitting in a chair at the bedside on the first postoperative day. c. The patient will drink polyethylene glycol lavage solution (GoLYTELY) preoperatively. d. IV antibiotics will be started at least 24 hours before surgery to reduce the bowel bacteria.

ANS: C A bowel-cleansing agent is used to empty the bowel before surgery to reduce the risk for infection. A permanent colostomy is created with this surgery. Sitting is contraindicated after an abdominal-perineal resection. Oral antibiotics (rather than IV antibiotics) are given to reduce colonic and rectal bacteria.

15. Following an acute myocardial infarction (AMI), a patient ambulates in the hospital hallway. When the nurse is evaluating the patient's response to the activity, which assessment data would indicate that the exercise level should be decreased? a. Blood pressure (BP) changes from 118/60 to 126/68 mm Hg. b. Oxygen saturation drops from 99% to 95%. c. Heart rate increases from 66 to 92 beats/minute. d. Respiratory rate goes from 14 to 20 breaths/minute.

ANS: C A change in heart rate of more than 20 beats over the resting heart rate indicates that the patient should stop and rest. The increases in BP and respiratory rate, and the slight decrease in oxygen saturation, are normal responses to exercise.

4. A 26-year-old woman is being evaluated for vomiting and abdominal pain. Which question from the nurse will be most useful in determining the cause of the patient's symptoms? a. "What type of foods do you eat?" b. "Is it possible that you are pregnant?" c. "Can you tell me more about the pain?" d. "What is your usual elimination pattern?"

ANS: C A complete description of the pain provides clues about the cause of the problem. Although the nurse should ask whether the patient is pregnant to determine whether the patient might have an ectopic pregnancy and before any radiology studies are done, this information is not the most useful in determining the cause of the pain. The usual diet and elimination patterns are less helpful in determining the reason for the patient's symptoms.

The nurse notes that a patient has incisional pain, a poor cough effort, and scattered rhonchi after a thoracotomy. Which action should the nurse take first? a. Assist the patient to sit upright in a chair. b. Splint the patient's chest during coughing. c. Medicate the patient with prescribed morphine. d. Observe the patient use the incentive spirometer.

ANS: C A major reason for atelectasis and poor airway clearance in patients after chest surgery is incisional pain (which increases with deep breathing and coughing). The first action by the nurse should be to medicate the patient to minimize incisional pain. The other actions are all appropriate ways to improve airway clearance but should be done after the morphine is given

17. A 38-yr-old patient who had a kidney transplant 8 years ago is receiving the immunosuppressants tacrolimus (Prograf), cyclosporine (Sandimmune), and prednisone . Which assessment data will be of most concern to the nurse? a. Skin is thin and fragile. b. Blood pressure is 150/92. c. A nontender axillary lump. d. Blood glucose is 144 mg/dL.

ANS: C A nontender lump suggests a malignancy such as a lymphoma, which could occur as a result of chronic immunosuppressive therapy. The elevated glucose, skin change, and hypertension are possible side effects of the prednisone and should be addressed, but they are not as great a concern as the possibility of a malignancy. DIF: Cognitive Level: Analyze (analysis) REF: 1096 OBJ: Special Questions: Prioritization TOP: Nursing Process: Assessment

The nurse is caring for a patient following an adrenalectomy. The highest priority in the immediate postoperative period is to a. protect the patient's skin. b. monitor for signs of infection. c. balance fluids and electrolytes. d. prevent emotional disturbances.

ANS: C After adrenalectomy, the patient is at risk for circulatory instability caused by fluctuating hormone levels, and the focus of care is to assess and maintain fluid and electrolyte status through the use of IV fluids and corticosteroids. The other goals are also important for the patient but are not as immediately life threatening as the circulatory collapse that can occur with fluid and electrolyte disturbances

A patient with extensive electrical burn injuries is admitted to the emergency department. Which prescribed intervention should the nurse implement first? a. Assess oral temperature. b. Check a potassium level. c. Place on cardiac monitor. d. Assess for pain at contact points.

ANS: C After an electrical burn, the patient is at risk for fatal dysrhythmias and should be placed on a cardiac monitor. Assessing the oral temperature is not as important as assessing for cardiac dysrhythmias. Checking the potassium level is important. However, it will take time before the laboratory results are back. The first intervention is to place the patient on a cardiac monitor and assess for dysrhythmias, so that they can be treated if occurring. A decreased or increased potassium level will alert the nurse to the possibility of dysrhythmias. The cardiac monitor will alert the nurse immediately of any dysrhythmias. Assessing for pain is important, but the patient can endure pain until the cardiac monitor is attached. Cardiac dysrhythmias can be lethal

A patient has just arrived in the emergency department after an electrical burn from exposure to a high-voltage current. What is the priority nursing assessment? a. Oral temperature b. Peripheral pulses c. Extremity movement d. Pupil reaction to light

ANS: C All patients with electrical burns should be considered at risk for cervical spine injury, and assessments of extremity movement will provide baseline data. The other assessment data are also necessary but not as essential as determining the cervical spine status

Which teaching should the nurse provide about intradermal skin testing to a patient with possible allergies? a. "Do not eat anything for about 6 hours before the testing." b. "Take an oral antihistamine about an hour before the testing." c. "Plan to wait in the clinic for 20 to 30 minutes after the testing." d. "Reaction to the testing will take about 48 to 72 hours to occur."

ANS: C Allergic reactions usually occur within minutes after injection of an allergen, and the patient will be monitored for at least 20 minutes for anaphylactic reactions after the testing. Medications that might modify the response, such as antihistamines, should be avoided before allergy testing. There is no reason to be NPO for skin testing. Results with intradermal testing occur within minutes.

Family members are in the patient's room when the patient has a cardiac arrest and the staff start resuscitation measures. Which action should the nurse take next? a. Keep the family in the room and assign a staff member to explain the care given and answer questions. b. Ask the family to wait outside the patient's room with a designated staff member to provide emotional support. c. Ask the family members whether they would prefer to remain in the patient's room or wait outside the room. d. Tell the family members that patients are comforted by having family members present during resuscitation efforts.

ANS: C Although many family members and patients report benefits from family presence during resuscitation efforts, the nurse's initial action should be to determine the preference of these family members. The other actions may be appropriate, but this will depend on what is learned when assessing family preferences

The sister of a patient diagnosed with BRCA gene-related breast cancer asks the nurse, "Do you think I should be tested for the gene?" Which response by the nurse is most appropriate? a. "In most cases, breast cancer is not caused by the BRCA gene." b. "It depends on how you will feel if the test is positive for the BRCA gene." c. "There are many things to consider before deciding to have genetic testing." d. "You should decide first whether you are willing to have a bilateral mastectomy."

ANS: C Although presymptomatic testing for genetic disorders allows patients to take action (such as mastectomy) to prevent the development of some genetically caused disorders, patients also need to consider that test results in their medical record may affect insurance, employability, etc. Telling a patient that a decision about mastectomy should be made before testing implies that the nurse has made a judgment about what the patient should do if the test is positive. Although the patient may need to think about her reaction if the test is positive, other issues (e.g., insurance) also should be considered. Although most breast cancers are not related to BRCA gene mutations, the patient with a BRCA gene mutation has a markedly increased risk for breast cancer

The health care provider asks the nurse whether a patient's angioedema has responded to prescribed therapies. Which assessment should the nurse perform? a. Ask the patient about any clear nasal discharge. b. Obtain the patient's blood pressure and heart rate. c. Check for swelling of the patient's lips and tongue. d. Assess the patient's extremities for wheal and flare lesions.

ANS: C Angioedema is characterized by swelling of the eyelids, lips, and tongue. Wheal and flare lesions, clear nasal drainage, and hypotension and tachycardia are characteristic of other allergic reactions.

During discharge teaching with an older patient who had a mitral valve replacement with a mechanical valve, the nurse must instruct the patient on the a. use of daily aspirin for anticoagulation. b. correct method for taking the radial pulse. c. need for frequent laboratory blood testing. d. need to avoid any physical activity for 1 month.

ANS: C Anticoagulation with warfarin (Coumadin) is needed for a patient with mechanical valves to prevent clotting on the valve. This will require frequent international normalized ratio testing. Daily aspirin use will not be effective in reducing the risk for clots on the valve. Monitoring of the radial pulse is not necessary after valve replacement. Patients should resume activities of daily living as tolerated. DIF: Cognitive Level: Apply (application)

After a 22-year-old female patient with a pituitary adenoma has had a hypophysectomy, the nurse will teach about the need for a. sodium restriction to prevent fluid retention. b. insulin to maintain normal blood glucose levels. c. oral corticosteroids to replace endogenous cortisol. d. chemotherapy to prevent malignant tumor recurrence.

ANS: C Antidiuretic hormone (ADH), cortisol, and thyroid hormone replacement will be needed for life after hypophysectomy. Without the effects of adrenocorticotropic hormone (ACTH) and cortisol, the blood glucose and serum sodium will be low unless cortisol is replaced. An adenoma is a benign tumor, and chemotherapy will not be needed

Immediately after the nurse administers an intracutaneous injection of an allergen on the forearm, a patient complains of itching at the site and of weakness and dizziness. What action should the nurse take first? a. Remind the patient to remain calm. b. Administer subcutaneous epinephrine. c. Apply a tourniquet above the injection site. d. Rub a local antiinflammatory cream on the site.

ANS: C Application of a tourniquet will decrease systemic circulation of the allergen and should be the first reaction. A local antiinflammatory cream may be applied to the site of a cutaneous test if the itching persists. Epinephrine will be needed if the allergic reaction progresses to anaphylaxis. The nurse should assist the patient to remain calm, but this is not an adequate initial nursing action.

16. Which assessment finding may indicate that a patient is experiencing adverse effects to a corticosteroid prescribed after kidney transplantation? a. Postural hypotension b. Recurrent tachycardia c. Knee and hip joint pain d. Increased serum creatinine

ANS: C Aseptic necrosis of the weight-bearing joints can occur when patients take corticosteroids over a prolonged period. Increased creatinine level, orthostatic dizziness, and tachycardia are not caused by corticosteroid use. DIF: Cognitive Level: Apply (application) REF: 1096 TOP: Nursing Process: Evaluation MSC: NCLEX: Physiological Integrity

A patient who has deep human bite wounds on the left hand is being treated in the urgent care center. Which action will the nurse plan to take? a. Prepare to administer rabies immune globulin (BayRab). b. Assist the health care provider with suturing of the bite wounds. c. Teach the patient the reason for the use of prophylactic antibiotics. d. Keep the wounds dry until the health care provider can assess them.

ANS: C Because human bites of the hand frequently become infected, prophylactic antibiotics are usually prescribed to prevent infection. To minimize infection, deep bite wounds on the extremities are left open. Rabies immune globulin might be used after an animal bite. Initial treatment of bite wounds includes copious irrigation to help clean out contaminants and microorganisms.

The nurse receives change-of-shift report on the oncology unit. Which patient should the nurse assess first? a. 35-year-old patient who has wet desquamation associated with abdominal radiation b. 42-year-old patient who is sobbing after receiving a new diagnosis of ovarian cancer c. 24-year-old patient who received neck radiation and has blood oozing from the neck d. 56-year-old patient who developed a new pericardial friction rub after chest radiation

ANS: C Because neck bleeding may indicate possible carotid artery rupture in a patient who is receiving radiation to the neck, this patient should be seen first. The diagnoses and clinical manifestations for the other patients are not immediately life threatening

A 22-yr-old patient who experienced a drowning accident in a local pool, but now is awake and breathing spontaneously, is admitted for observation. Which assessment will be most important for the nurse to take during the observation period? a. Auscultate heart sounds. c. Auscultate breath sounds. b. Palpate peripheral pulses. d. Check mental orientation.

ANS: C Because pulmonary edema is a common complication after drowning, the nurse should assess the breath sounds frequently. The other information also will be obtained by the nurse, but it is not as pertinent to the patient's admission diagnosis.

13. Which diet choice by the patient with an acute exacerbation of inflammatory bowel disease (IBD) indicates a need for more teaching? a. Scrambled eggs b. White toast and jam c. Oatmeal with cream d. Pancakes with syrup

ANS: C During acute exacerbations of IBD, the patient should avoid high-fiber foods such as whole grains. High-fat foods also may cause diarrhea in some patients. The other choices are low residue and would be appropriate for this patient.

A patient who collects honey to earn supplemental income has developed a hypersensitivity to bee stings. Which statement, if made by the patient, would indicate a need for additional teaching? a. "I need to find another way to earn extra money." b. "I will get a prescription for epinephrine and learn to self-inject it." c. "I will plan to take oral antihistamines daily before going to work." d. "I should wear a Medic-Alert bracelet indicating my allergy to bee stings."

ANS: C Because the patient is at risk for bee stings and the severity of allergic reactions tends to increase with added exposure to allergen, taking oral antihistamines will not adequately control the patient's hypersensitivity reaction. The other patient statements indicate a good understanding of management of the problem

21. A 71-year-old patient had an abdominal-perineal resection for colon cancer. Which nursing action is most important to include in the plan of care for the day after surgery? a. Teach about a low-residue diet. b. Monitor output from the stoma. c. Assess the perineal drainage and incision. d. Encourage acceptance of the colostomy stoma.

ANS: C Because the perineal wound is at high risk for infection, the initial care is focused on assessment and care of this wound. Teaching about diet is best done closer to discharge from the hospital. There will be very little drainage into the colostomy until peristalsis returns. The patient will be encouraged to assist with the colostomy, but this is not the highest priority in the immediate postoperative period.

The nurse, who is reviewing a clinic patient's medical record, notes that the patient missed the previous appointment for weekly immunotherapy. Which action by the nurse is most appropriate? a. Schedule an additional dose that week. b. Administer the usual dosage of the allergen. c. Consult with the health care provider about giving a lower allergen dose. d. Re-evaluate the patient's sensitivity to the allergen with a repeat skin test.

ANS: C Because there is an increased risk for adverse reactions after a patient misses a scheduled dose of allergen, the nurse should check with the health care provider before administration of the injection. A skin test is used to identify the allergen and would not be used at this time. An additional dose for the week may increase the risk for a reaction.

29. A 62-year-old patient has had a hemorrhoidectomy at an outpatient surgical center. Which instructions will the nurse include in discharge teaching? a. Maintain a low-residue diet until the surgical area is healed. b. Use ice packs on the perianal area to relieve pain and swelling. c. Take prescribed pain medications before a bowel movement is expected. d. Delay having a bowel movement for several days until healing has occurred.

ANS: C Bowel movements may be very painful, and patients may avoid defecation unless pain medication is taken before the bowel movement. A high-residue diet will increase stool bulk and prevent constipation. Delay of bowel movements is likely to lead to constipation. Warm sitz baths rather than ice packs are used to relieve pain and keep the surgical area clean.

The nurse is caring for a 36-year-old patient with pancreatic cancer. Which nursing action is the highest priority? a. Offer psychologic support for depression. b. Offer high-calorie, high-protein dietary choices. c. Administer prescribed opioids to relieve pain as needed. d. Teach about the need to avoid scratching any pruritic areas.

ANS: C Effective pain management will be necessary in order for the patient to improve nutrition, be receptive to teaching, or manage anxiety or depression

A new mother expresses concern about her baby developing allergies and asks what the health care provider meant by "passive immunity." Which example should the nurse use to explain this type of immunity? a. Early immunization b. Bone marrow donation c. Breastfeeding her infant d. Exposure to communicable diseases

ANS: C Colostrum provides passive immunity through antibodies from the mother. These antibodies protect the infant for a few months. However, memory cells are not retained, so the protection is not permanent. Active immunity is acquired by being immunized with vaccinations or having an infection. It requires that the infant has an immune response after exposure to an antigen. Cell-mediated immunity is acquired through T lymphocytes and is a form of active immunity.

A patient who has a right-sided chest tube following a thoracotomy has continuous bubbling in the suction-control chamber of the collection device. Which action by the nurse is most appropriate? a. Document the presence of a large air leak. b. Notify the surgeon of a possible pneumothorax. c. Take no further action with the collection device. d. Adjust the dial on the wall regulator to decrease suction.

ANS: C Continuous bubbling is expected in the suction-control chamber and indicates that the suction-control chamber is connected to suction. An air leak would be detected in the water-seal chamber. There is no evidence of pneumothorax. Increasing or decreasing the vacuum source will not adjust the suction pressure. The amount of suction applied is regulated by the amount of water in this chamber and not by the amount of suction applied to the system

The nurse will plan discharge teaching about prophylactic antibiotics before dental procedures for which patient? a. Patient admitted with a large acute myocardial infarction b. Patient being discharged after an exacerbation of heart failure c. Patient who had a mitral valve replacement with a mechanical valve d. Patient being treated for rheumatic fever after a streptococcal infection

ANS: C Current American Heart Association guidelines recommend the use of prophylactic antibiotics before dental procedures for patients with prosthetic valves to prevent infective endocarditis (IE). The other patients are not at risk for IE. DIF: Cognitive Level: Apply (application)

A patient born in 1955 had hepatitis A infection 1 year ago. According to Centers for Disease Control and Prevention (CDC) guidelines, which action should the nurse include in care when the patient is seen for a routine annual physical exam? a. Start the hepatitis B immunization series. b. Teach the patient about hepatitis A immune globulin. c. Ask whether the patient has been screened for hepatitis C. d. Test for anti-hepatitis-A virus immune globulin M (anti-HAV-IgM).

ANS: C Current CDC guidelines indicate that all patients who were born between 1945 and 1965 should be screened for hepatitis C because many individuals who are positive have not been diagnosed. Although routine hepatitis B immunization is recommended for infants, children, and adolescents, vaccination for hepatitis B is recommended only for adults at risk for blood-borne infections. Because the patient has already had hepatitis A, immunization and anti-HAV IgM levels will not be needed.

40. Which activity in the care of a 48-year-old female patient with a new colostomy could the nurse delegate to unlicensed assistive personnel (UAP)? a. Document the appearance of the stoma. b. Place a pouching system over the ostomy. c. Drain and measure the output from the ostomy. d. Check the skin around the stoma for breakdown.

ANS: C Draining and measuring the output from the ostomy is included in UAP education and scope of practice. The other actions should be implemented by LPNs or RNs.

A patient is diagnosed with both human immunodeficiency virus (HIV) and active tuberculosis (TB) disease. Which information obtained by the nurse is most important to communicate to the health care provider? a. The Mantoux test had an induration of 7 mm. b. The chest-x-ray showed infiltrates in the lower lobes. c. The patient is being treated with antiretrovirals for HIV infection. d. The patient has a cough that is productive of blood-tinged mucus.

ANS: C Drug interactions can occur between the antiretrovirals used to treat HIV infection and the medications used to treat TB. The other data are expected in a patient with HIV and TB.

A patient with increased intracranial pressure after a head injury has a ventriculostomy in place. Which action can the nurse delegate to unlicensed assistive personnel (UAP) who regularly work in the intensive care unit? a. Document intracranial pressure every hour. b. Turn and reposition the patient every 2 hours. c. Check capillary blood glucose level every 6 hours. d. Monitor cerebrospinal fluid color and volume hourly.

ANS: C Experienced UAP can obtain capillary blood glucose levels when they have been trained and evaluated in the skill. Monitoring and documentation of cerebrospinal fluid (CSF) color and intracranial pressure (ICP) require registered nurse (RN)-level education and scope of practice. Although repositioning patients is frequently delegated to UAP, repositioning a patient with a ventriculostomy is complex and should be supervised by the RN.

3. A 67-year-old patient is receiving IV antibiotics at home to treat chronic osteomyelitis of the left femur. The nurse chooses a nursing diagnosis of ineffective health maintenance when the nurse finds that the patient a. is frustrated with the length of treatment required. b. takes and records the oral temperature twice a day. c. is unable to plantar flex the foot on the affected side. d. uses crutches to avoid weight bearing on the affected leg.

ANS: C Foot drop is an indication that the foot is not being supported in a neutral position by a splint. Using crutches and monitoring the oral temperature are appropriate self-care activities. Frustration with the length of treatment is not an indicator of ineffective health maintenance of the osteomyelitis.

A patient who has severe pain associated with terminal pancreatic cancer is being cared for at home by family members. Which finding by the nurse indicates that teaching regarding pain management has been effective? a. The patient uses the ordered opioid pain medication whenever the pain is greater than 5 (0 to 10 scale). b. The patient agrees to take the medications by the IV route in order to improve analgesic effectiveness. c. The patient takes opioids around the clock on a regular schedule and uses additional doses when breakthrough pain occurs. d. The patient states that nonopioid analgesics may be used when the maximal dose of the opioid is reached without adequate pain relief.

ANS: C For chronic cancer pain, analgesics should be taken on a scheduled basis, with additional doses as needed for breakthrough pain. Taking the medications only when pain reaches a certain level does not provide effective pain control. Although nonopioid analgesics also may be used, there is no maximum dose of opioid. Opioids are given until pain control is achieved. The IV route is not more effective than the oral route, and usually the oral route is preferred

The nurse provides preoperative instruction for a patient scheduled for a left pneumonectomy for cancer of the lung. Which information should the nurse include about the patient's postoperative care? a. Positioning on the right side b. Bed rest for the first 24 hours c. Frequent use of an incentive spirometer d. Chest tube placement with continuous drainage

ANS: C Frequent deep breathing and coughing are needed after chest surgery to prevent atelectasis. To promote gas exchange, patients after pneumonectomy are positioned on the surgical side. Early mobilization decreases the risk for postoperative complications such as pneumonia and deep vein thrombosis. In a pneumonectomy, chest tubes may or may not be placed in the space from which the lung was removed. If a chest tube is used, it is clamped and only released by the surgeon to adjust the volume of serosanguineous fluid that will fill the space vacated by the lung. If the cavity overfills, it could compress the remaining lung and compromise the cardiovascular and pulmonary function. Daily chest x-rays can be used to assess the volume and space

10. A 37-yr-old female patient is hospitalized with acute kidney injury (AKI). Which information will be most useful to the nurse in evaluating improvement in kidney function? a. Urine volume b. Creatinine level c. Glomerular filtration rate (GFR) d. Blood urea nitrogen (BUN) level

ANS: C GFR is the preferred method for evaluating kidney function. BUN levels can fluctuate based on factors such as fluid volume status and protein intake. Urine output can be normal or high in patients with AKI and does not accurately reflect kidney function. Creatinine alone is not an accurate reflection of renal function. DIF: Cognitive Level: Analyze (analysis) REF: 1079 TOP: Nursing Process: Evaluation MSC: NCLEX: Physiological Integrity

A patient with severe burns has crystalloid fluid replacement ordered using the Parkland formula. The initial volume of fluid to be administered in the first 24 hours is 30,000 mL. The initial rate of administration is 1875 mL/hr. After the first 8 hours, what rate should the nurse infuse the IV fluids? a. 350 mL/hour b. 523 mL/hour c. 938 mL/hour d. 1250 mL/hour

ANS: C Half of the fluid replacement using the Parkland formula is administered in the first 8 hours and the other half over the next 16 hours. In this case, the patient should receive half of the initial rate, or 938 mL/hr

12. Heparin is ordered for a patient with a non-ST-segment-elevation myocardial infarction (NSTEMI). What is the purpose of the heparin? a. Heparin enhances platelet aggregation. b. Heparin decreases coronary artery plaque size. c. Heparin prevents the development of new clots in the coronary arteries. d. Heparin dissolves clots that are blocking blood flow in the coronary arteries.

ANS: C Heparin helps prevent the conversion of fibrinogen to fibrin and decreases coronary artery thrombosis. It does not change coronary artery plaque, dissolve already formed clots, or enhance platelet aggregation.

21. A 55-yr-old patient with end-stage kidney disease (ESKD) is scheduled to receive a prescribed dose of epoetin alfa (Procrit). Which information should the nurse report to the health care provider before giving the medication? a. Creatinine 1.6 mg/dL b. Oxygen saturation 89% c. Hemoglobin level 13 g/dL d. Blood pressure 98/56 mm Hg

ANS: C High hemoglobin levels are associated with a higher rate of thromboembolic events and increased risk of death from serious cardiovascular events (heart attack, heart failure, stroke) when erythropoietin (EPO) is administered to a target hemoglobin of greater than 12 g/dL. Hemoglobin levels higher than 12 g/dL indicate a need for a decrease in epoetin alfa dose. The other information also will be reported to the health care provider but will not affect whether the medication is administered. DIF: Cognitive Level: Apply (application) REF: 1081 TOP: Nursing Process: Assessment MSC: NCLEX: Physiological Integrity

9. Before administration of calcium carbonate to a patient with chronic kidney disease (CKD), the nurse should check laboratory results for a. potassium level. b. total cholesterol. c. serum phosphate. d. serum creatinine.

ANS: C If serum phosphate is elevated, the calcium and phosphate can cause soft tissue calcification. Calcium carbonate should not be given until the phosphate level is lowered. Total cholesterol, creatinine, and potassium values do not affect whether calcium carbonate should be administered. DIF: Cognitive Level: Apply (application) REF: 1081 TOP: Nursing Process: Implementation MSC: NCLEX: Physiological Integrity

A 19-yr-old student comes to the student health center at the end of the semester complaining that, "My heart is skipping beats." An electrocardiogram (ECG) shows occasional unifocal premature ventricular contractions (PVCs). What action should the nurse take next? a. Insert an IV catheter for emergency use. b. Start supplemental O2 at 2 to 3 L/min via nasal cannula. c. Ask the patient about current stress level and caffeine use. d. Have the patient taken to the nearest emergency department (ED)

ANS: C In a patient with a normal heart, occasional PVCs are a benign finding. The timing of the PVCs suggests stress or caffeine as possible etiologic factors. The patient is hemodynamically stable, so there is no indication that the patient needs supplemental O2, an IV, or to be seen in the ED. DIF: Cognitive Level: Apply (application)

The nurse is caring for a patient who has been diagnosed with stage I cancer of the colon. When assessing the need for psychologic support, which question by the nurse will provide the most information? a. "How long ago were you diagnosed with this cancer?" b. "Do you have any concerns about body image changes?" c. "Can you tell me what has been helpful to you in the past when coping with stressful events?" d. "Are you familiar with the stages of emotional adjustment to a diagnosis like cancer of the colon?"

ANS: C Information about how the patient has coped with past stressful situations helps the nurse determine usual coping mechanisms and their effectiveness. The length of time since the diagnosis will not provide much information about the patient's need for support. The patient's knowledge of typical stages in adjustment to a critical diagnosis does not provide insight into patient needs for assistance. Because surgical interventions for stage I cancer of the colon may not cause any body image changes, this question is not appropriate at this time

When a brain-injured patient responds to nail bed pressure with internal rotation, adduction, and flexion of the arms, the nurse reports the response as a. flexion withdrawal. c. decorticate posturing. b. localization of pain. d. decerebrate posturing.

ANS: C Internal rotation, adduction, and flexion of the arms in an unconscious patient is documented as decorticate posturing. Extension of the arms and legs is decerebrate posturing. Because the flexion is generalized, it does not indicate localization of pain or flexion withdrawal.

2. Which nursing intervention will be most effective when assisting the patient with coronary artery disease (CAD) to make appropriate dietary changes? a. Give the patient a list of low-sodium, low-cholesterol foods that should be included in the diet. b. Emphasize the increased risk for heart problems unless the patient makes the dietary changes. c. Help the patient modify favorite high-fat recipes by using monosaturated oils when possible. d. Inform the patient that a diet containing no saturated fat and minimal salt will be necessary.

ANS: C Lifestyle changes are more likely to be successful when consideration is given to the patient's values and preferences. The highest percentage of calories from fat should come from monosaturated fats. Although low-sodium and low-cholesterol foods are appropriate, providing the patient with a list alone is not likely to be successful in making dietary changes. Completely removing saturated fat from the diet is not a realistic expectation. Up to 7% of calories in the therapeutic lifestyle changes (TLC) diet can come from saturated fat. Telling the patient about the increased risk without assisting further with strategies for dietary change is unlikely to be successful.

18. The nurse in the dialysis clinic is reviewing the home medications of a patient with chronic kidney disease (CKD). Which medication reported by the patient indicates that patient teaching is required? a. Acetaminophen b. Calcium phosphate c. Magnesium hydroxide d. Multivitamin with iron

ANS: C Magnesium is excreted by the kidneys, and patients with CKD should not use over-the-counter products containing magnesium. The other medications are appropriate for a patient with CKD. DIF: Cognitive Level: Apply (application) REF: 1081 TOP: Nursing Process: Assessment MSC: NCLEX: Physiological Integrity

Which information will the nurse teach a 48-year-old patient who has been newly diagnosed with Graves' disease? a. Exercise is contraindicated to avoid increasing metabolic rate. b. Restriction of iodine intake is needed to reduce thyroid activity. c. Antithyroid medications may take several months for full effect. d. Surgery will eventually be required to remove the thyroid gland.

ANS: C Medications used to block the synthesis of thyroid hormones may take 2 to 3 months before the full effect is seen. Large doses of iodine are used to inhibit the synthesis of thyroid hormones. Exercise using large muscle groups is encouraged to decrease the irritability and hyperactivity associated with high levels of thyroid hormones. Radioactive iodine is the most common treatment for Graves' disease although surgery may be used

When rewarming a patient who arrived in the emergency department (ED) with a temperature of 87° F (30.6° C), which finding indicates that the nurse should discontinue active rewarming? a. The patient begins to shiver. b. The BP decreases to 86/42 mm Hg. c. The patient develops atrial fibrillation. d. The core temperature is 94° F (34.4° C).

ANS: D A core temperature of at least 89.6° F to 93.2° F (32° C to 34° C) indicates that sufficient rewarming has occurred. Dysrhythmias, hypotension, and shivering may occur during rewarming, and should be treated but are not an indication to stop rewarming the patient.

A patient with hypotension and an elevated temperature after working outside on a hot day is treated in the emergency department (ED). The nurse determines that discharge teaching has been effective when the patient makes which statement? a. "I'll take salt tablets when I work outdoors in the summer." b. "I should take acetaminophen (Tylenol) if I start to feel too warm." c. "I need to drink extra fluids when working outside in hot weather." d. "I'll move to a cool environment if I notice that I'm feeling confused"

ANS: C Oral fluids and electrolyte replacement solutions such as sports drinks help replace fluid and electrolytes lost when exercising in hot weather. Salt tablets are not recommended because of the risks of gastric irritation and hypernatremia. Antipyretic drugs are not effective in lowering body temperature elevations caused by excessive exposure to heat. A patient who is confused is likely to have more severe hyperthermia and will be unable to remember to take appropriate action.

29. A licensed practical/vocational nurse (LPN/LVN) is caring for a patient with stage 2 chronic kidney disease. Which observation by the RN requires an intervention? a. The LPN/LVN administers the erythropoietin subcutaneously. b. The LPN/LVN assists the patient to ambulate out in the hallway. c. The LPN/LVN administers the iron supplement and phosphate binder with lunch. d. The LPN/LVN carries a tray containing low-protein foods into the patient's room.

ANS: C Oral phosphate binders should not be given at the same time as iron because they prevent the iron from being absorbed. The phosphate binder should be given with a meal and the iron given at a different time. The other actions by the LPN/LVN are appropriate for a patient with renal insufficiency. DIF: Cognitive Level: Apply (application) REF: 1082 OBJ: Special Questions: Delegation TOP: Nursing Process: Implementation MSC: NCLEX: Safe and Effective Care Environment

A patient who is taking rifampin (Rifadin) for tuberculosis calls the clinic and reports having orange discolored urine and tears. Which is the best response by the nurse? a. Ask if the patient is experiencing shortness of breath, hives, or itching. b. Ask the patient about any visual abnormalities such as red-green color discrimination. c. Explain that orange discolored urine and tears are normal while taking this medication. d. Advise the patient to stop the drug and report the symptoms to the health care provider.

ANS: C Orange-colored body secretions are a side effect of rifampin. The patient does not have to stop taking the medication. The findings are not indicative of an allergic reaction. Alterations in red-green color discrimination commonly occurs when taking ethambutol (Myambutol), which is a different TB medication

14. Which action by a patient who is using peritoneal dialysis (PD) indicates that the nurse should provide more teaching about PD? a. The patient leaves the catheter exit site without a dressing. b. The patient plans 30 to 60 minutes for a dialysate exchange. c. The patient cleans the catheter while taking a bath each day. d. The patient slows the inflow rate when experiencing abdominal pain.

ANS: C Patients are encouraged to take showers rather than baths to avoid infections at the catheter insertion side. The other patient actions indicate good understanding of peritoneal dialysis. DIF: Cognitive Level: Apply (application) REF: 1086 TOP: Nursing Process: Evaluation MSC: NCLEX: Physiological Integrity

8. Which menu choice by the patient who is receiving hemodialysis indicates that the nurse's teaching has been successful? a. Split-pea soup, English muffin, and nonfat milk b. Oatmeal with cream, half a banana, and herbal tea c. Poached eggs, whole-wheat toast, and apple juice d. Cheese sandwich, tomato soup, and cranberry juice

ANS: C Poached eggs would provide high-quality protein, and apple juice is low in potassium. Cheese is high in salt and phosphate, and tomato soup is high in potassium. Split-pea soup is high in potassium, and dairy products are high in phosphate. Bananas are high in potassium, and cream is high in phosphate. DIF: Cognitive Level: Apply (application) REF: 1087 TOP: Nursing Process: Evaluation MSC: NCLEX: Physiological Integrity

External-beam radiation is planned for a patient with cervical cancer. What instructions should the nurse give to the patient to prevent complications from the effects of the radiation? a. Test all stools for the presence of blood. b. Maintain a high-residue, high-fiber diet. c. Clean the perianal area carefully after every bowel movement. d. Inspect the mouth and throat daily for the appearance of thrush.

ANS: C Radiation to the abdomen will affect organs in the radiation path, such as the bowel, and cause frequent diarrhea. Careful cleaning of this area will help decrease the risk for skin breakdown and infection. Stools are likely to have occult blood from the inflammation associated with radiation, so routine testing of stools for blood is not indicated. Radiation to the abdomen will not cause stomatitis. A low-residue diet is recommended to avoid irritation of the bowel when patients receive abdominal radiation.

Which information will the nurse include when teaching a patient who is scheduled for a radiofrequency catheter ablation for treatment of atrial flutter? a. The procedure prevents or minimizes the risk for sudden cardiac death. b. The procedure uses cold therapy to stop the formation of the flutter waves. c. The procedure uses electrical energy to destroy areas of the conduction system. d. The procedure stimulates the growth of new conduction pathways between the atria.

ANS: C Radiofrequency catheter ablation therapy uses electrical energy to "burn" or ablate areas of the conduction system as definitive treatment of atrial flutter (i.e., restore normal sinus rhythm) and tachydysrhythmias. All other statements regarding the procedure are incorrect. DIF: Cognitive Level: Apply (application)

18. Which nursing action included in the care of a patient after laminectomy can the nurse delegate to experienced unlicensed assistive personnel (UAP)? a. Check ability to plantar and dorsiflex the foot. b. Determine the patients readiness to ambulate. c. Log roll the patient from side to side every 2 hours. d. Ask about pain control with the patient-controlled analgesia (PCA).

ANS: C Repositioning a patient is included in the education and scope of practice of UAP, and experienced UAP will be familiar with how to maintain alignment in the postoperative patient. Evaluation of the effectiveness of pain medications, assessment of neurologic function, and evaluation of a patients readiness to ambulate after surgery require higher level nursing education and scope of practice.

38. Four hours after a bowel resection, a 74-year-old male patient with a nasogastric tube to suction complains of nausea and abdominal distention. The first action by the nurse should be to a. auscultate for hypotonic bowel sounds. b. notify the patient's health care provider. c. reposition the tube and check for placement. d. remove the tube and replace it with a new one.

ANS: C Repositioning the tube will frequently facilitate drainage. Because this is a common occurrence, it is not appropriate to notify the health care provider unless other interventions do not resolve the problem. Information about the presence or absence of bowel sounds will not be helpful in improving drainage. Removing the tube and replacing it are unnecessarily traumatic to the patient, so that would only be done if the tube was completely occluded.

6. An appropriate nursing intervention for a patient who has acute low back pain and muscle spasms is to teach the patient to a. keep both feet flat on the floor when prolonged standing is required. b. twist gently from side to side to maintain range of motion in the spine. c. keep the head elevated slightly and flex the knees when resting in bed. d. avoid the use of cold packs because they will exacerbate the muscle spasms.

ANS: C Resting with the head elevated and knees flexed will reduce the strain on the back and decrease muscle spasms. Twisting from side to side will increase tension on the lumbar area. A pillow placed under the upper back will cause strain on the lumbar spine. Alternate application of cold and heat should be used to decrease pain.

Which patient should the nurse assess first? a. Patient with urticaria after receiving an IV antibiotic b. Patient who has graft-versus-host disease and severe diarrhea c. Patient who is sneezing after having subcutaneous immunotherapy d. Patient with multiple chemical sensitivities who has muscle stiffness

ANS: C Sneezing after subcutaneous immunotherapy may indicate impending anaphylaxis and assessment and emergency measures should be initiated. The other patients also have findings that need assessment and intervention by the nurse, but do not have evidence of life-threatening complications.

Which statement by a patient would alert the nurse to a possible immunodeficiency disorder? a. "I take one baby aspirin every day to prevent stroke." b. "I usually eat eggs or meat for at least 2 meals a day." c. "I had my spleen removed many years ago after a car accident." d. "I had a chest x-ray 6 months ago when I had walking pneumonia."

ANS: C Splenectomy increases the risk for septicemia from bacterial infections. The patient's protein intake is good and should improve immune function. Daily aspirin use does not affect immune function. A chest x-ray does not have enough radiation to suppress immune function.

The health care provider writes an order for bacteriologic testing for a patient who has a positive tuberculosis skin test. Which action should the nurse take? a. Teach about the reason for the blood tests. b. Schedule an appointment for a chest x-ray. c. Teach about the need to get sputum specimens for 2 to 3 consecutive days. d. Instruct the patient to expectorate three specimens as soon as possible.

ANS: C Sputum specimens are obtained on 2 to 3 consecutive days for bacteriologic testing for M. tuberculosis. The patient should not provide all the specimens at once. Blood cultures are not used for tuberculosis testing. A chest x-ray is not bacteriologic testing. Although the findings on chest x-ray examination are important, it is not possible to make a diagnosis of TB solely based on chest x-ray findings because other diseases can mimic the appearance of TB

The charge nurse observes an inexperienced staff nurse caring for a patient who has had a craniotomy for resection of a brain tumor. Which action by the inexperienced nurse requires the charge nurse to intervene? a. The staff nurse assesses neurologic status every hour. b. The staff nurse elevates the head of the bed to 30 degrees. c. The staff nurse suctions the patient routinely every 2 hours. d. The staff nurse administers an analgesic before turning the patient.

ANS: C Suctioning increases intracranial pressure and should only be done when the patient's respiratory condition indicates it is needed. The other actions by the staff nurse are appropriate.

Which information obtained by the nurse in the endocrine clinic about a patient who has been taking prednisone (Deltasone) 40 mg daily for 3 weeks is most important to report to the health care provider? a. Patient's blood pressure is 148/94 mm Hg. b. Patient has bilateral 2+ pitting ankle edema. c. Patient stopped taking the medication 2 days ago. d. Patient has not been taking the prescribed vitamin D.

ANS: C Sudden cessation of corticosteroids after taking the medication for a week or more can lead to adrenal insufficiency, with problems such as severe hypotension and hypoglycemia. The patient will need immediate evaluation by the health care provider to prevent and/or treat adrenal insufficiency. The other information will also be reported, but does not require rapid treatment

The nurse is caring for a patient with left-sided lung cancer. Which finding would be most important for the nurse to report to the health care provider? a. Hematocrit 32% b. Pain with deep inspiration c. Serum sodium 126 mEq/L d. Decreased breath sounds on left side

ANS: C Syndrome of inappropriate antidiuretic hormone (and the resulting hyponatremia) is an oncologic metabolic emergency and will require rapid treatment in order to prevent complications such as seizures and coma. The other findings also require intervention, but are common in patients with lung cancer and not immediately life threatening

The nurse is caring for a patient who smokes 2 packs/day. To reduce the patient's risk of lung cancer, which action by the nurse is best? a. Teach the patient about the seven warning signs of cancer. b. Plan to monitor the patient's carcinoembryonic antigen (CEA) level. c. Discuss the risks associated with cigarettes during every patient encounter. d. Teach the patient about the use of annual chest x-rays for lung cancer screening.

ANS: C Teaching about the risks associated with cigarette smoking is recommended at every patient encounter because cigarette smoking is associated with multiple health problems. A tumor must be at least 0.5 cm large before it is detectable by current screening methods and may already have metastasized by that time. Oncofetal antigens such as CEA may be used to monitor therapy or detect tumor reoccurrence, but are not helpful in screening for cancer. The seven warning signs of cancer are actually associated with fairly advanced disease

After change-of-shift report on the oncology unit, which patient should the nurse assess first? a. Patient who has a platelet count of 82,000/µL after chemotherapy b. Patient who has xerostomia after receiving head and neck radiation c. Patient who is neutropenic and has a temperature of 100.5° F (38.1° C) d. Patient who is worried about getting the prescribed long-acting opioid on time

ANS: C Temperature elevation is an emergency in neutropenic patients because of the risk for rapid progression to severe infections and sepsis. The other patients also require assessments or interventions, but do not need to be assessed as urgently. Patients with thrombocytopenia do not have spontaneous bleeding until the platelets are 20,000/µL. Xerostomia does not require immediate intervention. Although breakthrough pain needs to be addressed rapidly, the patient does not appear to have breakthrough pain

Which assessment finding obtained by the nurse when assessing a patient with acute pericarditis should be reported immediately to the health care provider? a. Pulsus paradoxus 8 mm Hg b. Blood pressure (BP) of 168/94 mm Hg c. Jugular venous distention (JVD) to jaw level d. Level 6 (0 to 10 scale) chest pain with a deep breath

ANS: C The JVD indicates that the patient may have developed cardiac tamponade and may need rapid intervention to maintain adequate cardiac output. Hypertension would not be associated with complications of pericarditis, and the BP is not high enough to indicate that there is any immediate need to call the health care provider. A pulsus paradoxus of 8 mm Hg is normal. Level 6/10 chest pain should be treated but is not unusual with pericarditis. DIF: Cognitive Level: Analyze (analysis)

The nurse in the outpatient clinic has obtained health histories for these new patients. Which patient may need referral for genetic testing? a. 35-year-old patient whose maternal grandparents died after strokes at ages 90 and 96 b. 18-year-old patient with a positive pregnancy test whose first child has cerebral palsy c. 34-year-old patient who has a sibling with newly diagnosed polycystic kidney disease d. 50-year-old patient with a history of cigarette smoking who is complaining of dyspnea

ANS: C The adult form of polycystic kidney disease is an autosomal dominant disorder and frequently it is asymptomatic until the patient is older. Presymptomatic testing will give the patient information that will be useful in guiding lifestyle and childbearing choices. The other patients do not have any indication of genetic disorders or need for genetic testing

16. During the administration of the thrombolytic agent to a patient with an acute myocardial infarction (AMI), the nurse should stop the drug infusion if the patient experiences a. bleeding from the gums. b. increase in blood pressure. c. a decrease in level of consciousness. d. a nonsustained episode of ventricular tachycardia.

ANS: C The change in level of consciousness indicates that the patient may be experiencing intracranial bleeding, a possible complication of thrombolytic therapy. Some bleeding of the gums is an expected side effect of the therapy but not an indication to stop infusion of the thrombolytic medication. A decrease in blood pressure could indicate internal bleeding. A nonsustained episode of ventricular tachycardia is a common reperfusion dysrhythmia and may indicate that the therapy is effective.

Which action should the nurse take when the low pressure alarm sounds for a patient who has an arterial line in the left radial artery? a. Fast flush the arterial line. b. Check the left hand for pallor. c. Assess for cardiac dysrhythmias. d. Re-zero the monitoring equipment.

ANS: C The low pressure alarm indicates a drop in the patient's blood pressure, which may be caused by cardiac dysrhythmias. There is no indication to re-zero the equipment. Pallor of the left hand would be caused by occlusion of the radial artery by the arterial catheter, not by low pressure. There is no indication of a need for flushing the line.

35. The nurse obtains the following data when assessing a patient who experienced an ST-segment-elevation myocardial infarction (STEMI) 2 days previously. Which information is most important to report to the health care provider? a. The troponin level is elevated. b. The patient denies ever having a heart attack. c. Bilateral crackles are auscultated in the mid-lower lobes. d. The patient has occasional premature atrial contractions (PACs).

ANS: C The crackles indicate that the patient may be developing heart failure, a possible complication of myocardial infarction (MI). The health care provider may need to order medications such as diuretics or angiotensin-converting enzyme (ACE) inhibitors for the patient. Elevation in troponin level at this time is expected. PACs are not life-threatening dysrhythmias. Denial is a common response in the immediate period after the MI.

Which assessment finding of a 42-year-old patient who had a bilateral adrenalectomy requires the most rapid action by the nurse? a. The blood glucose is 176 mg/dL. b. The lungs have bibasilar crackles. c. The blood pressure (BP) is 88/50 mm Hg. d. The patient reports 5/10 incisional pain.

ANS: C The decreased BP indicates possible adrenal insufficiency. The nurse should immediately notify the health care provider so that corticosteroid medications can be administered. The nurse should also address the elevated glucose, incisional pain, and crackles with appropriate collaborative or nursing actions, but prevention and treatment of acute adrenal insufficiency is the priority after adrenalectomy.

5. After the nurse has finished teaching a patient about the use of sublingual nitroglycerin (Nitrostat), which patient statement indicates that the teaching has been effective? a. "I can expect some nausea as a side effect of nitroglycerin." b. "I should only take the nitroglycerin if I start to have chest pain." c. "I will call an ambulance if I still have pain after taking 3 nitroglycerin 5 minutes apart." d. "Nitroglycerin helps prevent a clot from forming and blocking blood flow to my heart."

ANS: C The emergency medical services (EMS) system should be activated when chest pain or other symptoms are not completely relieved after 3 sublingual nitroglycerin tablets taken 5 minutes apart. Nitroglycerin can be taken to prevent chest pain or other symptoms from developing (e.g., before intercourse). Gastric upset (e.g., nausea) is not an expected side effect of nitroglycerin. Nitroglycerin does not impact the underlying pathophysiology of coronary artery atherosclerosis.

The nurse is caring for a patient with idiopathic pulmonary arterial hypertension (IPAH) who is receiving epoprostenol (Flolan). Which assessment information requires the most immediate action by the nurse? a. The oxygen saturation is 94%. b. The blood pressure is 98/56 mm Hg. c. The patient's central IV line is disconnected. d. The international normalized ratio (INR) is prolonged.

ANS: C The half-life of this drug is 6 minutes, so the nurse will need to restart the infusion as soon as possible to prevent rapid clinical deterioration. The other data also indicate a need for ongoing monitoring or intervention, but the priority action is to reconnect the infusion

A patient reports dizziness and shortness of breath for several days. During heart monitoring in the emergency department (ED), the nurse obtains the following electrocardiographic (ECG) tracing. The nurse interprets this heart rhythm as a. junctional escape rhythm. b. accelerated idioventricular rhythm. c. third-degree atrioventricular (AV) block. d. sinus rhythm with premature atrial contractions (PACs).

ANS: C The inconsistency between the atrial and ventricular rates and the variable PR interval indicate that the rhythm is third-degree AV block. Sinus rhythm with PACs will have a normal rate and consistent PR intervals with occasional PACs. An accelerated idioventricular rhythm will not have visible P waves. DIF: Cognitive Level: Apply (application)

Which action by the nurse will be most effective in decreasing the spread of pertussis in a community setting? a. Providing supportive care to patients diagnosed with pertussis b. Teaching family members about the need for careful hand washing c. Teaching patients about the need for adult pertussis immunizations d. Encouraging patients to complete the prescribed course of antibiotics

ANS: C The increased rate of pertussis in adults is thought to be due to decreasing immunity after childhood immunization. Immunization is the most effective method of protecting communities from infectious diseases. Hand washing should be taught, but pertussis is spread by droplets and contact with secretions. Supportive care does not shorten the course of the disease or the risk for transmission. Taking antibiotics as prescribed does assist with decreased transmission, but patients are likely to have already transmitted the disease by the time the diagnosis is made

47. A 76-year-old patient with obstipation has a fecal impaction and is incontinent of liquid stool. Which action should the nurse take first? a. Administer bulk-forming laxatives. b. Assist the patient to sit on the toilet. c. Manually remove the impacted stool. d. Increase the patient's oral fluid intake.

ANS: C The initial action with a fecal impaction is manual disimpaction. The other actions will be used to prevent future constipation and impactions.

39. A 19-year-old female is brought to the emergency department with a knife handle protruding from the abdomen. During the initial assessment of the patient, the nurse should a. remove the knife and assess the wound. b. determine the presence of Rovsing sign. c. check for circulation and tissue perfusion. d. insert a urinary catheter and assess for hematuria.

ANS: C The initial assessment is focused on determining whether the patient has hypovolemic shock. The knife should not be removed until the patient is in surgery, where bleeding can be controlled. Rovsing sign is assessed in the patient with suspected appendicitis. A patient with a knife in place will be taken to surgery and assessed for bladder trauma there.

A patient who is scheduled for a right breast biopsy asks the nurse the difference between a benign tumor and a malignant tumor. Which answer by the nurse is correct? a. "Benign tumors do not cause damage to other tissues." b. "Benign tumors are likely to recur in the same location." c. "Malignant tumors may spread to other tissues or organs." d. "Malignant cells reproduce more rapidly than normal cells."

ANS: C The major difference between benign and malignant tumors is that malignant tumors invade adjacent tissues and spread to distant tissues and benign tumors never metastasize. The other statements are inaccurate. Both types of tumors may cause damage to adjacent tissues. Malignant cells do not reproduce more rapidly than normal cells. Benign tumors do not usually recur

An unconscious patient is admitted to the emergency department (ED) with a head injury. The patient's spouse and teenage children stay at the patient's side and ask many questions about the treatment being given. What action is best for the nurse to take? a. Call the family's pastor or spiritual advisor to take them to the chapel. b. Ask the family to stay in the waiting room until the assessment is completed. c. Allow the family to stay with the patient and briefly explain all procedures to them. d. Refer the family members to the hospital counseling service to deal with their anxiety.

ANS: C The need for information about the diagnosis and care is very high in family members of acutely ill patients. The nurse should allow the family to observe care and explain the procedures unless they interfere with emergent care needs. A pastor or counseling service can offer some support, but research supports information as being more effective. Asking the family to stay in the waiting room will increase their anxiety.

A patient with right lower-lobe pneumonia has been treated with IV antibiotics for 3 days. Which assessment data obtained by the nurse indicates that the treatment has been effective? a. Bronchial breath sounds are heard at the right base. b. The patient coughs up small amounts of green mucus. c. The patient's white blood cell (WBC) count is 9000/µL. d. Increased tactile fremitus is palpable over the right chest.

ANS: C The normal WBC count indicates that the antibiotics have been effective. All the other data suggest that a change in treatment is needed

Employee health test results reveal a tuberculosis (TB) skin test of 16-mm induration and a negative chest x-ray for a staff nurse working on the pulmonary unit. The nurse has no symptoms of TB. Which information should the occupational health nurse plan to teach the staff nurse? a. Standard four-drug therapy for TB b. Need for annual repeat TB skin testing c. Use and side effects of isoniazid (INH) d. Bacille Calmette-Guérin (BCG) vaccine

ANS: C The nurse is considered to have a latent TB infection and should be treated with INH daily for 6 to 9 months. The four-drug therapy would be appropriate if the nurse had active TB. TB skin testing is not done for individuals who have already had a positive skin test. BCG vaccine is not used in the United States for TB and would not be helpful for this individual, who already has a TB infection

Which action should the nurse in the emergency department take first for a new patient who is vomiting blood? a. Insert a large-gauge IV catheter. b. Draw blood for coagulation studies. c. Check blood pressure (BP), heart rate, and respirations. d. Place the patient in the supine position.

ANS: C The nurse's first action should be to determine the patient's hemodynamic status by assessing vital signs. Drawing blood for coagulation studies and inserting an IV catheter are also appropriate. However, the vital signs may indicate the need for more urgent actions. Because aspiration is a concern for this patient, the nurse will need to assess the patient's vital signs and neurologic status before placing the patient in a supine position

A patient recovering from heart surgery develops pericarditis and complains of level 6 (0 to 10 scale) chest pain with deep breathing. Which prescribed PRN medication will be the most appropriate for the nurse to give? a. Fentanyl 1 mg IV c. Oral ibuprofen (Motrin) 600 mg b. IV morphine sulfate 4 mg d. Oral acetaminophen (Tylenol) 650 mg

ANS: C The pain associated with pericarditis is caused by inflammation, so nonsteroidal antiinflammatory drugs (e.g., ibuprofen) are most effective. Opioid analgesics and acetaminophen are not very effective for the pain associated with pericarditis. DIF: Cognitive Level: Analyze (analysis)

42. After reviewing information shown in the accompanying figure from the medical records of a 43-year-old, which risk factor modification for coronary artery disease should the nurse include in patient teaching? a. Importance of daily physical activity b. Effect of weight loss on blood pressure c. Dietary changes to improve lipid levels d. Ongoing cardiac risk associated with history of tobacco use

ANS: C The patient has an elevated low-density lipoprotein (LDL) cholesterol and low high-density lipoprotein (HDL) cholesterol, which will increase the risk of coronary artery disease. Although the blood pressure is in the prehypertensive range, the patient's waist circumference and body mass index (BMI) indicate an appropriate body weight. The risk for coronary artery disease a year after quitting smoking is the same as a nonsmoker. The patient's occupation indicates that daily activity is at the levels suggested by national guidelines.

An 82-year-old patient in a long-term care facility has several medications prescribed. After the patient is newly diagnosed with hypothyroidism, the nurse will need to consult with the health care provider before administering a. docusate (Colace). b. ibuprofen (Motrin). c. diazepam (Valium). d. cefoxitin (Mefoxin).

ANS: C Worsening of mental status and myxedema coma can be precipitated by the use of sedatives, especially in older adults. The nurse should discuss the use of diazepam with the health care provider before administration. The other medications may be given safely to the patient

A patient's heart monitor shows sinus rhythm, rate 64. The PR interval is 0.18 seconds at 1:00 AM, 0.22 seconds at 2:30 PM, and 0.28 seconds at 4:00 PM. Which action should the nurse take next? a. Place the transcutaneous pacemaker pads on the patient. b. Give atropine sulfate 1 mg IV per agency dysrhythmia protocol. c. Call the health care provider before giving scheduled metoprolol (Lopressor). d. Document the patient's rhythm and assess the patient's response to the rhythm.

ANS: C The patient has progressive first-degree atrioventricular (AV) block, and the -blocker should be held until discussing the drug with the health care provider. Documentation and assessment are appropriate but not fully adequate responses. The patient with first-degree AV block usually is asymptomatic and a pacemaker is not indicated. Atropine is sometimes used for symptomatic bradycardia, but there is no indication that this patient is symptomatic. DIF: Cognitive Level: Analyze (analysis)

A patient who is complaining of a "racing" heart and feeling "anxious" comes to the emergency department. The nurse places the patient on a heart monitor and obtains the following electrocardiographic (ECG) tracing. Which action should the nurse take next? a. Prepare to perform electrical cardioversion. b. Have the patient perform the Valsalva maneuver. c. Obtain the patient's vital signs including O2 saturation. d. Prepare to give a -blocker medication to slow the heart rate.

ANS: C The patient has sinus tachycardia, which may have multiple etiologies such as pain, dehydration, anxiety, and myocardial ischemia. Further assessment is needed before determining the treatment. Vagal stimulation or -blockade may be used after further assessment of the patient. Electrical cardioversion is used for some tachydysrhythmias but would not be used for sinus tachycardia. DIF: Cognitive Level: Analyze (analysis)

30. Which electrocardiographic (ECG) change is most important for the nurse to report to the health care provider when caring for a patient with chest pain? a. Inverted P wave b. Sinus tachycardia c. ST-segment elevation d. First-degree atrioventricular block

ANS: C The patient is likely to be experiencing an ST-segment-elevation myocardial infarction (STEMI). Immediate therapy with percutaneous coronary intervention (PCI) or thrombolytic medication is indicated to minimize myocardial damage. The other ECG changes may also suggest a need for therapy, but not as rapidly.

During a routine health examination, a 40-year-old patient tells the nurse about a family history of colon cancer. Which action should the nurse take next? a. Teach the patient about the need for a colonoscopy at age 50. b. Teach the patient how to do home testing for fecal occult blood. c. Obtain more information from the patient about the family history. d. Schedule a sigmoidoscopy to provide baseline data about the patient.

ANS: C The patient may be at increased risk for colon cancer, but the nurse's first action should be further assessment. The other actions may be appropriate, depending on the information that is obtained from the patient with further questioning.

Which intervention by a new nurse who is caring for a patient who has just had an implantable cardioverter-defibrillator (ICD) inserted indicates a need for more teaching about the care of patients with ICDs? a. The nurse administers amiodarone (Cordarone) to the patient. b. The nurse helps the patient fill out the application for obtaining a Medic Alert device. c. The nurse encourages the patient to do active range of motion exercises for all extremities. d. The nurse teaches the patient that sexual activity can be resumed when the incision is healed.

ANS: C The patient should avoid moving the arm on the ICD insertion site until healing has occurred to prevent displacement of the ICD leads. The other actions by the new nurse are appropriate for this patient. DIF: Cognitive Level: Apply (application)

To prepare a 56-year-old male patient with ascites for paracentesis, the nurse a. places the patient on NPO status. b. assists the patient to lie flat in bed. c. asks the patient to empty the bladder. d. positions the patient on the right side.

ANS: C The patient should empty the bladder to decrease the risk of bladder perforation during the procedure. The patient would be positioned in Fowler's position and would not be able to lie flat without compromising breathing. Because no sedation is required for paracentesis, the patient does not need to be NPO.

18. In preparation for discharge, the nurse teaches a patient with chronic stable angina how to use the prescribed short-acting and long-acting nitrates. Which patient statement indicates that the teaching has been effective? a. "I will check my pulse rate before I take any nitroglycerin tablets." b. "I will put the nitroglycerin patch on as soon as I get any chest pain." c. "I will stop what I am doing and sit down before I put the nitroglycerin under my tongue." d. "I will be sure to remove the nitroglycerin patch before taking any sublingual nitroglycerin."

ANS: C The patient should sit down before taking the nitroglycerin to decrease cardiac workload and prevent orthostatic hypotension. Transdermal nitrates are used prophylactically rather than to treat acute pain and can be used concurrently with sublingual nitroglycerin. Although the nurse should check blood pressure before giving nitroglycerin, patients do not need to check the pulse rate before taking nitrates.

A patient with cancer has a nursing diagnosis of imbalanced nutrition: less than body requirements related to altered taste sensation. Which nursing action is most appropriate? a. Add strained baby meats to foods such as casseroles. b. Teach the patient about foods that are high in nutrition. c. Avoid giving the patient foods that are strongly disliked. d. Add extra spice to enhance the flavor of foods that are served.

ANS: C The patient will eat more if disliked foods are avoided and foods that the patient likes are included instead. Additional spice is not usually an effective way to enhance taste. Adding baby meats to foods will increase calorie and protein levels, but does not address the issue of taste. The patient's poor intake is not caused by a lack of information about nutrition

5. Which statement by a patient with stage 5 chronic kidney disease (CKD) indicates that the nurse's teaching about management of CKD has been effective? a. "I need to get most of my protein from low-fat dairy products." b. "I will increase my intake of fruits and vegetables to 5 per day." c. "I will measure my urinary output each day to help calculate the amount I can drink." d. "I need to take erythropoietin to boost my immune system and help prevent infection."

ANS: C The patient with end-stage renal disease is taught to measure urine output as a means of determining an appropriate oral fluid intake. Erythropoietin is given to increase the red blood cell count and will not offer any benefit for immune function. Dairy products are restricted because of the high phosphate level. Many fruits and vegetables are high in potassium and should be restricted in the patient with CKD. DIF: Cognitive Level: Apply (application) REF: 1082 TOP: Nursing Process: Evaluation MSC: NCLEX: Physiological Integrity

While obtaining a health history from a patient, the nurse learns that the patient has a history of allergic rhinitis and multiple food allergies. Which action by the nurse is most appropriate? a. Encourage the patient to carry an epinephrine kit in case a type IV allergic reaction to latex develops. b. Advise the patient to use oil-based hand creams to decrease contact with natural proteins in latex gloves. c. Document the patient's allergy history and be alert for any clinical manifestations of a type I latex allergy. d. Recommend that the patient use vinyl gloves instead of latex gloves in preventing blood-borne pathogen contact.

ANS: C The patient's allergy history and occupation indicate a risk of developing a latex allergy. The nurse should be prepared to manage any symptoms that may occur. Epinephrine is not an appropriate treatment for contact dermatitis that is caused by a type IV allergic reaction to latex. Oil-based creams will increase the exposure to latex from latex gloves. Vinyl gloves are appropriate to use when exposure to body fluids is unlikely.

1. After the insertion of an arteriovenous graft (AVG) in the right forearm, a patient complains of pain and coldness of the right fingers. Which action should the nurse take? a. Teach the patient about normal AVG function. b. Remind the patient to take a daily low-dose aspirin tablet. c. Report the patient's symptoms to the health care provider. d. Elevate the patient's arm on pillows to above the heart level.

ANS: C The patient's complaints suggest the development of distal ischemia (steal syndrome) and may require revision of the AVG. Elevation of the arm above the heart will further decrease perfusion. Pain and coolness are not normal after AVG insertion. Aspirin therapy is not used to maintain grafts. DIF: Cognitive Level: Apply (application) REF: 1088 TOP: Nursing Process: Implementation MSC: NCLEX: Physiological Integrity

On admission to the burn unit, a patient with an approximate 25% total body surface area (TBSA) burn has the following initial laboratory results: Hct 58%, Hgb 18.2 mg/dL (172 g/L), serum K+ 4.9 mEq/L (4.8 mmol/L), and serum Na+ 135 mEq/L (135 mmol/L). Which action will the nurse anticipate taking now? a. Monitor urine output every 4 hours. b. Continue to monitor the laboratory results. c. Increase the rate of the ordered IV solution. d. Type and crossmatch for a blood transfusion.

ANS: C The patient's laboratory data show hemoconcentration, which may lead to a decrease in blood flow to the microcirculation unless fluid intake is increased. Because the hematocrit and hemoglobin are elevated, a transfusion is inappropriate, although transfusions may be needed after the emergent phase once the patient's fluid balance has been restored. On admission to a burn unit, the urine output would be monitored more often than every 4 hours; likely every1 hour

17. A patient is recovering from a myocardial infarction (MI) and develops chest pain on day 3 that increases when taking a deep breath and is relieved by leaning forward. Which action should the nurse take next? a. Assess the feet for pedal edema. b. Palpate the radial pulses bilaterally. c. Auscultate for a pericardial friction rub. d. Check the heart monitor for dysrhythmias.

ANS: C The patient's symptoms are consistent with the development of pericarditis, a possible complication of MI. The other assessments listed are not consistent with the description of the patient's symptoms.

A patient develops carpopedal spasms and tingling of the lips following a parathyroidectomy. Which action should the nurse take first? a. Administer the ordered muscle relaxant. b. Give the ordered oral calcium supplement. c. Have the patient rebreathe from a paper bag. d. Start the PRN oxygen at 2 L/min per cannula.

ANS: C The patient's symptoms suggest mild hypocalcemia. The symptoms of hypocalcemia will be temporarily reduced by having the patient breathe into a paper bag, which will raise the PaCO2 and create a more acidic pH. The muscle relaxant will have no impact on the ionized calcium level. Although severe hypocalcemia can cause laryngeal stridor, there is no indication that this patient is experiencing laryngeal stridor or needs oxygen. Calcium supplements will be given to normalize calcium levels quickly, but oral supplements will take time to be absorbed

After obtaining the information shown in the accompanying figure regarding a patinet with addisons disease, which prescribed action will the nurse take first? a. Give 4 oz of fruit juice orally b. recheck the blood glucose level c. infuse 5% dextrose and 0.9% saline d. administer oxygen therapy as needed

ANS: C The patients poor skin turgor, hypotension, and hyponatremia indicate an addisonian crisis. Immediate correction of the hypovolemia and hyponatremia is needed. The other actions may also be needed but are not the initial action for the patient

49. A 72-year-old male patient with dehydration caused by an exacerbation of ulcerative colitis is receiving 5% dextrose in normal saline at 125 mL/hour. Which assessment finding by the nurse is most important to report to the health care provider? a. Patient has not voided for the last 4 hours. b. Skin is dry with poor turgor on all extremities. c. Crackles are heard halfway up the posterior chest. d. Patient has had 5 loose stools over the last 6 hours.

ANS: C The presence of crackles in an older patient receiving IV fluids at a high rate suggests volume overload and a need to reduce the rate of the IV infusion. The other data will also be reported, but are consistent with the patient's age and diagnosis and do not require a change in the prescribed treatment.

When caring for a patient with mitral valve stenosis, it is most important that the nurse assess for a. diastolic murmur. c. shortness of breath on exertion. b. peripheral edema. d. right upper quadrant tenderness.

ANS: C The pressure gradient changes in mitral stenosis lead to fluid backup into the lungs, resulting in hypoxemia and dyspnea. The other findings also may be associated with mitral valve disease but are not indicators of possible hypoxemia, which is a priority. DIF: Cognitive Level: Analyze (analysis)

A 38-year-old patient with cirrhosis has ascites and 4+ edema of the feet and legs. Which nursing action will be included in the plan of care? a. Restrict daily dietary protein intake. b. Reposition the patient every 4 hours. c. Place the patient on a pressure-relieving mattress. d. Perform passive range of motion daily.

ANS: C The pressure-relieving mattress will decrease the risk for skin breakdown for this patient. Adequate dietary protein intake is necessary in patients with ascites to improve oncotic pressure. Repositioning the patient every 4 hours will not be adequate to maintain skin integrity. Passive range of motion will not take the pressure off areas such as the sacrum that are vulnerable to breakdown

37. A patient who has chest pain is admitted to the emergency department (ED) and all of the following are ordered. Which one should the nurse arrange to be completed first? a. Chest x-ray b. Troponin level c. Electrocardiogram (ECG) d. Insertion of a peripheral IV

ANS: C The priority for the patient is to determine whether an acute myocardial infarction (AMI) is occurring so that reperfusion therapy can begin as quickly as possible. ECG changes occur very rapidly after coronary artery occlusion, and an ECG should be obtained as soon as possible. Troponin levels will increase after about 3 hours. Data from the chest x-ray may impact the patient's care but are not helpful in determining whether the patient is experiencing a myocardial infarction (MI). Peripheral access will be needed but not before the ECG.

41. A patient with diabetes mellitus and chronic stable angina has a new order for captopril (Capoten). The nurse should teach the patient that the primary purpose of captopril is to a. lower heart rate. b. control blood glucose levels. c. prevent changes in heart muscle. d. reduce the frequency of chest pain.

ANS: C The purpose for angiotensin-converting enzyme (ACE) inhibitors in patients with chronic stable angina who are at high risk for a cardiac event is to decrease ventricular remodeling. ACE inhibitors do not directly impact angina frequency, blood glucose, or heart rate.

Which finding is most important for the nurse to communicate to the health care provider about a patient who received a liver transplant 1 week ago? a. Dry palpebral and oral mucosa b. Crackles at bilateral lung bases c. Temperature 100.8° F (38.2° C) d. No bowel movement for 4 days

ANS: C The risk of infection is high in the first few months after liver transplant and fever is frequently the only sign of infection. The other patient data indicate the need for further assessment or nursing actions and might be communicated to the health care provider, but they do not indicate a need for urgent action

8. The nurse should reposition the patient who has just had a laminectomy and diskectomy by a. instructing the patient to move the legs before turning the rest of the body. b. having the patient turn by grasping the side rails and pulling the shoulders over. c. placing a pillow between the patients legs and turning the entire body as a unit. d. turning the patients head and shoulders first, followed by the hips, legs, and feet.

ANS: C The spine should be kept in correct alignment after laminectomy. The other positions will create misalignment of the spine.

A 62-year-old patient with hyperthyroidism is to be treated with radioactive iodine (RAI). The nurse instructs the patient a. about radioactive precautions to take with all body secretions. b. that symptoms of hyperthyroidism should be relieved in about a week. c. that symptoms of hypothyroidism may occur as the RAI therapy takes effect. d. to discontinue the antithyroid medications taken before the radioactive therapy.

ANS: C There is a high incidence of postradiation hypothyroidism after RAI, and the patient should be monitored for symptoms of hypothyroidism. RAI has a delayed response, with the maximum effect not seen for 2 to 3 months, and the patient will continue to take antithyroid medications during this time. The therapeutic dose of radioactive iodine is low enough that no radiation safety precautions are needed

The nurse needs to quickly estimate the heart rate for a patient with a regular heart rhythm. Which method will be best to use? a. Count the number of large squares in the R-R interval and divide by 300. b. Print a 1-minute electrocardiogram (ECG) strip and count the number of QRS complexes. c. Use the 3-second markers to count the number of QRS complexes in 6 seconds and multiply by 10. d. Calculate the number of small squares between one QRS complex and the next and divide into 1500.

ANS: C This is the quickest way to determine the ventricular rate for a patient with a regular rhythm. All the other methods are accurate but take longer. Cognitive Level: Analyze (analysis)

The charge nurse is assigning rooms for new admissions. Which patient would be the most appropriate roommate for a patient who has acute rejection of an organ transplant? a. A patient who has viral pneumonia b. A patient with second-degree burns c. A patient who is recovering from an anaphylactic reaction to a bee sting d. A patient with graft-versus-host disease after a recent bone marrow transplant

ANS: C Treatment for a patient with acute rejection includes administration of additional immunosuppressants, and the patient should not be exposed to increased risk for infection as would occur from patients with viral pneumonia, graft-versus-host disease, and burns. There is no increased exposure to infection from a patient who had an anaphylactic reaction.

A patient with metastatic cancer of the colon experiences severe vomiting following each administration of chemotherapy. Which action, if taken by the nurse, is most appropriate? a. Have the patient eat large meals when nausea is not present. b. Offer dry crackers and carbonated fluids during chemotherapy. c. Administer prescribed antiemetics 1 hour before the treatments. d. Give the patient two ounces of a citrus fruit beverage during treatments.

ANS: C Treatment with antiemetics before chemotherapy may help prevent nausea. The patient should eat small, frequent meals. Offering food and beverages during chemotherapy is likely to cause nausea. The acidity of citrus fruits may be further irritating to the stomach

While caring for a 23-yr-old patient with mitral valve prolapse (MVP) without valvular regurgitation, the nurse determines that discharge teaching has been effective when the patient states that it will be necessary to a. take antibiotics before any dental appointments. b. limit physical activity to avoid stressing the heart. c. avoid over-the-counter (OTC) drugs that contain stimulants. d. take an aspirin a day to prevent clots from forming on the valve.

ANS: C Use of stimulant drugs should be avoided by patients with MVP because they may exacerbate symptoms. Daily aspirin and restricted physical activity are not needed by patients with mild MVP. Antibiotic prophylaxis is needed for patients with MVP with regurgitation but will not be necessary for this patient. DIF: Cognitive Level: Apply (application)

12. A 22-year-old female patient with an exacerbation of ulcerative colitis is having 15 to 20 stools daily and has excoriated perianal skin. Which patient behavior indicates that teaching regarding maintenance of skin integrity has been effective? a. The patient uses incontinence briefs to contain loose stools. b. The patient asks for antidiarrheal medication after each stool. c. The patient uses witch hazel compresses to decrease irritation. d. The patient cleans the perianal area with soap after each stool.

ANS: C Witch hazel compresses are suggested to reduce anal irritation and discomfort. Incontinence briefs may trap diarrhea and increase the incidence of skin breakdown. Antidiarrheal medications are not given 15 to 20 times a day. The perianal area should be washed with plain water after each stool.

The nurse notes thick, white secretions in the endotracheal tube (ET) of a patient who is receiving mechanical ventilation. Which intervention will most directly treat this finding? a. Reposition the patient every 1 to 2 hours. b. Increase suctioning frequency to every hour. c. Add additional water to the patient's enteral feedings. d. Instill 5 mL of sterile saline into the ET before suctioning.

ANS: C Because the patient's secretions are thick, better hydration is indicated. Suctioning every hour without any specific evidence for the need will increase the incidence of mucosal trauma and would not address the etiology of the ineffective airway clearance. Instillation of saline does not liquefy secretions and may decrease the SpO2. Repositioning the patient is appropriate but will not decrease the thickness of secretions.

The family members of a patient who has been admitted to the intensive care unit (ICU) with multiple traumatic injuries have just arrived in the ICU waiting room. Which action should the nurse take first? a. Explain ICU visitation policies and encourage family visits. b. Escort the family from the waiting room to the patient's bedside. c. Describe the patient's injuries and the care that is being provided. d. Invite the family to participate in an interprofessional care conference.

ANS: C Lack of information is a major source of anxiety for family members and should be addressed first. Family members should be prepared for the patient's appearance and the ICU environment before visiting the patient for the first time. ICU visiting should be individualized to each patient and family rather than being dictated by rigid visitation policies. Inviting the family to participate in a multidisciplinary conference is appropriate but should not be the initial action by the nurse.

A nurse is weaning a 68-kg patient who has chronic obstructive pulmonary disease (COPD) from mechanical ventilation. Which patient assessment finding indicates that the weaning protocol should be stopped? a. The patient's heart rate is 97 beats/min. b. The patient's oxygen saturation is 93%. c. The patient respiratory rate is 32 breaths/min. d. The patient's spontaneous tidal volume is 450 mL.

ANS: C Tachypnea is a sign that the patient's work of breathing is too high to allow weaning to proceed. The patient's heart rate is within normal limits, but the nurse should continue to monitor it. An O2 saturation of 93% is acceptable for a patient with COPD. A spontaneous tidal volume of 450 mL is within the acceptable range.

After change-of-shift report on a ventilator weaning unit, which patient should the nurse assess first? a. Patient who failed a spontaneous breathing trial and has been placed in a rest mode on the ventilator b. Patient who is intubated and has continuous partial pressure end-tidal CO2 (PETCO2) monitoring c. Patient who was successfully weaned and extubated 4 hours ago and has no urine output for the last 6 hours d. Patient with a central venous O2 saturation (ScvO2) of 69% while on bilevel positive airway pressure (BiPAP

ANS: C The decreased urine output may indicate acute kidney injury or that the patient's cardiac output and perfusion of vital organs have decreased. Any of these causes would require rapid action. The data about the other patients indicate that their conditions are stable and do not require immediate assessment or changes in their care. Continuous PETCO2 monitoring is frequently used when patients are intubated. The rest mode should be used to allow patient recovery after a failed SBT, and an ScvO2 of 69% is within normal limits.

Which assessment finding obtained by the nurse when caring for a patient receiving mechanical ventilation indicates the need for suctioning? a. The patient was last suctioned 6 hours ago. b. The patient's oxygen saturation drops to 93%. c. The patient's respiratory rate is 32 breaths/min. d. The patient has occasional audible expiratory wheezes.

ANS: C The increase in respiratory rate indicates that the patient may have decreased airway clearance and requires suctioning. Suctioning is done when patient assessment data indicate that it is needed and not on a scheduled basis. Occasional expiratory wheezes do not indicate poor airway clearance, and suctioning the patient may induce bronchospasm and increase wheezing. An O2 saturation of 93% is acceptable and does not suggest that immediate suctioning is needed.

The nurse notes that a patient's endotracheal tube (ET), which was at the 22-cm mark, is now at the 25-cm mark, and the patient is anxious and restless. Which action should the nurse take next? a. Check the O2 saturation. b. Offer reassurance to the patient. c. Listen to the patient's breath sounds. d. Notify the patient's health care provider.

ANS: C The nurse should first determine whether the ET tube has been displaced into the right mainstem bronchus by listening for unilateral breath sounds. If so, assistance will be needed to reposition the tube immediately. The other actions are also appropriate, but detection and correction of tube malposition are the most critical actions.

The nurse is caring for a patient receiving a continuous norepinephrine IV infusion. Which patient assessment finding indicates that the infusion rate may need to be adjusted? a. Heart rate is slow at 58 beats/min. b. Mean arterial pressure (MAP) is 56 mm Hg. c. Systemic vascular resistance (SVR) is elevated. d. Pulmonary artery wedge pressure (PAWP) is low.

ANS: C Vasoconstrictors such as norepinephrine will increase SVR, and this will increase the work of the heart and decrease peripheral perfusion. The infusion rate may need to be decreased. Bradycardia, hypotension (MAP of 56 mm Hg), and low PAWP are not associated with norepinephrine infusion.

2. Which information will the nurse include when teaching a patient with acute low back pain (select all that apply)? a. Sleep in a prone position with the legs extended. b. Keep the knees straight when leaning forward to pick something up. c. Avoid activities that require twisting of the back or prolonged sitting. d. Symptoms of acute low back pain frequently improve in a few weeks. e. Ibuprofen (Motrin, Advil) or acetaminophen (Tylenol) can be used to relieve pain.

ANS: C, D, E Acute back pain usually starts to improve within 2 weeks. In the meantime, the patient should use medications such as nonsteroidal antiinflammatory drugs (NSAIDs) or acetaminophen to manage pain and avoid activities that stress the back. Sleeping in a prone position and keeping the knees straight when leaning forward will place stress on the back, and should be avoided.

The emergency department (ED) nurse is starting therapeutic hypothermia in a patient who has been resuscitated after a cardiac arrest. Which actions in the hypothermia protocol can be delegated to an experienced licensed practical/vocational nurse (LPN/LVN) (select all that apply)? a. Continuously monitor heart rhythm. b. Assess neurologic status every 2 hours. c. Give acetaminophen (Tylenol) 650 mg. d. Place cooling blankets above and below patient. e. Attach rectal temperature probe to cooling blanket control panel.

ANS: C, D, E Experienced LPN/LVNs have the education and scope of practice to implement hypothermia measures (e.g., cooling blanket, temperature probe) and administer medications under the supervision of a registered nurse (RN). Assessment of neurologic status and monitoring the heart rhythm require RN-level education and scope of practice and should be done by the RN.

A patient with a large stomach tumor that is attached to the liver is scheduled to have a debulking procedure. Which information should the nurse teach the patient about the outcome of this procedure? a. Pain will be relieved by cutting sensory nerves in the stomach. b. Relief of pressure in the stomach will promote better nutrition. c. Tumor growth will be controlled by the removal of malignant tissue. d. Tumor size will decrease and this will improve the effects of other therapy.

ANS: D A debulking surgery reduces the size of the tumor and makes radiation and chemotherapy more effective. Debulking surgeries do not control tumor growth. The tumor is debulked because it is attached to the liver, a vital organ (not to relieve pressure on the stomach). Debulking does not sever the sensory nerves, although pain may be lessened by the reduction in pressure on the abdominal organs

3. A 64-year-old woman who has chronic constipation asks the nurse about the use of psyllium (Metamucil). Which information will the nurse include in the response? a. Absorption of fat-soluble vitamins may be reduced by fiber-containing laxatives. b. Dietary sources of fiber should be eliminated to prevent excessive gas formation. c. Use of this type of laxative to prevent constipation does not cause adverse effects. d. Large amounts of fluid should be taken to prevent impaction or bowel obstruction.

ANS: D A high fluid intake is needed when patients are using bulk-forming laxatives to avoid worsening constipation. Although bulk-forming laxatives are generally safe, the nurse should emphasize the possibility of constipation or obstipation if inadequate fluid intake occurs. Although increased gas formation is likely to occur with increased dietary fiber, the patient should gradually increase dietary fiber and eventually may not need the psyllium. Fat-soluble vitamin absorption is blocked by stool softeners and lubricants, not by bulk-forming laxatives.

A patient who is receiving immunotherapy has just received an allergen injection. Which assessment finding is most important to communicate to the health care provider? a. The patient's IgG level is increased. b. The injection site is red and swollen. c. The patient's allergy symptoms have not improved. d. There is a 2-cm wheal at the site of the allergen injection.

ANS: D A local reaction larger than quarter size may indicate that a decrease in the allergen dose is needed. An increase in IgG indicates that the therapy is effective. Redness and swelling at the site are not unusual. Because immunotherapy usually takes 1 to 2 years to achieve an effect, an improvement in the patient's symptoms is not expected after a few months

Which assessment finding is of most concern for a 46-year-old woman with acute pancreatitis? a. Absent bowel sounds b. Abdominal tenderness c. Left upper quadrant pain d. Palpable abdominal mass

ANS: D A palpable abdominal mass may indicate the presence of a pancreatic abscess, which will require rapid surgical drainage to prevent sepsis. Absent bowel sounds, abdominal tenderness, and left upper quadrant pain are common in acute pancreatitis and do not require rapid action to prevent further complications

27. A 42-year-old male patient has had a herniorrhaphy to repair an incarcerated inguinal hernia. Which patient teaching will the nurse provide before discharge? a. Soak in sitz baths several times each day. b. Cough 5 times each hour for the next 48 hours. c. Avoid use of acetaminophen (Tylenol) for pain. d. Apply a scrotal support and ice to reduce swelling.

ANS: D A scrotal support and ice are used to reduce edema and pain. Coughing will increase pressure on the incision. Sitz baths will not relieve pain and would not be of use after this surgery. Acetaminophen can be used for postoperative pain.

Which information is most important for the nurse to communicate rapidly to the health care provider about a patient admitted with possible syndrome of inappropriate antidiuretic hormone (SIADH)? a. The patient has a recent weight gain of 9 lb. b. The patient complains of dyspnea with activity. c. The patient has a urine specific gravity of 1.025. d. The patient has a serum sodium level of 118 mEq/L.

ANS: D A serum sodium of less than 120 mEq/L increases the risk for complications such as seizures and needs rapid correction. The other data are not unusual for a patient with SIADH and do not indicate the need for rapid action

17. A 73-year-old patient with diverticulosis has a large bowel obstruction. The nurse will monitor for a. referred back pain. b. metabolic alkalosis. c. projectile vomiting. d. abdominal distention.

ANS: D Abdominal distention is seen in lower intestinal obstruction. Referred back pain is not a common clinical manifestation of intestinal obstruction. Metabolic alkalosis is common in high intestinal obstruction because of the loss of HCl acid from vomiting. Projectile vomiting is associated with higher intestinal obstruction.

The nurse is obtaining a health history from a 24-yr-old patient with hypertrophic cardiomyopathy (CMP). Which information obtained by the nurse is most important? a. The patient has a history of a recent upper respiratory infection. b. The patient has a family history of coronary artery disease (CAD). c. The patient reports using cocaine a "couple of times" as a teenager. d. The patient's 29-yr-old brother died from a sudden cardiac arrest.

ANS: D About half of all cases of hypertrophic CMP have a genetic basis, and it is the most common cause of sudden cardiac death in otherwise healthy young people. The information about the patient's brother will be helpful in planning care (e.g., an automatic implantable cardioverter-defibrillator [AICD]) for the patient and in counseling other family members. The patient should be counseled against the use of stimulant drugs, but the limited past history indicates that the patient is not currently at risk for cocaine use. Viral infections and CAD are risk factors for dilated cardiomyopathy but not for hypertrophic CMP. DIF: Cognitive Level: Analyze (analysis)

A patient who has burns on the arms, legs, and chest from a house fire has become agitated and restless 8 hours after being admitted to the hospital. Which action should the nurse take first? a. Stay at the bedside and reassure the patient. b. Administer the ordered morphine sulfate IV. c. Assess orientation and level of consciousness. d. Use pulse oximetry to check the oxygen saturation.

ANS: D Agitation in a patient who may have suffered inhalation injury might indicate hypoxia, and this should be assessed by the nurse first. Administration of morphine may be indicated if the nurse determines that the agitation is caused by pain. Assessing level of consciousness and orientation is also appropriate but not as essential as determining whether the patient is hypoxemic. Reassurance is not helpful to reduce agitation in a hypoxemic patient

A patient with pneumonia has a fever of 101.4° F (38.6° C), a nonproductive cough, and an oxygen saturation of 88%. The patient complains of weakness, fatigue, and needs assistance to get out of bed. Which nursing diagnosis should the nurse assign as the highest priority? a. Hyperthermia related to infectious illness b. Impaired transfer ability related to weakness c. Ineffective airway clearance related to thick secretions d. Impaired gas exchange related to respiratory congestion

ANS: D All these nursing diagnoses are appropriate for the patient, but the patient's oxygen saturation indicates that all body tissues are at risk for hypoxia unless the gas exchange is improved

6. Which statement made by a patient with coronary artery disease after the nurse has completed teaching about therapeutic lifestyle changes (TLC) diet indicates that further teaching is needed? a. "I will switch from whole milk to 1% milk." b. "I like salmon and I will plan to eat it more often." c. "I can have a glass of wine with dinner if I want one." d. "I will miss being able to eat peanut butter sandwiches."

ANS: D Although only 30% of the daily calories should come from fats, most of the fat in the TLC diet should come from monosaturated fats such as are found in nuts, olive oil, and canola oil. The patient can include peanut butter sandwiches as part of the TLC diet. The other patient comments indicate a good understanding of the TLC diet.

15. Which assessment finding for a patient who has had a surgical reduction of an open fracture of the right radius is most important to report to the health care provider? a. Serous wound drainage b. Right arm muscle spasms c. Right arm pain with movement d. Temperature 101.4 F (38.6 C)

ANS: D An elevated temperature is suggestive of possible osteomyelitis. The other clinical manifestations are typical after a repair of an open fracture.

The nurse obtains information about a hospitalized patient who is receiving chemotherapy for colorectal cancer. Which information about the patient alerts the nurse to discuss a possible change in therapy with the health care provider? a. Poor oral intake b. Frequent loose stools c. Complaints of nausea and vomiting d. Increase in carcinoembryonic antigen (CEA)

ANS: D An increase in CEA indicates that the chemotherapy is not effective for the patient's cancer and may need to be modified. The other patient findings are common adverse effects of chemotherapy. The nurse may need to address these, but they would not necessarily indicate a need for a change in therapy

A patient being admitted with bacterial meningitis has a temperature of 102.5° F (39.2° C) and a severe headache. Which order should the nurse implement first? a. Administer ceftizoxime (Cefizox) 1 g IV. b. Give acetaminophen (Tylenol) 650 mg PO. c. Use a cooling blanket to lower temperature. d. Swab the nasopharyngeal mucosa for cultures.

ANS: D Antibiotic therapy should be instituted rapidly in bacterial meningitis, but cultures must be done before antibiotics are started. As soon as the cultures are done, the antibiotic should be started. Hypothermia therapy and acetaminophen administration are appropriate but can be started after the other actions are implemented.

Which action will the nurse include in the plan of care for a patient in the rehabilitation phase after a burn injury to the right arm and chest? a. Keep the right arm in a position of comfort. b. Avoid the use of sustained-release narcotics. c. Teach about the purpose of tetanus immunization. d. Apply water-based cream to burned areas frequently.

ANS: D Application of water-based emollients will moisturize new skin and decrease flakiness and itching. To avoid contractures, the joints of the right arm should be positioned in an extended position, which is not the position of comfort. Patients may need to continue the use of opioids during rehabilitation. Tetanus immunization would have been given during the emergent phase of the burn injury

1. When developing a teaching plan for a 61-year-old man with the following risk factors for coronary artery disease (CAD), the nurse should focus on the a. family history of coronary artery disease. b. increased risk associated with the patient's gender. c. increased risk of cardiovascular disease as people age. d. elevation of the patient's low-density lipoprotein (LDL) level.

ANS: D Because family history, gender, and age are nonmodifiable risk factors, the nurse should focus on the patient's LDL level. Decreases in LDL will help reduce the patient's risk for developing CAD.

A male patient with hemophilia asks the nurse if his children will be hemophiliacs. Which response by the nurse is appropriate? a. "All of your children will be at risk for hemophilia." b. "Hemophilia is a multifactorial inherited condition." c. "Only your male children are at risk for hemophilia." d. "Your female children will be carriers for hemophilia."

ANS: D Because hemophilia is caused by a mutation of the X chromosome, all female children of a man with hemophilia are carriers of the disorder and can transmit the mutated gene to their offspring. Sons of a man with hemophilia will not have the disorder. Hemophilia is caused by a single genetic mutation and is not a multifactorial inherited condition

The nurse assesses a patient who is receiving interleukin-2. Which finding should the nurse report immediately to the health care provider? a. Generalized muscle aches b. Complaints of nausea and anorexia c. Oral temperature of 100.6° F (38.1° C) d. Crackles heard at the lower scapular border

ANS: D Capillary leak syndrome and acute pulmonary edema are possible toxic effects of interleukin-2. The patient may need oxygen and the nurse should rapidly notify the health care provider. The other findings are common side effects of interleukin-2.

An alcoholic and homeless patient is diagnosed with active tuberculosis (TB). Which intervention by the nurse will be most effective in ensuring adherence with the treatment regimen? a. Arrange for a friend to administer the medication on schedule. b. Give the patient written instructions about how to take the medications. c. Teach the patient about the high risk for infecting others unless treatment is followed. d. Arrange for a daily noon meal at a community center where the drug will be administered.

ANS: D Directly observed therapy is the most effective means for ensuring compliance with the treatment regimen, and arranging a daily meal will help ensure that the patient is available to receive the medication. The other nursing interventions may be appropriate for some patients but are not likely to be as helpful for this patient

Esomeprazole (Nexium) is prescribed for a patient who incurred extensive burn injuries 5 days ago. Which nursing assessment would best evaluate the effectiveness of the medication? a. Bowel sounds b. Stool frequency c. Abdominal distention d. Stools for occult blood

ANS: D H2 blockers and proton pump inhibitors are given to prevent Curling's ulcer in the patient who has suffered burn injuries. Proton pump inhibitors usually do not affect bowel sounds, stool frequency, or appetite

Which laboratory result for a patient with multifocal premature ventricular contractions (PVCs) is most important for the nurse to communicate to the health care provider? a. Blood glucose of 243 mg/dL c. Serum sodium of 134 mEq/L b. Serum chloride of 92 mEq/L d. Serum potassium of 2.9 mEq/L

ANS: D Hypokalemia increases the risk for ventricular dysrhythmias such as PVCs, ventricular tachycardia, and ventricular fibrillation. The health care provider will need to prescribe a potassium infusion to correct this abnormality. Although the other laboratory values are also abnormal, they are not likely to be the etiology of the patient's PVCs and do not require immediate correction. DIF: Cognitive Level: Analyze (analysis)

A clinic patient is experiencing an allergic reaction to an unknown allergen. Which action is most appropriate for the registered nurse (RN) to delegate to a licensed practical/vocational nurse (LPN/LVN)? a. Perform a focused physical assessment. b. Obtain the health history from the patient. c. Teach the patient about the various diagnostic studies. d. Administer skin testing by the cutaneous scratch method.

ANS: D LPN/LVNs are educated and licensed to administer medications under the supervision of an RN. RN-level education and the scope of practice include assessment of health history, focused physical assessment, and patient teaching.

The nurse completes discharge teaching for a patient who has had a lung transplant. The nurse evaluates that the teaching has been effective if the patient makes which statement? a. "I will make an appointment to see the doctor every year." b. "I will stop taking the prednisone if I experience a dry cough." c. "I will not worry if I feel a little short of breath with exercise." d. "I will call the health care provider right away if I develop a fever."

ANS: D Low-grade fever may indicate infection or acute rejection so the patient should notify the health care provider immediately if the temperature is elevated. Patients require frequent follow-up visits with the transplant team. Annual health care provider visits would not be sufficient. Home oxygen use is not an expectation after lung transplant. Shortness of breath should be reported. Low-grade fever, fatigue, dyspnea, dry cough, and oxygen desaturation are signs of rejection. Immunosuppressive therapy, including prednisone, needs to be continued to prevent rejection

A patient has been admitted with meningococcal meningitis. Which observation by the nurse requires action? a. The patient receives a regular diet tray. b. The bedrails on both sides of the bed are elevated. c. Staff have turned off the lights in the patient's room. d. Staff have entered the patient's room without a mask.

ANS: D Meningococcal meningitis is spread by respiratory secretions, so it is important to maintain respiratory isolation as well as standard precautions. Because the patient may be confused and weak, bedrails should be elevated at both the foot and head of the bed. Low light levels in the room decrease pain caused by photophobia. Nutrition is an important aspect of care in a patient with meningitis.

A lobectomy is scheduled for a patient with stage I non-small cell lung cancer. The patient tells the nurse, "I would rather have chemotherapy than surgery." Which response by the nurse is most appropriate? a. "Are you afraid that the surgery will be very painful?" b. "Did you have bad experiences with previous surgeries?" c. "Surgery is the treatment of choice for stage I lung cancer." d. "Tell me what you know about the various treatments available."

ANS: D More assessment of the patient's concerns about surgery is indicated. An open-ended response will elicit the most information from the patient. The answer beginning, "Surgery is the treatment of choice" is accurate, but it discourages the patient from sharing concerns about surgery. The remaining two answers indicate that the nurse has jumped to conclusions about the patient's reasons for not wanting surgery. Chemotherapy is the primary treatment for small cell lung cancer. In non-small cell lung cancer, chemotherapy may be used in the treatment of nonresectable tumors or as adjuvant therapy to surgery

The emergency department (ED) triage nurse is assessing four victims involved in a motor vehicle collision. Which patient has the highestpriority for treatment? a. A patient with no pedal pulses b. A patient with an open femur fracture c. A patient with bleeding facial lacerations d. A patient with paradoxical chest movement

ANS: D Most immediate deaths from trauma occur because of problems with ventilation, so the patient with paradoxical chest movements should be treated first. Face and head fractures can obstruct the airway, but the patient with facial injuries only has lacerations. The other two patients also need rapid intervention but do not have airway or breathing problems.

34. A patient complains of leg cramps during hemodialysis. The nurse should a. massage the patient's legs. b. reposition the patient supine. c. give acetaminophen (Tylenol). d. infuse a bolus of normal saline.

ANS: D Muscle cramps during dialysis are caused by rapid removal of sodium and water. Treatment includes infusion of normal saline. The other actions do not address the reason for the cramps. DIF: Cognitive Level: Apply (application) REF: 1091 TOP: Nursing Process: Implementation MSC: NCLEX: Physiological Integrity

Which assessment finding would the nurse need to report most quickly to the health care provider regarding a patient with acute pancreatitis? a. Nausea and vomiting b. Hypotonic bowel sounds c. Abdominal tenderness and guarding d. Muscle twitching and finger numbness

ANS: D Muscle twitching and finger numbness indicate hypocalcemia, which may lead to tetany unless calcium gluconate is administered. Although the other findings should also be reported to the health care provider, they do not indicate complications that require rapid action

A 67-year-old male patient with acute pancreatitis has a nasogastric (NG) tube to suction and is NPO. Which information obtained by the nurse indicates that these therapies have been effective? a. Bowel sounds are present. b. Grey Turner sign resolves. c. Electrolyte levels are normal. d. Abdominal pain is decreased.

ANS: D NG suction and NPO status will decrease the release of pancreatic enzymes into the pancreas and decrease pain. Although bowel sounds may be hypotonic with acute pancreatitis, the presence of bowel sounds does not indicate that treatment with NG suction and NPO status has been effective. Electrolyte levels may be abnormal with NG suction and must be replaced by appropriate IV infusion. Although Grey Turner sign will eventually resolve, it would not be appropriate to wait for this to occur to determine whether treatment was effective

Which finding by the nurse when assessing a patient with a large pituitary adenoma is most important to report to the health care provider? a. Changes in visual field b. Milk leaking from breasts c. Blood glucose 150 mg/dL d. Nausea and projectile vomiting

ANS: D Nausea and projectile vomiting may indicate increased intracranial pressure, which will require rapid actions for diagnosis and treatment. Changes in the visual field, elevated blood glucose, and galactorrhea are common with pituitary adenoma, but these do not require rapid action to prevent life-threatening complications

A patient is admitted with active tuberculosis (TB). The nurse should question a health care provider's order to discontinue airborne precautions unless which assessment finding is documented? a. Chest x-ray shows no upper lobe infiltrates. b. TB medications have been taken for 6 months. c. Mantoux testing shows an induration of 10 mm. d. Three sputum smears for acid-fast bacilli are negative.

ANS: D Negative sputum smears indicate that Mycobacterium tuberculosis is not present in the sputum, and the patient cannot transmit the bacteria by the airborne route. Chest x-rays are not used to determine whether treatment has been successful. Taking medications for 6 months is necessary, but the multidrug-resistant forms of the disease might not be eradicated after 6 months of therapy. Repeat Mantoux testing would not be done because the result will not change even with effective treatment

The nurse is caring for a patient undergoing plasmapheresis. The nurse should assess the patient for which clinical manifestation? a. Shortness of breath b. High blood pressure c. Transfusion reaction d. Numbness and tingling

ANS: D Numbness and tingling may occur as the result of the hypocalcemia caused by the citrate used to prevent coagulation. The other clinical manifestations are not associated with plasmapheresis.

34. Which question from the nurse would help determine if a patient's abdominal pain might indicate irritable bowel syndrome? a. "Have you been passing a lot of gas?" b. "What foods affect your bowel patterns?" c. "Do you have any abdominal distention?" d. "How long have you had abdominal pain?"

ANS: D One criterion for the diagnosis of irritable bowel syndrome (IBS) is the presence of abdominal discomfort or pain for at least 3 months. Abdominal distention, flatulence, and food intolerance are also associated with IBS, but are not diagnostic criteria.

The nurse is reviewing the medication administration record (MAR) on a patient with partial-thickness burns. Which medication is best for the nurse to administer before scheduled wound debridement? a. Ketorolac (Toradol) b. Lorazepam (Ativan) c. Gabapentin (Neurontin) d. Hydromorphone (Dilaudid)

ANS: D Opioid pain medications are the best choice for pain control. The other medications are used as adjuvants to enhance the effects of opioids

The nurse is caring for a patient who was admitted the previous day with a basilar skull fracture after a motor vehicle crash. Which assessment finding indicates a possible complication that should be reported to the health care provider? a. Complaint of severe headache b. Large contusion behind left ear c. Bilateral periorbital ecchymosis d. Temperature of 101.4° F (38.6° C)

ANS: D Patients who have basilar skull fractures are at risk for meningitis, so the elevated temperature should be reported to the health care provider. The other findings are typical of a patient with a basilar skull fracture.

The nurse is performing tuberculosis (TB) skin tests in a clinic that has many patients who have immigrated to the United States. Which question is most important for the nurse to ask before the skin test? a. "Is there any family history of TB?" b. "How long have you lived in the United States?" c. "Do you take any over-the-counter (OTC) medications?" d. "Have you received the bacille Calmette-Guérin (BCG) vaccine for TB?"

ANS: D Patients who have received the BCG vaccine will have a positive Mantoux test. Another method for screening (such as a chest x-ray) will need to be used in determining whether the patient has a TB infection. The other information also may be valuable but is not as pertinent to the decision about doing TB skin testing

Which statement by a patient with restrictive cardiomyopathy indicates that the nurse's discharge teaching about self-management has been effective? a. "I will avoid taking aspirin or other antiinflammatory drugs." b. "I can restart my exercise program that includes hiking and biking." c. "I will need to limit my intake of salt and fluids even in hot weather." d. "I will take antibiotics before my teeth are cleaned at the dental office."

ANS: D Patients with restrictive cardiomyopathy are at risk for infective endocarditis and should use prophylactic antibiotics for any procedure that may cause bacteremia. The other statements indicate a need for more teaching by the nurse. Dehydration and vigorous exercise impair ventricular filling in patients with restrictive cardiomyopathy. There is no need to avoid salt (unless ordered), aspirin, or nonsteroidal antiinflammatory drugs. DIF: Cognitive Level: Apply (application)

11. A 54-year-old woman who recently reached menopause and has a family history of osteoporosis is diagnosed with osteopenia following densitometry testing. In teaching the woman about her osteoporosis, the nurse explains that a. estrogen replacement therapy must be started to prevent rapid progression to osteoporosis. b. continuous, low-dose corticosteroid treatment is effective in stopping the course of osteoporosis. c. with a family history of osteoporosis, there is no way to prevent or slow gradual bone resorption. d. calcium loss from bones can be slowed by increasing calcium intake and weight-bearing exercise.

ANS: D Progression of osteoporosis can be slowed by increasing calcium intake and weight-bearing exercise. Estrogen replacement therapy does help prevent osteoporosis, but it is not the only treatment and is not appropriate for some patients. Corticosteroid therapy increases the risk for osteoporosis.

43. After reviewing a patient's history, vital signs, physical assessment, and laboratory data, which information shown in the accompanying figure is most important for the nurse to communicate to the health care provider? a. Q waves on ECG b. Elevated troponin levels c. Fever and hyperglycemia d. Tachypnea and crackles in lungs

ANS: D Pulmonary congestion and tachypnea suggest that the patient may be developing heart failure, a complication of myocardial infarction (MI). Mild fever and hyperglycemia are common after MI because of the inflammatory process that occurs with tissue necrosis. Troponin levels will be elevated for several days after MI. Q waves often develop with ST-segment-elevation MI.

After having a craniectomy and left anterior fossae incision, a 64-yr-old patient has impaired physical mobility related to decreased level of consciousness and weakness. An appropriate nursing intervention is to a. cluster nursing activities to allow longer rest periods. b. turn and reposition the patient side to side every 2 hours. c. position the bed flat and log roll to reposition the patient. d. perform range-of-motion (ROM) exercises every 4 hours.

ANS: D ROM exercises will help prevent the complications of immobility. Patients with anterior craniotomies are positioned with the head elevated. The patient with a craniectomy should not be turned to the operative side. When the patient is weak, clustering nursing activities may lead to more fatigue and weakness.

An older adult patient who has colorectal cancer is receiving IV fluids at 175 mL/hour in conjunction with the prescribed chemotherapy. Which finding by the nurse is most important to report to the health care provider? a. Patient complains of severe fatigue. b. Patient needs to void every hour during the day. c. Patient takes only 50% of meals and refuses snacks. d. Patient has audible crackles to the midline posterior chest.

ANS: D Rapid fluid infusions may cause heart failure, especially in older patients. The other findings are common in patients who have cancer and/or are receiving chemotherapy

When assessing a 53-yr-old patient with bacterial meningitis, the nurse obtains the following data. Which finding requires the most immediate intervention? a. The patient exhibits nuchal rigidity. b. The patient has a positive Kernig's sign. c. The patient's temperature is 101° F (38.3° C). d. The patient's blood pressure is 88/42 mm Hg.

ANS: D Shock is a serious complication of meningitis, and the patient's low blood pressure indicates the need for interventions such as fluids or vasopressors. Nuchal rigidity and a positive Kernig's sign are expected with bacterial meningitis. The nurse should intervene to lower the temperature, but this is not as life threatening as the hypotension.

Which finding indicates to the nurse that a patient's transjugular intrahepatic portosystemic shunt (TIPS) placed 3 months ago has been effective? a. Increased serum albumin level b. Decreased indirect bilirubin level c. Improved alertness and orientation d. Fewer episodes of bleeding varices

ANS: D TIPS is used to lower pressure in the portal venous system and decrease the risk of bleeding from esophageal varices. Indirect bilirubin level and serum albumin levels are not affected by shunting procedures. TIPS will increase the risk for hepatic encephalopathy

To determine whether there is a delay in impulse conduction through the ventricles, the nurse will measure the duration of the patient's a. P wave. c. PR interval. b. Q wave. d. QRS complex.

ANS: D The QRS complex represents ventricular depolarization. The P wave represents the depolarization of the atria. The PR interval represents depolarization of the atria, atrioventricular node, a bundle of His, bundle branches, and the Purkinje fibers. The Q wave is the first negative deflection following the P wave and should be narrow and short. Cognitive Level: Understand (comprehension)Chapter

The nurse obtains a rhythm strip on a patient who has had a myocardial infarction and makes the following analysis: no visible P waves, PR interval not measurable, ventricular rate of 162, R-R interval regular, and QRS complex wide and distorted, and QRS duration of 0.18 second. The nurse interprets the patient's cardiac rhythm as a. atrial flutter. c. ventricular fibrillation. b. sinus tachycardia. d. ventricular tachycardia.

ANS: D The absence of P waves, wide QRS, rate greater than 150 beats/min, and the regularity of the rhythm indicate ventricular tachycardia. Atrial flutter is usually regular, has a narrow QRS configuration, and has flutter waves present representing atrial activity. Sinus tachycardia has P waves. Ventricular fibrillation is irregular and does not have a consistent QRS duration. DIF: Cognitive Level: Apply (application)

A patient who has ovarian cancer is crying and tells the nurse, "My husband rarely visits. He just doesn't care." The husband indicates to the nurse that he never knows what to say to help his wife. Which nursing diagnosis is most appropriate for the nurse to add to the plan of care? a. Compromised family coping related to disruption in lifestyle b. Impaired home maintenance related to perceived role changes c. Risk for caregiver role strain related to burdens of caregiving responsibilities d. Dysfunctional family processes related to effect of illness on family members

ANS: D The data indicate that this diagnosis is most appropriate because poor communication among the family members is affecting family processes. No data suggest a change in lifestyle or its role as an etiology. The data do not support impairment in home maintenance or a burden caused by caregiving responsibilities

After the emergency department nurse has received a status report on the following patients who have been admitted with head injuries, which patient should the nurse assess first? a. A 20-yr-old patient whose cranial x-ray shows a linear skull fracture b. A 50-yr-old patient who has an initial Glasgow Coma Scale score of 13 c. A 30-yr-old patient who lost consciousness for a few seconds after a fall d. A 40-yr-old patient whose right pupil is 10 mm and unresponsive to light

ANS: D The dilated and nonresponsive pupil may indicate an intracerebral hemorrhage and increased intracranial pressure. The other patients are not at immediate risk for complications such as herniation.

The nurse has received change-of-shift report about the following patients on the progressive care unit. Which patient should the nurse see first? a. A patient with atrial fibrillation, rate 88 and irregular, who has a dose of warfarin (Coumadin) due b. A patient with second-degree atrioventricular (AV) block, type 1, rate 60, who is dizzy when ambulating c. A patient who is in a sinus rhythm, rate 98 and regular, recovering from an elective cardioversion 2 hours ago d. A patient whose implantable cardioverter-defibrillator (ICD) fired twice today and has a dose of amiodarone (Cordarone) due

ANS: D The frequent firing of the ICD indicates that the patient's ventricles are very irritable and the priority is to assess the patient and give the amiodarone. The other patients can be seen after the amiodarone is given. DIF: Cognitive Level: Analyze (analysis)

26. A patient with acute kidney injury (AKI) has longer QRS intervals on the electrocardiogram (ECG) than were noted on the previous shift. Which action should the nurse take first? a. Notify the patient's health care provider. b. Document the QRS interval measurement. c. Review the chart for the patient's current creatinine level. d. Check the medical record for the most recent potassium level.

ANS: D The increasing QRS interval is suggestive of hyperkalemia, so the nurse should check the most recent potassium and then notify the patient's health care provider. The BUN and creatinine will be elevated in a patient with AKI, but they would not directly affect the electrocardiogram (ECG). Documentation of the QRS interval is also appropriate, but interventions to decrease the potassium level are needed to prevent life-threatening dysrhythmias. DIF: Cognitive Level: Analyze (analysis) REF: 1072 OBJ: Special Questions: Prioritization TOP: Nursing Process: Implementation MSC: NCLEX: Physiological Integrity

A patient arrives in the emergency department (ED) after topical exposure to powdered lime at work. Which action should the nurse take first? a. Obtain the patient's vital signs. b. Obtain a baseline complete blood count. c. Decontaminate the patient by showering with water. d. Brush off any visible powder on the skin and clothing.

ANS: D The initial action should be to protect staff members and decrease the patient's exposure to the toxin by decontamination. Patients exposed to powdered lime should not be showered; instead, any and all visible powder should be brushed off. The other actions can be done after the decontamination is completed.

The nurse reviews the laboratory results of a patient who is receiving chemotherapy. Which laboratory result is most important to report to the health care provider? a. Hematocrit of 30% b. Platelets of 95,000/µL c. Hemoglobin of 10 g/L d. White blood cell (WBC) count of 2700/µL

ANS: D The low WBC count places the patient at risk for severe infection and is an indication that the chemotherapy dose may need to be lower or that WBC growth factors such as filgrastim (Neupogen) are needed. Although the other laboratory data indicate decreased levels, they do not indicate any immediate life-threatening adverse effects of the chemotherapy

Which data will the nurse monitor in relation to the 4+ pitting edema assessed in a patient with cirrhosis? a. Hemoglobin b. Temperature c. Activity level d. Albumin level

ANS: D The low oncotic pressure caused by hypoalbuminemia is a major pathophysiologic factor in the development of edema. The other parameters should also be monitored, but they are not directly associated with the patient's current symptoms

A 44-year-old female patient with Cushing syndrome is admitted for adrenalectomy. Which intervention by the nurse will be most helpful for a nursing diagnosis of disturbed body image related to changes in appearance? a. Reassure the patient that the physical changes are very common in patients with Cushing syndrome. b. Discuss the use of diet and exercise in controlling the weight gain associated with Cushing syndrome. c. Teach the patient that the metabolic impact of Cushing syndrome is of more importance than appearance. d. Remind the patient that most of the physical changes caused by Cushing syndrome will resolve after surgery.

ANS: D The most reassuring communication to the patient is that the physical and emotional changes caused by the Cushing syndrome will resolve after hormone levels return to normal postoperatively. Reassurance that the physical changes are expected or that there are more serious physiologic problems associated with Cushing syndrome are not therapeutic responses. The patient's physiological changes are caused by the high hormone levels, not by the patient's diet or exercise choices

9. The nurse will determine that more teaching is needed if a patient with discomfort from a bunion says, I will a. give away my high-heeled shoes. b. take ibuprofen (Motrin) if I need it. c. use the bunion pad to cushion the area. d. only wear sandals, no closed-toe shoes.

ANS: D The patient can wear shoes that have a wide forefoot. The other patient statements indicate that the teaching has been effective.

A patient with a possible pulmonary embolism complains of chest pain and difficulty breathing. The nurse finds a heart rate of 142 beats/minute, blood pressure of 100/60 mmHg, and respirations of 42 breaths/minute. Which action should the nurse take first? a. Administer anticoagulant drug therapy. b. Notify the patient's health care provider. c. Prepare patient for a spiral computed tomography (CT). d. Elevate the head of the bed to a semi-Fowler's position.

ANS: D The patient has symptoms consistent with a pulmonary embolism (PE). Elevating the head of the bed will improve ventilation and gas exchange. The other actions can be accomplished after the head is elevated (and oxygen is started). A spiral CT may be ordered by the health care provider to identify PE. Anticoagulants may be ordered after confirmation of the diagnosis of PE

A patient develops sinus bradycardia at a rate of 32 beats/min, has a blood pressure (BP) of 80/42 mm Hg, and is complaining of feeling faint. Which action should the nurse take next? a. Recheck the heart rhythm and BP in 5 minutes. b. Have the patient perform the Valsalva maneuver. c. Give the scheduled dose of diltiazem (Cardizem). d. Apply the transcutaneous pacemaker (TCP) pads.

ANS: D The patient is experiencing symptomatic bradycardia and treatment with TCP is appropriate. Continued monitoring of the rhythm and BP is an inadequate response. Calcium channel blockers will further decrease the heart rate and the diltiazem should be held. The Valsalva maneuver will further decrease the rate. DIF: Cognitive Level: Apply (application)

The nurse knows that discharge teaching about the management of a new permanent pacemaker has been most effective when the patient states a. "It will be several weeks before I can return to my usual activities." b. "I will avoid cooking with a microwave oven or being near one in use." c. "I will notify the airlines when I make a reservation that I have a pacemaker." d. "I won't lift the arm on the pacemaker side until I see the health care provider."

ANS: D The patient is instructed to avoid lifting the arm on the pacemaker side above the shoulder to avoid displacing the pacemaker leads. The patient should notify airport security about the presence of a pacemaker before going through the metal detector, but there is no need to notify the airlines when making a reservation. Microwave oven use does not affect the pacemaker. The insertion procedure involves minor surgery that will have a short recovery period. DIF: Cognitive Level: Apply (application)

An hour after a thoracotomy, a patient complains of incisional pain at a level 7 (based on 0 to 10 scale) and has decreased left-sided breath sounds. The pleural drainage system has 100 mL of bloody drainage and a large air leak. Which action is best for the nurse to take next? a. Milk the chest tube gently to remove any clots. b. Clamp the chest tube momentarily to check for the origin of the air leak. c. Assist the patient to deep breathe, cough, and use the incentive spirometer. d. Set up the patient controlled analgesia (PCA) and administer the loading dose of morphine.

ANS: D The patient is unlikely to take deep breaths or cough until the pain level is lower. A chest tube output of 100 mL is not unusual in the first hour after thoracotomy and would not require milking of the chest tube. An air leak is expected in the initial postoperative period after thoracotomy

A patient with leukemia is considering whether to have hematopoietic stem cell transplantation (HSCT). The nurse will include which information in the patient's teaching plan? a. Transplant of the donated cells is painful because of the nerves in the tissue lining the bone. b. Donor bone marrow cells are transplanted through an incision into the sternum or hip bone. c. The transplant procedure takes place in a sterile operating room to minimize the risk for infection. d. Hospitalization will be required for several weeks after the stem cell transplant procedure is performed.

ANS: D The patient requires strict protective isolation to prevent infection for 2 to 4 weeks after HSCT while waiting for the transplanted marrow to start producing cells. The transplanted cells are infused through an IV line, so the transplant is not painful, nor is an operating room or incision required

A patient admitted with acute dyspnea is newly diagnosed with dilated cardiomyopathy. Which information will the nurse plan to teach the patient about managing this disorder? a. A heart transplant should be scheduled as soon as possible. b. Elevating the legs above the heart will help relieve dyspnea. c. Careful compliance with diet and medications will prevent heart failure. d. Notify the health care provider about symptoms such as shortness of breath.

ANS: D The patient should be instructed to notify the health care provider about any worsening of heart failure symptoms. Because dilated cardiomyopathy does not respond well to therapy, even patients with good compliance with therapy may have recurrent episodes of heart failure. Elevation of the legs above the heart will worsen symptoms (although this approach is appropriate for a patient with hypertrophic cardiomyopathy). The patient with terminal or end-stage cardiomyopathy may consider heart transplantation. DIF: Cognitive Level: Apply (application)

A patient receiving head and neck radiation for larynx cancer has ulcerations over the oral mucosa and tongue and thick, ropey saliva. Which instructions should the nurse give to this patient? a. Remove food debris from the teeth and oral mucosa with a stiff toothbrush. b. Use cotton-tipped applicators dipped in hydrogen peroxide to clean the teeth. c. Gargle and rinse the mouth several times a day with an antiseptic mouthwash. d. Rinse the mouth before and after each meal and at bedtime with a saline solution.

ANS: D The patient should rinse the mouth with a saline solution frequently. A soft toothbrush is used for oral care. Hydrogen peroxide may damage tissues. Antiseptic mouthwashes may irritate the oral mucosa and are not recommended

36. After receiving change-of-shift report, which patient should the nurse assess first? a. Patient who is scheduled for the drain phase of a peritoneal dialysis exchange b. Patient with stage 4 chronic kidney disease who has an elevated phosphate level c. Patient with stage 5 chronic kidney disease who has a potassium level of 3.4 mEq/L d. Patient who has just returned from having hemodialysis and has a heart rate of 124/min

ANS: D The patient who has tachycardia after hemodialysis may be bleeding or excessively hypovolemic and should be assessed immediately for these complications. The other patients also need assessments or interventions but are not at risk for life-threatening complications. DIF: Cognitive Level: Analyze (analysis) REF: 1091 OBJ: Special Questions: Prioritization | Special Questions: Multiple Patients TOP: Nursing Process: Assessment MSC: NCLEX: Safe and Effective Care Environment

44. Which patient should the nurse assess first after receiving change-of-shift report? a. 60-year-old patient whose new ileostomy has drained 800 mL over the previous 8 hours b. 50-year-old patient with familial adenomatous polyposis who has occult blood in the stool c. 40-year-old patient with ulcerative colitis who has had six liquid stools in the previous 4 hours d. 30-year-old patient who has abdominal distention and an apical heart rate of 136 beats/minute

ANS: D The patient's abdominal distention and tachycardia suggest hypovolemic shock caused by problems such as peritonitis or intestinal obstruction, which will require rapid intervention. The other patients should also be assessed as quickly as possible, but the data do not indicate any life-threatening complications associated with their diagnoses.

7. A 27-year-old female patient is admitted to the hospital for evaluation of right lower quadrant abdominal pain with nausea and vomiting. Which action should the nurse take? a. Encourage the patient to sip clear liquids. b. Assess the abdomen for rebound tenderness. c. Assist the patient to cough and deep breathe. d. Apply an ice pack to the right lower quadrant.

ANS: D The patient's clinical manifestations are consistent with appendicitis, and application of an ice pack will decrease inflammation at the area. Checking for rebound tenderness frequently is unnecessary and uncomfortable for the patient. The patient should be NPO in case immediate surgery is needed. The patient will need to know how to cough and deep breathe postoperatively, but coughing will increase pain at this time.

When assessing a patient who has just arrived after an automobile accident, the emergency department nurse notes tachycardia and absent breath sounds over the right lung. For which intervention will the nurse prepare the patient? a. Emergency pericardiocentesis b. Stabilization of the chest wall with tape c. Administration of an inhaled bronchodilator d. Insertion of a chest tube with a chest drainage system

ANS: D The patient's history and absent breath sounds suggest a right-sided pneumothorax or hemothorax, which will require treatment with a chest tube and drainage. The other therapies would be appropriate for an acute asthma attack, flail chest, or cardiac tamponade, but the patient's clinical manifestations are not consistent with these problems

43. After several days of antibiotic therapy, an older hospitalized patient develops watery diarrhea. Which action should the nurse take first? a. Notify the health care provider. b. Obtain a stool specimen for analysis. c. Teach the patient about handwashing. d. Place the patient on contact precautions.

ANS: D The patient's history and new onset diarrhea suggest a C. difficile infection, which requires implementation of contact precautions to prevent spread of the infection to other patients. The other actions are also appropriate but can be accomplished after contact precautions are implemented.

A patient who was found unconscious in a burning house is brought to the emergency department by ambulance. The nurse notes that the patient's skin color is bright red. Which action should the nurse take first? a. Insert two large-bore IV lines. b. Check the patient's orientation. c. Assess for singed nasal hair and dark oral mucous membranes. d. Place the patient on 100% oxygen using a non-rebreather mask.

ANS: D The patient's history and skin color suggest carbon monoxide poisoning, which should be treated by rapidly starting oxygen at 100%. The other actions can be taken after the action to correct gas exchange

After change-of-shift report, which patient should the nurse assess first? a. 72-year-old with cor pulmonale who has 4+ bilateral edema in his legs and feet b. 28-year-old with a history of a lung transplant and a temperature of 101° F (38.3° C) c. 40-year-old with a pleural effusion who is complaining of severe stabbing chest pain d. 64-year-old with lung cancer and tracheal deviation after subclavian catheter insertion

ANS: D The patient's history and symptoms suggest possible tension pneumothorax, a medical emergency. The other patients also require assessment as soon as possible, but tension pneumothorax will require immediate treatment to avoid death from inadequate cardiac output or hypoxemia

37. A 45-year-old patient is admitted to the emergency department with severe abdominal pain and rebound tenderness. Vital signs include temperature 102° F (38.3° C), pulse 120, respirations 32, and blood pressure (BP) 82/54. Which prescribed intervention should the nurse implement first? a. Administer IV ketorolac (Toradol) 15 mg. b. Draw blood for a complete blood count (CBC). c. Obtain a computed tomography (CT) scan of the abdomen. d. Infuse 1 liter of lactated Ringer's solution over 30 minutes.

ANS: D The priority for this patient is to treat the patient's hypovolemic shock with fluid infusion. The other actions should be implemented after starting the fluid infusion.

The nurse teaches a postmenopausal patient with stage III breast cancer about the expected outcomes of cancer treatment. Which patient statement indicates that the teaching has been effective? a. "After cancer has not recurred for 5 years, it is considered cured." b. "The cancer will be cured if the entire tumor is surgically removed." c. "Cancer is never considered cured, but the tumor can be controlled with surgery, chemotherapy, and radiation." d. "I will need to have follow-up examinations for many years after I have treatment before I can be considered cured."

ANS: D The risk of recurrence varies by the type of cancer. Some cancers are considered cured after a shorter time span or after surgery, but stage III breast cancer will require additional therapies and ongoing follow-up

22. A 47-year-old female patient is transferred from the recovery room to a surgical unit after a transverse colostomy. The nurse observes the stoma to be deep pink with edema and a small amount of sanguineous drainage. The nurse should a. place ice packs around the stoma. b. notify the surgeon about the stoma. c. monitor the stoma every 30 minutes. d. document stoma assessment findings.

ANS: D The stoma appearance indicates good circulation to the stoma. There is no indication that surgical intervention is needed or that frequent stoma monitoring is required. Swelling of the stoma is normal for 2 to 3 weeks after surgery, and an ice pack is not needed.

Eight hours after a thermal burn covering 50% of a patient's total body surface area (TBSA) the nurse assesses the patient. Which information would be a priority to communicate to the health care provider? a. Blood pressure is 95/48 per arterial line. b. Serous exudate is leaking from the burns. c. Cardiac monitor shows a pulse rate of 108. d. Urine output is 20 mL per hour for the past 2 hours.

ANS: D The urine output should be at least 0.5 to 1.0 mL/kg/hr during the emergent phase, when the patient is at great risk for hypovolemic shock. The nurse should notify the health care provider because a higher IV fluid rate is needed. BP during the emergent phase should be greater than 90 systolic, and the pulse rate should be less than 120. Serous exudate from the burns is expected during the emergent phase

A young adult patient who is in the rehabilitation phase after having deep partial-thickness face and neck burns has a nursing diagnosis of disturbed body image. Which statement by the patient indicates that the problem is resolving? a. "I'm glad the scars are only temporary." b. "I will avoid using a pillow, so my neck will be OK." c. "I bet my boyfriend won't even want to look at me anymore." d. "Do you think dark beige makeup foundation would cover this scar on my cheek?"

ANS: D The willingness to use strategies to enhance appearance is an indication that the disturbed body image is resolving. Expressing feelings about the scars indicates a willingness to discuss appearance, but not resolution of the problem. Because deep partial-thickness burns leave permanent scars, a statement that the scars are temporary indicates denial rather than resolution of the problem. Avoiding using a pillow will help prevent contractures, but it does not address the problem of disturbed body image

38. After receiving change-of-shift report about the following four patients, which patient should the nurse assess first? a. 39-year-old with pericarditis who is complaining of sharp, stabbing chest pain b. 56-year-old with variant angina who is to receive a dose of nifedipine (Procardia) c. 65-year-old who had a myocardial infarction (MI) 4 days ago and is anxious about the planned discharge d. 59-year-old with unstable angina who has just returned to the unit after having a percutaneous coronary intervention (PCI)

ANS: D This patient is at risk for bleeding from the arterial access site for the PCI, so the nurse should assess the patient's blood pressure, pulse, and the access site immediately. The other patients should also be assessed as quickly as possible, but assessment of this patient has the highest priority.

8. A patient who has had chest pain for several hours is admitted with a diagnosis of rule out acute myocardial infarction (AMI). Which laboratory test should the nurse monitor to help determine whether the patient has had an AMI? a. Myoglobin b. Homocysteine c. C-reactive protein d. Cardiac-specific troponin

ANS: D Troponin levels increase about 4 to 6 hours after the onset of myocardial infarction (MI) and are highly specific indicators for MI. Myoglobin is released within 2 hours of MI, but it lacks specificity and its use is limited. The other laboratory data are useful in determining the patient's risk for developing coronary artery disease (CAD) but are not helpful in determining whether an acute MI is in progress.

Which action could the nurse delegate to unlicensed assistive personnel (UAP) trained as electrocardiogram (ECG) technicians working on the cardiac unit? a. Select the best lead for monitoring a patient with an admission diagnosis of Dressler syndrome. b. Obtain a list of herbal medications used at home while admitting a new patient with pericarditis. c. Teach about the need to monitor the weight daily for a patient who has hypertrophic cardiomyopathy. d. Watch the heart monitor for changes in rhythm while a patient who had a valve replacement ambulates.

ANS: D Under the supervision of registered nurses (RNs), UAPs check the patient's cardiac monitor and obtain information about changes in heart rate and rhythm with exercise. Teaching and obtaining information about home medications (prescribed or complementary) and selecting the best leads for monitoring patients require more critical thinking and should be done by the RN. DIF: Cognitive Level: Apply (application)

Which action should the nurse take when caring for a patient who is receiving chemotherapy and complains of problems with concentration? a. Teach the patient to rest the brain by avoiding new activities. b. Teach that "chemo-brain" is a short-term effect of chemotherapy. c. Report patient symptoms immediately to the health care provider. d. Suggest use of a daily planner and encourage adequate rest and sleep.

ANS: D Use of tools to enhance memory and concentration such as a daily planner and adequate rest are helpful for patients who develop "chemo-brain" while receiving chemotherapy. Patients should be encouraged to exercise the brain through new activities. Chemo-brain may be short- or long-term. There is no urgent need to report common chemotherapy side effects to the provider

During the emergent phase of burn care, which assessment will be most useful in determining whether the patient is receiving adequate fluid infusion? a. Check skin turgor. b. Monitor daily weight. c. Assess mucous membranes. d. Measure hourly urine output.

ANS: D When fluid intake is adequate, the urine output will be at least 0.5 to 1 mL/kg/hour. The patient's weight is not useful in this situation because of the effects of third spacing and evaporative fluid loss. Mucous membrane assessment and skin turgor also may be used, but they are not as adequate in determining that fluid infusions are maintaining adequate perfusion

When monitoring the effectiveness of treatment for a patient with a large anterior wall myocardial infarction, the most pertinent measurement for the nurse to obtain is a. central venous pressure (CVP). b. systemic vascular resistance (SVR). c. pulmonary vascular resistance (PVR). d. pulmonary artery wedge pressure (PAWP).

ANS: D PAWP reflects left ventricular end diastolic pressure (or left ventricular preload) and is a sensitive indicator of cardiac function. Because the patient is high risk for left ventricular failure, the PAWP must be monitored. An increase will indicate left ventricular failure. The other values would also provide useful information, but the most definitive measurement of changes in cardiac function is the PAWP.

After change-of-shift report, which patient should the progressive care nurse assess first? a. Patient who was extubated this morning and has a temperature of 101.4°F (38.6°C) b. Patient with bilevel positive airway pressure (BiPAP) for obstructive sleep apnea and a respiratory rate of 16 c. Patient with arterial pressure monitoring who is 2 hours post-percutaneous coronary intervention and needs to void d. Patient who is receiving IV heparin for a venous thromboembolism and has a partial thromboplastin time (PTT) of 101 sec

ANS: D The findings for this patient indicate high risk for bleeding from an elevated (nontherapeutic) PTT. The nurse needs to adjust the rate of the infusion (dose) per the health care provider's parameters. The patient with BiPAP for sleep apnea has a normal respiratory rate. The patient recovering from the percutaneous coronary intervention will need to be assisted with voiding and this task could be delegated to unlicensed assistive personnel. The patient with a fever may be developing ventilator-associated pneumonia, but addressing the bleeding risk is a higher priority.

The nurse educator is evaluating the care that a new registered nurse (RN) provides to a patient receiving mechanical ventilation. Which action by the new RN indicates the need for more education? a. The RN increases the FIO2 to 100% before suctioning. b. The RN secures a bite block in place using adhesive tape. c. The RN asks for assistance to resecure the endotracheal tube. d. The RN positions the patient with the head of bed at 10 degrees.

ANS: D The head of the patient's bed should be positioned at 30 to 45 degrees to prevent ventilator-associated pneumonia. The other actions by the new RN are appropriate

While waiting for heart transplantation, a patient with severe cardiomyopathy has a ventricular assist device (VAD) implanted. When planning care for this patient, the nurse should anticipate a. preparing the patient for a permanent VAD. b. administering immunosuppressive medications. c. teaching the patient the reason for complete bed rest. d. monitoring the surgical incision for signs of infection.

ANS: D The insertion site for the VAD provides a source for transmission of infection to the circulatory system and requires frequent monitoring. Patients with VADs are able to have some mobility and may not be on bed rest. The VAD is a bridge to transplantation, not a permanent device. Immunosuppression is not necessary for nonbiologic devices such as the VAD.

To maintain proper cuff pressure of an endotracheal tube (ET) when the patient is on mechanical ventilation, the nurse should a. inflate the cuff with a minimum of 10 mL of air. b. inflate the cuff until the pilot balloon is firm on palpation. c. inject air into the cuff until a manometer shows 15 mm Hg pressure. d. inject air into the cuff until a slight leak is heard only at peak inflation.

ANS: D The minimal occluding volume technique involves injecting air into the cuff until an air leak is present only at peak inflation. The volume to inflate the cuff varies with the ET and the patient's size. Cuff pressure should be maintained at 20 to 25 mm Hg. An accurate assessment of cuff pressure cannot be obtained by palpating the pilot balloon.

The nurse responds to a ventilator alarm and finds the patient lying in bed gasping and holding the endotracheal tube (ET) in her hand. Which action should the nurse take next? a. Activate the rapid response team. b. Provide reassurance to the patient. c. Call the health care provider to reinsert the tube. d. Manually ventilate the patient with 100% oxygen.

ANS: D The nurse should ensure maximal patient oxygenation by manually ventilating with a bag-valve-mask system. Offering reassurance to the patient, notifying the health care provider about the need to reinsert the tube, and activating the rapid response team are also appropriate after the nurse has stabilized the patient's oxygenation.

Four hours after mechanical ventilation is initiated, a patient's arterial blood gas (ABG) results include a pH of 7.51, PaO2 of 82 mm Hg, PaCO2 of 26 mm Hg, and HCO3- of 23 mEq/L (23 mmol/L). The nurse will anticipate the need to a. increase the FIO2. b. increase the tidal volume. c. increase the respiratory rate. d. decrease the respiratory rate.

ANS: D The patient's PaCO2 and pH indicate respiratory alkalosis caused by too high a respiratory rate. The PaO2 is appropriate for a patient with COPD and increasing the respiratory rate and tidal volume would further lower the PaCO2

Which patient requires the most rapid assessment and care by the emergency department nurse? a. The patient with hemochromatosis who reports abdominal pain b. The patient with neutropenia who has a temperature of 101.8° F c. The patient with thrombocytopenia who has oozing gums after a tooth extraction d. The patient with sickle cell anemia who has had nausea and diarrhea for 24 hours

B A neutropenic patient with a fever is assumed to have an infection and is at risk for rapidly developing sepsis. Rapid assessment, cultures, and initiation of antibiotic therapy are needed. The other patients also require rapid assessment and care but not as urgently as the neutropenic patient.

A patient who has been receiving IV heparin infusion and oral warfarin (Coumadin) for a deep vein thrombosis (DVT) is diagnosed with heparin-induced thrombocytopenia (HIT) when the platelet level drops to 110,000/μL. Which action will the nurse include in the plan of care? a. Prepare for platelet transfusion. b. Discontinue the heparin infusion. c. Administer prescribed warfarin (Coumadin). d. Use low-molecular-weight heparin (LMWH).

B All heparin is discontinued when HIT is diagnosed. The patient should be instructed to never receive heparin or LMWH. Warfarin is usually not given until the platelet count has returned to 150,000/μL. The platelet count does not drop low enough in HIT for a platelet transfusion, and platelet transfusions increase the risk for thrombosis.

After receiving change-of-shift report for several patients with neutropenia, which patient should the nurse assess first? a. A 56-yr-old with frequent explosive diarrhea b. A 33-yr-old with a fever of 100.8° F (38.2° C) c. A 66-yr-old who has white pharyngeal lesions d. A 23-yr-old who is complaining of severe fatigue

B Any fever in a neutropenic patient indicates infection and can quickly lead to sepsis and septic shock. Rapid assessment and (if prescribed) initiation of antibiotic therapy within 1 hour are needed. The other patients also need to be assessed but do not exhibit symptoms of potentially life-threatening problems.

A routine complete blood count for an active older man indicates possible myelodysplastic syndrome. The nurse will plan to teach the patient about a. blood transfusion. b. bone marrow biopsy. c. filgrastim (Neupogen) administration. d. erythropoietin (Epogen) administration.

B Bone marrow biopsy is needed to make the diagnosis and determine the specific type of myelodysplastic syndrome. The other treatments may be necessary if there is progression of the myelodysplastic syndrome, but the initial action for this asymptomatic patient will be a bone marrow biopsy.

The nurse notes scleral jaundice in a patient being admitted with hemolytic anemia. The nurse will plan to check the laboratory results for the a. Schilling test. c. gastric analysis. b. bilirubin level. d. stool occult blood.

B Jaundice is caused by the elevation of bilirubin level associated with red blood cell hemolysis. The other tests would not be helpful in monitoring or treating a hemolytic anemia.

Which problem reported by a patient with hemophilia is most important for the nurse to communicate to the health care provider? a. Leg bruises c. Skin abrasions b. Tarry stools d. Bleeding gums

B Melena is a sign of gastrointestinal bleeding and requires collaborative actions such as checking hemoglobin and hematocrit and administration of coagulation factors. The other problems indicate a need for patient teaching about how to avoid injury but are not indicators of possible serious blood loss.

A patient who is receiving methotrexate for severe rheumatoid arthritis develops a megaloblastic anemia. The nurse will anticipate teaching the patient about increasing oral intake of a. iron. b. folic acid. c. cobalamin (vitamin B12). d. ascorbic acid (vitamin C).

B Methotrexate use can lead to folic acid deficiency. Supplementation with oral folic acid supplements is the usual treatment. The other nutrients would not correct folic acid deficiency, although they would be used to treat other types of anemia.

An appropriate nursing intervention for a hospitalized patient with severe hemolytic anemia is to a. provide a diet high in vitamin K. b. alternate periods of rest and activity. c. teach the patient how to avoid injury. d. place the patient on protective isolation.

B Nursing care for patients with anemia should alternate periods of rest and activity to encourage activity without causing undue fatigue. There is no indication that the patient has a bleeding disorder, so a diet high in vitamin K or teaching about how to avoid injury is not needed. Protective isolation might be used for a patient with aplastic anemia, but it is not indicated for hemolytic anemia.

A patient has a 6-cm thoracic aortic aneurysm that was discovered during routine chest x-ray. When obtaining an admission history from the patient, it will be most important for the nurse to ask about a.low back pain. b.trouble swallowing. c.abdominal tenderness. d.changes in bowel habits.

B Difficulty swallowing may occur with a thoracic aneurysm because of pressure on the esophagus. The other symptoms will be important to assess for in patients with abdominal aortic aneurysms.

A patient who has acute myelogenous leukemia (AML) is considering treatment with a hematopoietic stem cell transplant (HSCT). The best approach for the nurse to assist the patient with a treatment decision is to a. discuss the need for insurance to cover post-HSCT care. b. ask whether there are questions or concerns about HSCT. c. emphasize the positive outcomes of a bone marrow transplant. d. explain that a cure is not possible with any treatment except HSCT.

B Offering the patient an opportunity to ask questions or discuss concerns about HSCT will encourage the patient to voice concerns about this treatment and will allow the nurse to assess whether the patient needs more information about the procedure. Treatment of AML using chemotherapy is another option for the patient. It is not appropriate for the nurse to ask the patient to consider insurance needs in making this decision.

It is important for the nurse providing care for a patient with sickle cell crisis to a. limit the patient's intake of oral and IV fluids. b. evaluate the effectiveness of opioid analgesics. c. encourage the patient to ambulate as much as tolerated. d. teach the patient about high-protein, high-calorie foods.

B Pain is the most common clinical manifestation of a crisis and usually requires large doses of continuous opioids for control. Fluid intake should be increased to reduce blood viscosity and improve perfusion. Rest is usually ordered to decrease metabolic requirements. Patients are instructed about the need for dietary folic acid, but high-protein, high-calorie diets are not emphasized.

A patient who has a history of a transfusion-related acute lung injury (TRALI) is to receive a transfusion of packed red blood cells (PRBCs). Which action by the nurse will decrease the risk for TRALI for this patient? a. Infuse the PRBCs slowly over 4 hours. b. Transfuse only leukocyte-reduced PRBCs. c. Administer the scheduled diuretic before the transfusion. d. Give the PRN dose of antihistamine before the transfusion.

B TRALI is caused by a reaction between the donor and the patient leukocytes that causes pulmonary inflammation and capillary leaking. The other actions may help prevent respiratory problems caused by circulatory overload or by allergic reactions, but they will not prevent TRALI.

A young adult who has von Willebrand disease is admitted to the hospital for minor knee surgery. The nurse will review the coagulation survey to check the a. platelet count. c. thrombin time. b. bleeding time. d. prothrombin time.

B The bleeding time is affected by von Willebrand disease. Platelet count, prothrombin time, and thrombin time are normal in von Willebrand disease.

Which action will the admitting nurse include in the care plan for a patient who has neutropenia? a. Avoid intramuscular injections. b. Check temperature every 4 hours. c. Omit fruits or vegetables from the diet. d. Place a "No Visitors" sign on the door.

B The earliest sign of infection in a neutropenic patient is an elevation in temperature. Although unpeeled fresh fruits and vegetables should be avoided, fruits and vegetables that are peeled or cooked are acceptable. Injections may be required for administration of medications such as filgrastim (Neupogen). The number of visitors may be limited and visitors with communicable diseases should be avoided, but a "no visitors" policy is not needed.

The nurse is caring for a patient with type A hemophilia being admitted to the hospital with severe pain and swelling in the right knee. The nurse should a. apply heat to the knee. b. immobilize the knee joint. c. assist the patient with light weight bearing. d. perform passive range of motion to the knee.

B The initial action should be total rest of the knee to minimize bleeding. Ice packs are used to decrease bleeding. Range of motion (ROM) and weight-bearing exercise are contraindicated initially, but after the bleeding stops, ROM and physical therapy are started.

A patient who has acute myelogenous leukemia develops an absolute neutrophil count of 850/μL while receiving outpatient chemotherapy. Which action by the outpatient clinic nurse is most appropriate? a. Discuss the need for hospital admission to treat the neutropenia. b. Teach the patient to administer filgrastim (Neupogen) injections. c. Plan to discontinue the chemotherapy until the neutropenia resolves. d. Order a high-efficiency particulate air (HEPA) filter for the patient's home.

B The patient may be taught to self-administer filgrastim injections. Although chemotherapy may be stopped with severe neutropenia (neutrophil count <500/μL), administration of filgrastim usually allows the chemotherapy to continue. Patients with neutropenia are at higher risk for infection when exposed to other patients in the hospital. HEPA filters are expensive and are used in the hospital, where the number of pathogens is much higher than in the patient's home environment.

A patient who has acute myelogenous leukemia (AML) asks the nurse whether the planned chemotherapy will be worth undergoing. Which response by the nurse is appropriate? a. "If you do not want to have chemotherapy, other treatment options include stem cell transplantation." b. "The side effects of chemotherapy are difficult, but AML frequently goes into remission with chemotherapy." c. "The decision about treatment is one that you and the doctor need to make rather than asking what I would do." d. "You don't need to make a decision about treatment right now because leukemias in adults tend to progress slowly."

B This response uses therapeutic communication by addressing the patient's question and giving accurate information. The other responses either give inaccurate information or fail to address the patient's question, which will discourage the patient from asking the nurse for information.

The nurse is planning to administer a transfusion of packed red blood cells (PRBCs) to a patient with blood loss from gastrointestinal hemorrhage. Which action can the nurse delegate to unlicensed assistive personnel (UAP)? a. Verify the patient identification (ID) according to hospital policy. b. Obtain the temperature, blood pressure, and pulse before the transfusion. c. Double-check the product numbers on the PRBCs with the patient ID band. d. Monitor the patient for shortness of breath or chest pain during the transfusion.

B UAP education includes measurement of vital signs. UAP would report the vital signs to the registered nurse (RN). The other actions require more education and a larger scope of practice and should be done by licensed nursing staff members.

Which nursing action should be included in the plan of care after endovascular repair of an abdominal aortic aneurysm? a.Record hourly chest tube drainage. b.Monitor fluid intake and urine output. c.Assess the abdominal incision for redness. d.Teach the patient to plan for a long recovery period.

B Because renal artery occlusion can occur after endovascular repair, the nurse should monitor parameters of renal function such as intake and output. Chest tubes will not be needed for endovascular surgery, the recovery period will be short, and there will not be an abdominal wound.

The nurse is admitting a patient newly diagnosed with peripheral artery disease. Which admission order should the nurse question? a.Cilostazol drug therapy b.Omeprazole drug therapy c.Use of treadmill for exercise d.Exercise to the point of discomfort

B Because the antiplatelet effect of clopidogrel is reduced when it is used with omeprazole, the nurse should clarify this order with the health care provider. The other interventions are appropriate for a patient with peripheral artery disease.

While working in the outpatient clinic, the nurse notes that a patient has a history of intermittent claudication. Which statement by the patient would support this information? a."When I stand too long, my feet start to swell." b."My legs cramp when I walk more than a block." c."I get short of breath when I climb a lot of stairs." d."My fingers hurt when I go outside in cold weather."

B Cramping that is precipitated by a consistent level of exercise is descriptive of intermittent claudication. Finger pain associated with cold weather is typical of Raynaud's phenomenon. Shortness of breath that occurs with exercise is not typical of intermittent claudication, which is reproducible. Swelling associated with prolonged standing is typical of venous disease.

The health care provider prescribes an infusion of heparin and daily partial thromboplastin time (PTT) testing for a patient with venous thromboembolism (VTE). The nurse will plan to a.decrease the infusion when the PTT value is 65 seconds. b.avoid giving IM medications to prevent localized bleeding. c.have vitamin K available in case reversal of the heparin is needed. d.monitor posterior tibial and dorsalis pedis pulses with the Doppler.

B Intramuscular injections are avoided in patients receiving anticoagulation to prevent hematoma formation and bleeding from the site. A PTT of 65 seconds is within the therapeutic range. Vitamin K is used to reverse warfarin. Pulse quality is not affected by VTE.

The nurse who works in the vascular clinic has several patients with venous insufficiency scheduled today. Which patient should the nurse assign to an experienced licensed practical/vocational nurse (LPN/LVN)? a.Patient who has been complaining of increased edema and skin changes in the legs b.Patient who needs wound care for a chronic venous stasis ulcer on the right lower leg c.Patient who has a history of venous thromboembolism and is complaining of dyspnea d.Patient who needs teaching about elastic compression stockings for venous insufficiency

B LPN education and scope of practice includes wound care. The other patients, which require more complex assessments or education, should be managed by the RN.

A patient with a venous thromboembolism (VTE) is started on enoxaparin (Lovenox) and warfarin (Coumadin). The patient asks the nurse why two medications are necessary. Which response by the nurse is most accurate? a."Taking two blood thinners greatly reduces the risk for another clot to form." b."Enoxaparin will work right away, but warfarin takes several days to begin preventing clots." c."Enoxaparin will start to dissolve the clot, and warfarin will prevent any more clots from forming." d."Because of the risk for a blood clot in the lungs, it is important for you to take more than one blood thinner."

B Low molecular weight heparin (LMWH) is used because of the immediate effect on coagulation and discontinued once the international normalized ratio (INR) value indicates that the warfarin has reached a therapeutic level. LMWH has no thrombolytic properties. The use of two anticoagulants is not related to the risk for pulmonary embolism, and two are not necessary to reduce the risk for another VTE. Anticoagulants do not thin the blood.

The nurse has started discharge teaching for a patient who is to continue warfarin (Coumadin) after hospitalization for venous thromboembolism (VTE). The nurse determines that additional teaching is needed when the patient says which of the following? a."I should get a Medic Alert device stating that I take warfarin." b."I should reduce the amount of green, leafy vegetables that I eat." c."I will need routine blood tests to monitor the effects of the warfarin." d."I will check with my health care provider before I begin any new drugs."

B Patients taking warfarin are taught to follow a consistent diet with regard to foods that are high in vitamin K, such as green, leafy vegetables. The other patient statements are accurate.

Which instructions should the nurse include in a teaching plan for an older patient newly diagnosed with peripheral artery disease (PAD)? a."Exercise only if you do not experience any pain." b."It is very important that you stop smoking cigarettes." c."Try to keep your legs elevated whenever you are sitting." d."Put elastic compression stockings on early in the morning."

B Smoking cessation is essential for slowing the progression of PAD to critical limb ischemia and reducing the risk of myocardial infarction and death. Circulation to the legs will decrease if the legs are elevated. Patients with PAD are taught to exercise to the point of feeling pain, rest, and then resume walking. Support hose are not used for patients with PAD.

After receiving change of shift report, which patient admitted to the emergency department should the nurse assess first? a.A 67-yr-old patient who has a gangrenous left foot ulcer with a weak pedal pulse b.A 50-yr-old patient who is complaining of sudden sharp and severe upper back pain c.A 39-yr-old patient who has right calf tenderness, redness, and swelling after a plane ride d.A 58-yr-old patient who is taking anticoagulants for atrial fibrillation and has black stools

B The patient's presentation of sudden sharp and severe upper back pain is consistent with dissecting thoracic aneurysm, which will require the most rapid intervention. The other patients also require rapid intervention but not before the patient with severe pain.

Which actions could the nurse delegate to unlicensed assistive personnel (UAP) who are providing care for a patient who is at risk for venous thromboembolism? a.Monitor for any bleeding after anticoagulation therapy is started. b.Apply sequential compression device whenever the patient is in bed. c.Ask the patient about use of herbal medicines or dietary supplements. d.Instruct the patient to call immediately if any shortness of breath occurs.

B UAP training includes the use of equipment that requires minimal nursing judgment, such as sequential compression devices. Patient assessment and teaching require more education and critical thinking and should be done by the registered nurse (RN).

Which action for a patient with neutropenia is appropriate for the registered nurse (RN) to delegate to a licensed practical/vocational nurse (LPN/LVN)? a. Assessing the patient for signs and symptoms of infection b. Teaching the patient the purpose of neutropenic precautions c. Administering subcutaneous filgrastim (Neupogen) injection d. Developing a discharge teaching plan for the patient and family

C Administration of subcutaneous medications is included in LPN/LVN education and scope of practice. Patient teaching, assessment, and developing the plan of care require RN level education and scope of practice.

Which instruction will the nurse plan to include in discharge teaching for a patient admitted with a sickle cell crisis? a. Take a daily multivitamin with iron. b. Limit fluids to 2 to 3 quarts per day. c. Avoid exposure to crowds when possible. d. Drink only two caffeinated beverages daily.

C Exposure to crowds increases the patient's risk for infection, the most common cause of sickle cell crisis. There is no restriction on caffeine use. Iron supplementation is generally not recommended. A high-fluid intake is recommended.

Which action will the nurse include in the plan of care for a patient who has thalassemia major? a. Teach the patient to use iron supplements. b. Avoid the use of intramuscular injections. c. Administer iron chelation therapy as needed. d. Notify health care provider of hemoglobin 11 g/dL.

C The frequent transfusions used to treat thalassemia major lead to iron toxicity in patients unless iron chelation therapy is consistently used. Iron supplementation is avoided in patients with thalassemia. There is no need to avoid intramuscular injections. The goal for patients with thalassemia major is to maintain a hemoglobin of 10 g/dL or greater.

A patient with possible disseminated intravascular coagulation arrives in the emergency department with a blood pressure of 82/40, temperature of 102° F (38.9° C), and severe back pain. Which prescribed action will the nurse implement first? a. Administer morphine sulfate 4 mg IV. b. Give acetaminophen (Tylenol) 650 mg. c. Infuse normal saline 500 mL over 30 minutes. d. Schedule complete blood count and coagulation studies.

C The patient's blood pressure indicates hypovolemia caused by blood loss and should be addressed immediately to improve perfusion to vital organs. The other actions are also appropriate and should be rapidly implemented, but improving perfusion is the priority for this patient.

A postoperative patient receiving a transfusion of packed red blood cells develops chills, fever, headache, and anxiety 35 minutes after the transfusion is started. After stopping the transfusion, what action should the nurse take? a. Give the PRN diphenhydramine . b. Send a urine specimen to the laboratory. c. Administer PRN acetaminophen (Tylenol). d. Draw blood for a new type and crossmatch.

C The patient's clinical manifestations are consistent with a febrile, nonhemolytic transfusion reaction. The transfusion should be stopped and antipyretics administered for the fever as ordered. A urine specimen is needed if an acute hemolytic reaction is suspected. Diphenhydramine is used for allergic reactions. This type of reaction does not indicate incorrect crossmatching.

A patient with septicemia develops prolonged bleeding from venipuncture sites and blood in the stools. Which action is most important for the nurse to take? a. Avoid other venipunctures. b. Apply dressings to the sites. c. Notify the health care provider. d. Give prescribed proton-pump inhibitors.

C The patient's new onset of bleeding and diagnosis of sepsis suggest that disseminated intravascular coagulation (DIC) may have developed, which will require collaborative actions such as diagnostic testing, blood product administration, and heparin administration. The other actions are also appropriate, but the most important action should be to notify the health care provider so that DIC treatment can be initiated rapidly.

The nurse performing an assessment of a patient who has chronic peripheral artery disease (PAD) of the legs and an ulcer on the right second toe would expect to find a.dilated superficial veins. b.swollen, dry, scaly ankles. c.prolonged capillary refill in all the toes. d.serosanguineous drainage from the ulcer.

C Capillary refill is prolonged in PAD because of the slower and decreased blood flow to the periphery. The other listed clinical manifestations are consistent with chronic venous disease.

A 46-yr-old service-counter worker undergoes sclerotherapy for treatment of superficial varicose veins at an outpatient center. Which instructions should the nurse provide to the patient before discharge? a.Sitting at the work counter, rather than standing, is recommended. b.Exercise, such as walking or jogging, can cause recurrence of varicosities. c.Elastic compression stockings should be applied before getting out of bed. d.Taking an aspirin daily will help prevent clots from forming around venous valves.

C Elastic compression stockings are applied with the legs elevated to reduce pressure in the lower legs. Walking is recommended to prevent recurrent varicosities. Sitting and standing are both risk factors for varicose veins and venous insufficiency. An aspirin a day is not adequate to prevent venous thrombosis and would not be recommended for a patient who had just had sclerotherapy.

Which assessment finding for a patient who has been admitted with a right calf venous thromboembolism (VTE) requires immediate action by the nurse? a.Erythema of right lower leg b.Complaint of right calf pain c.New onset shortness of breath d.Temperature of 100.4°F (38°C)

C New onset dyspnea suggests a pulmonary embolus, which will require rapid actions such as O2 administration and notification of the health care provider. The other findings are typical of VTE.

A young adult patient tells the health care provider about experiencing cold, numb fingers when running during the winter, and Raynaud's phenomenon is suspected. The nurse will anticipate teaching the patient about tests for a.hyperglycemia. b.hyperlipidemia. c.autoimmune disorders. d.coronary artery disease.

C Secondary Raynaud's phenomenon may occur in conjunction with autoimmune diseases such as rheumatoid arthritis. Patients should be screened for autoimmune disorders. Raynaud's phenomenon is not associated with hyperlipidemia, hyperglycemia, or coronary artery disease.

Which action by a new nurse who is giving fondaparinux (Arixtra) to a patient with a lower leg venous thromboembolism (VTE) indicates that more education about the drug is needed? a.The nurse avoids rubbing the injection site after giving the drug. b.The nurse injects the drug into the abdominal subcutaneous tissue. c.The nurse ejects the air bubble from the syringe before giving the drug. d.The nurse does not check partial thromboplastin time (PTT) before giving the drug.

C The air bubble is not ejected before giving fondaparinux to avoid loss of drug. The other actions by the nurse are appropriate for subcutaneous administration of a low molecular weight heparin (LMWH). LMWHs typically do not require ongoing PTT monitoring and dose adjustment.

The nurse is caring for a patient with a descending aortic dissection. Which assessment finding is most important to report to the health care provider? a.Weak pedal pulses b.Absent bowel sounds c.Blood pressure of 138/88 mm Hg d.25 mL of urine output over the past hour

C The blood pressure is typically kept at less than 120 mm Hg systolic to minimize extension of the dissection. The nurse will need to notify the health care provider so that β-blockers or other antihypertensive drugs can be prescribed. The other findings are typical with aortic dissection and should also be reported but do not require immediate action.

Following a cauda equina spinal cord injury, which action will the nurse include in the plan of care? a. Catheterize patient every 3 to 4 hours. b. Assist patient to ambulate several times daily. c. Administer medications to reduce bladder spasm. d. Stabilize the neck when repositioning the patient.

Catheterize patient every 3 to 4 hours. Patients with cauda equina syndrome have areflexic bladder, and intermittent catheterization will be used for emptying the bladder. Because the bladder is flaccid, antispasmodic medications will not be used. The legs are flaccid with cauda equina syndrome and the patient will be unable to ambulate. The head and neck will not need to be stabilized following a cauda equina injury, which affects the lumbar and sacral nerve roots.

Which statement by a patient indicates good understanding of the nurse's teaching about prevention of sickle cell crisis? a. "Home oxygen therapy is frequently used to decrease sickling." b. "There are no effective medications that can help prevent sickling." c. "Routine continuous dosage narcotics are prescribed to prevent a crisis." d. "Risk for a crisis is decreased by having an annual influenza vaccination."

D Because infection is the most common cause of a sickle cell crisis, influenza, Haemophilus influenzae, pneumococcal pneumonia, and hepatitis immunizations should be administered. Although continuous dose opioids and oxygen may be administered during a crisis, patients do not receive these therapies to prevent crisis. Hydroxyurea (Hydrea) is a medication used to decrease the number of sickle cell crises.

Which laboratory test will the nurse use to determine whether filgrastim (Neupogen) is effective for a patient with acute lymphocytic leukemia who is receiving chemotherapy? a. Platelet count b. Reticulocyte count c. Total lymphocyte count d. Absolute neutrophil count

D Filgrastim increases the neutrophil count and function in neutropenic patients. Although total lymphocyte, platelet, and reticulocyte counts are also important to monitor in this patient, the absolute neutrophil count is used to evaluate the effects of filgrastim.

A patient who has non-Hodgkin's lymphoma is receiving combination treatment with rituximab (Rituxan) and chemotherapy. Which patient assessment finding requires the most rapid action by the nurse? a. Anorexia c. Oral ulcers b. Vomiting d. Lip swelling

D Lip swelling in angioedema may indicate a hypersensitivity reaction to the rituximab. The nurse should stop the infusion and further assess for anaphylaxis. The other findings may occur with chemotherapy but are not immediately life threatening.

An expected action by the nurse caring for a patient who has an acute exacerbation of polycythemia vera is to a. place the patient on bed rest. b. administer iron supplements. c. avoid use of aspirin products. d. monitor fluid intake and output.

D Monitoring hydration status is important during an acute exacerbation because the patient is at risk for fluid overload or underhydration. Aspirin therapy is used to decrease risk for thrombosis. The patient should be encouraged to ambulate to prevent deep vein thrombosis. Iron is contraindicated in patients with polycythemia vera.

Which laboratory result will the nurse expect to show a decreased value if a patient develops heparin-induced thrombocytopenia (HIT)? a. Prothrombin time b. Erythrocyte count c. Fibrinogen degradation products d. Activated partial thromboplastin time

D Platelet aggregation in HIT causes neutralization of heparin, so the activated partial thromboplastin time will be shorter, and more heparin will be needed to maintain therapeutic levels. The other data will not be affected by HIT.

A 62-year old man with chronic anemia is experiencing increased fatigue and occasional palpitations at rest. The nurse would expect the patient's laboratory test findings to include a. an RBC count of 4,500,000/mL. b. a hematocrit (Hct) value of 38%. c. normal red blood cell (RBC) indices. d. a hemoglobin (Hgb) of 8.6 g/dL (86 g/L).

D The patient's clinical manifestations indicate moderate anemia, which is consistent with a Hgb of 6 to 10 g/dL. The other values are all within the range of normal.

A patient in the emergency department complains of back pain and difficulty breathing 15 minutes after a transfusion of packed red blood cells is started. The nurse's first action should be to a. administer oxygen therapy at a high flow rate. b. obtain a urine specimen to send to the laboratory. c. notify the health care provider about the symptoms. d. disconnect the transfusion and infuse normal saline.

D The patient's symptoms indicate a possible acute hemolytic reaction caused by the transfusion. The first action should be to disconnect the transfusion and infuse normal saline. The other actions also are needed but are not the highest priority.

When discussing risk factor modification for a patient who has a 5-cm abdominal aortic aneurysm, the nurse will focus teaching on which patient risk factor? a.Male gender b.Turner syndrome c.Abdominal trauma history d.Uncontrolled hypertension

D All of the factors contribute to the patient's risk, but only hypertension can potentially be modified to decrease the patient's risk for further expansion of the aneurysm.

Which nursing intervention for a patient who had an open repair of an abdominal aortic aneurysm 2 days previously is appropriate for the nurse to delegate to unlicensed assistive personnel (UAP)? a.Monitor the quality and presence of the pedal pulses. b.Teach the patient the signs of possible wound infection. c.Check the lower extremities for strength and movement. d.Help the patient to use a pillow to splint while coughing.

D Assisting a patient who has already been taught how to cough is part of routine postoperative care and within the education and scope of practice for UAP. Patient teaching and assessment of essential postoperative functions such as circulation and movement should be done by RNs.

Which topic should the nurse include in patient teaching for a patient with a venous stasis ulcer on the left lower leg? a.Need to increase carbohydrate intake b.Methods of keeping the wound area dry c.Purpose of prophylactic antibiotic therapy d.Application of elastic compression stockings

D Compression of the leg is essential to healing of venous stasis ulcers. High dietary intake of protein, rather than carbohydrates, is needed. Prophylactic antibiotics are not routinely used for venous ulcers. Moist dressings are used to hasten wound healing.

Several hours after a patient had an open surgical repair of an abdominal aortic aneurysm, the UAP reports to the nurse that urinary output for the past 2 hours has been 45 mL. The nurse notifies the health care provider and anticipates an order for a(n) a.hemoglobin count. b.additional antibiotic. c.serum creatinine level. d.increased IV infusion rate.

D The decreased urine output suggests decreased renal perfusion and monitoring of renal function is needed. There is no indication that infection is a concern, so antibiotic therapy and a WBC count are not needed. The IV rate may be increased because hypovolemia may be contributing to the patient's decreased urinary output.

A patient at the clinic says, "I always walk after dinner, but lately my leg cramps and hurts after just a few minutes of starting. The pain goes away after I stop walking, though." The nurse should a.look for the presence of tortuous veins bilaterally on the legs. b.ask about any skin color changes that occur in response to cold. c.assess for unilateral swelling, redness, and tenderness of either leg. d.palpate for the presence of dorsalis pedis and posterior tibial pulses.

D The nurse should assess for other clinical manifestations of peripheral arterial disease in a patient who describes intermittent claudication. Changes in skin color that occur in response to cold are consistent with Raynaud's phenomenon. Tortuous veins on the legs suggest venous insufficiency. Unilateral leg swelling, redness, and tenderness indicate venous thromboembolism.

An older patient with chronic atrial fibrillation develops sudden severe pain, pulselessness, pallor, and coolness in the right leg. The nurse should notify the health care provider and immediately a.apply a compression stocking to the leg. b.elevate the leg above the level of the heart. c.assist the patient in gently exercising the leg. d.keep the patient in bed in the supine position.

D The patient's history and clinical manifestations are consistent with acute arterial occlusion, and resting the leg will decrease the O2 demand of the tissues and minimize ischemic damage until circulation can be restored. Elevating the leg or applying an elastic wrap will further compromise blood flow to the leg. Exercise will increase oxygen demand for the tissues of the leg.

The health care provider has prescribed bed rest with the feet elevated for a patient admitted to the hospital with venous thromboembolism. Which action by the nurse to elevate the patient's feet is best? a.The patient is placed in the Trendelenburg position. b.Two pillows are positioned under the affected leg. c.The bed is elevated at the knee and pillows are placed under the feet. d.One pillow is placed under the thighs and two pillows are placed under the lower legs.

D The purpose of elevating the feet is to enhance venous flow from the feet to the right atrium, which is best accomplished by placing two pillows under the feet and one under the thighs. Placing the patient in the Trendelenburg position will lower the head below heart level, which is not indicated for this patient. Placing pillows under the calf or elevating the bed at the knee may cause blood stasis at the calf level.

A nurse who works on the neurology unit just received change-of-shift report. Which patient will the nurse assess first? a. Patient with botulism who is experiencing difficulty swallowing b. Patient with Bell's palsy who has herpes vesicles in front of the ear c. Patient with neurosyphilis who has tabes dorsalis and decreased deep tendon reflexes d. Patient with an abscess caused by injectable drug use who needs tetanus immune globulin

Patient with botulism who is experiencing difficulty swallowing The patient's diagnosis and difficulty swallowing indicate that the nurse should rapidly assess for respiratory distress. The information about the other patients is consistent with their diagnoses and does not indicate any immediate need for assessment or intervention.

A patient has an incomplete left spinal cord lesion at the level of T7, resulting in Brown-Séquard syndrome. Which nursing action should be included in the plan of care? a. Assessment of the patient for right arm weakness b. Assessment of the patient for increased right leg pain c. Positioning the patient's left leg when turning the patient d. Teaching the patient to look at the right leg to verify its position

Positioning the patient's left leg when turning the patient The patient with Brown-Séquard syndrome has loss of motor function on the ipsilateral side and will require the nurse to move the left leg. Pain sensation will be lost on the patient's right leg. Arm weakness will not be a problem for a patient with a T7 injury. The patient will retain position sense for the right leg.

The nurse reviews the EMR for a patient scheduled for a total hip replacement. Which assessment data shown in the accompanying figure increase the patient's risk for respiratory complications after surgery? a. Albumin level and recent weight loss. b. Mild confusion and recent weight loss. c. Age and recent arthroscopic procedure. d. Anemia and recent arthroscopic procedure.

a. Albumin level and recent weight loss. -(Albumin level in the picture is 2.7 mg/dL and the patient recently lost 15 lbs). The patient's recent weight loss and low protein stores indicate possible muscle weakness which make it more difficult for an older patient to recover from the effects of general anesthesia and immobility associated with hip surgery. The other information will also be noted by the nurse but does not place the patient at higher risk for respiratory failure.

Which finding in a patient hospitalized with bronchiectasis is most important to report to the health care provider? a. Cough productive of bloody, purulent mucus b. Scattered crackles and wheezes heard bilaterally c. Complaint of sharp chest pain with deep breathing d. Respiratory rate 28 breaths/minute while ambulating

a. Cough productive of bloody, purulent mucus Hemoptysis may indicate life-threatening hemorrhage, and should be reported immediately to the health care provider.

A 46-year-old patient tells the nurse using acetaminophen (Tylenol) several times every day for recurrent bilateral headaches. Which action will the nurse plan to take first? a. Discuss the need to stop taking acetaminophen. b. Suggest the use of biofeedback for headache control. c. Describe the use of botulism toxin (Botox) for headaches. d. Teach the patient about magnetic resonance imaging (MRI).

a. Discuss the need to stop taking acetaminophen. -The headache description suggests that the patient is experiencing medication overuse headache. The initial action will be withdrawal of the medication. The other actions may be needed if the headache persist.

A nurse is caring for a patient who is orally intubated and receiving mechanical ventilation. To decrease the risk for ventilator-associated pneumonia, which action will the nurse include in the plan of care? a. Elevate head of bed to 30 to 45 degrees. b. Suction the endotracheal tube every 2 to 4 hours. c. Limit the use of PEEP. d. Give enteral feedings at no more than 10 mL/hr.

a. Elevate head of bed to 30 to 45 degrees. -Elevation of the head decreases the risk for aspiration. PEEP is frequently needed to improve oxygenation in patients receiving mechanical ventilation. Suctioning should be done only when the patient assessment indicates that it is necessary. Enteral feedings should provide adequate calories for the patient's high energy needs.

A patient reports feeling numbness and tingling of the left arm before experiencing a tonic-clonic seizure. The nurse determines that this history is consistent with what type of seizure? a. Focal. b. Atonic. c. Absence. d. Myoclonic.

a. Focal. -The initial symptoms of a focal seizure involve clinical manifestations that are localized to a particular part of the body or brain. Symptoms of an absence seizure are staring and a brief loss consciousness. In an atonic seizure, the patient loses muscle tone and (typically) falls to the ground. Myoclonic seizures are characterized by a sudden jerk of the body or extremities.

While caring for a patient who has been admitted with a pulmonary embolism, the nurse notes a change in the patient's oxygen saturation (SpO2) from 95% to 88%. Which action should the nurse take next? a. Increase the oxygen flow rate. b. Suction the patient's oropharynx. c. Instruct the patient to cough and deep breathe. d. Help the patient to sit in a more upright position.

a. Increase the oxygen flow rate. -Increasing oxygen flow rate will usually improve oxygen saturation in patients with ventilation-perfusion mismatch, as occurs with pulmonary embolism. Because the problem is with perfusion, actions that improve ventilation, such as deep breathing and coughing, sitting upright, and suctioning, are not likely to improve oxygenation.

A nurse is caring for an obese patient with right lower lobe pneumonia. Which position will be best to improve gas exchange? a. On the left side. b. On the right side. c. In the tripod position. d. In the high-Fowler's position.

a. On the left side. -The patient should be positioned with the "good" lung in the dependent position to improve the match between ventilation and perfusion. The obese patient's abdomen will limit respiratory excursion when sitting in the high-Fowler's or tripod positions.

Which information about a patient who is receiving cisatracurium (Nimbex) to prevent asynchronous breathing with the positive pressure ventilator requires immediate action by the nurse? a. Only continuous IV opioids have been ordered. b. The patient does not respond to verbal stimulation. c. There is not cough or gag when the patient is suctioned. d. The patient's oxygen saturation fluctuates between 90% to 93%.

a. Only continuous IV opioids have been ordered. -Because neuromuscular blockage is extremely anxiety provoking, it is essential that patients who are receiving neuromuscular blockage receive concurrent sedation and analgesia. Absence of response to stimuli is expected in patients receiving neuromuscular blockage. The oxygen saturation is adequate.

A nurse is caring for a patient with ARDS who is receiving mechanical ventilation using synchronized intermittent mandatory ventilation (SIMV). The settings include fraction of inspired oxygen (FIO2) 80%, tidal volume 450, rate 16/minute, and PEEP 5 cm. Which assessment finding is most important for the nurse to report to the HCP? a. Oxgen saturation 99%. b. Respiratory rate 22 breaths/minute. c. Crackles audible at lung bases. d. Heart rate 106 beats/minute.

a. Oxgen saturation 99%. -The FIO2 of 80% increases the risk for oxygen toxicity. Because the patient's O2 saturation is 99%, a decrease in FIO2 is indicated to avoid toxicity. The other patient data would be typical for a patient with ARDS and would not need to be urgently reported to the HCP.

A 27-year-old who has been treated for status epilepticus in the ED will be transferred to the medical nursing unit. Which equipment should the nurse have available in the patient's assigned room (select all that apply)? a. Side-rail pads. b. Tongue blade. c. Oxygen mask. d. Suction tubing. e. Urinary catheter. f. Nasogastric tube.

a. Side-rail pads. c. Oxygen mask. d. Suction tubing. -The patient is at risk for further seizures, and oxygen and suctioning may be needed after any seizures to clear the airway and maximize oxygenation. The bed's side rails should be padded to minimize the risk for patient injury during a seizure. Use of tongue blades during a seizure is contraindicated. Insertion of a NG tube is not indicated because the airway problem is not caused by vomiting or abdominal distention. A urinary catheter is not required unless there is urinary retention.

When admitting a patient with possible respiratory failure with a high PaCO2, which assessment information should be immediately reported to the HCP? a. The patient is somnolent. b. The patient complains of weakness. c. The patient's blood pressure is 164/98. d. The patient's oxygen saturation is 90%.

a. The patient is somnolent. -Increasing somnolence will decrease the patient's respiratory rate and further increase the PaCO2 and respiratory failure. Rapid action is needed to prevent respiratory arrest. An SpO2 of 90%, weakness, and elevated BP all require ongoing monitoring but are not indicators of possible impending respiratory arrest.

A patient with Parkinson's disease is admitted to the hospital for treatment of pneumonia. Which nursing interventions will be included in the plan of care (select all that apply)? a. Use an elevated toilet seat. b. Cut the patient's food into small pieces. c. Provide high-protein foods at each meal. d. Place an armchair at the patient's bedside. e. Observe for sudden exacerbation of symptoms.

a. Use an elevated toilet seat. b. Cut the patient's food into small pieces. d. Place an armchair at the patient's bedside. -Because the patient with Parkinson's has difficulty chewing, food should be cut into smal pieces. An armchair should be used when the patient is seated so that the patient can use the arms to assist with getting up from the chair. An elevated toilet seat will facilitate getting on and off the toilet. High-Protein foods will decrease the effectiveness of L-dopa. Parkinson's is a steadily progressive disease without acute exacerbations.

A young adult patient who denies any history of smoking is seen in the clinic with a new diagnosis of chronic obstructive pulmonary disease (COPD). The nurse should plan to teach the patient about a. a1-antitrypsin testing. c. use of the nicotine patch. b. leukotriene modifiers. d. continuous pulse oximetry.

a. a1-antitrypsin testing When *COPD* occurs in *young* patients, especially without a smoking history, a genetic deficiency in *a1-antitrypsin* should be suspected.

The nurse advises a patient with myasthenia graves (MG) to a. perform physically demanding activities early in the day. b. anticipate the need for weekly plasmapheresis treatments. c. do frequent weight-bearing exercise to prevent muscle atrophy. d. protect the extremities from injury due to poor sensory perception.

a. perform physically demanding activities early in the day. -Muscles are generally strongest in the morning, and activities involving muscle activity should be scheduled then. Plasmapheresis is not routinely scheduled, but is used for myasthenia crisis or for situations in which corticosteroid therapy must be avoided. There is no decrease in sensation with MG, and muscle atrophy does not occur because although there is muscle weakness, they are still used.

Before administering botulinum antitoxin to a patient in the emergency department, it is most important for the nurse to a. obtain the patient's temperature. b. administer an intradermal test dose. c. document the neurologic symptoms. d. ask the patient about an allergy to eggs.

administer an intradermal test dose. To assess for possible allergic reactions, an intradermal test dose of the antitoxin should be administered. Although temperature, allergy history, and symptom assessment and documentation are appropriate, these assessments will not affect the decision to administer the antitoxin.

A construction worker arrives at an urgent care center with a deep puncture wound after an old nail penetrated his boot.. The patient reports having had a tetanus booster 6 years ago. The nurse will anticipate a. IV infusion of tetanus immune globulin (TIG). b. administration of the tetanus-diphtheria (Td) booster. c. intradermal injection of an immune globulin test dose. d. initiation of the tetanus-diphtheria immunization series.

administration of the tetanus-diphtheria (Td) booster If the patient has not been immunized within 5 years, administration of the Td booster is indicated because the wound is deep. Immune globulin administration is given by the IM route if the patient has no previous immunization. Administration of a series of immunization is not indicated. TIG is not indicated for this patient, and a test dose is not needed for immune globulin.

A high school teacher who has just been diagnosed with epilepsy after having a generalized tonic-clonic seizure tells the nurse, "I cannot teach anymore, it will be too upsetting if I have a seizure at work." Which response by the nurse is best? a. "You might benefit from some psychologic counseling." b. "Epilepsy usually can be well controlled with mediations." c. "You will want to contact the Epilepsy Foundation for assistance." d. "The Department of Vocational Rehabilitation can help with work retraining."

b. "Epilepsy usually can be well controlled with mediations." -The nurse should inform the patient that most patients with seizure disorders are controlled with medication. The other information may be necessary if the seizures persist after treatment with antiseizure medications is implemented.

The nurse provides dietary teaching for a patient with chronic obstructive pulmonary disease (COPD) who has a low body mass index (BMI). Which patient statement indicates that the teaching has been effective? a. "I will drink lots of fluids with my meals." b. "I can have ice cream as a snack every day." c. "I will exercise for 15 minutes before meals." d. "I will decrease my intake of meat and poultry."

b. "I can have ice cream as a snack every day." High-calorie foods such as ice cream are an appropriate snack for patients with COPD. Fluid intake of 3 L/day is recommended, but fluids should be taken between meals rather than with meals to improve oral intake of solid foods.

Which statement by the nurse when explaining the purpose of positive end-expiratory pressure (PEEP) to the family members of a patient with ARDS is accurate? a. "PEEP will push more air into the lungs during inhalation." b. "PEEP prevents the lung air sacs from collapsing during exhalation." c. "PEEP will prevent lung damage while the patient is on the ventilator." d. "PEEP allows the breathing machine to deliver 100% oxygen to the lungs."

b. "PEEP prevents the lung air sacs from collapsing during exhalation." -By preventing alveolar collapse during expiration, PEEP improves gas exchange and oxygenation. PEEP will not prevent lung damage (e.g., fibrotic changes that occur with ARDS), push more air into the lungs, or change the fraction of inspire oxygen (FIO2) delivered to the patient.

A young adult female patient with cystic fibrosis (CF) tells the nurse that she is considering getting married and wondering about having children. Which initial response by the nurse is best? a. "Are you aware of the normal lifespan for patients with CF?" b. "Would like more information to help you with that decision?" c. "Many women with CF do not have difficulty conceiving children." d. "You will need to have genetic counseling before making a decision."

b. "Would like more information to help you with that decision?"

The nurse receives a change-of-shift report on the following patients with chronic obstructive pulmonary disease (COPD). Which patient should the nurse assess first? a. A patient with loud expiratory wheezes b. A patient with a respiratory rate of 38 breaths/min c. A patient who has a cough productive of thick, green mucus d. A patient with jugular venous distention and peripheral edema

b. A patient with a respiratory rate of 38 breaths/min A respiratory rate of 38/min indicates severe respiratory distress, and the patient needs immediate assessment and intervention to prevent possible respiratory arrest.

A hospitalized patient complains of a bilateral headache, 4/10 on the pain scale, that radiates from the base of the skull. Which prescribed medications should the nurse administer initially? a. Lorazepam (Ativan). b. Acetaminophen (Tylenol). c. Morphine sulfate (Roxanol). d. Butalbital and aspirin (Fiorinal).

b. Acetaminophen (Tylenol). -The patient's symptoms are consistent with a tension headache, and initial therapy usually involves a nonopioid analgesic such as acetaminophen, which is sometimes combined with a sedative or muscle relaxant. Lorazepam may be used in conjunction with acetaminophen but would not be appropriate as the initial mono therapy. Morphine sulfate and butalbital and aspirin would be more appropriate for a headache that did not respond to a nonopioid analgesic.

An oxygen saturation (SpO2) for a patient with left lower lobe pneumonia is 90%. The patient has rhonchi, a weak cough effort, and complains of fatigue. Which action is a priority for the nurse to take? a. Position the patient on the left side. b. Assist the patient with staged coughing. c. Place a humidifier in the patient's room. d. Schedule a 2-hour rest period for the patient.

b. Assist the patient with staged coughing. -The patient's assessment indicates that assisted coughing is needed to help remove secretions which will improve oxygenation. A 2-hour rest period at this time may allow the oxygen saturation to drop further. Humidification will not be helpful unless the secretions can be mobilized. Positioning on the left side may cause a further decrease in oxygen saturation because perfusion will be directed more word the more poorly ventilated lung.

A patient with respiratory failure has a respiratory rate of 6 breaths/minute and an oxygen saturation (SpO2) of 88%. The patient is increasingly lethargic. Which intervention will the nurse anticipate? a. Administration of 100% oxygen by non-rebreather mask. b. Endotracheal intubation and positive pressure ventilation. c. Insertion of a mini-tracheostomy with frequent suctioning. d. Initiation of continuous positive pressure ventilation (CPAP).

b. Endotracheal intubation and positive pressure ventilation. -The patient's lethargy, low respiratory rate, and SpO2 indicate the need for mechanical ventilation with ventilator-controlled respiratory rate. Administration of high flow oxygen will not be helpful because the patient's respiratory rate is so low. Insertion of a mini-tracheostomy will facilitate removal of secretions, but it will not improve the patient's respiratory rate or oxygenation. CPAP requires that the patient initiate an adequate respiratory rate to allow adequate gas exchange.

A patient with acute respiratory distress syndrome (ARDS) and acute kidney injury has the following medications ordered. Which medication should the nurse discuss with the HCP before giving? a. Pantoprazole (Protonix) 40 mg IV. b. Gentamicin (Garamycin) 60 mg IV. c. Sucralfate (Carafate) 1 g per NG tube. d. Methylprednisolone (Solu-Medrol) 60 mg IV.

b. Gentamicin (Garamycin) 60 mg IV. -Gentamicin, which is one of the amino glycoside antibiotics, is potentially nephrotoxic, and the nurse should clarify the drug and dosage with the HCP before administration. The other medications are appropriate for the patient with ARDS.

The clinic nurse makes a follow-up telephone call to a patient with asthma. The patient reports having a baseline peak flow reading of 600 L/min, and the current peak flow is 420 L/min. Which action should the nurse take first? a. Tell the patient to go to the hospital emergency department. b. Instruct the patient to use the prescribed albuterol (Ventolin HFA). c. Ask about recent exposure to any new allergens or asthma triggers. d. Question the patient about use of the prescribed inhaled corticosteroids.

b. Instruct the patient to use the prescribed albuterol (Ventolin HFA). The patient's peak flow is 70% of normal, indicating a need for immediate use of short-acting b2-adrenergic SABA medications.

A patient is receiving 35% O2 via a Venturi mask. To ensure the correct amount of O2 delivery, which action by the nurse is important? a. Teach the patient to keep the mask on during meals. b. Keep the air entrainment ports clean and unobstructed. c. Give a high enough flow rate to keep the bag from collapsing. d. Drain moisture condensation from the corrugated tubing every hour.

b. Keep the air entrainment ports clean and unobstructed. The air entrainment ports regulate the O2 percentage delivered to the patient, so they must be unobstructed.

Following a thymectomy, a 62-year-old male patient with myasthenia gravis receives the usual dose of pyridostigmine (Mestinon). An hour later, the patient complains of nausea and severe abdominal cramps. Which action should the nurse take first? a. Auscultate the patient's bowel sounds. b. Notify the patient's HCP. c. Administer the prescribed PRN antiemetic drug. d. Give the scheduled dose of prednisone (Deltasone).

b. Notify the patient's HCP. -The patient's history and symptoms indicate a possible cholinergic crisis. The HCP should be notified immediately, and it is likely that atropine will be prescribed. The other actions will be appropriate if the patient is not experiencing a cholinergic crisis.

A patient admitted with acute respiratory failure has a nursing diagnosis of ineffective airway clearance related to thick secretions. Which action is a priority for the nurse to include in the plan of care? a. Encourage use of incentive spirometer. b. Offer the patient fluids at frequent intervals. c. Teach the patient the importance of ambulation. d. Titrate oxygen level to keep O2 saturation >93%.

b. Offer the patient fluids at frequent intervals. -Because the reason for the poor airway clearance is the thick secretions, the best action will be to encourage the patient to improve oral fluid intake. Patients should be instructed to use the incentive spirometer on a regular basis (e.g., every hour) in order to facilitate the clearance of the secretions. The other actions may also be helpful in improving the patient's gas exchange, but they do not address the thick secretions that are causing the poor airway clearance.

Which finding by the nurse for a patient with a nursing diagnosis of *impaired gas exchange* will be most useful in evaluating the effectiveness of treatment? a. Even, unlabored respirations c. Absence of wheezes or crackles b. Pulse oximetry reading of 92% d. Respiratory rate of 18 breaths/min

b. Pulse oximetry reading of 92% For the nursing diagnosis of *impaired gas exchange*, the best data for evaluation are arterial blood gases (ABGs) or *pulse oximetry*.

The nurse is caring for a patient who is intubated and receiving positive pressure ventilation to treat ARDS. Which finding is most important to report to the HCP? a. Blood urea nitrogen (BUN) level 32 mg/dL. b. Red-brown drainage from orogastric tube. c. Scattered coarse crackles heard throughout lungs. d. Arterial blood gases: pH 7.31, PaCO2 50, PaO2 68.

b. Red-brown drainage from orogastric tube. -The nasogastric drainage indicates possible gastrointestinal bleeding and/or stress ulcer, and should be reported. The pH and PaCO2 are slightly abnormal, but current guidelines advocating for permissive hypercapnia indicate that these would not indicate an immediate need for a change in therapy. The BUN is slightly elevated but does not indicate an immediate need for action. Adventitious breath sounds are commonly heard in patients with ARDS.

The home health nurse is visiting a patient with *chronic obstructive pulmonary disease (COPD)*. Which nursing action is appropriate to implement for a nursing diagnosis of *impaired breathing pattern related to anxiety*? a. Titrate O2 to keep saturation at least 90%. b. Teach the patient how to use pursed-lip breathing. c. Discuss a high-protein, high-calorie diet with the patient. d. Suggest the use of over-the-counter sedative medications.

b. Teach the patient how to use pursed-lip breathing.

Which action will the nurse plan to take for a 40-year-old patient with MS who has urinary retention caused by a flaccid bladder? a. Decrease the patient's evening fluid intake. b. Teach the patient how to use the Crede method. c. Suggest the use of adult incontinence briefs for nighttime only. d. Assist the patient to the commode every 2 hours during the day.

b. Teach the patient how to use the Crede method. -The Crede method can be used to improve bladder emptying. Decreasing fluid intake will not improve bladder emptying and may increase risk for UTI and dehydration. The use of incontinence briefs and frequent toileting will not improve bladder emptying.

A patient with chronic obstructive pulmonary disease (COPD) has coarse crackles throughout the lung fields and a chronic, nonproductive cough. Which nursing intervention will be most effective? a. Change the O2 flow rate to the highest prescribed rate. b. Teach the patient to use the Flutter airway clearance device. c. Reinforce the ongoing use of pursed-lip breathing techniques. d. Teach the patient about consistent use of inhaled corticosteroids.

b. Teach the patient to use the Flutter airway clearance device. Airway clearance devices assist with moving mucus into larger airways, where it can more easily be expectorated.

A nurse is caring for a patient with ARDS who is being treated with mechanical ventilation and high levels of positive end-expiratory pressure (PEEP). Which assessment finding by the nurse indicates that the PEEP may need to be reduced? a. The patient's PaO2 is 50 mmHg and the SaO2 is 88%. b. The patient has subcutaneous emphysema on the upper thorax. c. The patient has bronchial breath sounds in both lung fields. d. The patient has a first-degree atrioventricular heart block with a heart rate of 58.

b. The patient has subcutaneous emphysema on the upper thorax. -The subcutaneous emphysema indicates barotrauma caused by positive pressure ventilation and PEEP. Bradycardia, hypoxemia, and bronchial breath sounds are all concerns and will need to be addressed, but they are not specific indications that PEEP should be reduced.

The nurse teaches a patient about *pursed-lip breathing*. Which action by the patient would indicate to the nurse that further teaching is needed? a. The patient inhales slowly through the nose. b. The patient puffs up the cheeks while exhaling. c. The patient practices by blowing through a straw. d. The patient's ratio of inhalation to exhalation is 1:3.

b. The patient puffs up the cheeks while exhaling. The patient should relax the facial muscles without puffing the cheeks while doing pursed-lip breathing.

When a 74-year-old patient is seen in the health clinic with new development of a stooped posture, shuffling gait, and pill rolling-type trim, the nurse will anticipate teaching the patient about a. oral corticosteroids. b. antiparkinsonian drugs. c. magnetic resonance imagine (MRI). d. electroencephalogram (EEG) testing.

b. antiparkinsonian drugs. -The diagnosis of Parkinson's is made when two of the three characteristic manifestations of tremor, rigidity, and bradykinesia are present. The confirmation of the diagnosis is made on the basis of improvement when antiparkinsonian drugs are administered. This patient has symptoms of tremor and bradykinesia. The next anticipated step will be treatment with medications. MRI and EEG are not useful in diagnosing Parkinson's disease, and corticosteroid therapy is not used to treat it.

When obtaining a health history and physical assessment for a 36-year-old female patient with possible multiple sclerosis (MS), the nurse should a. assess for the presence of chest pain. b. inquire about urinary tract problems. c. inspect the skin for rashes or discoloration. d. ask the patient about any increase in libido.

b. inquire about urinary tract problems. -Urinary tract problems with incontinence or retention are common symptoms of MS. Chest pain and skin rashes are not symptoms of MS. A decrease in libido is common with MS.

The nurse will assess a 67-year-old patient who is experiencing a cluster headache for a. nuchal rigidity. b. unilateral ptosis. c. projectile vomiting. d. throbbing, bilateral facial pain.

b. unilateral ptosis. -Unilateral eye edema, tearing, and ptosis are characteristics of cluster headaches. Nuchal rigidity suggests meningeal irritation, such as occurs with meningitis. Although nausea and vomiting may occur with migraine headaches, projectile vomiting is more consistent with increased intracranial pressure (ICP). Unilateral sharp, stabbing pain, rather than throbbing pain, is characteristic of cluster headaches.

The nurse determines that teaching about management of migraine headaches has been effective when the patient says which of the following? a. "I can take the Topiramate (Topamax) as soon as a headache starts." b. "A glass of wine might help me relax and prevent a headache." c. "I will lie down someplace dark and quiet when the headache begins." d. "I should avoid taking aspirin and sumatriptan (Imitrex) at the same time."

c. "I will lie down someplace dark and quiet when the headache begins." -It is recommended that the patient with a migraine rest in a dark, quiet area. Topiramate (Topamax) is used to prevent migraines and must be taken several months to determine effectiveness. Aspirin or other NSAID's can be taken with the triptans. Alcohol may precipitate migraine headaches.

Which instruction should the nurse include in an exercise teaching plan for a patient with chronic obstructive pulmonary disease (COPD)? a. "Avoid upper body exercise to prevent dyspnea." b. "Stop exercising if you start to feel short of breath." c. "Use the bronchodilator before you start to exercise." d. "Breathe in and out through the mouth while you exercise."

c. "Use the bronchodilator before you start to exercise." Use of a bronchodilator before exercise improves airflow for some patients and is recommended.

A patient seen in the asthma clinic has recorded daily peak flow rates that are 75% of the baseline. Which action will the nurse plan to take next? a. Increase the dose of the leukotriene inhibitor. b. Teach the patient about the use of oral corticosteroids. c. Administer a bronchodilator and recheck the peak flow. d. Instruct the patient to keep the scheduled follow-up appointment.

c. Administer a bronchodilator and recheck the peak flow. The patient's peak flow reading indicates that the condition is worsening (yellow zone). The patient should take the bronchodilator and recheck the peak flow.

To evaluate the effectiveness of ordered interventions for a patient with ventilatory failure, which diagnostic test will be most useful to the nurse? a. Chest x-ray. b. Oxygen saturation. c. Arterial blood gas analysis. d. Central venous pressure monitoring.

c. Arterial blood gas analysis. -Arterial blood gas (ABG) analysis is most useful in this setting because ventilatory failure causes problems with CO2 retention, and ABGs provide information about the PaCO2 and pH. The other tests may also be done to help in assessing oxygenation or determining that cause of the patient's ventilatory failure.

A 22-year-old patient seen at the health clinic with a severe migraine headache tells the nurse about having other similar headaches recently. Which initial action should the nurse take? a. Teach about the use of triptan drugs. b. Refer the patient for stress counseling. c. Ask the patient to keep a headache diary. d. Suggest the use of muscle-relaxation techniques.

c. Ask the patient to keep a headache diary. -The initial nursing action should be further assessment of the precipitating causes of the headaches, quality, and location of pain, etc. Stress reduction, muscle relaxation, and the triptan drugs may be helpful, but more assessment is needed first.

A 49-year-old patient with multiple sclerosis (MS) is to begin treatment with glatiramer acetate (Copaxone). Which information will the nurse include in patient teaching? a. Recommendation to drink at least 4 L of fluid daily. b. Need to avoid driving or operating heavy machinery. c. How to draw up and administer injections of the medication. d. Use of contraceptive methods other than oral contraceptives.

c. How to draw up and administer injections of the medication. -Copaxone is administered by self-injection. Oral contraceptives are an appropriate choice for birth control. There is no need to avoid driving or drink large fluid volumes when taking glatiramer.

A patient hospitalized with chronic obstructive pulmonary disease (COPD) is being discharged home on O2 therapy. Which instruction should the nurse include in the discharge teaching? a. Travel is not possible with the use of O2 devices. b. O2 flow should be increased if the patient has more dyspnea. c. O2 use can improve the patient's prognosis and quality of life. d. Storage of O2 requires large metals tanks that each last 4 to 6 hours.

c. O2 use can improve the patient's prognosis and quality of life.

The nurse documents the vital signs for a patient admitted 2 days ago with gram-negative sepsis: temperature 101.2 F, blood pressure 90/56 mmHg, pulse 92, respirations 34. Which action should the nurse take next? a. Give the scheduled IV antibiotic. b. Give the PRN acetaminophen (Tylenol). c. Obtain oxygen saturation using pulse oximetry. d. Notify the HCP of the patient's vital signs.

c. Obtain oxygen saturation using pulse oximetry. -The patient's increased respiratory rate in combination with the admission diagnosis of gram-negative sepsis indicates that acute respiratory distress syndrome (ARDS) may be developing. The nurse should check for hypoxemia, a hallmark of ARDS. The HCP should be notified after further assessment of the patient. Giving the scheduled antibiotic and the PRN acetaminophen will also be done, but they are not the highest priority for a patient who may be developing ARDS.

A patient with chronic obstructive pulmonary disease (COPD) has a nursing diagnosis of imbalanced nutrition: less than body requirements. Which intervention would be most appropriate for the nurse to include in the plan of care? a. Encourage increased intake of whole grains. b. Increase the patient's intake of fruits and fruit juices. c. Offer high-calorie protein snacks between meals and at bedtime. d. Assist the patient in choosing foods with high vegetable content.

c. Offer high-calorie protein snacks between meals and at bedtime. Eating small amounts more frequently (as occurs with snacking) will increase caloric intake by decreasing the fatigue and feelings of fullness associated with large meals.

Which assessment is most important for the nurse to make regarding a patient with myasthenia gravis? a. Pupil size. b. Grip strength. c. Respiratory effort. d. Level of consciousness.

c. Respiratory effort. -Because respiratory insufficiency may be life threatening, it will be most important to monitor respiratory function. The other data also will be assessed but are not as critical.

A patient newly diagnosed with asthma is being discharged. The nurse anticipates including which topic in the discharge teaching? a. Use of long-acting b-adrenergic medications c. Self-administration of inhaled corticosteroids b. Side effects of sustained-release theophylline d. Complications associated with O2 therapy

c. Self-administration of inhaled corticosteroids Inhaled corticosteroids are more effective in improving asthma than any other drug and are indicated for all patients with persistent asthma

The nurse interviews a patient with a new diagnosis of chronic obstructive pulmonary disease (COPD). Which information is *most* specific in confirming a diagnosis of *chronic bronchitis*? a. The patient tells the nurse about a family history of bronchitis. b. The patient indicates a 30 pack-year cigarette smoking history. c. The patient reports a productive cough for 3 months every winter. d. The patient denies having respiratory problems until the past 12 months.

c. The patient reports a productive cough for 3 months every winter. A diagnosis of *chronic bronchitis* is based on a history of having a *productive cough for 3 months* for at least *2 consecutive years*.

The nurse is caring for a patient with *chronic obstructive pulmonary disease (COPD)*. Which information obtained from the patient would prompt the nurse to consult with the health care provider before administering the prescribed *theophylline*? a. The patient reports a recent 15-lb weight gain. b. The patient denies shortness of breath at present. c. The patient takes cimetidine (Tagamet HB) daily. d. The patient complains of coughing up green mucus.

c. The patient takes cimetidine (Tagamet HB) *Cimetidine interferes* with the metabolism of *theophylline*, and concomitant administration may lead rapidly to *theophylline toxicity*.

The nurse takes an admission history on a patient with possible asthma who has new-onset wheezing and shortness of breath. Which information may indicate a need for a change in therapy? a. The patient has chronic inflammatory bowel disease. b. The patient has a history of pneumonia 6 months ago. c. The patient takes propranolol (Inderal) for hypertension. d. The patient uses acetaminophen (Tylenol) for headaches.

c. The patient takes propranolol (Inderal) for hypertension. b-Blockers such as propranolol can cause bronchospasm in some patients with asthma.

A patient with COPD arrives in the emergency department complaining of shortness of breath and dyspnea on minimal exertion. Which assessment finding by the nurse is most important to report to the HCP? a. The patient has bibasilar lung crackles. b. The patient is sitting in the tripod position. c. The patient's respirations have decreased from 30 to 10 breaths/minute. d. The patient's pulse oximetry indicates an O2 saturation of 91%.

c. The patient's respirations have decreased from 30 to 10 breaths/minute. -A decrease in the respiratory rate in a patient with respiratory distress suggests the onset of fatigue and a high risk for respiratory arrest. Therefore immediate action such as positive pressure ventilation is needed. Patients who are experiencing respiratory distress frequently sit in the tripod position because it decreases the work of breathing. Crackles in the lung bases may be the baseline for a patient with COPD. An oxygen saturation of 91% is common in patients with COPD and will provide adequate gas exchange and tissue oxygenation.

While the nurse is transporting a patient on a stretcher to the radiology department, the patient beings having a tonic-clonic seizure. Which action should the nurse take? a. Insert an oral airway during the seizure to maintain a patent airway. b. Restrain the patient's arms and legs to prevent injury during the seizure. c. Time and observe and record the details of the seizure and postictal state. d. Avoid touching the patient to prevent further devours system stimulation.

c. Time and observe and record the details of the seizure and postictal state. -Because the diagnosis and treatment of seizures frequently are based on the description of the seizure, recording the length and details of the seizure is important. Insertion of an oral airway and restraining the patient during the seizure are contraindicated. The nurse may need to move the patient to decrease the risk of injury during the seizure.

During change-of-shift report on a medical unit, the nurse learns that a patient with aspiration pneumonia who was admitted with respiratory distress has become increasingly agitated. Which action should the nurse take first? a. Give the prescribed PRN sedative drug. b. Offer reassurance and reorient the patient. c. Use pulse oximetry to check to oxygen saturation. d. Notify the HCP about the patient's status.

c. Use pulse oximetry to check to oxygen saturation. -Agitation may be an early indicator of hypoxemia. The other actions may also be appropriate depending on the findings about oxygen saturation.

A patient develops increasing dyspnea and hypoxemia 2 days after heart surgery. To determine whether the patient has ARDS or pulmonary edema caused by heart failure, the nurse will plan to assist with a. obtaining a ventilation-perfusion scan. b. drawing blood for ABGs. c. insertion of a pulmonary artery catheter. d. positioning the patient for a chest x-ray.

c. insertion of a pulmonary artery catheter. -Pulmonary artery wedge pressures are normal in the patient with ARDS because the fluid in the alveoli is caused by increased permeability of the alveolar-capillary membrane rather than by the backup of fluid from the lungs (as occurs in cardiogenic pulmonary edema). The other tests will not be helpful in differentiating cardiogenic from non-cardiogenic pulmonary edema.

A 40-year-old patient is diagnosed with early Huntington's disease (HD). When teaching the patient, spouse, and children about this disorder, the nurse will provide information about that a. use of levodopa-carbidopa (Sinemet) to help reduce HD symptoms. b. prophylactic antibiotics to decrease the risk for aspiration pneumonia. c. option of genetic testing for the patient's children to determine their own HD risks. d. lifestyle changes of improved nutrition and exercise that delay disease progression.

c. option of genetic testing for the patient's children to determine their own HD risks. -Genetic testing is available to determine whether an asymptomatic individual has the HD gene. The patient and family should be informed of the benefits and problems associated with genetic testing. Sinemet will increase symptoms of HD because HD involves an increase in dopamine. Antibiotic therapy will not reduce the risk for aspiration. There are no effective treatments or lifestyle changes that delay the progression of symptoms in HD.

After receiving change-of-shift report on a medical unit, which patient should the nurse assess first? a. A patient with cystic fibrosis who has thick, green-colored sputum. b. A patient with pneumonia who has crackles bilaterally in the lung bases. c. A patient with emphysema who has an oxygen saturation of 90% to 92%. d. A patient with septicemia who has intercostal and suprasternal retractions.

d. A patient with septicemia who has intercostal and suprasternal retractions. -This patient's history of septicemia and labored breathing suggest the onset of ARDS, which will require rapid intervention such as administration of oxygen and use of positive pressure ventilation. The other patients should also be assessed as quickly as possible, but their assessment data are typical of their disease processes and do not suggest deterioration in their status.

When assessing a patient with COPD, the nurse finds a new onset of agitation and confusion. Which action should the nurse take first? a. Notify the HCP. b. Check pupils for reaction to light. c. Attempt to calm and reorient the patient. d. Assess oxygenation using pulse oximetry.

d. Assess oxygenation using pulse oximetry. -Because agitation and confusion are frequently the initial indicators of hypoxemia, the nurse's initial action should be to assess oxygen saturation. The other actions are also appropriate, but assessment of oxygenation takes priority over other assessments and notification of the HCP.

A young adult patient with cystic fibrosis (CF) is admitted to the hospital with increased dyspnea. Which intervention should the nurse include in the plan of care? a. Schedule a sweat chloride test. b. Arrange for a hospice nurse visit. c. Place the patient on a low-sodium diet. d. Perform chest physiotherapy every 4 hours.

d. Perform chest physiotherapy every 4 hours. Routine scheduling of airway clearance techniques is an essential intervention for patients with CF.

Which information about a 60-year-old patient with MS indicates that the nurse should consult with the HCP before giving the prescribed dose of dalfampridine (Ampyra)? a. The patient has relapsing-remitting MS. b. The patient walks a mile a day for exercise. c. The patient complains of pain with neck flexion. d. The patient has an increased serum creatinine level.

d. The patient has an increased serum creatinine level. -Dalfampridine should not be given to patients with impaired renal function. The other information will not impact whether the dalfampridine should be administered.

The nurse teaches a patient who has *asthma* about *peak flow meter use*. Which action by the patient indicates that teaching was successful? a. The patient inhales rapidly through the peak flow meter mouthpiece. b. The patient takes montelukast (Singulair) for peak flows in the red zone. c. The patient calls the health care provider when the peak flow is in the green zone. d. The patient uses an albuterol (Ventolin HFA) inhaler for peak flows in the yellow zone.

d. The patient uses an *albuterol (Ventolin HFA)* inhaler for *peak flows* in the yellow zone. Readings in the yellow zone indicate a decrease in peak flow. The patient should use short-acting B2-adrenergic (SABA) medications.

A 62-year-old patient who has Parkinson's disease is taking bromocriptine (Parlodel). Which information obtained by the nurse may indicate a need for a decrease in the dose? a. The patient has a chronic dry cough. b. The patient has four loose stools in a day. c. The patient develops a deep vein thrombosis. d. The patient's blood pressure in 92/52 mmHg.

d. The patient's blood pressure in 92/52 mmHg. -Hypotension is an adverse effect of bromocriptine, and the nurse should check with the HCP before giving the medication. Diarrhea, cough, and DVT are not associated with bromocriptine use.

The nurse completes an admission assessment on a patient with asthma. Which information given by patient is indicates a need for a change in therapy? a. The patient uses albuterol (Ventolin HFA) before aerobic exercise. b. The patient says that the asthma symptoms are worse every spring. c. The patient's heart rate increases after using the albuterol (Ventolin HFA) inhaler. d. The patient's only medications are albuterol (Ventolin HFAl) and salmeterol

d. The patient's only medications are albuterol (Ventolin HFAl) and salmeterol Long-acting b2-agonists should be used only in patients who also are using an inhaled corticosteroid for long-term control

Which information will the nurse include in the asthma teaching plan for a patient being discharged? a. Use the inhaled corticosteroid when shortness of breath occurs. b. Inhale slowly and deeply when using the dry powder inhaler (DPI). c. Hold your breath for 5 seconds after using the bronchodilator inhaler. d. Tremors are an expected side effect of rapidly acting bronchodilators.

d. Tremors are an expected side effect of rapidly acting bronchodilators. *Tremors* are a common side effect of *short-acting 2-adrenergic (SABA)* medications and not a reason to avoid using the SABA inhaler.

A 76-year-old patient is being treated with carbidopa/levodopa (Sinemet) for Parkinson's disease. Which information is most important for the nurse to report to the the HCP? a. Shuffling gait. b. Tremor at rest. c. Cogwheel rigidity of limbs. d. Uncontrolled head movement.

d. Uncontrolled head movement. -Dyskinesia is an adverse effect of the Sinemet, indicating a need for a change in medication or decrease in dose. The other findings are typical with Parkinson's disease.

The nurse develops a teaching plan to help increase activity tolerance at home for an older adult with severe chronic obstructive pulmonary disease (COPD). Which instructions would be appropriate for the nurse to include in the plan of care? a. Stop exercising when you feel short of breath. b. Walk until pulse rate exceeds 130 beats/minute. c. Limit exercise to activities of daily living (ADLs). d. Walk 15 to 20 minutes a day at least 3 times/week.

d. Walk 15 to 20 minutes a day at least 3 times/week.

A 38-year-old patient has returned home following rehabilitation for a spinal cord injury. The home care nurse notes that the spouse is performing many of the activities that the patient had been managing unassisted during rehabilitation. The most appropriate action by the nurse at this time is to a. remind the patient about the importance of independence in daily activities. b. tell the spouse to stop because the patient is able to perform activities independently. c. develop a plan to increase the patient's independence in consultation with the patient and the spouse. d. recognize that it is important for the spouse to be involved in the patient's care and encourage that participation.

develop a plan to increase the patient's independence in consultation with the patient and the spouse. The best action by the nurse will be to involve all the parties in developing an optimal plan of care. Because family members who will be assisting with the patient's ongoing care need to feel that their input is important, telling the spouse that the patient can perform activities independently is not the best choice. Reminding the patient about the importance of independence may not change the behaviors of the spouse. Supporting the activities of the spouse will lead to ongoing dependency by the patient.

A patient who had a C7 spinal cord injury a week ago has a weak cough effort and audible rhonchi. The initial intervention by the nurse should be to a. administer humidified oxygen by mask. b. suction the patient's mouth and nasopharynx. c. push upward on the epigastric area as the patient coughs. d. encourage incentive spirometry every 2 hours during the day.

push upward on the epigastric area as the patient coughs. Because the cough effort is poor, the initial action should be to use assisted coughing techniques to improve the ability to mobilize secretions. Administration of oxygen will improve oxygenation, but the data do not indicate hypoxemia. The use of the spirometer may improve respiratory status, but the patient's ability to take deep breaths is limited by the loss of intercostal muscle function. Suctioning may be needed if the patient is unable to expel secretions by coughing but should not be the nurse's first action.

A 68-year-old patient hospitalized with a new diagnosis of Guillain-Barré syndrome has numbness and weakness of both feet. The nurse will anticipate teaching the patient about a. intubation and mechanical ventilation. b. administration of corticosteroid drugs. c. insertion of a nasogastric (NG) feeding tube. d. infusion of immunoglobulin (Sandoglobulin).

infusion of immunoglobulin (Sandoglobulin). Because the Guillain-Barré syndrome is in the earliest stages (as evidenced by the symptoms), use of high-dose immunoglobulin is appropriate to reduce the extent and length of symptoms. Mechanical ventilation and tube feedings may be used later in the progression of the syndrome but are not needed now. Corticosteroid use is not helpful in reducing the duration or symptoms of the syndrome.

When the nurse is developing a rehabilitation plan for a 30-year-old patient with a C6 spinal cord injury, an appropriate goal is that the patient will be able to a. drive a car with powered hand controls. b. push a manual wheelchair on a flat surface. c. turn and reposition independently when in bed. d. transfer independently to and from a wheelchair.

push a manual wheelchair on a flat surface. The patient with a C6 injury will be able to use the hands to push a wheelchair on flat, smooth surfaces. Because flexion of the thumb and fingers is minimal, the patient will not be able to grasp a wheelchair during transfer, drive a car with powered hand controls, or turn independently in bed.

A 32-year-old pregnant patient with Bell's palsy refuses to eat while others are present because of embarrassment about drooling. The best response by the nurse is to a. respect the patient's feelings and arrange for privacy at mealtimes. b. teach the patient to chew food on the unaffected side of the mouth. c. offer the patient liquid nutritional supplements at frequent intervals. d. discuss the patient's concerns with visitors who arrive at mealtimes.

respect the patient's feelings and arrange for privacy at mealtimes. The patient's desire for privacy should be respected to encourage adequate nutrition and reduce patient embarrassment. Liquid supplements will reduce the patient's enjoyment of the taste of food. It would be inappropriate for the nurse to discuss the patient's embarrassment with visitors unless the patient wishes to share this information. Chewing on the unaffected side of the mouth will enhance nutrition and enjoyment of food but will not decrease the drooling.


Set pelajaran terkait

Communication in Sport Final Exam

View Set

NCM 106: DRUGS AFFECTING THE REPRODUCTIVE SYSTEM (Doc Salinas)

View Set

<2> 生活・金融機関 漢字リスト

View Set